1. The ratification of the Sixteenth Amendment to the U.S. Constitution was necessary to validate the Federal on corporations.

a. True *b. False

2. Before the Sixteenth Amendment to the Constitution was ratified, there was no valid Federal income tax on individuals.

a. True *b. False

3. The first income tax on individuals (after the ratification of the Sixteenth Amendment to the Constitution) levied tax rates from a low of 2% to a high of 6%.

*a. True b. False

4. The Federal income tax on individuals generates more revenue than the Federal income tax on corporations.

*a. True b. False

5. The pay-as-you-go feature of the Federal income tax on individuals conforms to Adam Smith’s canon of certainty.

a. True *b. False

6. Because the law is complicated, most individual taxpayers are not able to complete their Federal income tax returns without outside assistance.

*a. True b. False

7. The FICA tax (Medicare component) on wages is progressive since the tax due increases as wages increase.

a. True *b. False

8. The Federal estate and gift taxes are examples of progressive taxes.

*a. True b. False

9. The Federal excise tax on cigarettes is an example of a proportional tax.

*a. True b. False

10. Currently, the Federal income tax is less progressive than it ever has been in the past.

a. True *b. False

11. Mona inherits her mother’s personal residence, which she converts to a furnished rent house. These changes should affect the amount of ad valorem property taxes levied on the properties.

*a. True b. False

12. A fixture will be subject to the ad valorem tax on personalty rather than the ad valorem tax on realty.

a. True *b. False

13. Even if property tax rates are not changed, the amount of ad valorem taxes imposed on realty may not remain the same.

*a. True b. False

14. The ad valorem tax on personal use personalty is more often avoided by taxpayers than the ad valorem tax on business use personalty.

*a. True b. False

15. A Federal excise tax is no longer imposed on admission to theaters.

*a. True b. False

16. There is a Federal excise tax on hotel occupancy.

a. True *b. False

17. The Federal gas-guzzler tax applies only to automobiles manufactured overseas and imported into the U.S.

a. True *b. False

18. Like the Federal counterpart, the amount of the state excise taxes on gasoline varies from state to state.

a. True *b. False

19. Not all of the states that impose a general sales tax also have a .

a. True *b. False

20. Sales made by mail order are not exempt from the application of a general sales (or use) tax.

*a. True b. False

21. Two persons who live in the same state but in different counties may not be subject to the same general sales tax rate.

*a. True b. False

22. States impose either a or a general sales tax, but not both types of taxes.

a. True *b. False

23. A safe and easy way for a taxpayer to avoid local and state sales taxes is to make the purchase in a state that levies no such taxes.

a. True *b. False

24. On transfers by death, the Federal government relies on an estate tax, while states impose an estate tax, an inheritance tax, both taxes, or neither tax.

*a. True b. False

25. An inheritance tax is a tax on a decedent’s right to pass property at death.

a. True *b. False

26. One of the major reasons for the enactment of the Federal estate tax was to prevent large amounts of wealth from being accumulated within the family unit.

*a. True b. False

27. Under Clint’s will, all of his property passes to either the Lutheran Church or to his wife. No Federal estate tax will be due on Clint’s death in 2013.

*a. True b. False

28. Under the usual state inheritance tax, two heirs, a cousin and a son of the deceased, would not be taxed at the same rate.

*a. True b. False

29. The annual exclusion, currently $14,000, is available for gift and estate tax purposes.

a. True *b. False

30. In 2012, José, a widower, sells land (fair market value of $100,000) to his daughter, Linda, for $50,000. José has made a taxable gift of $50,000.

a. True *b. False

31. Julius, a married taxpayer, makes gifts to each of his six children. A maximum of twelve annual exclusions could be allowed as to these gifts.

*a. True b. False

32. One of the motivations for making a gift is to save on income taxes.

*a. True b. False

33. The formula for the Federal income tax on corporations is the same as that applicable to individuals.

a. True *b. False

34. A state income tax can be imposed on nonresident taxpayers who earn income within the state or on an itinerant basis.

*a. True b. False

35. For state income tax purposes, a majority of states allow a deduction for Federal income taxes.

a. True *b. False

36. Some states use their state income tax return as a means of collecting unpaid sales and use taxes.

*a. True b. False

37. No state has offered an income tax amnesty program more than once.

a. True *b. False

38. For Federal income tax purposes, there never has been a general amnesty period.

*a. True b. False

39. Under state amnesty programs, all delinquent and unpaid income taxes are forgiven.

a. True *b. False

40. When a state decouples from a Federal tax provision, it means that this provision will not apply for state income tax purposes.

*a. True b. False

41. The principal objective of the FUTA tax is to provide some measure of retirement security.

a. True *b. False

42. Currently, the tax base for the Social Security component of the FICA is not limited to a dollar amount.

a. True *b. False

43. A parent employs his twin daughters, age 17, in his sole proprietorship. The daughters are not subject to FICA coverage.

*a. True b. False

44. Unlike FICA, FUTA requires that employers comply with state as well as Federal rules.

*a. True b. False

45. A major advantage of a type of income tax is its simplicity.

*a. True b. False

46. The value added tax (VAT) has not had wide acceptance in the international community.

a. True *b. False

47. If more IRS audits are producing a greater number of no change results, this indicates increased compliance on the part of taxpayers.

a. True *b. False

48. The amount of a taxpayer’s itemized deductions will increase the chance of being audited by the IRS.

*a. True b. False

49. In an office audit, the audit by the IRS takes place at the office of the taxpayer.

a. True *b. False

50. The IRS agent auditing the return will issue an RAR even if the taxpayer owes no additional taxes.

*a. True b. False

51. If a “special agent” becomes involved in the audit of a return, this indicates that the IRS suspects that fraud is involved.

*a. True b. False

52. If a taxpayer files early (i.e., before the due date of the return), the statute of limitations on assessments begins on the date the return is filed.

a. True *b. False

53. For omissions from gross income in excess of 25% of that reported, there is no statute of limitations on additional income tax assessments by the IRS.

a. True *b. False

54. If an income tax return is not filed by a taxpayer, there is no statute of limitations on assessments of tax by the IRS.

*a. True b. False

55. If fraud is involved, there is no time limit on the assessment of a deficiency by the IRS.

*a. True b. False

56. The IRS is required to redetermine the interest rate on underpayments and overpayments once a year.

a. True *b. False

57. A calendar year taxpayer files his 2012 Federal income tax return on March 5, 2013. The return reflects an overpayment of $6,000, and the taxpayer requests a refund of this amount. The refund is paid on May 17, 2013. The refund need not include interest.

*a. True b. False

58. For individual taxpayers, the interest rate for income tax refunds (overpayments) is the same as that applicable to assessments (underpayments).

*a. True b. False

59. During any month in which both the failure to file penalty and the failure to pay penalty apply, the failure to file penalty is increased by the amount of the failure to pay penalty.

a. True *b. False

60. When interest is charged on a deficiency, any part of a month counts as a full month.

a. True *b. False

61. For the negligence penalty to apply, the underpayment must be caused by intentional disregard of rules and regulations without intent to defraud.

*a. True b. False

62. Upon audit by the IRS, Faith is assessed a deficiency of $40,000 of which $25,000 is attributable to negligence. The 20% negligence penalty will apply to $25,000.

*a. True b. False

63. If the tax deficiency is attributable to fraud, the negligence penalty will not be imposed.

*a. True b. False

64. The civil fraud penalty can entail large fines and possible incarceration.

a. True *b. False

65. Even though a client refuses to correct an error on a past return, it may be possible for a practitioner to continue to prepare returns for the client.

*a. True b. False

66. In preparing an income tax return, the use of a client’s estimates is not permitted.

a. True *b. False

67. In preparing a tax return, all questions on the return must be answered.

a. True *b. False

68. A CPA firm in sends many of its less complex tax returns to be prepared by a group of accountants in India. If certain procedures are followed, this outsourcing of tax return preparation is proper.

*a. True b. False

69. The objective of pay-as-you-go (paygo) is to achieve revenue neutrality.

*a. True b. False

70. When Congress enacts a tax cut that is phased in over a period of years, revenue neutrality is achieved.

a. True *b. False

71. A tax cut enacted by Congress that contains a sunset provision will make the tax cut temporary.

*a. True b. False

72. The tax law provides various tax credits, deductions, and exclusions that are designed to encourage taxpayers to obtain additional education. These provisions can be justified on both economic and equity grounds.

a. True *b. False

73. Various tax provisions encourage the creation of certain types of retirement plans. Such provisions can be justified on both economic and social grounds.

*a. True b. False

74. To lessen, or eliminate, the effect of multiple taxation, a taxpayer who is subject to both foreign and U.S. income taxes on the same income is allowed either a deduction or a credit for the foreign tax paid.

*a. True b. False

75. To mitigate the effect of the annual accounting period concept, the tax law permits the carryforward to other years of the excess charitable contributions of a particular year.

*a. True b. False

76. Jason’s business warehouse is destroyed by fire. As the insurance proceeds exceed the basis of the property, a gain results. If Jason shortly reinvests the proceeds in a new warehouse, no gain is recognized due to the application of the wherewithal to pay concept.

*a. True b. False

77. As it is consistent with the wherewithal to pay concept, the tax law requires a seller to recognize gain in the year the installment sale occurs.

a. True *b. False

78. Stealth taxes have the effect of generating additional taxes from all taxpayers.

a. True *b. False

79. A provision in the law that compels accrual basis taxpayers to pay a tax on prepaid income in the year received and not when earned is consistent with generally accepted accounting principles.

a. True *b. False

80. As a matter of administrative convenience, the IRS would prefer to have Congress decrease (rather than increase) the amount of the standard deduction allowed to individual taxpayers.

a. True *b. False

81. In cases of doubt, courts have held that tax relief provisions should be broadly construed in favor of taxpayers.

a. True *b. False

82. On occasion, Congress has to enact legislation that clarifies the tax law in order to change a result reached by the U.S. Supreme Court.

*a. True b. False

83. Which, if any, of the following statements best describes the history of the Federal income tax?

a. It did not exist during the Civil War. *b. The Federal income tax on corporations was held by the U.S. Supreme Court to be allowable under the U.S. Constitution. c. The Federal income tax on individuals was held by the U.S. Supreme Court to be allowable under the U.S. Constitution. d. Both the Federal income tax on individuals and on corporations was held by the U.S. Supreme Court to be contrary to the U.S. Constitution. e. None of the above.

84. Which, if any, is not one of Adam Smith’s canons of taxation?

a. Economy. b. Certainty. c. Convenience. *d. Simplicity. e. Equality.

85. Which, if any, of the following taxes are proportional (rather than progressive)?

*a. State general sales tax. b. Federal corporate income tax. c. Federal estate tax. d. Federal gift tax. e. All of the above.

86. Which, if any, of the following transactions will increase a taxing jurisdiction’s revenue from the ad valorem tax imposed on real estate?

a. A resident dies and leaves his farm to his church. b. A large property owner issues a conservation easement as to some of her land. *c. A tax holiday issued 10 years ago has expired. d. A bankrupt motel is acquired by the Red Cross and is to be used to provide housing for homeless persons. e. None of the above.

87. Which, if any, of the following transactions will decrease a taxing jurisdiction’s ad valorem tax revenue imposed on real estate?

*a. A tax holiday is granted to an out-of-state business that is searching for a new factory site. b. An abandoned church is converted to a restaurant. c. A public school is razed and turned into a city park. d. A local university sells a dormitory that will be converted for use as an apartment building. e. None of the above.

88. Which, if any, of the following is a typical characteristic of an ad valorem tax on personalty?

a. Taxpayer compliance is greater for personal use property than for business use property. *b. The tax on automobiles sometimes considers the age of the vehicle. c. Most states impose a tax on intangibles. d. The tax on intangibles generates considerable revenue since it is difficult for taxpayers to avoid. e. None of the above.

89. Federal excise taxes that are no longer imposed include:

a. Tax on air travel. b. Tax on wagering. c. Tax on the manufacture of sporting equipment. d. Tax on alcohol. *e. None of the above.

90. Taxes not imposed by the Federal government include:

a. Tobacco excise tax. b. Customs duties (tariffs on imports). *c. Tax on rent cars. d. Gas guzzler tax. e. None of the above.

91. Taxes levied by both states and the Federal government include:

a. General sales tax. b. Custom duties. c. Hotel occupancy tax. d. Franchise tax. *e. None of the above.

92. Taxes levied by all states include:

*a. Tobacco excise tax. b. Individual income tax. c. Inheritance tax. d. General sales tax. e. None of the above.

93. A use tax is imposed by:

a. The Federal government and all states. b. The Federal government and a majority of the states. c. All states and not the Federal government. *d. Most of the states and not the Federal government. e. None of the above.

94. Burt and Lisa are married and live in a common law state. Burt wants to make gifts to their four children in 2013. What is the maximum amount of the annual exclusion they will be allowed for these gifts?

a. $14,000. b. $28,000. c. $56,000. *d. $112,000. e. None of the above.

95. Property can be transferred within the family group by gift or at death. One motivation for preferring the gift approach is:

a. To take advantage of the higher unified transfer tax credit available under the gift tax. b. To avoid a future decline in value of the property transferred. *c. To take advantage of the per donee annual exclusion. d. To shift income to higher bracket donees. e. None of the above.

96. Indicate which, if any, statement is incorrect. State income taxes:

a. Can piggyback to the Federal version. *b. Cannot apply to visiting nonresidents. c. Can decouple from the Federal version. d. Can provide occasional amnesty programs. e. None of the above.

97. State income taxes generally can be characterized by:

*a. The same date for filing as the Federal income tax. b. No provision for withholding procedures. c. Allowance of a deduction for Federal income taxes paid. d. Applying only to individuals and not applying to corporations. e. None of the above.

98. A characteristic of FICA is that:

a. It does not apply when one spouse works for the other spouse. b. It is imposed only on the employer. c. It provides a modest source of income in the event of loss of employment. d. It is administered by both state and Federal governments. *e. None of the above.

99. A characteristic of FUTA is that:

a. It is imposed on both employer and employee. b. It is imposed solely on the employee. *c. Compliance requires following guidelines issued by both state and Federal regulatory authorities. d. It is applicable to spouses of employees but not to any children under age 18. e. None of the above.

100. The U.S. (either Federal, state, or local) does not impose:

a. Franchise taxes. b. Severance taxes. c. Occupational fees. d. Custom duties. *e. Export duties.

101. The proposed flat tax:

a. Would eliminate the income tax. *b. Would simplify the income tax. c. Would tax the increment in value as goods move through the production and manufacturing stages to the marketplace. d. Is a tax on consumption. e. None of the above.

102. A VAT (value added tax):

*a. Is regressive in its effect. b. Has not proved popular outside of the U.S. c. Is not a tax on consumption. d. Is used exclusively by third world (less developed) countries. e. None of the above.

103. Characteristics of the “Fair Tax” (i.e., national sales tax) include which, if any, of the following:

a. Abolition of the Federal individual (but not the corporate) income tax. b. Abolition of all Federal income taxes but retention of payroll taxes (including the self-employment tax). c. Abolition of all Federal income taxes and payroll taxes but retention of the Federal estate and gift taxes. *d. Abolition of all Federal income and payroll taxes as well as the Federal estate and gift taxes. e. None of the above.

104. In terms of probability, which of the following taxpayers would be least likely to be audited by the IRS?

a. Taxpayer owns and operates a check-cashing service. *b. Taxpayer is an employed electrician. c. Taxpayer just received a $3 million personal injury award as a result of a lawsuit. d. Taxpayer just won a $1 million slot machine jackpot at a Las Vegas casino. e. Taxpayer has been audited several times before.

105. Which of the following is a characteristic of the audit process?

a. Most taxpayer audits involve “special” agents. b. Self-employed taxpayers are less likely to be selected for audit than employed taxpayers. *c. Less important issues are handled by means of a correspondence audit. d. If a taxpayer disagrees with the IRS auditor’s finding, the only resort is to the courts. e. None of the above.

106. David files his tax return 45 days after the due date. Along with the return, David remits a check for $40,000 which is the balance of the tax owed. Disregarding the interest element, David’s total failure to file and to pay penalties are:

a. $400. b. $3,600. *c. $4,000. d. $4,400. e. None of the above.

107. A characteristic of the fraud penalties is:

a. When negligence and civil fraud apply to a deficiency, the negligence penalty predominates. *b. Criminal fraud can result in a fine and a prison sentence. c. The criminal fraud penalty is 75% of the deficiency attributable to the fraud. d. The IRS has the same burden of proof in the case of criminal fraud than with civil fraud. e. None of the above.

108. Regarding proper ethical guidelines, which (if any) of the following is correct?

*a. The use of client estimates in preparing a return may be acceptable. b. Under no circumstances should a question on a tax return be left unanswered. c. If a client has made a mistake in a prior year’s return and refuses to correct it, you should withdraw from the engagement. d. If the exact amount of a deduction is not certain (e.g., around mid-$600s) , it should be recorded as an odd amount (i.e., $649) so as to increase the appearance of greater certainty. e. None of the above.

109. Both economic and social considerations can be used to justify:

a. Favorable tax treatment for accident and health plans provided for employees and financed by employers. b. Disallowance of any deduction for expenditures deemed to be contrary to public policy (e.g., fines, penalties, illegal kickbacks, bribes to government officials). *c. Various tax credits, deductions, and exclusions that are designed to encourage taxpayers to obtain additional education. d. Allowance of a deduction for state and local income taxes paid. e. None of the above.

110. Social considerations can be used to justify:

*a. Allowance of a credit for child care expenses. b. Allowing excess capital losses to be carried over to other years. c. Allowing accelerated amortization for the cost of installing pollution control facilities. d. Allowing a Federal income tax deduction for state and local sales taxes. e. None of the above.

111. Allowing a domestic production activities deduction for certain manufacturing income can be justified:

a. As mitigating the effect of the annual accounting period concept. b. As promoting administrative feasibility. *c. By economic considerations. d. Based on the wherewithal to pay concept. e. None of the above.

112. Provisions in the tax law that promote energy conservation and more use of alternative (non-fossil) fuels can be justified by:

a. Political considerations. *b. Economic and social considerations. c. Promoting administrative feasibility. d. Encouragement of small business. e. None of the above.

113. Which, if any, of the following provisions cannot be justified as mitigating the effect of the annual accounting period concept?

*a. Nonrecognition of gain allowed for involuntary conversions. b. Net operating loss carryback and carryover provisions. c. Carry over of excess charitable contributions. d. Use of the installment method to recognize gain. e. Carry over of excess capital losses.

114. Which, if any, of the following provisions of the tax law cannot be justified as promoting administrative feasibility (simplifying the task of the IRS)?

a. Penalties are imposed for failure to file a return or pay a tax on time. b. Prepaid income is taxed in the year received and not in the year earned. c. Annual adjustments for indexation increases the amount of the standard deduction allowed. d. Casualty losses must exceed 10% of AGI to be deductible. *e. A deduction is allowed for charitable contributions.

115. A landlord leases property upon which the tenant makes improvements. The improvements are significant and are not made in lieu of rent. At the end of the lease, the value of the improvements are not income to the landlord. This rule is an example of:

a. A clear reflection of income result. b. The tax benefit rule. c. The arm’s length concept. *d. The wherewithal to pay concept. e. None of the above.

116. Match the statements that relate to each other. Note: Some choices may be used more than once or not at all.Jock taxDecouplingDIFTax fraud suspectedRevenue neutralityRARWherewithal to pay conceptMitigation of the annual accounting period conceptTax on transfers at death (inheritance type)Excise tax on tobaccoUse taxIncome tax amnestyImport taxes (customs duties)“Pay-as-you-go” (paygo)Export taxesState income tax applied to visiting nonresident Undoing the “piggyback” result No correct match provided IRS special agent Ideal budget goal as to new tax legislation “No change” is one possible result Deferral of gains from involuntary conversions Carryback and carryforward of net operating losses Imposed by some states but not the Federal government Imposed by all states and the Federal government Every state that has a general sales tax has one Imposed by some states but not the Federal government Imposed only by the Federal government Ideal budget goal as to new tax legislation No correct match provided

[a] 1. Jock tax [b] 2. Decoupling [c] 3. DIF [d] 4. Tax fraud suspected [e] 5. Revenue neutrality [f] 6. RAR [g] 7. Wherewithal to pay concept [h] 8. Mitigation of the annual accounting period concept [i] 9. Tax on transfers at death (inheritance type) [j] 10. Excise tax on tobacco [k] 11. Use tax [l] 12. Income tax amnesty [m] 13. Import taxes (customs duties) [n] 14. “Pay-as-you-go” (paygo) [o] 15. Export taxes

a. State income tax applied to visiting nonresident b. Undoing the “piggyback” result c. No correct match provided d. IRS special agent e. Ideal budget goal as to new tax legislation f. “No change” is one possible result g. Deferral of gains from involuntary conversions h. Carryback and carryforward of net operating losses i. Imposed by some states but not the Federal government j. Imposed by all states and the Federal government k. Every state that has a general sales tax has one l. Imposed by some states but not the Federal government m. Imposed only by the Federal government n. Ideal budget goal as to new tax legislation o. No correct match provided

117. Match the statements that relate to each other. Note: Some choices may be used more than once.Office auditField auditFailure to file penaltyFailure to pay penaltyNegligence penaltyCriminal fraud penaltyFraud and statute of limitationsEarly filing and statute of limitations (deficiency situations)Late filing and statute limitations (deficiency situations)No return and statute limitationsMore than 25% gross income omission and statute of limitationsInterest due on refundConducted at IRS office Conducted at taxpayer’s office 5% per month (25% limit) 0.5% per month (25% limit) 20% of underpayment No correct match provided No statute of limitations (period remains open) 3 years from due date of return 3 years from date return is filed No statute of limitations (period remains open) 6 years 45-day grace period allowed to IRS 75% of underpayment

[a] 1. Office audit [b] 2. Field audit [c] 3. Failure to file penalty [d] 4. Failure to pay penalty [e] 5. Negligence penalty [f] 6. Criminal fraud penalty [g] 7. Fraud and statute of limitations [h] 8. Early filing and statute of limitations (deficiency situations) [i] 9. Late filing and statute limitations (deficiency situations) [j] 10. No return and statute limitations [k] 11. More than 25% gross income omission and statute of limitations [l] 12. Interest due on refund

a. Conducted at IRS office b. Conducted at taxpayer’s office c. 5% per month (25% limit) d. 0.5% per month (25% limit) e. 20% of underpayment f. No correct match provided g. No statute of limitations (period remains open) h. 3 years from due date of return i. 3 years from date return is filed j. No statute of limitations (period remains open) k. 6 years l. 45-day grace period allowed to IRS m. 75% of underpayment

118. Using the choices provided below, show the justification for each provision of the tax law listed. A tax credit for amounts spent to furnish care for children while the parent is at work.Additional depreciation deduction allowed for the year the asset is acquired.Tax brackets are increased for inflation.A small business corporation can elect to avoid the corporate income tax.A deduction for contributions by an employee to certain retirement plans.A deduction for qualified tuition paid to obtain higher education.A deduction for certain expenses (interest and taxes) incident to home ownership.A Federal deduction for state and local income taxes paid.A deduction for certain income from manufacturing activities.A bribe to the local sheriff, although business related, is not deductible.Contributions to charitable organizations are deductible.A Federal deduction for state and local sales taxes paid.Tax credits available for the purchase of a vehicle that uses alternative (non-fossil) fuels.Tax credits for home improvements that conserve energy.More rapid expensing for tax purposes of the costs of installing pollution control devices.Social considerations Economic considerations Equity considerations Economic considerations Economic considerations Economic considerations Economic considerations Equity considerations Economic considerations Social considerations Social considerations Equity considerations Economic considerations Economic considerations Economic considerations

[a] 1. A tax credit for amounts spent to furnish care for children while the parent is at work. [b] 2. Additional depreciation deduction allowed for the year the asset is acquired. [c] 3. Tax brackets are increased for inflation. [d] 4. A small business corporation can elect to avoid the corporate income tax. [e] 5. A deduction for contributions by an employee to certain retirement plans. [f] 6. A deduction for qualified tuition paid to obtain higher education. [g] 7. A deduction for certain expenses (interest and taxes) incident to home ownership. [h] 8. A Federal deduction for state and local income taxes paid. [i] 9. A deduction for certain income from manufacturing activities. [j] 10. A bribe to the local sheriff, although business related, is not deductible. [k] 11. Contributions to charitable organizations are deductible. [l] 12. A Federal deduction for state and local sales taxes paid. [m] 13. Tax credits available for the purchase of a vehicle that uses alternative (non-fossil) fuels. [n] 14. Tax credits for home improvements that conserve energy. [o] 15. More rapid expensing for tax purposes of the costs of installing pollution control devices.

a. Social considerations b. Economic considerations c. Equity considerations d. Economic considerations e. Economic considerations f. Economic considerations g. Economic considerations h. Equity considerations i. Economic considerations j. Social considerations k. Social considerations l. Equity considerations m. Economic considerations n. Economic considerations o. Economic considerations

119. Taylor, a widow, makes cash gifts to her five married children (including their spouses) and to her seven grandchildren. What is the maximum amount Taylor can give for calendar year 2013 without using her unified transfer tax credit?

Correct Answer: $221,000. $14,000 (annual exclusion) × 17 donees = $238,000.

120. For the tax year 2013, Noah reported gross income of $300,000 on his timely filed Federal income tax return. a. Presuming the general rule applies, when does the statute of limitations on assessments normally expire? b. Suppose Noah inadvertently omitted gross income of $76,000. When does the statute of limitations on assessments expire? c. Suppose the omission was deliberate and not inadvertent. When does the statute of limitations on assessments expire?

Correct Answer: a. Three years from April 15, 2014. b. If more than 25% of gross income is omitted, a six-year statute applies (i.e., 6 years from April 15, 2014). Here, it does as $76,000 is more than $75,000 (25% × $300,000). c. If fraud is involved, the statute never expires.

121. Without obtaining an extension, Pam files her income tax return 55 days after the due date. With her return, she pays an additional tax of $60,000. Disregarding any interest element, what is Pam’s penalty for failure to pay and to file?

Correct Answer: $6,000. Disregarding the interest element, Pam’s total penalties are as follows:

Failure to pay penalty (0.5% × $ 600 $60,000 × 2 months) Plus: Failure to file $6,000 penalty (5% × $60,000 × 2 months) Less failure to pay (6 5,400 penalty for same 00) period Total penalties $6,000

122. On his 2013 income tax return, Andrew omitted income and overstated deductions to the extent that his income tax was understated by $500,000. Disregarding any interest element, what is Andrew’s penalty if the understatement was due to: a. Negligence. b. Civil fraud. c. Criminal fraud.

Correct Answer: a. $100,000 (20% × $500,000). b. $375,000 (75% × $500,000). c. Various fines and/or prison sentence.

123. Several years ago, Logan purchased extra grazing land for his ranch at a cost of $240,000. In 2013, the land is condemned by the state for development as a highway maintenance depot. Under the condemnation award, Logan receives $600,000 for the land. Within the same year, he replaces the property with other grazing land. What is Logan’s tax situation if the replacement land cost: a. $210,000? b. $360,000? c. $630,000? d. Why?

Correct Answer: a. The full realized gain of $360,000 [$600,000 (condemnation proceeds) – $240,000 (cost of land)] must be recognized, as only $210,000 was reinvested. The condemnation proceeds of $600,000 exceed the amount reinvested by more than $360,000. b. As only $360,000 was reinvested in replacement property, $240,000 ($600,000 – $360,000) of the gain must be recognized. c. As the full $600,000 was reinvested, no realized gain need be recognized. d. If some of the gain is not reinvested, consistent with the wherewithal to pay concept there exists the ability to pay the tax.

124. Paige is the sole shareholder of Citron Corporation. During the year, Paige leases a building to Citron for a monthly rental of $80,000. If the fair rental value of the building is $60,000, what are the income tax consequences to the parties involved?

Correct Answer: The rent charged by Paige is not “arms length”; as such, Citron Corporation’s rent deduction is $60,000 (not $80,000). The $20,000 difference is a nondeductible dividend distribution. For Paige, the change merely requires reclassification. Instead of $80,000 of rent income, she has $60,000 of rent income and $20,000 of dividend income.

125. In 1985, Roy leased real estate to Drab Corporation for 20 years. Drab Corporation made significant capital improvements to the property. In 2005, Roy decides not to renew the lease and vacates the property. At that time, the value of the improvements is $800,000. Roy sells the real estate in 2013 for $1,200,000 of which $900,000 is attributable to the improvements. How and when is Roy taxed on the improvements made by Drab Corporation?

Correct Answer: Roy is not subject to taxation on the improvements until he disposes of the property (i.e., 2013). After a controversial Supreme Court decision years ago, Congress clarified the tax law to make it more consistent with the wherewithal to pay concept.

126. The Federal income tax is based on a pay-as-you-go system and has become a “mass tax.” Explain this statement.

Correct Answer: The pay-as-you-go system is present in the wage and other withholding procedures. In the case of self-employed persons, it is manifested in the required quarterly payments for estimated taxes. The income tax became a mass tax during World War II when its coverage was extended to 74% of the population (from less than 6% in 1939).

127. In terms of Adam Smith’s canons of taxation, how does the Federal income tax fare as far as economy is concerned?

Correct Answer: Economy is present only if the collection procedure of the IRS is considered. Economy is not present, however, if the focus is on taxpayer compliance effort and costs.

128. Due to the population change, the Goose Creek School District has decided to close one of its high schools. Since it has no further need of the property, the school is listed for sale. The two bids it receives are as follows:

United Methodist $1,700,0 Church 00 Planet Motors 1,600,00 0

The United Methodist Church would use the property to establish a sectarian middle school. Planet, a well-known car dealership, would revamp the property and operate it as a branch location.

If you were a member of the School District board, what factors would you consider in evaluating the two bids?

Correct Answer: Although the bid from the United Methodist Church is higher, several other factors need to be considered. Does, for example, Goose Creek School district exempt property owned by churches from its ad valorem taxes? If so, losing this property from the tax base could prove very costly over the long run. Also, it is probable that income-producing property (such as a car dealership) would be taxed at a higher rate than that owned by a nonprofit organization (a school operated by a church). This assumes, of course, that the school would be taxed at all. The auto dealership also would generate sales tax.

129. Morgan inherits her father’s personal residence including all of the furnishings. She plans to add a swimming pool and sauna to the property and rent it as a furnished house. What are some of the ad valorem property tax problems Morgan can anticipate?

Correct Answer: The real estate taxes probably will increase for several reasons. The capital improvements and the conversion from residential to rental will trigger the increase. Furthermore, the furnishings may generate an ad valorem tax on personalty. (Depending on applicable law, furniture might not be subject to tax unless used for business purposes—such as in this case.)

130. In 2011, Deborah became 65 years old. In 2012 she added a swimming pool, and in 2013 she converted the residence to rental property and moved into an assisted living facility. Since 2010, Deborah’s ad valorem property taxes have decreased once and increased twice. Explain.

Correct Answer: The decrease probably came in 2011 when Deborah reached age 65. The increases probably occurred in 2012 when she added the pool and in 2013 when the residence was converted to rental property.

131. A lack of compliance in the payment of use taxes can be resolved by several means. In this regard, comment on the following: a. Registration of automobiles. b. Reporting of Internet purchases on state income tax returns.

Correct Answer: a. As reflected in Example 5 in the text, re- registration of a car purchased out-of-state is the occasion for the owner’s home state to collect the use tax. b. Completing the state income tax return reminds (or forces) the taxpayer to pay use tax on out-of-state-purchases.

132. What are the pros and cons of the following state and local tax provisions? a. An ad valorem property tax holiday made available to a manufacturing plant that is relocating. b. Hotel occupancy tax and a rental car surcharge. c. A back-to-school sales tax holiday.

Correct Answer: a. Such a holiday is designed to attract new industry to the area. This means more jobs and growth in consumption. On the other hand, if the tax holiday is too generous, this places a strain on available public revenue. The result could be that schools and capital maintenance (roads, public services) will suffer. b. The hotel occupancy tax and car rental surcharges are popular because they mainly impact visitors. Also, they can generate considerable revenue to finance major capital improvements. If these taxes become excessive, however, they could discourage major events (such as conventions). c. Such holidays are very popular with both merchants and consumers and serve the social need of defraying some of the costs of sending children to school. Once established, however, they are difficult to get rid of. Thus, they become an annual drain on sales tax revenue.

133. What is a severance tax? How productive can it be in terms of generating revenue?

Correct Answer: A severance tax is one imposed when natural resources (e.g., oil, gas, iron ore, coal) are extracted. It is based on the notion that the state has an interest in such resources. For some states, the revenue from severance taxes can be significant. Alaska, for example, relies heavily on its severance taxes and has been able to avoid both a state income tax and a general sales tax.

134. What is the difference between an inheritance tax and an estate tax? Who imposes these taxes?

Correct Answer: An inheritance tax is a tax on the right to receive property from a decedent. An estate tax is imposed on the right to pass property at death. The Federal government imposes estate taxes, while states impose inheritance taxes. Some states impose both, while others impose neither.

135. Logan dies with an estate worth $20 million. Under his will, $10 million passes to his wife while $10 million goes to his church. What is Logan’s Federal estate tax result?

Correct Answer: None. After a marital deduction of $10 million and a charitable deduction of $10 million, Logan’s taxable estate is $0.

136. With regard to state income taxes, explain what is meant by the “jock tax”?

Correct Answer: Although states have a right to levy an income tax on all nonresidents who earn income within the state, they usually do so only on highly paid visitors. Such persons are often athletes, hence the designation of “jock tax.”

137. Virtually all state income tax returns contain checkoff boxes for donations to various causes. On what grounds has this procedure been criticized?

Correct Answer: In many cases the procedure is overused (i.e., a multiplicity of boxes). This overuse adds complexity to the return. Also, in most cases the donation is being drawn from any income tax refund that might be due. Thus, taxpayers may not fully appreciate that they are paying for such checkoffs.

138. State and local governments are sometimes forced to find ways to generate additional revenue. Comment on the pros and cons of the following procedures: a. Decouple what would be part of the piggyback format of the state income tax. b. Tax amnesty provisions. c. Internet shaming.

Correct Answer: a. The decoupling process is easily accomplished as to new Federal tax changes that have never taken effect at the state level. Taxpayers are not apt to miss what they never have enjoyed. b. Tax amnesty provisions generate considerable revenue. It also unmasks many taxpayers who have not previously paid taxes. Now that the taxing jurisdiction is aware of their existence, they will tend to pay taxes in the future. c. By use of a public Web site, the taxing authority posts the names of those taxpayers that are delinquent as to various taxes (e.g., sales, income). This public humiliation (or threat of) very often results in compliance.

139. Briana lives in one state and works in the adjoining state. Both states tax the income she earns from her job. Does Briana have any relief from this apparent double taxation of the same income?

Correct Answer: Most states allow their residents some form of tax credit for the income taxes paid to other states. In Briana’s case, the credit would be allowed by the state where she lives for the taxes paid to the state where she works.

140. In late June 2013, Art is audited by the state and a large deficiency is assessed. In November of the same year, his Federal income tax return is audited by the IRS. What has probably happened?

Correct Answer: The IRS has been notified by the state concerning the results of the June audit.

141. Two months after the burglary of his personal residence, Eric is audited by the IRS. Among the items taken in the burglary was a shoe box containing approximately $50,000 in cash. Eric is the owner and operator of a cash-and-carry liquor store. Eric wonders why he was audited. Can you help explain?

Correct Answer: Although Eric’s audit by the IRS could be the result of sheer chance, this appears unlikely. Press coverage of the burglary, particularly if the items stolen were enumerated, could have put the IRS on notice. Why would anyone keep such a large amount of cash at his personal residence? Also, Eric is in a business where tax evasion is easily accomplished.

142. Rick, the sole proprietor of an adult entertainment club, is audited by the IRS. On the third day of the field audit, the regular IRS agent is accompanied by a special agent. Should Rick be concerned by this new development? Explain.

Correct Answer: Yes, he should. Special agents rarely appear during an audit unless the regular agent suspects that fraud may be involved. Considering the type of business Rick conducts, the heavy use of cash probably exists. With cash involved, tax evasion is easier to carry out.

143. Tracy has just been audited and the IRS agent has issued an RAR that assesses a large deficiency. Since Tracy disagrees with the result, her next step is to go to court. Do you agree?

Correct Answer: Tracy might save herself time and expense by going to the Appeals Division of the IRS. Here, the IRS has the authority to negotiate a settlement based on the “hazards of litigation” (i.e., the probabilities of winning or losing). If a settlement is reached, resort to the courts is avoided.

144. Can a taxpayer start the 3-year statute of limitations on additional assessments by the IRS by filing his income tax return early (i.e., before the due date)? Can the period be shortened by filing late (i.e., after the due date)?

Correct Answer: The answer is no in both cases. When filing early, the statute starts to run on the due date of the return. When filing late however, the filing date controls.

145. Brayden files his Federal income tax return by April 15, but does not pay the tax. Although he expects to pay interest on the large amount of tax he still owes, he feels that the timely filing has avoided any penalties. Is Brayden’s assumption correct?

Correct Answer: Although Brayden has avoided the failure to file penalty, the failure to pay penalty will apply. It is 0.5% per month up to a maximum of 25% of the tax due as shown on the return.

146. Melinda has been referred to you by one of your clients. In the past, she has prepared her own income tax returns, but she has become overwhelmed by the increased complexity of the tax law. Consequently, Melinda wants you to prepare her return for calendar year 2013. In reviewing her 2012 return, you note that she has claimed as a deduction the entire cost of a business building that should have been capitalized and depreciated. What course of action should you follow?

Correct Answer: You should recommend to Melinda that an amended return be filed for 2012 correcting the error. If she refuses, you should assess the gravity of the error and how it impacts on your ability to file an accurate return for 2013. If you cannot do so, then you must decline the engagement.

147. Your client, Connie, won $12,000 in a football office pool. She sees no reason to include it in her income for several reasons. First, the amount won will not be reported to the IRS. Second, as an average income employee, she is unlikely to be audited by the IRS. Third, she feels that she has probably lost this much in other past office pools. How do you respond?

Correct Answer: As a practitioner, you cannot play the audit lottery. You must presume she will be audited irrespective of the probabilities. Although the use of estimates is allowed, Connie’s assumptions as to her losses are not realistic. Even if they were reliable, gambling losses cannot be offset against gambling winnings but must be separately deducted. Thus, the $12,000 must be reported as income or you cannot prepare Connie’s return.

148. Under what conditions is it permissible, from an ethical standpoint, for a CPA firm to outsource tax return preparation to a third party?

Correct Answer: First, the clients’ confidentiality must be preserved. Second, the CPA firm must verify the accuracy of the work. Third, the clients must be advised as to the practice.

149. In terms of revenue neutrality, comment on a tax cut enacted by Congress that: a. contains revenue offsets. b. includes a sunset provision.

Correct Answer: a. Ideally, to achieve revenue neutrality all tax cuts should be accompanied by revenue offsets. b. A sunset provision does not account for the immediate revenue losses generated by a tax cut. It merely provides that such losses will not continue beyond a specified date when the tax cut expires and the former tax law is reinstated.

150. The tax law contains various tax credits, deductions, and exclusions that are designed to encourage taxpayers to obtain additional education. On what grounds can these provisions be justified?

Correct Answer: Social and economic considerations. As to the latter, a better educated workforce carries a positive economic impact.

151. The tax law contains various provisions that encourage home ownership. a. On what basis can this objective be justified? b. Are there any negative considerations? Explain.

Correct Answer: a. Home ownership can be justified on economic and social grounds. b. Granting tax advantages to persons who are purchasing their homes places the taxpayers who rent at a disadvantage. The result is inequality in treatment.

152. The tax law allows an income tax deduction (or a credit) for foreign income taxes. Explain why.

Correct Answer: The deduction (or a credit) for foreign income taxes can be justified on the grounds that it mitigates the double tax imposed on the same income.

153. The tax law allows, under certain conditions, deferral of gain recognition for involuntary conversions. a. What is the justification for this relief measure? b. What happens if the proceeds are not entirely reinvested?

Correct Answer: a. By recognizing that the taxpayer’s relative economic situation has not changed and that he or she lacks the wherewithal to pay a tax, any recognition of realized gain is deferred. b. If the proceeds from an involuntary conversion are not fully reinvested in property that is similar or related in service or use, recognized gain results. Such recognized gain cannot exceed realized gain and will be limited to the amount of the proceeds not reinvested. Recognition is based on the notion that the taxpayer now has the wherewithal to pay the tax that results.

154. How do the net operating loss provisions in the tax law mitigate the effect of the annual accounting concept?

Correct Answer: Without the allowance of carryback and/or carryover provisions that apply the excess losses to profitable years, the losses would disappear. As shown by Example 25, this result places a business with profit and loss fluctuations on a more level playing field with one that maintains a stable income pattern.

155. In connection with facilitating the function of the IRS in the administration of the tax laws, comment on the utility of the following: a. An increase in the amount of the standard deduction. b. Dollar and percentage limitations on the deduction of personal casualty losses. c. Availability of interest and penalties for taxpayer noncompliance.

Correct Answer: a. An increase in the amount of the standard deduction reduces the number of taxpayers who choose to itemize their personal deductions. This, in turn, cuts down on the deductions the IRS has to check. b. Limitations placed on casualty and theft losses curtail the number of taxpayers who can claim the deduction. c. The imposition of extra penalties, in addition to the tax owed, definitely deters taxpayer noncompliance.

156. Congress reacts to judicial decisions that interpret the tax law in different ways. When it approves of a decision, Congress may act to amend the Code to incorporate the holding. When it disapproves, Congress may amend the Code to nullify its effect. Give an example of each one of these congressional reactions.

Correct Answer: Congress approved of the judicial conclusion that most stock dividends should be nontaxable and amended the Code to this effect. However, it disagreed as to when leasehold improvements should be taxed to a lessor. Consistent with the wherewithal to pay concept, the improvements are to be taxed on the termination of the lease. Thus, Congress overturned a judicial holding that would have taxed such improvements in the year they are made by the lessee.

157. Rules of tax law do not include Revenue Rulings and Revenue Procedures.

a. True *b. False

158. A tax professional need not worry about the relative weight of authority within the various tax law sources.

a. True *b. False

159. In recent years, Congress has been relatively successful in simplifying the .

a. True *b. False

160. A taxpayer should always minimize his or her tax liability.

a. True *b. False

161. The first codification of the tax law occurred in 1954.

a. True *b. False

162. The Code section citation is incorrect: § 212(1).

a. True *b. False

163. Subchapter D refers to the “Corporate Distributions and Adjustments” section of the Internal Revenue Code.

a. True *b. False

164. Regulations are generally issued immediately after a statute is enacted.

a. True *b. False

165. Temporary Regulations are only published in the Internal Revenue Bulletin.

a. True *b. False

166. Revenue Rulings issued by the National Office of the IRS carry the same legal force and effect as Regulations.

a. True *b. False

167. A Revenue Ruling is a judicial source of Federal tax law.

a. True *b. False

168. The following citation can be a correct citation: Rev. Rul. 95-271, I.R.B. No. 54, 18.

a. True *b. False

169. Revenue Procedures deal with the internal management practices and procedures of the IRS.

*a. True b. False

170. Post-1984 letter rulings may be substantial authority for purposes of the accuracy-related penalty in § 6662.

*a. True b. False

171. A letter ruling applies only to the taxpayer who asks for and obtains a letter ruling.

*a. True b. False

172. The IRS is not required to make a letter ruling public.

a. True *b. False

173. Determination letters usually involve finalized transactions.

*a. True b. False

174. Technical Advice Memoranda deal with completed transactions.

*a. True b. False

175. Technical Advice Memoranda may not be cited as precedents by taxpayers.

*a. True b. False

176. A taxpayer must pay any tax deficiency assessed by the IRS and sue for a refund to bring suit in the U.S. Court of Federal Claims. Only in the Tax Court can jurisdiction be obtained without first paying the assessed tax deficiency.

*a. True b. False

177. In a U.S. District Court, a jury can decide both questions of fact and questions of law.

a. True *b. False

178. Three judges will normally hear each U.S. Tax Court case.

a. True *b. False

179. A taxpayer can obtain a jury trial in the U.S. Tax Court.

a. True *b. False

180. A taxpayer must pay any tax deficiency assessed by the IRS and sue for a refund to bring suit in the U.S. District Court.

*a. True b. False

181. Arizona is in the jurisdiction of the Eighth Circuit Court of Appeals.

a. True *b. False

182. is in the jurisdiction of the Second Circuit Court of Appeals.

a. True *b. False

183. The Golsen rule has been overturned by the U.S. Supreme Court.

a. True *b. False

184. The granting of a Writ of Certiorari indicates that at least four members of the Supreme Court believe that an issue is of sufficient importance to be heard by the full court.

*a. True b. False

185. The “petitioner” refers to the party against whom a suit is brought.

a. True *b. False

186. The term “petitioner” is a synonym for “defendant.”

a. True *b. False

187. The U.S. Tax Court meets most often in , D.C.

a. True *b. False

188. There are 11 geographic U.S. Circuit Court of Appeals.

*a. True b. False

189. The following citation is correct: Larry G. Mitchell, 131 T.C. 215 (2008).

*a. True b. False

190. The IRS issues an acquiescence or nonacquiescence only for regular Tax Court decisions.

a. True *b. False

191. There is a direct conflict between a Code section adopted in 2008 and a treaty with France (signed in 2012). The Code section controls.

a. True *b. False

192. The Index to Federal Tax Articles (published by Warren, Gorham, and Lamont) is available in print and electronic formats.

a. True *b. False

193. A U.S. District Court is the lowest trial court.

*a. True b. False

194. The research process should begin with a tax service.

a. True *b. False

195. Electronic databases are most frequently searched by the keyword approach.

*a. True b. False

196. The test for whether a child qualifies for dependency status is first conducted under the qualified child requirement.

*a. True b. False

197. A Bluebook opinion is substantial authority for purposes of the accuracy related penalty.

*a. True b. False

198. The primary purpose of effective tax planning is to reduce or defer the tax in the current tax year.

a. True *b. False

199. Deferring income to a subsequent year is considered to be tax avoidance.

*a. True b. False

200. Tax planning usually involves a completed transaction.

a. True *b. False

201. The Regulation section of the CPA exam is 60% Taxation and 40% Law & Professional Responsibilities.

*a. True b. False

202. The Internal Revenue Code was first codified in what year?

a. 1913. b. 1923. *c. 1939. d. 1954. e. 1986.

203. Tax bills are handled by which committee in the U.S. House of Representatives?

a. Taxation Committee. *b. Ways and Means Committee. c. Finance Committee. d. Budget Committee. e. None of the above.

204. Federal tax legislation generally originates in what body?

a. . b. Senate Finance Committee. *c. House Ways and Means Committee. d. Senate Floor. e. None of the above.

205. Subtitle A of the Internal Revenue Code covers which of the following taxes?

*a. Income taxes. b. Estate and gift taxes. c. Excise taxes. d. Employment taxes. e. All of the above.

206. In § 212(1), the number (1) stands for the:

a. Section number. b. Subsection number. *c. Paragraph designation. d. Subparagraph designation. e. None of the above.

207. Which of these is not a correct citation to the Internal Revenue Code?

a. Section 211. b. Section 1222(1). c. Section 2(a)(1)(A). d. Section 280B. *e. All of above are correct cites.

208. Which of the following is not an administrative source of tax law?

a. Field Service Advice. b. Revenue Procedure. c. Technical Advice Memoranda. d. General Counsel Memorandum. *e. All of the above are administrative sources.

209. Which of the following sources has the highest tax validity?

a. Revenue Ruling. b. Revenue Procedure. c. Regulations. *d. Internal Revenue Code section. e. None of the above.

210. Which of the following types of Regulations has the highest tax validity?

a. Temporary. *b. Legislative. c. Interpretive. d. Procedural. e. None of the above.

211. Which statement is not true with respect to a Regulation that interprets the tax law?

*a. Issued by the U.S. Congress. b. Issued by the U.S. Treasury Department. c. Designed to provide an interpretation of the tax law. d. Carries more legal force than a Revenue Ruling. e. All of the above statements are true.

212. In addressing the importance of a Regulation, an IRS agent must:

*a. Give equal weight to the Code and the Regulations. b. Give more weight to the Code rather than to a Regulation. c. Give more weight to the Regulation rather than to the Code. d. Give less weight to the Code rather than to a Regulation. e. None of the above.

213. Which item may not be cited as a precedent?

a. Regulations. b. Temporary Regulations. *c. Technical Advice Memoranda. d. U.S. District Court decision. e. None of the above.

214. What statement is not true with respect to Temporary Regulations?

*a. May not be cited as precedent. b. Issued as Proposed Regulations. c. Automatically expire within three years after the date of issuance. d. Found in the Federal Register. e. All of the above statements are true.

215. What administrative release deals with a proposed transaction rather than a completed transaction?

*a. Letter Ruling. b. Technical Advice Memorandum. c. Determination Letter. d. Field Service Advice. e. None of the above.

216. Which of the following indicates that a decision has precedential value for future cases?

*a. Stare decisis. b. Golsen doctrine. c. En banc. d. Reenactment doctrine. e. None of the above.

217. A taxpayer who loses in a U.S. District Court may appeal directly to the:

a. U.S. Supreme Court. b. U.S. Tax Court. c. U.S. Court of Federal Claims. *d. U.S. Circuit Court of Appeals. e. All of the above.

218. If a taxpayer decides not to pay a tax deficiency, he or she must go to which court?

a. Appropriate U.S. Circuit Court of Appeals. b. U.S. District Court. *c. U.S. Tax Court. d. U.S. Court of Federal Claims. e. None of the above.

219. A jury trial is available in the following trial court:

a. U.S. Tax Court. b. U.S. Court of Federal Claims. *c. U.S. District Court. d. U.S. Circuit Court of Appeals. e. None of the above.

220. A taxpayer may not appeal a case from which court:

a. U.S. District Court. b. U.S. Circuit Court of Appeals. c. U.S. Court of Federal Claims. *d. Small Case Division of the U.S. Tax Court. e. None of the above.

221. The IRS will not acquiesce to the following tax decisions:

a. U.S. District Court. b. U.S. Tax Court. c. U.S. Court of Federal Claims. *d. Small Case Division of the U.S. Tax Court. e. All of the above.

222. Which publisher offers the Standard Federal Tax Reporter?

a. Research Institute of America. *b. Commerce Clearing House. c. Prentice-Hall. d. LexisNexis. e. None of the above.

223. Which is presently not a major tax service?

a. Standard Federal Tax Reporter. *b. Federal Taxes. c. United States Tax Reporter. d. Tax Management Portfolios. e. All of the above are major tax services.

224. Which publisher offers the United States Tax Reporter?

*a. Research Institute of America. b. Commerce Clearing House. c. LexisNexis. d. Tax Analysts. e. None of the above.

225. When searching on an online tax service, which approach is more frequently used?

a. Code section approach. *b. Keyword approach. c. Table of contents approach. d. Index. e. All are about the same.

226. A researcher can find tax information on home page sites of:

a. Governmental bodies. b. Tax academics. c. Publishers. d. CPA firms. *e. All of the above.

227. Tax research involves which of the following procedures:

a. Identifying and refining the problem. b. Locating the appropriate tax law sources. c. Assessing the validity of the tax law sources. d. Follow-up. *e. All of the above.

228. Which tax-related website probably gives the best policy- orientation results?

a. taxalmanac.org. b. irs.gov. c. taxsites.com. *d. taxanalyst.com. e. ustaxcourt.gov.

229. Which court decision would probably carry more weight?

a. Regular U.S. Tax Court decision. *b. Reviewed U.S. Tax Court decision. c. U.S. District Court decision. d. Memorandum Tax Court decision. e. U.S. Court of Federal Claims.

230. Which Regulations have the force and effect of law?

a. Procedural Regulations. b. Finalized Regulations. *c. Legislative Regulations. d. Interpretive Regulations. e. All of the above.

231. Which items tell taxpayers the IRS’s reaction to certain court decisions?

a. Notices. b. Revenue Procedures. c. Revenue Rulings. *d. Actions on Decisions. e. Legislative Regulations.

232. Which court decision carries more weight?

a. Federal District Court. *b. Second Circuit Court of Appeals. c. Memorandum U.S. Tax Court decision. d. Small Cases Division of U.S. Tax Court. e. U.S. Court of Federal Claims.

233. Which company does not publish citators for tax purposes?

*a. John Wiley & Sons. b. Commerce Clearing House. c. Research Institute of America. d. Westlaw. e. Shepard’s.

234. Which is not a primary source of tax law?

a. Notice 89-99, 1989-2 C.B. 422. b. Estate of Harry Holmes v. Comm., 326 U.S. 480 (1946). c. Rev. Rul. 79-353, 1979-2 C.B. 325. d. Prop. Reg. § 1.752-4T(f). *e. All of the above are primary sources.

235. Which statement is incorrect with respect to taxation on the CPA exam?

a. The CPA exam now has only four parts. *b. There are no longer case studies on the exam. c. A candidate may not go back after exiting a testlet. d. Simulations include a four-function pop-up calculator. e. None of the above are incorrect.

236. How can Congressional committee reports be used by a tax researcher?

Correct Answer: Congressional committee reports often explain the provisions of proposed legislation and are a valuable source of ascertaining the intent of Congress. The intent of Congress is the key to interpreting new legislation by taxpayers, especially before Regulations are published.

237. What are Treasury Department Regulations?

Correct Answer: Regulations are issued by the U.S. Treasury Department under authority granted by Congress. Interpretive by nature, they provide taxpayers with considerable guidance on the meaning and application of the Code. Regulations may be issued in proposed, temporary, or final form. Regulations carry considerable authority as the official interpretation of tax statutes. They are an important factor to consider in complying with the tax law. Courts generally ignore Proposed Regulations.

238. Compare Revenue Rulings with Revenue Procedures.

Correct Answer: Revenue Rulings are official pronouncements of the National Office of the IRS. They typically provide one or more examples of how the IRS would apply a law to specific fact situations. Like Regulations, Revenue Rulings are designed to provide interpretation of the tax law. However, they do not carry the same legal force and effect as Regulations and usually deal with more restricted problems. Regulations are approved by the Secretary of the Treasury, whereas Revenue Rulings generally are not.

Revenue Procedures are issued in the same manner as Revenue Rulings, but deal with the internal management practices and procedures of the IRS. Familiarity with these procedures can increase taxpayer compliance and help the IRS administer the tax laws more efficiently. A taxpayer’s failure to follow a Revenue Procedure can result in unnecessary delay or, in a discretionary situation, can cause the IRS to decline to act on behalf of the taxpayer.

239. What is a Technical Advice Memorandum?

Correct Answer: The National Office of the IRS releases Technical Advice Memoranda (TAMs) weekly. TAMs resemble letter rulings in that they give the IRS’s determination of an issue. However, they differ in several respects. Letter rulings deal with proposed transactions and are issued to taxpayers at their request. In contrast, TAMs deal with completed transactions. Furthermore, TAMs arise from questions raised by IRS personnel during audits and are issued by the National Office of the IRS to its field personnel. TAMs are often requested for questions relating to exempt organizations and employee plans. TAMs are not officially published and may not be cited or used as precedent.

240. Discuss the advantages and disadvantages of the Small Cases Division of the U.S. Tax Court.

Correct Answer: There is no appeal from the Small Cases Division. The jurisdiction of the Small Cases Division is limited to cases involving amounts of $50,000 or less. The proceedings of the Small Cases Division are informal (e.g., no necessity for the taxpayer to be represented by a lawyer or other tax adviser). Special trial judges rather than Tax Court judges preside over these proceedings. The decisions of the Small Cases Division are not precedents for any other court decision and are not reviewable by any higher court. Proceedings can be more timely and less expensive in the Small Cases Division. Some of these cases can now be found on the U.S. Tax Court Internet Website.

241. Distinguish between the jurisdiction of the U.S. Tax Court and a U.S. District Court.

Correct Answer: The U.S. Tax Court hears only tax cases and is the most popular tax forum. The U.S. District Court hears a wide variety of nontax cases, including drug crimes and other Federal violations, as well as tax cases. Some Tax Court justices have been appointed from IRS or Treasury Department positions. For these reasons, some people suggest that the U.S. Tax Court has more expertise in tax matters.

242. How do treaties fit within tax sources?

Correct Answer: The U.S signs certain tax treaties (sometimes called tax conventions) with foreign countries to render mutual assistance in tax enforcement and to avoid double taxation. Tax legislation enacted in 1988 provided that neither a tax law nor a tax treaty takes general precedence. Thus, when there is a direct conflict with the Code and a treaty, the most recent item will take precedence. A taxpayer must disclose on the tax return any position where a treaty overrides a tax law. There is a $1,000 penalty per failure to disclose for individuals and a $10,000 per failure penalty for corporations.

243. Under the Federal income tax formula for individuals, a choice must be made between claiming deductions for AGI and itemized deductions.

a. True *b. False

244. Under the Federal income tax formula for individuals, the determination of adjusted gross income (AGI) precedes that of taxable income (TI).

*a. True b. False

245. Under the income tax formula, a taxpayer must choose between deductions for AGI and the standard deduction.

a. True *b. False

246. After Ellie moves out of the apartment she had rented as her personal residence, she recovers her damage deposit of $1,000. The $1,000 is not income to Ellie.

*a. True b. False

247. An “above the line” deduction refers to a deduction for AGI.

*a. True b. False

248. Because they appear on page 1 of Form 1040, itemized deductions are also referred to as “page 1 deductions.”

a. True *b. False

249. A decrease in a taxpayer’s AGI could increase the amount of medical expenses that can be deducted.

*a. True b. False

250. An increase in a taxpayer’s AGI could decrease the amount of charitable contribution that can be claimed.

a. True *b. False

251. Adjusted gross income (AGI) appears at the bottom of page 1 and at the top of page 2 of Form 1040.

*a. True b. False

252. All exclusions from gross income are reported on Form 1040.

a. True *b. False

253. The filing status of a taxpayer (e.g., single, head of household) must be identified before the applicable standard deduction is determined.

*a. True b. False

254. Lee, a citizen of Korea, is a resident of the U.S. Any rent income Lee receives from land he owns in Korea is not subject to the U.S. income tax.

a. True *b. False

255. The additional standard deduction for age and blindness is greater for married taxpayers than for single taxpayers.

a. True *b. False

256. The basic and additional standard deductions both are subject to an annual adjustment for inflation.

*a. True b. False

257. Many taxpayers who previously itemized will start claiming the standard deduction when they purchase a home.

a. True *b. False

258. Once they reach age 65, many taxpayers will switch from itemizing their deductions from AGI and start claiming the standard deduction.

*a. True b. False

259. Claude’s deductions from AGI exceed the standard deduction allowed for 2013. Under these circumstances, Claude cannot claim the standard deduction.

a. True *b. False

260. As opposed to itemizing deductions from AGI, the majority of individual taxpayers choose the standard deduction.

*a. True b. False

261. Howard, age 82, dies on January 2, 2013. On Howard’s final income tax return, the full amount of the basic and additional standard deductions will be allowed even though Howard lived for only 2 days during the year.

*a. True b. False

262. In 2013, Ed is 66 and single. If he has itemized deductions of $7,300, he should not claim the standard deduction alternative.

a. True *b. False

263. Jason and Peg are married and file a joint return. Both are over 65 years of age and Jason is blind. Their standard deduction for 2013 is $15,800 ($12,200 + $1,200 + $1,200 + $1,200).

*a. True b. False

264. Derek, age 46, is a surviving spouse. If he has itemized deductions of $12,500 for 2013, Derek should not claim the standard deduction.

*a. True b. False

265. Buddy and Hazel are ages 72 and 71 and file a joint return. If they have itemized deductions of $14,300 for 2013, they should not claim the standard deduction.

a. True *b. False

266. Clara, age 68, claims head of household filing status. If she has itemized deductions of $10,100 for 2013, she should not claim the standard deduction.

a. True *b. False

267. Monique is a resident of the U.S. and a citizen of France. If she files a U.S. income tax return, Monique cannot claim the standard deduction.

a. True *b. False

268. Dan and Donna are husband and wife and file separate returns for the year. If Dan itemizes his deductions from AGI, Donna cannot claim the standard deduction.

*a. True b. False

269. Benjamin, age 16, is claimed as a dependent by his parents. During 2013, he earned $700 at a car wash. Benjamin’s standard deduction is $1,350 ($1,000 + $350).

a. True *b. False

270. Debby, age 18, is claimed as a dependent by her mother. During 2013, she earned $1,100 in interest income on a savings account. Debby’s standard deduction is $1,450 ($1,100 + $350).

a. True *b. False

271. Katrina, age 16, is claimed as a dependent by her parents. During 2013, she earned $5,600 as a checker at a grocery store. Her standard deduction is $5,950 ($5,600 earned income + $350).

*a. True b. False

272. A dependent cannot claim a personal exemption on his or her own return.

*a. True b. False

273. When separate income tax returns are filed by married taxpayers, one spouse cannot claim the other spouse as an exemption.

a. True *b. False

274. Butch and Minerva are divorced in December of 2013. Since they were married for more than one-half of the year, they are considered as married for 2013.

a. True *b. False

275. For the year a spouse dies, the surviving spouse is considered married for the entire year for income tax purposes.

*a. True b. False

276. In determining whether the gross income test is met for dependency exemption purposes, only the taxable portion of a scholarship is considered.

*a. True b. False

277. Albert buys his mother a TV. For purposes of meeting the support test, Albert cannot include the cost of the TV.

a. True *b. False

278. If an individual does not spend funds that have been received from another source (e.g., interest on municipal bonds), the unexpended amounts are not considered for purposes of the support test.

*a. True b. False

279. Using borrowed funds from a mortgage on her home, Leah provides 52% of her own support, while her sons furnished the rest. Leah can be claimed as a dependent under a multiple support agreement.

a. True *b. False

280. Roy and Linda were divorced in 2012. The divorce decree awards custody of their children to Linda but is silent as to who is entitled to claim them as dependents. If Roy furnished more than half of their support, he can claim them as dependents in 2013.

a. True *b. False

281. In 2013, Hal furnishes more than half of the support of his ex- wife and her father, both of whom live with him. The divorce occurred in 2012. Hal may claim the father-in-law and the ex-wife as dependents.

*a. True b. False

282. After her divorce, Hope continues to support her ex-husband’s sister, Cindy, who does not live with her. Hope can claim Cindy as a dependent.

*a. True b. False

283. Darren, age 20 and not disabled, earns $4,000 during 2013. Darren’s parents cannot claim him as a dependent unless he is a full- time student.

*a. True b. False

284. Lucas, age 17 and single, earns $6,000 during 2013. Lucas’s parents cannot claim him as a dependent if he does not live with them.

*a. True b. False

285. Sarah furnishes more than 50% of the support of her son and daughter-in-law who live with her. If the son and daughter-in-law file a joint return, Sarah cannot claim them as dependents.

a. True *b. False

286. Kim, a resident of Oregon, supports his parents who are residents of Canada but citizens of Korea. Kim can claim his parents as dependents.

*a. True b. False

287. Stealth taxes are directed at lower income taxpayers.

a. True *b. False

288. In determining the filing requirement based on gross income received, both additional standard deductions (i.e., age and blindness) are taken into account.

a. True *b. False

289. For dependents who have income, special filing requirements apply.

*a. True b. False

290. A taxpayer who itemizes must use Form 1040, and cannot use Form 1040EZ or Form 1040A.

*a. True b. False

291. An individual taxpayer uses a fiscal year March 1-February 28. The due date of this taxpayer’s Federal income tax return is May 15 of each tax year.

a. True *b. False

292. Married taxpayers who file a joint return cannot later (i.e., after the filing due date) switch to separate returns for that year.

*a. True b. False

293. Married taxpayers who file separately cannot later (i.e., after the due date for filing) change to a joint return.

a. True *b. False

294. Surviving spouse filing status begins in the year in which the deceased spouse died.

a. True *b. False

295. In January 2013, Jake’s wife dies and he does not remarry. For tax year 2013, Jake may not be able to use the filing status available to married persons filing joint returns.

*a. True b. False

296. For tax purposes, married persons filing separate returns are treated the same as single taxpayers.

a. True *b. False

297. Katelyn is divorced and maintains a household in which she and her daughter, Crissa, live. Crissa, age 22, earns $11,000 during 2013 as a model. Katelyn does not qualify for head of household filing status.

*a. True b. False

298. Ed is divorced and maintains a home in which he and a dependent friend live. Ed does not qualify for head of household filing status.

*a. True b. False

299. In terms of income tax consequences, abandoned spouses are treated the same way as married persons filing separate returns.

a. True *b. False

300. Since an abandoned spouse is treated as single and has one or more dependent children, he or she qualifies for the standard deduction available to head of household.

*a. True b. False

301. Currently, the top income tax rate in effect is not the highest it has ever been.

*a. True b. False

302. In terms of timing as to any one year, the Tax Tables are available before the Tax Rate Schedules.

a. True *b. False

303. The kiddie tax does not apply to a child whose earned income is more than one-half of his or her support.

*a. True b. False

304. Once a child reaches age 19, the kiddie tax no longer applies.

a. True *b. False

305. When the kiddie tax applies and the parents are divorced, the applicable parent (for determining the parental tax) is the one with the greater taxable income.

a. True *b. False

306. When the kiddie tax applies, the child need not file an income tax return because the child’s income will be reported on the parents’ return.

a. True *b. False

307. A child who has unearned income of $2,000 or less cannot be subject to the kiddie tax.

*a. True b. False

308. A child who is married cannot be subject to the kiddie tax.

a. True *b. False

309. In 2013, Frank sold his personal use automobile for a loss of $9,000. He also sold a personal coin collection for a gain of $10,000. As a result of these sales, $10,000 is subject to income tax.

*a. True b. False

310. Gain on the sale of collectibles held for more than 12 months always is subject to a tax rate of 28%.

a. True *b. False

311. For 2013, Stuart has a short-term capital loss, a collectible long-term capital gain, and a long-term capital gain from land held as investment. The short-term loss is first applied to the collectible capital gain.

*a. True b. False

312. In terms of the tax formula applicable to individual taxpayers, which, if any, of the following statements is correct?

*a. In arriving at taxable income, a taxpayer must choose between the standard deduction and deductions from AGI. b. In arriving at AGI, personal and dependency exemptions must be subtracted from gross income. c. In arriving at taxable income, a taxpayer must choose between the standard deduction and claiming personal and dependency exemptions. d. The formula does not apply if a taxpayer elects to claim the standard deduction. e. None of the above.

313. In terms of the tax formula applicable to individual taxpayers, which, if any, of the following statements is correct?

a. In arriving at AGI, a taxpayer must elect between claiming deductions for AGI and deductions from AGI. b. In arriving at taxable income, a taxpayer must elect between claiming deductions for AGI and deductions from AGI. c. If a taxpayer has deductions for AGI, the standard deduction is not available. d. In arriving at taxable income, a taxpayer must elect between deductions for AGI and the standard deduction. *e. None of the above.

314. Regarding the tax formula and its relationship to Form 1040, which, if any, of the following statements is correct?

a. Most exclusions from gross income are reported on page 2 of Form 1040. b. An “above the line deduction” refers to a deduction from AGI. *c. A “page 1 deduction” refers to a deduction for AGI. d. The taxable income (TI) amount appears both at the bottom of page 1 and at the top of page 2 of Form 1040. e. None of the above.

315. Which of the following items, if any, is deductible?

a. Parking expenses incurred in connection with jury duty— taxpayer is a dentist. *b. Substantiated gambling losses (not in excess of gambling winnings) from state lottery. c. Contributions to mayor’s reelection campaign. d. Speeding ticket incurred while on business. e. Premiums paid on personal life insurance policy.

316. Which, if any, of the following is a deduction for AGI?

*a. Contributions to a traditional Individual Retirement Account. b. Child support payments. c. Funeral expenses. d. Loss on the sale of a personal residence. e. Medical expenses.

317. Which, if any, of the following is a deduction for AGI?

a. State and local sales taxes. b. Interest on home mortgage. c. Charitable contributions. *d. Unreimbursed moving expenses of an employee. e. None of the above.

318. Which, if any, of the statements regarding the standard deduction is correct?

*a. Some taxpayers may qualify for two types of standard deductions. b. Not available to taxpayers who choose to deduct their personal and dependency exemptions. c. Not available to taxpayers who choose to claim their deduction for AGI. d. The basic standard deduction is indexed for inflation but the additional standard deduction is not. e. None of the above.

319. Which, if any, of the following statements relating to the standard deduction is correct?

a. If a taxpayer dies during the year, his (or her) standard deduction must be prorated. *b. If a taxpayer is claimed as a dependent of another, his (or her) additional standard deduction is allowed in full (i.e., no adjustment is necessary). c. If spouses file separate returns, both spouses must claim the standard deduction (rather than itemize their deductions from AGI). d. If a taxpayer is claimed as a dependent of another, no basic standard deduction is allowed. e. None of the above.

320. During 2013, Marvin had the following transactions:

Salary $50,000 Bank loan (proceeds used to buy 10,000 personal auto) Alimony paid 12,000 Child support paid 6,000 Gift from aunt 20,000

Marvin’s AGI is:

a. $32,000. *b. $38,000. c. $44,000. d. $56,000. e. $64,000.

321. During 2013, Esther had the following transactions:

Salary $70,000 Interest income on Xerox bonds 2,000 Inheritance from uncle 40,000 Contribution to traditional IRA 5,500 Capital losses 2,500

Esther’s AGI is:

a. $62,000. *b. $64,000. c. $67,000. d. $102,000. e. $104,000.

322. During 2013, Sarah had the following transactions:

Salary $ 80,00 0 Interest income on City of Baltimore 1,000 bonds Damages for personal injury (car 100,000 accident) Punitive damages (same car accident) 200,000 Cash dividends from Chevron 7,000 Corporation stock

Sarah’s AGI is:

a. $185,000. b. $187,000. c. $285,000. *d. $287,000. e. $387,000.

323. In 2013, Cindy had the following transactions:

Salary $90,000 Short-term capital gain from a stock 4,000 investment Moving expense to change jobs (11,000) Received repayment of $20,000 loan she made to her sister in 2009 20,000 (includes no interest) State income taxes (5,000)

Cindy’s AGI is:

a. $114,000. b. $103,000. c. $98,000. d. $94,000. *e. $83,000.

324. Sylvia, age 17, is claimed by her parents as a dependent. During 2013, she had interest income from a bank savings account of $2,000 and income from a part-time job of $4,200. Sylvia’s taxable income is:

a. $4,200 – $4,550 = $0. b. $6,200 – $5,700 = $500. *c. $6,200 – $4,550 = $1,650. d. $6,200 – $1,000 = $5,200. e. None of the above.

325. Tony, age 15, is claimed as a dependent by his grandmother. During 2013, Tony had interest income from Boeing Corporation bonds of $1,000 and earnings from a part-time job of $700. Tony’s taxable income is:

a. $1,700. b. $1,700 – $700 – $1,000 = $0. *c. $1,700 – $1,050 = $650. d. $1,700 – $1,000 = $700. e. None of the above.

326. Merle is a widow, age 80 and blind, who is claimed as a dependent by her son. During 2013, she received $4,800 in Social Security benefits, $2,350 in bank interest, and $1,800 in cash dividends from stocks. Merle’s taxable income is:

*a. $4,150 – $1,000 – $3,000 = $150. b. $4,150 – $3,000 = $1,150. c. $4,150 – $1,000 – $1,500 = $1,650. d. $8,950 – $1,000 – $3,000 = $4,950. e. None of the above.

327. Wilma, age 70 and single, is claimed as a dependent on her daughter’s tax return. During 2013, she had interest income of $2,500 and $800 of earned income from baby sitting. Wilma’s taxable income is:

a. $700. b. $850. c. $1,800. d. $2,250. *e. None of the above.

328. Kyle and Liza are married and under 65 years of age. During 2013, they furnish more than half of the support of their 19-year old daughter, May, who lives with them. She graduated from high school in May 2012. May earns $15,000 from a part-time job, most of which she sets aside for future college expenses. Kyle and Liza also provide more than half of the support of Kyle’s cousin who lives with them. Liza’s father, who died on January 3, 2013, at age 90, has for many years qualified as their dependent. How many personal and dependency exemptions should Kyle and Liza claim?

a. Two. b. Three. *c. Four. d. Five. e. None of the above.

329. Evan and Eileen Carter are husband and wife and file a joint return for 2013. Both are under 65 years of age. They provide more than half of the support of their daughter, Pamela (age 25), who is a full- time medical student. Pamela receives a $5,000 scholarship covering her tuition at college. They furnish all of the support of Belinda (Evan’s grandmother), who is age 80 and lives in a nursing home. They also support Peggy (age 66), who is a friend of the family and lives with them. How many dependency exemptions may the Carters claim?

a. Two. *b. Three. c. Four. d. Five. e. None of the above.

330. In which, if any, of the following situations may the individual not be claimed as a dependent of the taxpayer?

a. A former spouse who lives with the taxpayer (divorce took place last year). b. A stepmother who does not live with the taxpayer. c. A married daughter who lives with the taxpayer. d. A half brother who does not live with the taxpayer and is a citizen and resident of Canada. *e. A cousin who does not live with the taxpayer.

331. During 2013, Lisa (age 66) furnished more than 50% of the support of the following persons:

• Lisa’s current husband who has no income and is not claimed by someone else as a dependent. • Lisa’s stepson (age 19) who lives with her and earns $6,000 as a dance instructor. He dropped out of school a year ago. • Lisa’s ex-husband who does not live with her. The divorce occurred two years ago. • Lisa’s former brother-in-law who does not live with her.

Presuming all other dependency tests are met, on a separate return how many personal and dependency exemptions may Lisa claim?

a. Two. *b. Three. c. Four. d. Five. e. None of the above.

332. A qualifying child cannot include:

a. A nonresident alien. b. A married son who files a joint return. c. A daughter who is away at college. d. A brother who is 28 years of age and disabled. *e. A grandmother.

333. Ellen, age 12, lives in the same household with her father, grandfather, and uncle. The cost of maintaining the household is provided by her grandfather (40%) and her uncle (60%). Disregarding tie-breaker rules, Ellen is a qualifying child as to:

a. Only her father. b. Only her grandfather and uncle. c. Only her uncle. *d. All parties involved (i.e., father, grandfather, and uncle). e. None of the above.

334. Millie, age 80, is supported during the current year as follows:

Percent of Support Weston (a son) 20% Faith (a daughter) 35% Jake (a cousin) 25% Brayden (unrelated close 20% family friend)

During the year, Millie lives in an assisted living facility. Under a multiple support agreement, indicate which parties can qualify to claim Millie as a dependent.

*a. Weston and Faith. b. Faith. c. Weston, Faith, Jake, and Brayden. d. Faith, Jake, and Brayden. e. None of the above.

335. The Hutters filed a joint return for 2013. They provide more than 50% of the support of Carla, Melvin, and Aaron. Carla (age 18) is a cousin and earns $2,800 from a part-time job. Melvin (age 25) is their son and is a full-time law student. He received from the university a $3,800 scholarship for tuition. Aaron is a brother who is a citizen of Israel but resides in France. Carla and Melvin live with the Hutters. How many personal and dependency exemptions can the Hutters claim on their Federal income tax return?

a. Two. b. Three. *c. Four. d. Five. e. None of the above.

336. Which of the following characteristics correctly describes the procedure for the phaseout of exemptions?

a. The threshold amounts are different and depend on filing status (e.g., joint return, single). b. The threshold amounts are indexed for inflation each year. c. The phaseout procedure is known as a “stealth tax.” d. For the phaseout procedure to be applied, a taxpayer’s AGI must exceed the threshold amount. *e. All of the above.

337. Regarding the rules applicable to filing of income tax returns, which, if any, of the following is an incorrect statement:

a. Married persons who file joint returns cannot later (after the due date of the return) substitute separate returns. b. Married persons who file separate returns can later (after the due date of the return) substitute a joint return. *c. The usual test as to when a taxpayer must file a return is based on the total of the following: personal exemption + basic standard deduction + both additional standard deductions. d. Special filing requirement rules exist for taxpayers who are claimed as dependents of another. e. None of the above.

338. Kyle, whose wife died in December 2010, filed a joint tax return for 2010. He did not remarry, but has continued to maintain his home in which his two dependent children live. What is Kyle’s filing status as to 2013?

*a. Head of household. b. Surviving spouse. c. Single. d. Married filing separately. e. None of the above.

339. Emily, whose husband died in December 2012, maintains a household in which her dependent mother lives. Which (if any) of the following is her filing status for the tax year 2013? (Note: Emily is the executor of her husband’s estate.)

a. Single. b. Married, filing separately. c. Surviving spouse. *d. Head of household. e. Married, filing jointly.

340. Which of the following taxpayers may file as a head of household in 2013?

Ron provides all the support for his mother, Betty, who lives by herself in an apartment in Fort Lauderdale. Ron pays the rent and other expenses for the apartment and properly claims his mother as a dependent.

Tammy provides over one-half the support for her 18-year old brother, Dan. Dan earned $4,200 in 2013 working at a fast food restaurant and is saving his money to attend college in 2014. Dan lives in Tammy’s home.

Joe’s wife left him late in December of 2012. No legal action was taken and Joe has not heard from her in 2013. Joe supported his 6-year-old son, who lived with him throughout 2013.

a. Ron only. b. Tammy only. c. Joe only. d. Ron and Joe only. *e. Ron, Tammy, and Joe.

341. Nelda is married to Chad, who abandoned her in early June of 2013. She has not seen or communicated with him since then. She maintains a household in which she and her two dependent children live. Which of the following statements about Nelda’s filing status in 2013 is correct?

a. Nelda can use the rates for single taxpayers. b. Nelda can file a joint return with Chad. c. Nelda can file as a surviving spouse. *d. Nelda can file as a head of household. e. None of the above statements is appropriate.

342. Arnold is married to Sybil, who abandoned him in 2012. He has not seen or communicated with her since April of that year. He maintains a household in which their son, Evans, lives. Evans is age 25 and earns over $6,000 each year. For tax year 2013, Arnold’s filing status is:

a. Married, filing jointly. b. Head of household. *c. Married, filing separately. d. Surviving spouse. e. Single.

343. Regarding the Tax Tables applicable to the Federal income tax, which of the following statements is correct?

*a. For any one year, the Tax Tables are issued by the IRS after the Tax Rate Schedules. b. The Tax Tables will always yield the same amount of tax as the Tax Rate Schedules. c. Taxpayers can elect as to whether the use the Tax Tables or the Tax Rate Schedules. d. The Tax Tables can be used by an estate but not by a trust. e. No correct answer given.

344. In which, if any, of the following situations will the kiddie tax not apply?

a. The child is married but does not file a joint return. *b. The child has unearned income of $2,000 or less. c. The child has unearned income that exceeds more than half of his (or her) support. d. The child is under age 24 and a full-time student. e. None of the above.

345. Which, if any, of the following is a correct statement relating to the kiddie tax?

a. If the parents are divorced, the income of the noncustodial parent is used to determine the allocable parental tax. b. The components for the application of the kiddie tax are not subject to adjustment for inflation. c. If the kiddie tax applies, the parents must include the income of the child on their own income tax return. *d. The kiddie tax does not apply if both parents of the child are deceased. e. None of the above.

346. During the year, Kim sold the following assets: business auto for a $1,000 loss, stock investment for a $1,000 loss, and pleasure yacht for a $1,000 loss. Presuming adequate income, how much of these losses may Kim claim?

a. $0. b. $1,000. *c. $2,000. d. $3,000. e. None of the above.

347. Perry is in the 33% tax bracket. During 2013, he had the following capital asset transactions:

Gain from the sale of a stamp $30,000 collection (held for 10 years) Gain from the sale of an investment 10,000 in land (held for 4 years) Gain from the sale of stock 4,000 investment (held for 8 months)

Perry’s tax consequences from these gains are as follows:

a. (15% × $30,000) + (33% × $4,000). *b. (15% × $10,000) + (28% × $30,000) + (33% × $4,000). c. (0% × $10,000) + (28% × $30,000) + (33% × $4,000). d. (15% × $40,000) + (33% × $4,000). e. None of the above.

348. Kirby is in the 15% tax bracket and had the following capital asset transactions during 2013:

Long-term gain from the sale of a $11,000 coin collection Long-term gain from the sale of a 10,000 land investment Short-term gain from the sale of a 2,000 stock investment

Kirby’s tax consequences from these gains are as follows:

a. (5% × $10,000) + (15% × $13,000). b. (15% × $13,000) + (28% × $11,000). *c. (0% × $10,000) + (15% × $13,000). d. (15% × $23,000). e. None of the above.

349. For the current year, David has salary income of $80,000 and the following property transactions:

Stock investment sales— Long-term capital gain $ 9,000 Short-term capital loss (12,000) Loss on sale of camper (purchased 4 (2,000) years ago and used for family vacations)

What is David’s AGI for the current year?

a. $76,000. *b. $77,000. c. $78,000. d. $89,000. e. None of the above.

350. During 2013, Trevor has the following capital transactions:

LTCG $ 6,00 0 Long-term collectible gain 2,000 STCG 4,000 STCL 10,000

After the netting process, the following results:

a. Long-term collectible gain of $2,000. b. LTCG of $6,000, Long-term collectible gain of $2,000, and a STCL of $6,000. c. LTCG of $6,000, Long-term collectible gain of $2,000, and a STCL carryover to 2014 of $3,000. *d. LTCG of $2,000. e. None of the above.

351. Regarding dependency exemptions, classify each statement in one of the four categories:A son lives with taxpayer and earns $3,000.A daughter who does not live with taxpayer.A granddaughter, who lives with taxpayer, is 19 years old, earns $5,000, and is not a full-time student.An uncle who lives with taxpayer.A nephew who lives with taxpayer.A niece who lives with taxpayer, is 20 years old, earns $5,000, and is a full-time student.A half brother who lives with taxpayer.A cousin who does not live with taxpayer.A stepdaughter who does not live with taxpayer.A daughter-in-law who lives with taxpayer.A family friend who is supported by and lives with the taxpayer.An ex-husband (divorce occurred last year) who lives with taxpayer.Could be either a qualifying child or a qualifying relative. Could be a qualifying relative. Could be neither a qualifying child nor a qualifying relative. Could be a qualifying relative. Could be either a qualifying child or a qualifying relative. Could be a qualifying child. Could be either a qualifying child or a qualifying relative. Could be neither a qualifying child nor a qualifying relative. Could be a qualifying relative. Could be a qualifying relative. Could be a qualifying relative. Could be a qualifying relative.

[a] 1. A son lives with taxpayer and earns $3,000. [b] 2. A daughter who does not live with taxpayer. [c] 3. A granddaughter, who lives with taxpayer, is 19 years old, earns $5,000, and is not a full-time student. [d] 4. An uncle who lives with taxpayer. [e] 5. A nephew who lives with taxpayer. [f] 6. A niece who lives with taxpayer, is 20 years old, earns $5,000, and is a full-time student. [g] 7. A half brother who lives with taxpayer. [h] 8. A cousin who does not live with taxpayer. [i] 9. A stepdaughter who does not live with taxpayer. [j] 10. A daughter-in-law who lives with taxpayer. [k] 11. A family friend who is supported by and lives with the taxpayer. [l] 12. An ex-husband (divorce occurred last year) who lives with taxpayer.

a. Could be either a qualifying child or a qualifying relative. b. Could be a qualifying relative. c. Could be neither a qualifying child nor a qualifying relative. d. Could be a qualifying relative. e. Could be either a qualifying child or a qualifying relative. f. Could be a qualifying child. g. Could be either a qualifying child or a qualifying relative. h. Could be neither a qualifying child nor a qualifying relative. i. Could be a qualifying relative. j. Could be a qualifying relative. k. Could be a qualifying relative. l. Could be a qualifying relative.

352. Match the statements that relate to each other. Note: Choice L may be used more than once.Surviving spouseScholarship funds for tuitionAdditional standard deductionScholarship funds for room and boardAbandoned spouseBasic standard deductionResident of Canada or MexicoAge of a qualifying child$1,000Kiddie tax appliesKiddie tax does not applyMultiple support agreementUnmarried taxpayer who can use the same tax rates as married persons filing jointly. Not considered in applying the gross income test (for dependency exemption purposes). Not available to 65-year old taxpayer who itemizes. Considered in applying gross income test (for dependency exemption purposes). Qualifies for head of household filing status. Not available to 65-year old taxpayer who itemizes. Exception for U.S. citizenship or residency test (for dependency exemption purposes). Considered for dependency exemption purposes. Largest basic standard deduction available to a dependent who has no earned income. A child (age 16) who is a dependent and has only unearned income of $4,500. A child (age 15) who is a dependent and has only earned income. Exception to the support test (for dependency exemption purposes). No correct match provided.

[a] 1. Surviving spouse [b] 2. Scholarship funds for tuition [c] 3. Additional standard deduction [d] 4. Scholarship funds for room and board [e] 5. Abandoned spouse [f] 6. Basic standard deduction [g] 7. Resident of Canada or Mexico [h] 8. Age of a qualifying child [i] 9. $1,000 [j] 10. Kiddie tax applies [k] 11. Kiddie tax does not apply [l] 12. Multiple support agreement

a. Unmarried taxpayer who can use the same tax rates as married persons filing jointly. b. Not considered in applying the gross income test (for dependency exemption purposes). c. Not available to 65-year old taxpayer who itemizes. d. Considered in applying gross income test (for dependency exemption purposes). e. Qualifies for head of household filing status. f. Not available to 65-year old taxpayer who itemizes. g. Exception for U.S. citizenship or residency test (for dependency exemption purposes). h. Considered for dependency exemption purposes. i. Largest basic standard deduction available to a dependent who has no earned income. j. A child (age 16) who is a dependent and has only unearned income of $4,500. k. A child (age 15) who is a dependent and has only earned income. l. Exception to the support test (for dependency exemption purposes). m. No correct match provided.

353. Match the statements that relate to each other. Note: Choice L may be used more than once.Multiple support agreementKiddie tax may be imposedNonresident alienTax Rate ScheduleGain on collectibles (held more than one year)Average income tax rateMarginal income tax rateAdditional standard deductionRelationship test (for dependency exemption purposes)Long-term capital gainsGlobal system of taxationTerritorial system of taxationNo one qualified taxpayer meets the support test. A dependent child (age 18) who has only unearned income. Not eligible for the standard deduction. Highest applicable rate is 35%. Maximum rate is 28%. Equal to tax liability divided by taxable income. The highest income tax rate applicable to a taxpayer. Available to a 70-year-old father claimed as a dependent by his son. Taxpayer’s ex-husband does not qualify. Applicable rate could be as low as 0%. No correct match provided. Income from foreign sources is not subject to tax.

[a] 1. Multiple support agreement [b] 2. Kiddie tax may be imposed [c] 3. Nonresident alien [d] 4. Tax Rate Schedule [e] 5. Gain on collectibles (held more than one year) [f] 6. Average income tax rate [g] 7. Marginal income tax rate [h] 8. Additional standard deduction [i] 9. Relationship test (for dependency exemption purposes) [j] 10. Long-term capital gains [k] 11. Global system of taxation [l] 12. Territorial system of taxation

a. No one qualified taxpayer meets the support test. b. A dependent child (age 18) who has only unearned income. c. Not eligible for the standard deduction. d. Highest applicable rate is 35%. e. Maximum rate is 28%. f. Equal to tax liability divided by taxable income. g. The highest income tax rate applicable to a taxpayer. h. Available to a 70-year-old father claimed as a dependent by his son. i. Taxpayer’s ex-husband does not qualify. j. Applicable rate could be as low as 0%. k. No correct match provided. l. Income from foreign sources is not subject to tax.

354. Emily had the following transactions during 2013:

Salary $90,000 Interest income on bonds— Issued by City of $4,000 Nashville Issued by Chevron 5,00 9,000 Corporation 0 Alimony received 5,000 Child support received 20,000 City and state income taxes (5,000) paid Bank loan obtained to pay for 15,000 car purchase

What is Emily’s AGI for 2013?

Correct Answer: $100,000. $90,000 (salary) + $5,000 (interest on Chevron Corporation bonds) + $5,000 (alimony received). Interest on the City of Nashville bonds is an exclusion from gross income. The bank loan has no tax effect, as Emily is obligated to repay the amount borrowed. City and state income taxes are deductions from AGI.

355. Edgar had the following transactions for 2013:

Salary $ 80,000 Alimony paid (4,000) Recovery from car accident— Personal injury $40,000 damages Punitive damages 70,000 110,000 Gift from parents 20,000 Property sales— Loss on sale of boat ($ 4,000) (used for pleasure and owned 4 years) Gain on sale of ADM 4,000 (–0–) stock (held for 10 months as an investment)

What is Edgar’s AGI for 2013?

Correct Answer: $150,000. $80,000 (salary) – $4,000 (alimony paid) + $70,000 (punitive damage award) + $4,000 (short-term capital gain on the sale of stock investment). The personal injury recovery and the gift from Edgar’s parents are exclusions from gross income. The loss from the sale of the boat is personal and, therefore, nondeductible. The short-term capital gain on the sale of the ADM stock is taxed in full as ordinary income.

356. Taylor had the following transactions for 2013:

Salary $ 85,000

Moving expenses incurred to (12,000) change jobs Inheritance received from 300,000 deceased uncle Life insurance proceeds from 200,000 policy on uncle’s life (Taylor was named the beneficiary) Cash prize from church raffle 3,000 Payment of church pledge (4,500)

What is Taylor’s AGI for 2013?

Correct Answer: $76,000. $85,000 (salary) + $3,000 (raffle prize) – $12,000 (moving expenses). The inheritance and life insurance proceeds are exclusions from gross income. The payment by Taylor of her church pledge is a deduction from AGI. Thus, it does not enter into the determination of AGI.

357. Tom is single and for 2013 has AGI of $50,000. He is age 70 and has no dependents. For 2013, he has itemized deductions from AGI of $7,000. Determine Tom’s taxable income for 2013.

Correct Answer: $38,500. Tom’s standard deduction is $6,100 (basic) + $1,500 (additional) for a total of $7,600. Consequently, he should select the standard deduction option since it exceeds his itemized deductions of $7,000. Thus, his taxable income is determined as follows: $50,000 (AGI) – $7,600 (standard deduction) – $3,900 (personal exemption) = $38,500.

358. Warren, age 17, is claimed as a dependent by his father. In 2013, Warren has dividend income of $1,500 and earns $400 from a part-time job. a. What is Warren’s taxable income for 2013? b. Suppose Warren earned $1,200 (not $400) from the part-time job. What is Warren’s taxable income for 2013?

Correct Answer: a. $900. Warren’s standard deduction is the greater of $400 (earned income) + $350 or $1,000. Thus, $1,500 + $400 – $1,000 = $900 taxable income. b. $1,150. Warren’s standard deduction now becomes $1,550 ($1,200 + $350). Thus, $1,500 + $1,200 – $1,550 = $1,150 taxable income.

359. Meg, age 23, is a full-time law student and is claimed by her parents as a dependent. During 2013, she received $1,400 interest income from a bank savings account and $5,900 from a part-time job. What is Meg’s taxable income for 2013?

Correct Answer: $1,200. Meg’s standard deduction is the greater of $5,900 (earned income) + $350 or $1,000. But the $6,250 is limited to $6,100 (the standard deduction allowed a single person). Thus, $1,400 + $5,900 – $6,100 = $1,200 taxable income.

360. Heloise, age 74 and a widow, is claimed as a dependent by her daughter. For 2013, she had income as follows: $2,500 interest on municipal bonds; $3,200 Social Security benefits; $3,000 income from a part-time job; and $2,800 dividends on stock investments. What is Heloise’s taxable income for 2013?

Correct Answer: $950. $3,000 (income from job) + $2,800 (dividends) – $3,350 (basic standard deduction is $3,000 + $350) – $1,500 (additional standard deduction for age) = $950. The Social Security benefits of $3,200 and the interest on municipal bonds of $2,500 are not taxable.

361. Pedro is married to Consuela, who lives with him. Both are U.S. citizens and residents of Nebraska. Pedro furnishes all of the support of his parents, who are citizens and residents of Mexico. He also furnishes all of the support of Consuela’s parents, who are citizens and residents of El Salvador. Consuela has no gross income for the year. If Pedro files as a married person filing separately, how many personal and dependency exemptions can he claim on his return?

Correct Answer: Four. A personal exemption for Pedro and Consuela and dependency exemptions for Pedro’s parents. Consuela can be claimed because she has no income. Presumably she is not being claimed as a dependent by another. Although Pedro’s parents are neither U.S. citizens nor residents, they are residents of Mexico. Consuela’s parents meet neither the citizenship nor residency tests.

362. Homer (age 68) and his wife Jean (age 70) file a joint return. They furnish all of the support of Luther (Homer’s 90-year old father), who lives with them. For 2013, they received $6,000 of interest income on city of Chicago bonds and interest income on corporate bonds of $48,000. Compute Homer and Jean’s taxable income for 2013.

Correct Answer: $21,700. Their gross income is $48,000 since the $6,000 interest on municipal bonds is an exclusion. They are entitled to a basic standard deduction of $12,200 and additional standard deductions of $1,200 each for being age 65 or older. They can claim a dependency exemption of $3,900 for Luther and two personal exemptions for themselves. Thus, $48,000 – $12,200 – $2,400 (2 × $1,200) – $11,700 (3 × $3,900) = $21,700.

363. Ellen, age 39 and single, furnishes more than 50% of the support of her parents, who do not live with her. Ellen practices as a self- employed interior decorator and has gross income in 2013 of $120,000. Her deductions are as follows: $30,000 business and $8,100 itemized. a. What is Ellen’s taxable income for 2013? b. Can Ellen qualify for head of household filing status? Explain.

Correct Answer: a. $69,350. $120,000 (gross income) - $30,000 (business deductions for AGI) = $90,000 (AGI) – $8,950 (standard deduction) – $3,900 (personal exemption) – $7,800 (dependency exemptions for parents) = $69,350 taxable income. The answer presumes that the parents meet the other dependency exemption tests (e.g., gross income) besides support. b. Ellen can qualify for head of household filing status if she furnishes more than half of the cost of maintaining her parents’ household. Also, at least one of Ellen’s parents must qualify as her dependent (see part a. above).

364. Ashley earns a salary of $55,000, has capital gains of $3,000, and interest income of $5,000 in 2013. Her husband died in 2012. Ashley has a dependent son, Tyrone, who is age 8. Her itemized deductions are $9,000. a. Calculate Ashley’s taxable income for 2013. b. What is her filing status?

Correct Answer: a. Salary $55,000 Capital gains 3,000 Interest 5,00 0 AGI $63,000 Less: Standard deduction (12,200) Less: Personal exemption and (7,800) dependency deduction ($3,900 × 2) Taxable income $43,000

b. Ashley satisfies the requirements for a surviving spouse.

365. During the year, Irv had the following transactions:

Long-term loss on the sale of $7,000 business use equipment Long-term loss on the sale of 6,000 personal use camper Long-term gain on the sale of 3,000 personal use boat Short-term loss on the sale of stock 4,000 investment Long-term loss on the sale of land 5,000 investment

How are these transactions handled for income tax purposes?

Correct Answer: Ordinary loss of $7,000 on the business equipment. The $6,000 loss on the camper is personal and not deductible. However, the $3,000 gain on the boat is taxable and is applied against the long-term capital loss on the land, reducing it to $2,000. The $4,000 short-term capital loss on the stock offsets ordinary income up to $3,000. The unused remaining $1,000 short-term capital loss and the $2,000 long-term capital loss from the land sale are carried over to future years.

366. During 2013, Addison has the following gains and losses:

LTCG $10,000 LTCL 3,000 STCG 2,000 STCL 7,000

a. How much is Addison’s tax liability if she is in the 15% tax bracket? b. If her tax bracket is 33% (not 15%)?

Correct Answer: a. $0. After the initial netting process, there is a LTCG of $7,000, and a STCL of $5,000. The $5,000 of STCL is applied to the LTCG of $7,000. The final result is a net LTCG of $2,000 taxed at 0% for a tax liability of $0. b. $300. See part a. for the netting process. Now the $2,000 is taxed at 15% for a tax liability of $300.

367. During 2013, Jackson had the following capital gains and losses:

Gain from the sale of coin $12,000 collection (held three years) Gain from the sale of land held as 9,000 an investment for six years Gain from the sale of stock held as 3,000 an investment (held for 10 months)

a. How much is Jackson’s tax liability if he is in the 15% tax bracket? b. If his tax bracket is 33% (not 15%)?

Correct Answer: a. $2,250. Gain of $12,000 on the sale of the coin collection is taxed at 15% (lesser of 28% or 15%). The same is true for the short- term gain of $3,000. The gain of $9,000 on the sale of the land is taxed at 0%. Thus, (15% × $15,000) + (0% × $9,000) = $2,250. b. $5,700. (33% × $3,000) + (28% × $12,000) + (15% × $9,000) = $5,700.

368. During 2013, Madison had salary income of $80,000 and the following capital transactions:

LTCG $13,000 LTCL 15,000 STCG 13,000 STCL 6,000

How are these transactions handled for income tax purposes?

Correct Answer: Combining the long-term transactions yields a net LTCL of $2,000 ($13,000 – $15,000), while the short-term process results in a net STCG of $7,000 ($13,000 – $6,000). A further combination leaves a net STCG of $5,000 ($7,000 – $2,000) which is taxed as ordinary income. Only net LTCG results in preferential tax treatment.

369. Mr. Lee is a citizen and resident of Hong Kong, while Mr. Anderson is a citizen and resident of the U.S. In the taxation of income, Hong Kong uses a territorial approach, while the U.S. follows the global system. In terms of effect, explain what this means to Mr. Lee and Mr. Anderson.

Correct Answer: Mr. Lee is taxed only on the income he receives from Hong Kong, while Mr. Anderson is taxed on his global income. Under the U.S. approach, a citizen or resident is taxed on a worldwide basis. Since the U.S. system could lead to the same income being taxed twice, various relief provisions are necessitated (e.g., foreign tax credit).

370. The Deweys are expecting to save on their taxes for 2013. Not only have both incurred large medical expenses, but both reached age 65. During the year, they also recognized a $30,000 loss on some land they sold which was purchased as an investment several years ago. Are the Deweys under a mistaken understanding regarding their tax position? Explain.

Correct Answer: The Deweys are expecting to qualify for two additional standard deductions and anticipating a deduction for medical expenses. The two objectives cannot coexist. Claiming a medical deduction requires that they itemize. Taxpayers who itemize, however, cannot claim any type of standard deduction. Regarding the capital loss, and presuming no capital gains, only $3,000 can be deducted against their other income. The balance of $27,000 must be carried over to future years.

371. Deductions for AGI are often referred to as “above-the-line” or “page 1” deductions. Explain.

Correct Answer: “Above the line” means before the bottom line of page 1 of Form 1040, which is AGI. These deductions appear on page 1 of Form 1040.

372. Adjusted gross income (AGI) sets the ceiling or the floor for certain deductions. Explain and illustrate what this statement means.

Correct Answer: By a ceiling what is meant is that the deduction cannot exceed a percentage of AGI. Thus, the charitable contribution deduction cannot exceed 50% of a taxpayer’s AGI. By a floor what is meant is that a deduction is allowed only if it exceeds a percentage of AGI. Thus, the deduction for medical expenses is limited to the excess of these expenses over 10% (or 7.5%) of AGI.

373. During the current year, Doris received a large gift from her parents and a sizeable inheritance from an uncle. She also paid premiums on an insurance policy on her life. Doris is confused because she cannot find any place on Form 1040 to report these items. Explain.

Correct Answer: Gifts and inheritances are exclusions from gross income. Like most exclusions, they are not reported on Form 1040. Premiums on a personal life insurance policy are nondeductible. Nondeductible items, such as these premiums, are not reported on Form 1040

374. Mel is not quite sure whether an expenditure he made is a deduction for AGI or a deduction from AGI. Since he plans to choose the standard deduction option for the year, does the distinction matter? Explain.

Correct Answer: It makes a great deal of difference if the expenditure is a deduction for AGI. If it is, Mel will benefit taxwise. It makes no difference, however, if it is a deduction from. The standard deduction is in lieu of itemized deductions.

375. When filing their Federal income tax returns, the Youngs always claimed the standard deduction. After they purchased a home, however, they started to itemize their deductions from AGI.

a. Explain the reason for the change. b. Suppose they purchased the home in November 2012, but did not start itemizing until tax year 2013. Why the delay as to itemizing?

Correct Answer: a. The interest on the home mortgage and the property taxes gave the Youngs itemized deductions in excess of the applicable standard deduction. b. The home mortgage interest and property taxes for two months (i.e., November and December) may not have been enough to place the Youngs in a position to exceed the applicable standard deduction for 2012. In 2013, however, a full 12 months worth of home mortgage interest and property taxes in involved.

376. The Dargers have itemized deductions that exceed the standard deduction. However, when they file their joint return, they choose the standard deduction option. a. Is this proper procedure? b. Aside from a possible misunderstanding as to the tax law, what might be the reason for the Darger’s choice?

Correct Answer: a. Yes. The choice between itemizing and claiming the standard deduction is elective and up to the taxpayer. b. The excess of the itemized deductions over the standard deduction may be marginal, and the Dargers are willing to forgo the effort of itemizing for a small tax savings. Also, they may not maintain the records (i.e., substantiation) that some itemized deductions require.

377. Under what circumstances, if any, may an ex-spouse be claimed as a dependent?

Correct Answer: As an ex-spouse does not meet the relationship test, he or she must be a member of the taxpayer’s household. The association cannot be in violation of local law and the year involved cannot be the year of the divorce.

378. In resolving qualified child status for dependency exemption purposes, why are tiebreaker rules necessary? Can these rules be waived?

Correct Answer: A person being claimed as a dependent may satisfy qualified child status as to more than one taxpayer. See Example 16 in the text. The tiebreaker rules can be waived.

379. In satisfying the support test and the gross income test for claiming a dependency exemption, a scholarship received by the person being claimed is handled the same way for each test. Do you agree or disagree with this statement? Why?

Correct Answer: Disagree. For purposes of the support test, all of the scholarship is disregarded. For purposes of the gross income test, only the taxable part is considered (i.e., the nontaxable part is disregarded).

380. In order to claim a dependency exemption for other than a qualifying child, a taxpayer must meet the support test. Generally, this is done by furnishing more than 50% of a dependent’s support. What exceptions exist, if any, where the support furnished need not be more than 50%?

Correct Answer: One exception involves the multiple support agreement. Here, family members collectively furnish more than 50% of the support, but no one person does so. For those qualified individuals who contribute more than 10%, the group can designate which person may claim the dependency exemption.

The second exception involves the divorced parents of children. The custodial parent is entitled to the dependency exemptions for the children. If this parent agrees not to claim the exemption(s), then the noncustodial parent may do so.

381. In applying the gross income test in the case of dependents that are married, could the application of community property laws have any effect? Explain.

Correct Answer: Most often, the application of community property laws will impact on the dependency status of the spouse of a qualifying child. Suppose, for example, Roger maintains a household that includes his 18-year-old daughter, Alice, and her husband, Craig. Assume further that Alice earns $8,000 from a part-time job while Craig has no income. In a common law state, Craig meets the gross income test (i.e., $0) while Alice’s gross income, as a qualifying child, is immaterial. In a community property state, however, Craig now violates the gross income test with $4,000 (50% × $8,000) of income, while Alice remains immune.

382. In meeting the criteria of a qualifying child for dependency exemption purposes, when if ever, might the child’s income become relevant?

Correct Answer: The amount of income earned by the qualifying child normally is of no consequence. If, however, such income is used to make the child self- supporting, then he or she can no longer be a qualifying child. Such child also would not be a qualifying relative due to the gross income and support tests.

383. Lena is 66 years of age, single, and blind and is not claimed as a dependent. How much gross income must she have before she is required to file a Federal income tax return for 2013?

Correct Answer: $11,500. $6,100 (basic standard deduction) + $1,500 (additional standard deduction for age) + $3,900 (personal exemption). Note that no additional standard deduction is allowed for blindness.

384. When can a taxpayer not use Form 1040EZ? Form 1040A?

Correct Answer: Form 1040EZ cannot be used when the taxpayer claims any dependents; is age 65 or older (or blind); or, has taxable income of $100,000 or more. Form 1040A cannot be used if the taxpayer itemizes deductions from AGI.

385. Contrast the tax consequences resulting from the following filing status situations: a. Married filing jointly versus married filing separately. b. Married filing separately versus single filing separately. c. Married filing separately versus abandoned spouse status.

Correct Answer: a. Married persons filing jointly have a number of tax elections available to them that cannot be chosen if they file separate returns. For example, the credit for child and dependent care expenses and the earned income credit are not available unless married persons file joint returns. b. Married persons filing separately often will not fare as well as the couple that remains single. For one advantage, each single person has full flexibility in choosing between the standard deduction and itemizing and is not bound by what the companion does. A second advantage is the ability of each to apply a full $3,000 of excess capital losses against ordinary income. For married persons filing separate returns, the ordinary income offset is restricted to $1,500 each. c. Because abandoned spouse status means that the taxpayer is treated as being single, the same advantages mentioned in part a. above exist when compared to married persons filing separate returns. Even more advantageous is that abandoned spouse status permits the use of head of household filing status. Head of household tax rates are lower than those applicable to single persons (and married persons filing separate returns). Also, the standard deduction amount for head of household filing status is larger than that available to single persons (and married persons filing separate returns).

386. Jayden and Chloe Harper are husband and wife and use the calendar year for tax purposes. a. If the Harpers file a joint return for 2013, can they later switch to separate returns for 2013? b. If the Harpers file separate returns for 2013, can they later switch to a joint return for 2013?

Correct Answer: a. Unless the Harpers do so on or before the regular filing date (i.e., April 15, 2014), they cannot switch to separate returns. b. Yes, they can unless the statute of limitations has run (usually three years from the filing date).

387. When married persons file a joint return, joint and several liability results. What does this mean?

Correct Answer: Joint and several liability means that either spouse is fully liable for any income tax due for the year. Thus, if more tax is due, the IRS can pursue either spouse for the deficiency.

388. Regarding head of household filing status, comment on the following: a. A taxpayer qualifies even though he maintains a household which he and the dependent do not share. b. A taxpayer does not qualify even though the person sharing the household is a dependent. c. The usual eventual filing status of a surviving spouse.

Correct Answer: a. If the household is that of a dependent parent, it need not be taxpayer’s household. b. If the household does not include a dependent that meets the relationship test, head of household filing status is unavailable. An example would be a taxpayer who maintains a household for a cousin who lives with her. Even if the cousin is a dependent under the member of the household test, taxpayer does not qualify for head of household filing status. A cousin does not satisfy the relationship test. c. Once the two-year surviving spouse period terminates, the taxpayer usually will qualify for head of household filing status.

389. The major advantage of being classified as an abandoned spouse is that the taxpayer is treated for tax purposes as being single and not married. This means that an abandoned spouse can use the more favorable tax rates available to single persons than those available to married persons filing separately. Comment on the accuracy of this conclusion.

Correct Answer: The conclusion is incorrect. The classification of abandoned spouse allows the taxpayer to the use of the rates for head of household filing status which are more favorable than married filing separately.

390. For the past few years, Corey’s filing status has been as follows: 2009 (married/joint); 2010 (married/separate); 2011 (surviving spouse); 2012 (surviving spouse); and 2013 (head of household). Explain what probably has happened.

Correct Answer: One probable explanation is that Corey’s wife died in 2010 and the executor of her estate refused to agree to filing a joint return. As surviving spouse status does not continue beyond two years, Corey is relegated to head of household status in 2013.

391. DeWayne is a U.S. citizen and resident. He spends much of each year in the United Kingdom on business. He is married to Petula, a U.K. citizen and resident of London. DeWayne has heard that it is possible that he can file a joint income tax return for U.S. purposes. If this is so, what are the constraints he should consider in making any such decision?

Correct Answer: The election to file a joint return has the effect of treating Petula as a U.S. resident. Unfortunately, this places Petula on the U.S. global approach to taxation. If Petula has considerable income of her own, the election makes this income subject to U.S. taxation which may be disadvantageous.

392. For 2013, Tom has taxable income of $48,005. When he uses the Tax Tables, Tom finds that his tax liability is higher than under the Tax Rate Schedules. a. Why the difference? b. Can Tom use the Tax Rate Schedules?

Correct Answer: a. Even though the Tax Tables are based on the Tax Rate Schedules, minor differences in the tax liabilities will result. The variance is due to the fact that the tax for any table bracket amount is determined by using the midpoint amount. In Tom’s case, the tax on the $48,000 – $48,050 bracket is the tax on $48,025. Because Tom’s taxable income (i.e., $48,005) is below $48,025, his tax will be higher. b. No. Except in special situations, taxpayers must use the Tax Tables.

393. List at least three exceptions to the application of the kiddie tax.

Correct Answer: • Unearned income of $2,000 or less. • Age 19 (or age 24 if a full-time student) or older. • Both parents deceased. • Earned income in excess of 50% of support. • Married and filing a joint return with spouse.

394. The Martins have a teenage son who has become an accomplished bagpiper. With proper promotion and scheduling, the son has good income potential by charging for his services at special events (particularly funerals). However, the Martins are fearful that the income could generate a kiddie tax and cause them the loss of a dependency exemption deduction. Are the Martins’ concerns justified? Explain.

Correct Answer: The income received by the son would be earned income. Therefore, the kiddie tax is not a problem since it applies only to unearned income. As long as the son is under age 19 (or a full-time student under age 24), he is a dependent as a qualifying child. Under these rules, the amount of the son’s income does not matter (unless he becomes self- supporting). If the son is age 19 (or older) and not a student, any dependency exemption must satisfy the qualifying relative rules. Here, not meeting the gross income test would cause the dependency exemption to be lost.

395. In early 2013, Ben sold a yacht, held for 9 months and for pleasure, for a $5,000 gain. Concerned about offsetting the gain before year-end, Ben is considering selling one of the following—each of which would yield a $5,000 loss:

· Houseboat used for recreation.

· Truck used in business.

· Stock investment held for 13 months.

Evaluate each choice.

Correct Answer: The sale of the houseboat produces no benefit since losses on personal use property are not deductible. The sale of the truck yields an ordinary loss of $5,000. The ordinary loss result offsets the ordinary income caused by a short-term capital gain. The best choice, however, is the stock investment. A net long-term capital loss can neutralize a net short-term capital gain and prevent ordinary income from materializing. By itself, a net long-term capital loss can only be offset against regular income to the extent of $3,000. Also, it might obviate long-term capital gains which are taxed at preferential tax rates.

396. After paying down the mortgage on their personal residence, the Hills have found that their itemized deductions for each year are always slightly less than the standard deduction option. a. Explain what has happened. b. What remedy do you suggest?

Correct Answer: a. Paying down the mortgage reduced the interest expense deduction. With less interest expense, the Hills’ deductions from AGI no longer exceed the standard deduction amount. b. The Hills should begin concentrating their other itemized deductions (e.g., charitable contributions) by paying for multiple years in the same year. Being on a cash basis, the timing of the deduction is based on the year of payment. In alternate years, moreover, the standard deduction is claimed.

397. Maude’s parents live in another state and she cannot claim them as her dependents. If Maude pays their medical expenses, can she derive any tax benefit from doing so? Explain.

Correct Answer: If Maude could otherwise claim her parents as dependents except for not satisfying either the gross income or the joint return tests, she can claim any medical expenses paid on their behalf.

398. The realization requirement gives an incentive to own assets that have increased in value and to sell assets whose value has decreased.

a. True *b. False

399. A sole proprietorship purchased an asset for $1,000 in 2013 and its value was $1,500 at the end of 2013. In 2014, the sole proprietorship sold the asset for $1,400. The sole proprietorship realized a taxable gain of $400 in 2014 but an economic loss of $100 in 2014.

*a. True b. False

400. Judy is a cash basis attorney. In 2013, she performed services in connection with the formation of a corporation and received stock with a value of $4,000 for her services. By the end of the year, the value of the stock had decreased to $2,000. She continued to hold the stock. Judy must recognize $4,000 of gross income from the stock for 2013.

*a. True b. False

401. Barney painted his house which saved him $3,000. According to the realization requirement, Barney must recognize $3,000 of income.

a. True *b. False

402. Nicholas owned stock that decreased in value by $20,000 during the year, but he did not sell the stock. He earned $45,000 salary, but received only $34,000 because $11,000 in taxes were withheld. Nicholas saved $10,000 of his salary and used the remainder for personal living expenses. Nicholas’s economic income for the year exceeded his gross income for tax purposes.

a. True *b. False

403. The fact that the accounting method the taxpayer uses to measure income is consistent with GAAP does not assure that the method will be acceptable for tax purposes.

*a. True b. False

404. The financial accounting principle of conservatism is not well- suited to the task of measuring taxable income.

*a. True b. False

405. A cash basis taxpayer purchased a certificate of deposit for $1,000 on July 1, 2013 that will pay $1,100 upon its maturity on June 30, 2015. The taxpayer must recognize a portion of the income in 2013.

*a. True b. False

406. Ralph purchased his first Series EE bond during the year. He paid $709 for a 10-year bond with a $1,000 maturity value. The yield to maturity on the bonds was 3.5%. Ralph is not required to recognize the $291 ($1,000 – $709) original issue discount until the bond matures. However, Ralph can elect to amortize the discount over the ten-year period.

*a. True b. False

407. At the beginning of 2013, Mary purchased a 3-year certificate of deposit (CD) for $8,760. The maturity value of the certificate was $10,000 and it was to yield 4.5%. She also purchased a Series EE bond for $6,400 with a maturity value in 10 years of $10,000. Mary must recognize $1,240 of income from the certificate of deposit in 2013, and $3,600 from the Series EE bonds in 2022.

a. True *b. False

408. In 2005, Terry purchased land for $150,000. In 2013, Terry received $10,000 from a local cable television company in exchange for Terry allowing the company to run an underground cable across Terry’s property. Terry is not required to recognize income from receiving the $10,000 because it was a return of his capital invested in the land.

*a. True b. False

409. In December 2013, Mary collected the December 2013 and January 2014 rent from a tenant. Mary is a cash basis taxpayer. The amount collected in December 2013 for the 2014 rent should be included in her 2014 gross income.

a. True *b. False

410. On December 1, 2013, Daniel, an accrual basis taxpayer, collects $12,000 rent for December 2013 and $12,000 for January 2014. Daniel must include the $24,000 in 2013 gross income.

*a. True b. False

411. On January 1, 2013, an accrual basis taxpayer entered into a contract to provide termite inspection service each month for 36 months. The amount received for the contract was $2,400. The taxpayer should report $1,600 of income in 2014.

*a. True b. False

412. An advance payment received in June 2013 by an accrual basis and calendar year taxpayer for services to be provided over a 36-month period can be spread over four tax years.

a. True *b. False

413. An accrual basis taxpayer who owns and operates a professional basketball team is allowed to allocate income from season ticket sales on the basis of the number of games played during the year.

*a. True b. False

414. In 2013, Juan, a cash basis taxpayer, was offered $3 million for signing a professional baseball contract. He counteroffered that he would receive $900,000 per year for 4 years beginning in 2014. The team accepted the counteroffer. Juan constructively received $3 million in 2013.

a. True *b. False

415. The constructive receipt doctrine requires that income must be recognized when it is made available to the cash basis taxpayer, although it has not been actually received. The constructive receipt doctrine does not apply to accrual basis taxpayers.

*a. True b. False

416. Fred is a full-time teacher. He has written a book and receives royalties from it. Fred’s mother, Mabel, is age 65 and lives on her Social Security benefits and gifts from her son, Fred. This year Fred directed the publisher to make the royalty check payable to Mabel because she needs the money for support. Fred must include the amount of the royalty check in his gross income.

*a. True b. False

417. Jessica is a cash basis taxpayer. When Jessica failed to repay a loan, the bank garnished her salary. Each week $60 was withheld from Jessica’s salary and paid to the bank. Jessica is required to include the $60 each week in her gross income even though it is the creditor that benefits from the income.

*a. True b. False

418. ABC Corporation declared a dividend for taxpayers of record as of December 24, 2013. The dividend checks were mailed on December 31, 2013. Ed, a cash basis shareholder, received the dividend check on January 2, 2014. Ed cannot delay reporting the income from the dividend until 2014.

a. True *b. False

419. Tom, a cash basis taxpayer, purchased a bond on March 31 for $10,000, plus $100 accrued interest. In December, Tom collected $500 interest from the bond. Tom’s interest income from the bond for the year is $500.

a. True *b. False

420. When stock is sold after the date of declaration but before the record date, the buyer must recognize as income the dividend declared.

*a. True b. False

421. Linda delivers pizzas for a pizza shop. On Wednesday, December 31, 2013, Linda made several deliveries and collected $400 from customers. However, Linda forgot to turn in the proceeds for the day to her employer until the following Friday, January 2, 2014. The pizza shop owner recognizes the income of $400 when he receives it from Linda in 2014.

a. True *b. False

422. Jake is the sole shareholder of an S corporation that earned $60,000 in 2013. The corporation was short on cash and therefore distributed only $15,000 to Jake in 2013. Jake is required to recognize $60,000 of income from the S corporation in 2013.

*a. True b. False

423. The B & W Partnership earned taxable income of $140,000 for the year. Bryan is entitled to 50% of the profits, but Bryan withdrew only $60,000 during the year. Bryan’s gross income from the partnership for the year is $60,000.

a. True *b. False

424. When a business is operated as an S corporation, a disadvantage is that the shareholder must pay the tax on his or her share of the S corporation’s income even though the S corporation did not distribute the income to the shareholder.

*a. True b. False

425. Mark is a cash basis taxpayer. He is a partner in the M&M partnership, and his share of the partnership’s profits for 2013 is $90,000. Only $40,000 was distributed to him in January 2013, and this was his share of the 2012 partnership profits. None of the 2013 profits were distributed although Mark’s share of the 2013 profits was $90,000. Mark’s gross income from the partnership for 2013 is $40,000.

a. True *b. False

426. Rhonda has a 30% interest in the capital and profits of the ABC Partnership. In the first year of the partnership, 2013, it earned $150,000. However, the partners agreed that nothing would be distributed until after the end of March 2014, before Rhonda filed her 2013 tax return. The distributions were to be delayed because it was unclear as to whether business conditions would remain good in 2014. Things were going well in 2014 and therefore the partnership distributed $30,000 to Rhonda at the end of March, as a portion of her share of the partnership’s 2013 earnings. The partnership’s income for 2014 was $60,000. As a result, Rhonda must recognize $30,000 of gross income in 2013 and $18,000 in 2014.

a. True *b. False

427. April, a calendar year taxpayer, is a 40% partner in Pale Partnership, whose fiscal year ends on September 30th. For the fiscal year ending September 30, 2013, the partnership had $400,000 net income and for fiscal year ending September 30, 2014, the partnership had $300,000 net income. April withdrew $100,000 in December of each year. April’s gross income from the partnership for 2013 is $160,000 ($400,000 ´ 40%).

*a. True b. False

428. Alvin is the sole shareholder of an S corporation that earned $200,000 in 2013 and distributed $75,000 to Alvin. Alvin must recognize $75,000 as income from the S corporation in 2013.

a. True *b. False

429. Samantha and her son, Brent, are cash basis taxpayers. Samantha gave Brent a corporate bond with a face amount and fair market value of $10,000. On the date of the gift, March 31, 2013, the accrued interest on the bond was $100. On December 31, 2013, Brent collected $400 interest on the bond. Brent must include in gross income the $300 interest earned after the date of the gift.

*a. True b. False

430. In all community property states, the income from property that was inherited by a spouse after the marriage is treated as all earned by the spouse who inherited the property.

a. True *b. False

431. Ted earned $150,000 during the current year. He paid Alice, his former wife, $75,000 in alimony. Under these facts, the tax is paid by the person who benefits from the income rather than the person who earned the income.

*a. True b. False

432. George and Erin are divorced, and George is required to pay Erin $20,000 of alimony each year. George earns $75,000 a year. Erin is required to include the alimony payments in gross income although George earned the income.

*a. True b. False

433. After the divorce, Jeff was required to pay $18,000 per year to his former spouse, Darlene, who had custody of their child. Jeff’s payments will be reduced to $12,000 per year in the event the child dies or reaches age 21. During the year, Jeff paid the $18,000 required under the divorce agreement. Darlene must include the $12,000 in gross income.

*a. True b. False

434. Paula transfers stock to her former spouse, Fred. The transfer is pursuant to a divorce agreement. Paula’s cost of the stock was $75,000 and its fair market value on the date of the transfer is $95,000. Fred later sells the stock for $100,000. Fred’s recognized gain from the sale of the stock is $5,000.

a. True *b. False

435. Jacob and Emily were co-owners of a personal residence. As part of their divorce agreement, Emily paid Jacob cash for his interest in the personal residence. This cash payment results in a taxable gain to Jacob if he receives more cash than his share of the cost of the residence.

a. True *b. False

436. Alimony recapture may occur if there is a substantial decrease in the amount of the alimony payments in the second year.

*a. True b. False

437. If the alimony recapture rules apply, the recipient of the alimony decreases his or her AGI by a portion of the amount included in gross income as alimony in a prior year or years.

*a. True b. False

438. Father made an interest-free loan of $25,000 to Son who used the money to buy an SUV. Son had $1,600 interest income from a certificate of deposit for the year. Father is not required to impute interest income.

a. True *b. False

439. In the case of a below-market gift loan for which there is no exception to the imputed interest rules, the lender is deemed to have received interest income even though no interest is charged and collected.

*a. True b. False

440. In the case of a gift loan of less than $100,000, the imputed interest rules apply if the donee has net investment income of over $1,000.

*a. True b. False

441. Susan purchased an annuity for $200,000. She is to receive $18,000 each year and her life expectancy is 13 years. If Susan collects under the annuity for 14 years, the entire $18,000 received in the 14th year must be included in her gross income.

*a. True b. False

442. Terri purchased an annuity for $100,000. She was to receive $10,000 per year and her life expectancy was 20 years. She died after receiving 8 payments. Terri’s final return should reflect a loss of $20,000 ($100,000 – $80,000).

a. True *b. False

443. If a lottery prize winner transfers the prize to a qualified government unit or nonprofit organization, then the prize is excluded from the winner’s gross income if the amount of the prize does not exceed 30% of the winner’s AGI.

a. True *b. False

444. If the employer provides all employees with group term life insurance equal to twice the employee’s annual salary, an employee with a salary of $50,000 has no gross income from the life insurance protection provided by the employer.

a. True *b. False

445. In the case of a person with other income of $300,000, 15% of his or her Social Security benefits received are excluded from gross income.

*a. True b. False

446. Norma’s income for 2013 is $27,000 from part-time work and $9,000 of Social Security benefits. Norma is not married. A portion of her Social Security benefits must be included in her gross income.

*a. True b. False

447. Lois, who is single, received $9,000 of Social Security benefits. She also received $25,000 from dividends, interest, and her employer’s pension plan. If Lois sells a capital asset that produces a $1,000 recognized loss, Lois’s taxable income will decrease by more than $1,000.

*a. True b. False

448. On a particular Saturday, Tom had planned to paint a room in his house, but his employer gave him the opportunity to work that day. If Tom works, he must hire a painter for $120. For Tom to have a positive cash flow from working and hiring the painter:

*a. Tom must earn more than $160 if he is in the 25% marginal tax bracket. b. Tom must earn at least $160 if he is in the 33% marginal tax bracket. c. Tom must earn at least $150 if he is in the 25% marginal tax bracket. d. Tom must earn at least $135 if he is in the 15% marginal tax bracket. e. None of the above.

449. The tax concept and economic concept of income are in agreement on which of the following:

a. The fair rental value of an owner-occupied home should be included in income. b. The increase in value of assets held for the entire year should be included in income for the year. *c. Rent income for 2014 collected in 2013 is income for 2013. d. All of the above. e. None of the above.

450. The Blue Utilities Company paid Sue $2,000 for the right to lay an underground electric cable across her property anytime in the future.

a. Sue must recognize $2,000 gross income in the current year if the company did not install the cable during the year. *b. Sue is not required to recognize gross income from the receipt of the funds, but she must reduce her cost basis in the land by $2,000. c. Sue must recognize $2,000 gross income in the current year regardless of whether the company installed the cable during the year. d. Sue must recognize $2,000 gross income in the current year, and when the cable is installed, she must reduce her cost basis in the land by $2,000. e. None of the above.

451. For purposes of determining gross income, which of the following is true?

a. A mechanic completed repairs on an automobile during the year and collects money from the customer. The customer was not satisfied with the repairs and sued the mechanic for a refund. The mechanic can defer recognition of the income until the suit has been settled. *b. A taxpayer who finds a wallet full of money is required to recognize income even though someone may eventually ask for the return of the money. c. Embezzlement proceeds are not included in the embezzler’s gross income because the embezzler has an obligation to repay the owner. d. All of the above are false. e. All of the above are true.

452. Detroit Corporation sued Chicago Corporation for intentional damage to Detroit’s goodwill. Detroit had created its goodwill through providing high-quality services to its customers. Thus, no basis for the goodwill appeared on Detroit’s balance sheet. The suit was settled and Detroit received $1,500,000 for the damages to its goodwill.

a. The $1,500,000 is not taxable because it represents a recovery of capital. *b. The $1,500,000 is taxable because Detroit has no basis in the goodwill. c. The $1,500,000 is not taxable because Detroit did nothing to earn the money. d. The $1,500,000 is not taxable because Detroit settled the case. e. None of the above.

453. The annual increase in the cash surrender value of a life insurance policy:

a. Is taxed when the individual dies and the heirs collect the insurance proceeds. b. Must be included in gross income each year under the original issue discount rules. c. Reduces the deduction for life insurance expense. *d. Is not included in gross income each year because of the substantial restrictions on gaining access to the policy’s value. e. None of the above.

454. Turner, a successful executive, is negotiating a compensation plan with his potential employer. The employer has offered to pay Turner a $600,000 annual salary, payable at the rate of $50,000 per month. Turner counteroffers to receive a monthly salary of $40,000 ($480,000 annually) and a $180,000 bonus in 5 years when Turner will be age 65.

a. If the employer accepts Turner’s counteroffer, Turner will recognize $660,000 at the time the offer is accepted. b. If the employer accepts Turner’s counteroffer, Turner will recognize as gross income $55,000 per month [($480,000 + $180,000)/12]. *c. If the employer accepts Turner’s counteroffer, Turner will recognize $40,000 income each month for the year and $180,000 in year 5. d. If the employer accepts Turner’s counteroffer, Turner must recognize imputed interest income on the $180,000 to be received in 5 years. e. None of the above.

455. Maroon Corporation expects the employees’ income tax rates to increase next year. The employees use the cash method. The company presently pays on the last day of each month. The company is considering changing its policy so that the December salaries will be paid on the first day of the following year. What would be the effect on an employee of the proposed change in company policy for paying its salaries beginning for December 2013.

a. The employee would be required to recognize the income in December 2013 because it is constructively received at the end of the month. b. The employee would be required to recognize the income in December 2013 because the employee has a claim of right to the income when it is earned. *c. The employee will not be required to recognize the income until it is received, in 2014. d. The employee can elect to either include the pay in 2013 or 2014. e. None of the above.

456. The annual increase in the cash surrender value of a life insurance policy:

a. Is taxed according to the original issue discount rules. *b. Is not included in gross income because the policy must be surrendered to receive the cash surrender value. c. Reduces the deduction for life insurance expense. d. Is exempt because it is life insurance proceeds. e. None of the above.

457. Under the original issue discount (OID) rules as applied to a three-year certificate of deposit:

a. All of the income must be recognized in the year of maturity by a cash basis taxpayer. b. The OID will be included in gross income for the year of purchase. c. The interest income will be the same each year. *d. The interest income will be greater in the third year than in the first year. e. None of the above is correct.

458. Freddy purchased a certificate of deposit for $20,000 on July 1, 2013. The certificate’s maturity value in two years (June 30, 2015) is $21,218, yielding 3% before-tax interest.

a. Freddy must recognize $1,218 gross income in 2013. b. Freddy must recognize $1,218 gross income in 2015. c. Freddy must recognize $600 (.03 × $20,000) gross income in 2015. *d. Freddy must recognize $300 (.03 × $20,000 × .5) gross income in 2013. e. None of the above.

459. Jerry purchased a U.S. Series EE savings bond for $744. The bond has a maturity value in 10 years of $1,000 and yields 3% interest. This is the first Series EE bond that Jerry has ever owned.

*a. Jerry can defer the interest income until the bond matures in 10 years. b. Jerry must report ($1,000 – $744)/10 = $25.60 interest income each year he owns the bond. c. The interest on the bonds is exempt from Federal income tax. d. Jerry can report all of the $256 as a capital gain in the year it matures. e. None of the above.

460. Office Palace, Inc., leased an all-in-one printer to a new customer, Ashley, on December 27, 2013. The printer was to rent for $600 per month for a period of 36 months beginning January 1, 2014. Ashley was required to pay the first and last month’s rent at the time the lease was signed. Ashley was also required to pay a $1,500 damage deposit. Office Palace must recognize as income for the lease:

a. $0 in 2013, if Office Palace is an accrual basis taxpayer. b. $7,800 in 2014, if Office Palace is a cash basis taxpayer. c. $2,700 in 2013, if Office Palace is a cash basis taxpayer. *d. $1,200 in 2013, if Office Palace is an accrual basis taxpayer. e. None of the above.

461. The Maroon & Orange Gym, Inc., uses the accrual method of accounting. The corporation sells memberships that entitle the member to use the facilities at any time. A one-year membership costs $480 ($480/12 = $40 per month); a two-year membership costs $720 ($720/24 = $30 per month). Cash payment is required at the beginning of the membership period. On July 1, 2013, the company sold a one-year membership and a two-year membership. The company should report as gross income from the two contracts:

a. $420 in 2013. b. $960 in 2013. c. $180 in 2015. *d. $780 in 2014. e. None of the above.

462. Orange Cable TV Company, an accrual basis taxpayer, allows its customers to pay by the year in advance ($500 per year), or two years in advance ($950). In September 2012, the company collected the following amounts applicable to future services:

October 2012-September 2014 services $144,000 (two-year contracts) October 2012-September 2013 services 128,000 (one-year contracts) Total $272,000

As a result of the above, Orange Cable should report as gross income:

a. $272,000 in 2012. b. $128,000 in 2012. c. $168,000 in 2013. *d. $222,000 in 2013. e. None of the above.

463. With respect to the prepaid income from services, which of the following is true?

a. The treatment of prepaid income is the same for tax and financial accounting. b. A cash basis taxpayer can spread the income over the period services are to be provided if all of the services will be completed by the end of the tax year following the year of receipt. *c. An accrual basis taxpayer can spread the income over the period services are to be provided if all of the services will be completed by the end of the tax year following the year of receipt. d. An accrual basis taxpayer can spread the income over the period services are to be provided on a contract for three years or less. e. None of the above.

464. With respect to income from services, which of the following is true?

a. The income is always amortized over the period the services will be rendered by an accrual basis taxpayer. b. A cash basis taxpayer can spread the income from a 24-month service contract over the contract period. *c. If an accrual basis taxpayer sells a 36-month service contract on July 1, 2013 for $3,600, the taxpayer’s 2013 gross income from the contract is $600. d. If an accrual basis taxpayer sells a 24-month service contract on July 1, 2013, one-half (12/24) the income is recognized in 2014. e. None of the above.

465. The Green Company, an accrual basis taxpayer, provides business- consulting services. Clients generally pay a retainer at the beginning of a 12-month period. This entitles the client to no more than 40 hours of services. Once the client has received 40 hours of services, Green charges $500 per hour. Green Company allocates the retainer to income based on the number of hours worked on the contract. At the end of the tax year, the company had $50,000 of unearned revenues from these contracts. The company also had $10,000 in unearned rent income received from excess office space leased to other companies. Based on the above, Green must include in gross income for the current year:

a. $60,000. b. $50,000. *c. $10,000. d. $0. e. None of the above.

466. Teal company is an accrual basis taxpayer. On December 1, 2013, a customer paid for an item that was on hand, but the customer wanted the item delivered in early January 2014. Teal delivered the item on January 4, 2014. Teal included the sale in its 2013 income for financial accounting purposes.

*a. Teal must recognize the income in 2013. b. Teal must recognize the income in the year title to the goods passed to the customer, as determined under the state laws in which the store is located. c. Teal can elect to recognize the income in either 2013 or 2014. d. Teal must recognize the income in 2014. e. None of the above.

467. On January 5, 2013, Tim purchased a bond paying interest at 6% for $30,000. On March 31, 2013, he gave the bond to Jane. The bond pays $1,800 interest on December 31. Tim and Jane are cash basis taxpayers. When Jane collects the interest in December 2013:

a. Tim must include all of the interest in his gross income. b. Jane must report $1,800 gross income for 2013. *c. Jane reports $1,350 of interest income in 2013, and Tim reports $450 of interest income in 2013. d. Jane reports $450 of interest income in 2013, and Tim reports $1,350 of interest income in 2013. e. None of the above is correct.

468. Mike contracted with Kram Company, Mike’s controlled corporation. Mike was a medical doctor and the contract provided that he would work exclusively for the corporation. No other doctor worked for the corporation. The corporation contracted to perform an operation for Rosa for $8,000. The corporation paid Mike $6,500 to perform the operation under the terms of his employment contract.

*a. Mike’s gross income is $6,500. b. Mike must recognize the $8,000 gross income because he provided the service. c. Mike must recognize $8,000 gross income since the patient obviously wanted him to perform the operation. d. The Kram Company corporation’s gross income is $1,500. e. None of the above.

469. As a general rule: I. Income from property is taxed to the person who owns the property. II. Income from services is taxed to the person who earns the income. III. The assignee of income from property must pay tax on the income. IV. The person who receives the benefit of the income must pay the tax on the income.

*a. Only I and II are true. b. Only III and IV are true. c. I, II, and III are true, but IV is false. d. I, II, III, and IV are true. e. None of the above is true.

470. On November 1, 2013, Bob, a cash basis taxpayer, gave Dave common stock. On October 30, 2013, the corporation had declared the dividend payable to shareholders of record as of November 22, 2013. The dividend was paid on December 15, 2013. The corporation has paid the $1,200 dividend once each year for the past ten years, during which Bob owned the stock. When Dave collected the dividend on December 15, 2013:

a. Bob must include $1,000 (10/12 x $1,200) of the dividend in his gross income. *b. Bob must include all of the dividend in his gross income. c. Dave must include all of the dividend in his gross income. d. Dave should treat the $1,200 as a recovery of capital. e. None of the above is correct.

471. Daniel purchased a bond on July 1, 2013, at par of $10,000 plus accrued interest of $300. On December 31, 2013, Daniel collected the $600 interest for the year. On January 1, 2014, Daniel sold the bond for $10,200.

*a. Daniel must recognize $300 interest income for 2013 and a $200 gain on the sale of the bond in 2014. b. Daniel must recognize $600 interest income for 2013 and a $200 gain on the sale of the bond in 2014. c. Daniel must recognize $600 interest income for 2013 and a $100 loss on the sale of the bond in 2014. d. Daniel must recognize $300 interest income for 2013 and a $100 loss on the sale of the bond in 2014. e. None of the above.

472. Theresa, a cash basis taxpayer, purchased a bond on July 1, 2009, for $10,000, plus $400 of accrued interest. The bond paid $800 of interest each December 31. On March 31, 2013, she sold the bond for $9,800, which included $200 of accrued interest.

*a. Theresa has $200 interest income and a $400 loss from the bond in 2013. b. Theresa has $200 interest income and a $200 gain from the bond in 2013. c. Theresa has a $100 loss from the sale of the bond and no interest income. d. Theresa’s loss on the sale of the bond is $600. e. None of the above.

473. Darryl, a cash basis taxpayer, gave 1,000 shares of Copper Company common stock to his daughter on September 29, 2013. Copper Company is a publicly held company that has declared a $2.00 per share dividend on September 30th every year for the last 20 years. Just as Darryl had expected, Copper Company declared a $2.00 per share dividend on September 30th, payable on October 15th, to stockholders of record as of October 10th. The daughter received the $2,000 dividend on October 18, 2013.

*a. The daughter must recognize the income because she owned the stock when the dividend was declared and she received the $2,000. b. Darryl must recognize the income of $2,000 because the purpose of the gift was to avoid taxes. c. Darryl must recognize $1,500 of the dividend because he owned the stock for three-fourths of the year. d. Darryl must recognize the $2,000 dividend as his income because he constructively received the dividend. e. None of the above.

474. Wayne owns a 30% interest in the capital and profits of Emerald Company (a calendar year partnership). For tax year 2013, the partnership earned revenue of $900,000 and had operating expenses of $660,000. During the year, Wayne withdrew from the partnership a total of $90,000. He also invested an additional $30,000 in the partnership. For 2013, Wayne’s gross income from the partnership is:

*a. $72,000. b. $90,000. c. $132,000. d. $162,000. e. None of the above.

475. Harry and Wanda were married in Texas, a community property state, but moved to Virginia, a common law state. The calculation of their income on a joint return:

a. Will increase as a result of changing their state of residence. b. Will decrease as a result of changing their state of residence. *c. Will not change as a result of changing their state of residence. d. Will not be permitted. e. None of the above.

476. Jim and Nora, residents of a community property state, were married in early 2011. Late in 2011 they separated, and in 2013 they were divorced. Each earned a salary, and they received income from community owned investments in all relevant years. They filed separate returns in 2011 and 2012.

*a. In 2012, Nora must report only her salary and one-half of the income from community property on her separate return. b. In 2012, Nora must report on her separate return one-half of the Jim and Nora salary and one-half of the community property income. c. In 2013, Nora must report on her separate return one-half of the Jim and Nora salary for the period they were married as well as one-half of the community property income and her income earned after the divorce. d. In 2013, Nora must report only her salary on her separate return. e. None of the above.

477. Under the alimony rules:

a. To determine whether a cash payment is alimony, one must consult the state laws that define alimony. b. A person who receives a property division has experienced an increase in wealth and thus should be subject to tax. *c. The income is included in the gross income of the recipient of the payments. d. A person who earns $90,000 and pays $20,000 in alimony is taxed on $90,000 because the $20,000 alimony is income assigned to the former spouse. e. None of the above.

478. Travis and Andrea were divorced. Their only marital property consisted of a personal residence (fair market value of $400,000, cost of $200,000), and publicly-traded stocks (fair market value of $800,000, cost basis of $500,000). Under the terms of the divorce agreement, Andrea received the personal residence and Travis received the stocks. In addition, Andrea was to receive $50,000 for eight years. I. If the $50,000 annual payments are to be made to Andrea or her estate (if she dies before the end of the eight years), the payments will qualify as alimony. II. Andrea has a taxable gain from an exchange of her one-half interest in the stocks for Travis’ one-half interest in the house and cash. III. If Travis sells the stocks for $900,000, he must recognize a $400,000 gain.

*a. Only III is true. b. Only I and III are true. c. Only I and II are true. d. I, II, and III are true. e. None of the above are true.

479. Which of the following is not a requirement for an alimony deduction?

a. The payments must be in cash. b. The payments must cease upon the death of the payee. *c. The payments must extend over at least three years. d. The payor and payee must not live in the same household at the time of the payments. e. All of the above are requirements for an alimony deduction.

480. Tim and Janet were divorced. Their only marital property was a personal residence with a value of $120,000 and cost of $50,000. Under the terms of the divorce agreement, Janet would receive the house and Janet would pay Tim $15,000 each year for 5 years, or until Tim’s death, whichever should occur first. Tim and Janet lived apart when the payments were made to Tim. The divorce agreement did not contain the word “alimony.”

a. Tim must recognize a $35,000 [$60,000 – 1/2($50,000)] gain on the sale of his interest in the house. b. Tim does not recognize any income from the above transactions. c. Janet is not allowed any alimony deductions. *d. Janet is allowed to deduct $15,000 each year for alimony paid. e. None of the above.

481. Thelma and Mitch were divorced. The couple had a joint brokerage account that included stocks with a basis of $600,000 and a fair market value of $1,000,000. Under the terms of the divorce agreement, Mitch would receive the stocks and Mitch would pay Thelma $100,000 each year for 6 years, or until Thelma’s death, whichever should occur first. Thelma and Mitch lived apart when the payments were made by Mitch. Mitch paid the $600,000 to Thelma over the six-year period. The divorce agreement did not contain the word “alimony.” Then, Mitch sold the stocks for $1,300,000. Mitch’s recognized gain from the sale is:

a. $0. b. $1,000,000 ($1,300,000 – $300,000). *c. $700,000 ($1,300,000 – $600,000). d. $300,000 ($1,300,000 – $1,000,000). e. None of the above.

482. The alimony recapture rules are intended to:

a. Assist former spouses in collecting alimony when the other spouse moves to another state. *b. Prevent tax deductions for property divisions. c. Reduce the net cash outflow for the payor. d. Distinguish child support payments from alimony. e. None of the above.

483. The alimony rules:

a. Are based on the principle that the person who earns the income should pay the tax. b. Permit tax deductions for property divisions. c. Look to state law to determine the definition of alimony. *d. Distinguish child support payments from alimony. e. None of the above.

484. Under the terms of a divorce agreement, Kim was to pay her husband Tom $7,000 per month in alimony. Kim’s payments will be reduced to $3,000 per month when their 9 year-old son becomes 21. The husband has custody of their son. For a twelve-month period, Kim can deduct from gross income (and Tom must include in gross income):

a. $60,000. b. $48,000. *c. $36,000. d. $0. e. None of the above.

485. Under the terms of a divorce agreement, Lanny was to pay his wife Joyce $2,000 per month in alimony and $500 per month in child support. For a twelve-month period, Lanny can deduct from gross income (and Joyce must include in gross income):

a. $0. b. $6,000. *c. $24,000. d. $30,000. e. None of the above.

486. Under the terms of a divorce agreement, Ron is to pay his former wife Jill $10,000 per month. The payments are to be reduced to $7,000 per month when their 15 year-old child reaches age 18. During the current year, Ron paid $120,000 under the agreement. Assuming all of the other conditions for alimony are satisfied, Ron can deduct from gross income (and Jill must include in gross income) as alimony:

a. $120,000. *b. $84,000. c. $36,000. d. $0. e. None of the above is correct.

487. The purpose of the tax rules that apply to below-market loans between family members is to:

a. Discourage loans between related parties. *b. Prevent shifting of income among family members. c. Prevent gifts from being disguised as bad debt expenses. d. Prevent gift tax avoidance. e. None of the above is true.

488. On January 1, Father (Dave) loaned Daughter (Debra) $100,000 to purchase a new car and to pay off college loans. There were no other loans outstanding between Dave and Debra. The relevant Federal rate on interest was 6 percent. The loan was outstanding for the entire year.

*a. If Debra has $15,000 of investment income, Dave must recognize $6,090 of imputed interest income. b. Dave must recognize $6,090 of imputed interest income regardless of the amount of Debra’s investment income. c. Debra must recognize $6,090 of imputed interest income. d. Debra must recognize $6,090 of imputed interest income if Dave has at least $6,090 of investment income. e. None of the above.

489. The effects of a below-market loan for $100,000 made by a corporation to its chief executive officer as an enticement to get him to remain with the company are:

a. The corporation has imputed interest income and the employee is deemed to have received a gift. b. The corporation has imputed interest income and dividends paid. c. The employee has no income unless the funds are invested and produce investment income for the year. *d. The employee has imputed compensation income and the corporation has imputed interest income. e. None of the above.

490. Sarah, a majority shareholder in Teal, Inc., made a $200,000 interest-free loan to the corporation. Sarah is not an employee of the corporation.

a. Sarah must recognize imputed interest expense and the corporation must recognize imputed interest income. *b. Sarah must recognize imputed interest income and the corporation must recognize imputed interest expense. c. Sarah must recognize imputed dividend income and the corporation may recognize imputed interest expense. d. Neither Sarah’s nor the corporation’s gross income is affected by the loans because no interest was charged. e. None of the above.

491. Sharon made a $60,000 interest-free loan to her son, Todd, who used the money to start a new business. Todd’s only sources of income were $25,000 from the business and $490 of interest on his checking account. The relevant Federal interest rate was 5%. Based on the above information:

a. Todd’s business net profit will be reduced by $3,000 (.05 × $60,000) of interest expense. b. Sharon must recognize $3,000 (.05 × $60,000) of imputed interest income on the below- market loan. c. Todd’s gross income must be increased by the $3,000 (.05 × $60,000) imputed interest income on the below market loan. *d. Sharon does not recognize any imputed interest income and Todd does not recognize any imputed interest expense. e. None of the above is correct.

492. Jay, a single taxpayer, retired from his job as a public school teacher in 2013. He is to receive a retirement annuity of $1,200 each month and his life expectancy is 180 months. He contributed $36,000 to the pension plan during his 35-year career; so his adjusted basis is $36,000. Jay collected 192 payments before he died. What is the correct method for reporting the pension income?

a. Since Jay is no longer working, none of the pension payments must be included in his gross income. b. The first $36,000 received is a nontaxable recovery of capital, and all subsequent annuity payments are taxable. c. The first $180,000 he receives is taxable and the last $36,000 is a nontaxable recovery of capital. *d. All of the last 12 payments he received ($14,400) are taxable. e. None of the above.

493. In 2013, Todd purchased an annuity for $150,000. The annuity is to pay him $2,500 per month for the rest of his life. His life expectancy is 100 months. Which of the following is correct?

a. Todd is not required to recognize any income until he has collected 60 payments (60 × $2,500 = $150,000). b. If Todd collects 30 payments and then dies in 2015, Todd’s estate should amend his tax returns for 2013 and 2014 and eliminate all of the reported income from the annuity for those years. *c. For each $2,500 payment received in the first year, Todd must include $1,000 in gross income. d. For each $2,500 payment received in the first year, Todd must include $1,500 in gross income. e. None of the above.

494. Seth, a calendar year taxpayer, purchased an annuity for $50,000 in 2011. The annuity was to pay him $3,000 on the first day of each year, beginning in 2011, for the remainder of his life. Seth’s life expectancy at the time he purchased the annuity was 20 years. In 2013, Seth developed a deadly disease, and doctors estimated that he would live for no more than 24 months.

*a. If Seth dies in 2014, a loss can be claimed on his final return for his unrecovered cost of the annuity. b. If Seth dies in 2014, his returns for the two previous years can be amended to allocate the entire cost of the annuity to the years in which he received payments and reported gross income. c. If Seth is still alive at the end of 2013, he is not required to recognize any gross income because of his terminal illness. d. If Seth is still alive in 2033, his recovery of capital for that year is $500. e. None of the above.

495. Betty purchased an annuity for $24,000 in 2013. Under the contract, Betty will receive $300 each month for the rest of her life. According to the actuarial estimates, Betty will live to receive 96 payments and will receive a 3% return on her original investment.

a. If Betty collects $3,000 in 2013, her gross income is $630 (.03 × $21,000). b. Betty has no gross income until she has collected $24,000. *c. If Betty lives to collect more than 96 payments, all of the amounts collected after the 96th payment must be included in taxable income. d. If Betty lives to collect only 60 payments before her death, she will report a $6,000 loss from the annuity [$24,000 – (60 × $300) = $6,000] on her final return. e. None of the above.

496. Gordon, an employee, is provided group term life insurance coverage equal to twice his annual salary of $125,000 per year. According to the IRS Uniform Premium Table (based on Gordon’s age), the amount is $12 per year for $1,000 of protection. The cost of an individual policy would be $15 per year for $1,000 of protection. Since Gordon paid nothing towards the cost of the $250,000 protection, Gordon must include in his 2012 gross income which of the following amounts?

a. $1,350. *b. $2,400. c. $3,000. d. $3,750. e. None of the above.

497. Green, Inc., provides group term life insurance for all of its employees. The coverage equals twice the employee’s annual salary. Sam, a vice-president, worked all year for Green, Inc., and received $200,000 of coverage for the year at a cost to Green of $1,500. The Uniform Premiums (based on Sam’s age) are $.25 per month for $1,000 of protection. How much must Sam include in gross income this year?

a. $0. b. $375. *c. $450. d. $600. e. None of the above.

498. Turner, Inc., provides group term life insurance to the officers of the corporation only. Janet, a vice-president, received $400,000 of coverage for the year at a cost to Turner, Inc. of $5,600. The Uniform Premiums (based on Janet’s age) are $15 a year for $1,000 protection. How much of this must Janet include in gross income this year?

a. $0. b. $2,700. c. $5,600. *d. $6,000. e. None of the above.

499. The amount of Social Security benefits received by an individual that he or she must include in gross income:

a. Is computed in the same manner as an annuity [exclusion = (cost/expected return) × amount received]. b. May not exceed the portion contributed by the employer. c. May not exceed 50% of the Social Security benefits received. *d. May be zero or as much as 85% of the Social Security benefits received, depending upon the taxpayer’s Social Security benefits and other income. e. None of the above.

500. The taxable portion of Social Security benefits may be affected by:

a. The taxpayer’s itemized deductions. *b. The individual’s tax-exempt interest income. c. The number of quarters the individual worked. d. The individual’s standard deduction. e. None of the above.

501. Debbie is age 67 and unmarried and her only sources of income are $200,000 in taxable interest and $20,000 of Social Security benefits. Debbie’s adjusted gross income for the year is:

a. $220,000. *b. $217,000. c. $203,000. d. $200,000. e. None of the above.

502. Our tax laws encourage taxpayers to ____ assets that have appreciated in value and ____ assets that have declined in value.

a. sell, keep. b. sell, sell. *c. keep, sell. d. keep, keep. e. None of the above.

503. Margaret owns land that appreciates at the rate of 10% each year. Ralph owns a zero coupon (i.e., all of the interest is paid at maturity but is taxed annually) corporate bond with a yield to maturity of 10%. At the end of 10 years, the bond will mature and the land will be sold. At the end of the 10 years,

a. Margaret and Ralph will have accumulated the same after-tax amounts. b. Ralph will have accumulated a greater after-tax amount because the interest on the bond is tax-exempt. c. Margaret will have accumulated the greater after-tax amount because the gain on the land is tax-exempt. d. Margaret will have accumulated the greater after-tax amount but only if her marginal tax rate never exceeds 27%. *e. Margaret will accumulate the greater after-tax amount because she earns a return on the deferred taxes.

504. Ted was shopping for a new automobile. He found one that met his needs and agreed to purchase it for $23,000. He had shopped around and concluded that he could not get a better price from another dealer. After he had paid for the automobile, the dealer called to notify Ted that he was entitled to a manufacturer’s rebate of $1,500. The next week he received a $1,500 check from the manufacturer. How much should Ted include in gross income?

Correct Answer: Perhaps in Ted’s mind he is $1,500 richer as a result of the rebate, since he was willing to pay $23,000 for the automobile without any knowledge of the fact that he was entitled to the rebate. However, from the point of view of measuring gross income, one could reason that he purchased an automobile for a net cost of $21,500 ($23,000 – $1,500). The fact that the net cost is less than the amount Ted was willing to pay should not affect the determination of gross income.

505. Determine the proper tax year for gross income inclusion in each of the following cases. a. An automobile dealer has several new cars in inventory, but often does not have the right combination of body style, color, and accessories. In some cases the dealer makes an offer to sell a car at a certain price, accepts a deposit, and then orders the car from the manufacturer. When the car is received from the manufacturer, the sale is closed, and the dealer receives the balance of the sales price. At the end of the current year, the dealer has deposits totaling $8,200 for cars that have not been received from the manufacturer. When is the $8,200 subject to tax? b. Purple Corporation, an exterminating company, is a calendar year taxpayer. It contracts to provide service to homeowners once a month under a one-, two-, or three- year contract. On April 1 of the current year, the company sold a customer a one- year contract for $120. How much of the $120 is taxable in the current year if the company is an accrual basis taxpayer. If the $120 is payment on a two-year contract, how much is taxed in the year the contract is sold and in the following year? If the $120 is payment on a three-year contract, how much is taxed in the year the contract is sold and in the following year? c. Pink, Inc., an accrual basis taxpayer, owns an amusement park whose fiscal year ends September 30. To increase business during the fall and winter months, Pink sold passes that would allow the holder to ride “free” during the months of October through March. During the month of September, $6,000 was collected from the sale of passes for the upcoming fall and winter. When will the $6,000 be taxable to Pink? d. The taxpayer is in the office equipment rental business and uses the accrual basis of accounting. In December he collected $5,000 in rents for the following January. When is the $5,000 taxable?

Correct Answer: a. Reg. § 1.451-5 specifies that accrual basis taxpayers may defer the recognition of income from advance payments for the future sale of inventories that are not on hand the last day of the year and the amount collected is less than the seller’s cost of the goods. The $8,200 would not be includible in the gross income of the dealer for the current year and would be includible in gross income at the time the sale is consummated upon delivery of the car. b. Revenue Procedure 2004-34 permits the accrual basis taxpayer to amortize the prepaid income for the first year under the contract. However, the balance of the unearned income must be recognized in the tax year following the year of receipt. In the case of a contract sold on April 1 that was for services over the twelve-month period beginning on that date, the taxpayer would recognize 9/12 of the income in the year of sale, and the remaining balance (3/12) in the following year. In the case of a contract sold on April 1 that was for services over the 24-month period beginning on that date, the taxpayer would recognize 9/24 of the income in the year of sale and the remaining balance (15/24) in the following year. In the case of a contract sold on April 1 that was for services over the 36- month period beginning on that date, the taxpayer would recognize 9/36 of the income in the year of sale and the remaining balance (27/36) in the following year. c. Revenue Procedure 2004-34 would permit deferral of $6,000 from income until the following tax year since all services will be performed by the end of the tax year following the year of receipt. d. Prepaid rent is taxable in the year of receipt to both the accrual and cash basis taxpayers. Revenue Procedure 2004-34 is not applicable to prepaid rent income.

506. On January 1, 2013, Faye gave Todd, her son, a 36-month certificate of deposit she purchased December 31, 2011, for $8,638. Faye gave Todd 1,000 shares of ABC, Inc., on December 2, 2013. The certificate had a maturity value of $10,000 and the yield to maturity was 5%. On November 30, 2013, ABC, Inc., had declared a dividend of $1.00 payable to stockholders of record on December 5th. How much interest and dividends should Todd include in his gross income for 2013?

Correct Answer: Todd must report $454 of interest income and no dividends. The certificate of deposit is an original issue discount instrument. Therefore, Faye should have reported $432 (.05 × $8,638) of interest income in 2012, and thus the adjusted basis of the CD is $9,070 ($8,638 + $432). Todd must report interest income for 2013 calculated as follows: $454 (.05 × $9,070). The dividends had been declared before Todd received the stock. According to the IRS in this case, the income belongs to Faye since she was the owner of the stock when the dividend was declared and she assigned to Todd the right to receive it.

507. José, a cash method taxpayer, is a partner in J&T Accounting Services, a calendar year partnership. Under the partnership agreement, José is to receive 20% of the partnership’s profits or losses. Each partner is allowed to withdraw $10,000 each month for his or her living expenses. José withdrew $120,000 during the year as his monthly draw in 2013. However, in December the partnership was short on cash and José was required to invest an additional $10,000 in the partnership. In March 2013, José received $40,000 as his share of distributed 2012 profits. The partnership earnings before partners’ withdrawals for 2013 totaled $1 million. Compute José’s gross income from the partnership for 2013.

Correct Answer: José’s gross income from the partnership is his share of the partnership profits of $200,000 (.20 × $1 million). The amount of the distributions he receives (a recovery of capital normally) generally does not affect the amount he includes in his gross income. A withdrawal of profits is analogous to withdrawing cash from a bank account created with after-tax earnings. His investment is a $10,000 contribution to capital.

508. Ted and Alice were in the process of negotiating a divorce agreement. They own bonds with a basis of $800,000 and a fair market value of $800,000. They also own common stock with a basis of $600,000 and a fair market value of $800,000. Alice is trying to decide whether to bargain to receive the bonds or the stock. She has no plans for selling the bonds or stock, whichever she receives. a. Which would you advise Alice to receive? b. From Ted’s perspective, are the assets of equal value?

Correct Answer: a. The significant difference between the assets from a tax perspective is that the person who receives the stock will have a $600,000 basis when the value is $800,000. Therefore, a taxable gain will be recognized if the assets are sold for more than $600,000. The bonds do not have this taxable gain possibility. Alice cannot be absolutely certain that the property will never be sold; therefore, she should accept the bonds. b. The bonds and stock are not of equal value because the stock has the lower tax basis thus creating a potential tax liability upon their sale.

509. Margaret made a $90,000 interest-free loan to her son, Adam, who used the money to retire a mortgage on his personal residence and to buy a certificate of deposit. Adam’s only income for the year is his salary of $35,000 and $1,400 interest income on the certificate of deposit. The relevant Federal interest rate is 8% compounded semiannually. The loan is outstanding for the entire year. a. Based on the above information, what is the effect of the loan on Margaret’s gross income for the year? b. The facts are the same as above, except you discovered that Margaret had made an additional loan of $15,000 to Adam in the previous year. Adam used the funds to pay his child’s private school tuition. What are the effects of the loans on Margaret’s gross income?

Correct Answer: a. Margaret’s interest income from the loan is $1,400, which is equal to Adam’s net investment income for the year. Thus, Margaret’s imputed interest income for the year is $1,400, which is the lesser of the imputed interest at the Federal rate of $7,344 [($90,000 × 8% × 1/2) + ($93,600 × 8% × 1/2)] or Adam’s net investment income of $1,400. b. Margaret’s loans to Adam exceed $100,000 ($90,000 + $15,000). Therefore, Margaret must recognize interest income equal to the Federal rate times the outstanding loans.

(.08 × $105,000 × 1/2) = $4,200; (.08 × $109,200 × 1/2) = $4,368

Total = $8,568

510. Arnold was employed during the first six months of the year and earned a $90,000 salary. During the next 6 months, he collected $7,200 of unemployment compensation, borrowed $6,000 (using his personal residence as collateral), and withdrew $1,000 from his savings account (including $60 interest). When he left his former employer, he withdrew his retirement benefits (a qualified annuity) in a lump-sum of $50,000. He made no contributions to the plan. Arnold’s parents loaned him $10,000 (interest-free) on July 1 of the current year, when the Federal rate was 3%. Arnold did not repay the loan during the year and used the money for living expenses. Calculate Arnold’s adjusted gross income for the year.

Correct Answer: Salary $ 90,000 Unemployment compensation 7,200 Interest income 60 Retirement benefit 50,0 00 Adjusted gross income $147,260

The interest-free loan does not result in gross income to Arnold because of the $10,000 exception.

511. How does the taxation of Social Security benefits differ from the taxation of an annuity purchased by the taxpayer?

Correct Answer: In case of Social Security benefits, the taxpayer pays into the fund out of his or her before tax earnings, and the employer matches the employee’s contribution. The amount of Social Security benefits that are included in gross income (50% or 85%) is dependent upon the person,s other income combined with the Social Security benefit. The employee’s contribution to the Social Security fund does not directly enter into the calculation of the taxable portion.

In the case of an annuity purchased by the individual, the taxable portion is computed by subtracting from the amount received the actual amount paid by the individual. The taxable portion is spread over the individual’s life expectancy, and a loss is recognized if the employee dies before the end of his or her life expectancy.

512. Sarah, a widow, is retired and receives $20,000 interest income and dividends and $10,000 in Social Security benefits. Sarah is considering selling a stock at a $8,000 gain. What will be the increase in Sarah’s gross income as a result of the sale of the stock?

Correct Answer: None of Sarah’s Social Security benefits will be taxable if she does not sell the stock {50%[$20,000 + 50%($10,000) – $25,000] = $0}.

The $8,000 gain will cause Sarah’s taxable Social Security benefits to increase by $4,000, which is the lesser of the following:

• 50%($10,000) $5,000 • 50%[$28,000 + 50%($10,000) $4,000 – $25,000]

Therefore, the $8,000 gain will cause Sarah’s gross income to increase by $12,000 ($8,000 + $4,000).

513. Roy is considering purchasing land for $10,000. He expects the land to appreciate in value 8% each year (compounded) and he will sell it at the end of 10 years. He also is considering purchasing a bond for $10,000. The bond does not pay any annual interest, but will pay $21,589 at maturity in 10 years. The before-tax rate of return on the bond is 8%. Roy is in the 40% (combined Federal and State) marginal tax bracket. Roy has other investments that earn a 8% before-tax rate of return. Given that the compound interest factor at 8% is 2.1589, and at 4.8% the factor is 1.5981, which alternative should Roy choose?

Correct Answer: Roy should select the investment in the land. The investment in the bond earns a 4.8% after-tax rate of return. The tax on the original issue discount must be paid each year; therefore, owning the bond is equivalent to owning an investment that appreciates at an after-tax rate of 4.8%. At the end of 10 years, Roy will have accumulated $15,981 (1.5981 × $10,000). With the land, Roy’s investment will appreciate to $21,589 (2.1589 × $10,000) which exceeds the $15,981 amount accumulated with the bond.

514. In January 2013, Tammy purchased a bond due in 24 months. The cost of the bond is $857 and its maturity value is $1,000. No interest is paid each year, but the compound interest rate on the bond is 8%. Tammy also purchased a Series EE United States Government bond for $558, with a maturity value in 10 years of $1,000. This is the only Series EE bond she has ever owned. The Series EE bond is sold to yield 6% interest. Tammy is 13 years old and has no other source of income. She is claimed as a dependent by her parents. Compute Tammy’s gross income from the bond and Series EE bond for 2013.

Correct Answer: Tammy’s only recognized income is from the original issue discount of $69 ($857 × 8%) on the bond. The Series EE bonds are exempt from the original issue discount rules. However, Tammy could elect to include the original issue discount on the Series EE bond each year, and it appears that the election should be made. Because Tammy has no other sources of income, the effective tax rate on the accrued Series EE bond interest is zero because of the available standard deduction of $900. The interest reported will increase Tammy’s basis in the Series EE bond and, therefore, she will not have to recognize any income at maturity. The interest on the Series EE bond for 2013, if the election is made, is $33 ($558 × 6%).

515. In some foreign countries, the tax law specifically designates the types of income items that are includible in gross income. How does this approach compare with the U.S. Internal Revenue Code (§ 61)? What is a major advantage to the approach used in the U.S. tax law?

Correct Answer: The Internal Revenue Code defines gross income as all income unless specifically excluded. The advantage of the U.S. system is that an all- inclusive list of types of income does not have to be developed.

516. Melissa is a compulsive coupon clipper. She often brags about the time she purchased a cart full of groceries for $5.00, when the cost without coupons would have been $50. Discuss whether Melissa realizes gross income from her coupon clipping.

Correct Answer: Under the all-inclusive concept of gross income, one could reason that Melissa realizes income from her coupon collecting activities. She clearly increases her wealth, as she acquires food at 1/10th of the amount paid by those who do not spend their time and effort saving coupons. In some cases, the coupons may have been obtained when other goods were purchased. One could reason that the price paid included a cost of the coupon; thus, the price paid would be allocated between the goods received and the cost of the coupon. Then, when the coupon was utilized, the difference between Melissa’s cost and the redemption value would be income.

However, it seems unlikely that the IRS would attempt to tax the earnings from coupon clipping because of the time and effort required to collect small amounts of taxes.

517. Katherine is 60 years old and is bargaining with her employer over deferred compensation. In exchange for reducing her current year’s salary by $50,000, she can receive a lump-sum amount in 5 years, when she will retire. If she receives the $50,000 in the current year, she will invest in certificates of deposit that yield 5%. Katherine is in the 28% marginal tax bracket in all relevant years. What is the minimum amount Katherine should accept as a deferred pay option? [Hint: the compound interest factor is 1.1934.]

Correct Answer: $59,669

The $50,000 salary will be $36,000 [(1 – .28)($50,000)] after-tax. When this is invested in a CD that yields 3.6% [(1 – .28)(.05)] after-tax for five years, the compounded amount will be $42,962 ($36,000 × 1.1934). If a lump-sum is received in five years, it will be subject to tax. Therefore, Katherine should receive at least $59,669 [$42,962/(1 – .28)].

518. Dick and Jane are divorced in 2012. At the time of the divorce, Dick had a lawsuit pending. He had filed suit against a former employer for overtime pay. As part of a divorce agreement, Dick agreed to pay Jane one-half of the proceeds from the lawsuit. In 2013, Dick collected $250,000 from the former employer and paid Jane $125,000. What are the tax consequences for Dick receiving the $250,000 and then paying Jane the $125,000?

Correct Answer: The $250,000 payment is additional gross income to Dick. In order for Dick to avoid tax on the $125,000 he transferred to Jane, the payment must qualify as alimony. This means the payment must be made only if she is alive at the time Dick receives the award. Moreover, even if the payment qualifies as alimony, the large payment received in 2013 will probably result in some alimony recapture.

519. Rachel owns rental properties. When Rachel rents to a new tenant, she usually requires the tenant to pay an amount in addition to the first month’s rent. The additional amount serves as security for damages to the property and the tenant’s failure to pay future rents. How should the payments be characterized (e.g., on lease documents) to minimize Rachel’s current tax liability?

Correct Answer: The payments should be characterized as damage deposits. This will ensure that the IRS will not tax the payments as prepaid income. A payment to secure future rents would probably be treated as prepaid rent income.

520. Rachel, who is in the 35% marginal tax bracket, is considering purchasing an annuity that will pay her $10,000 per year for the remainder of her life. Her life expectancy is 15 years. The cost of the annuity is $97,120, and the cost is calculated to yield her an expected 6% return on her investment. As an alternative, Rachel could place the $97,120 in a savings account yielding 6% and she could withdraw $10,000 each year for 15 years (reducing the value of the account to zero at the end of 15 years). How might the tax laws applicable to annuities affect Rachel’s decision?

Correct Answer: The tax laws favor the purchase of the annuity. This results because the annuity rules allow the taxpayer to recover his or her investment more quickly than under the savings account plan. Each alternative yields the same taxable income over the 15 years. However, Rachel’s taxes are deferred with the annuity contract. Under the annuity rules, Rachel would treat as a recovery of capital $6,475 ($97,120/15) each year. With the savings account, Rachel has less recovery of capital in the early years than in the later years. For example, in the first year Rachel earns interest on the savings account of $5,827 ($97,120 × .06) and has a recovery of capital of $4,173 ($10,000 – $5,827). Thus, the present value of the taxes are greater with the savings account.

521. In the case of a zero interest below-market loan by a corporation to a shareholder-employee, what difference does it make to the corporation and the shareholder whether the loan is characterized as a corporation’s loan to its shareholder or a corporation’s loan to its employee?

Correct Answer: Imputed interest on the loan to an employee would create compensation expense equal to the amount of imputed interest that is not charged the employee. The compensation expense would be deductible by the corporation. On the other hand, the imputed interest on the shareholder loan creates a non-deductible dividend paid by the corporation. From the shareholder-employee’s perspective, dividend treatment might be preferable because the dividends are not subject to 1.45% Medicare tax and are eligible for the beneficial tax rate for qualified dividends.

522. Under the formula for taxing Social Security benefits, low income taxpayers are not required to include any of the Social Security benefits in gross income. But as income increases, 50% of the Social Security benefits may be included in gross income. Further increases in income will cause as much as 85% of the Social Security benefits being subject to tax. Does this mean that the taxation of Social Security benefits is more or less progressive than the taxation of other types of income?

Correct Answer: The formula for the taxation of Social Security benefits is more progressive than the taxation of other sources of income. Under a progressive system, as income increases the tax as a percent of income increases. This is accomplished by increasing the marginal tax rate as income increases. With the Social Security taxing formula, as income increases, and the taxpayer is subjected to higher marginal rates, the amount of taxable income increases as more of the Social Security benefits are subject to tax.

523. For a person who is in the 35% marginal tax bracket, $1,000 of tax-exempt income is equivalent to $1,350 of income that is subject to tax.

a. True *b. False

524. John told his nephew, Steve, “if you maintain my house when I cannot, I will leave the house to you when I die. Steve maintained the house and when John died Steve inherited the house. The value of the residence can be excluded from Steve’s gross income as an inheritance.

a. True *b. False

525. Brooke works part-time as a waitress in a restaurant. For groups of 7 or more customers, the customer is charged 15% of the bill for Brooke’s services. For parties of less than 7, the tips are voluntary. Brooke received $11,000 from the groups of 7 or more and $7,000 in voluntary tips from all other customers. Using the customary 15% rate, her voluntary tips would have been only $6,000. Brooke must include $18,000 ($11,000 + $7,000) in gross income.

*a. True b. False

526. Mel was the beneficiary of a $45,000 group term life insurance policy on his wife. His wife’s employer paid all of the premiums on the policy. Mel used the life insurance proceeds to purchase a United States Government bond, which paid him $2,500 interest during the current year. Mel’s Federal gross income from the above is $2,500.

*a. True b. False

527. Zack was the beneficiary of a life insurance policy on his wife. Zack had paid $20,000 in premiums on the policy. He collected $50,000 on the policy when his wife died from a terminal illness. Because it took several months to process the claim, the insurance company paid Zack $53,000, the face amount of the policy plus $3,000 interest. Zack must include $23,000 in his gross income.

a. True *b. False

528. Ed died while employed by Violet Company. His wife collected $40,000 on a group term life insurance policy that Violet provided its employees, and $6,000 of accrued salary Ed had earned prior to his death. All of the premiums on the group term life insurance policy were excluded from the Ed’s gross income. Ed’s wife is required to recognize as gross income only the $6,000 she received for the accrued salary.

*a. True b. False

529. Gary cashed in an insurance policy on his life. He needed the funds to pay for his terminally ill wife’s medical expenses. He had paid $12,000 in premiums and he collected $30,000 from the insurance company. Gary is not required to include the gain of $18,000 ($30,000 – $12,000) in gross income.

a. True *b. False

530. When Betty was diagnosed as having a terminal illness, she sold her life insurance policy to Insurance Purchase, Inc., a company that is licensed to invest in these types of contracts. Betty sold the policy for $32,000 and Insurance Purchase, Inc., became the beneficiary. She had paid total premiums of $19,000. Betty died 8 months after the sale. Insurance Purchase, Inc., collected $50,000 on the policy. The company had paid additional premiums of $4,000 on the policy. Betty is not required to recognize a $13,000 gain from the sale of her life insurance policy and Insurance Purchase, Inc., is required to recognize a $14,000 gain from the insurance policy.

*a. True b. False

531. Agnes receives a $5,000 scholarship which covers her tuition at Parochial High School. She may not exclude the $5,000 because the exclusion applies only to scholarships to attend college.

a. True *b. False

532. If a scholarship does not satisfy the requirements for a gift, the scholarship must be included in gross income.

a. True *b. False

533. Ashley received a scholarship to be used as follows: tuition $6,000; room and board $9,000; and books and laboratory supplies $2,000. Ashley is required to include only $9,000 in her gross income.

*a. True b. False

534. In December 2013, Emily, a cash basis taxpayer, received a $2,500 cash scholarship for the Spring semester of 2014. However, she did not use the funds to pay the tuition until January 2014. Emily can exclude the $2,500 from her gross income in 2013.

*a. True b. False

535. Betty received a graduate teaching assistantship that was awarded on the basis of academic achievement. The payments must be included in her gross income.

*a. True b. False

536. In 2013, Theresa was in an automobile accident and suffered physical injuries. The accident was caused by Ramon’s negligence. In 2014, Theresa collected from his insurance company. She received $15,000 for loss of income, $10,000 for pain and suffering, $50,000 for punitive damages, and $6,000 for medical expenses which she had deducted on her 2013 tax return (the amount in excess of 10% of adjusted gross income). As a result of the above, Theresa’s 2014 gross income is increased by $56,000.

*a. True b. False

537. Workers’ compensation benefits are included in gross income if the employer also pays the employee while the employee is recovering from his or her injury.

a. True *b. False

538. Sam was unemployed for the first two months of 2013. During that time, he received $4,000 of state unemployment benefits. He worked for the next six months and earned $14,000. In September, he was injured on the job and collected $5,000 of workers’ compensation benefits. Sam’s Federal gross income from the above is $18,000 ($4,000 + $14,000).

*a. True b. False

539. Sarah’s employer pays the hospitalization insurance premiums for a policy that covers all employees and retired former employees. After Sarah retires, the hospital insurance premiums paid for her by her employer can be excluded from her gross income.

*a. True b. False

540. Meg’s employer carries insurance on its employees that will pay an employee his or her regular salary while the employee is away from work due to illness. The premiums for Meg’s coverage were $1,800. Meg was absent from work for two months as a result of a kidney infection. Meg’s employer’s insurance company paid Meg’s regular salary of $8,000 while she was away from work. Meg also collected $2,000 on a wage continuation policy she had purchased. Meg must include $11,800 in her gross income.

a. True *b. False

541. Melody works for a company with only 22 employees. Her employer contributed $2,000 to her health savings account (HSA), and the account earned $100 in interest during the year. Melody withdrew only $1,200 to pay medical expenses during the year. Melody is not required to recognize any gross income from the HSA for the year.

*a. True b. False

542. If an employer pays for the employee’s long-term care insurance premiums, the employee can exclude from gross income the premiums but all of the benefits collected must be included in gross income.

a. True *b. False

543. Employees of a CPA firm located in Virginia may exclude from gross income the meals and lodging provided by the employer while they were on an audit in Texas.

a. True *b. False

544. Carla is a deputy sheriff. Her employer requires that she live in the county where she is employed. Housing is very expensive; so the county agreed to pay her $4,800 per year to cover the higher cost of housing. Carla must include the housing supplement in her gross income.

*a. True b. False

545. Roger is in the 35% marginal tax bracket. Roger’s employer has created a flexible spending account for medical and dental expenses that are not covered by the company’s health insurance plan. Roger had his salary reduced by $1,200 during the year for contributions to the flexible spending plan. However, Roger incurred only $1,100 in actual expenses for which he was reimbursed. Under the plan, he must forfeit the $100 unused amount. His after-tax cost of overfunding the plan is $65.

*a. True b. False

546. Mauve Company permits employees to occasionally use the copying machine for personal purposes. The copying machine is located in the office where the higher paid executives work, so they occasionally use the machine. However, the machine is not convenient for use by the lower paid warehouse employees and, thus, they never use the copier. The use of the copy machine may not be excluded from gross income because the benefit is discriminatory.

a. True *b. False

547. Fresh Bakery often has unsold donuts at the end of the day. The bakery allows employees to take the leftovers home. The employees are not required to recognize gross income because the bakery does not incur any additional cost.

a. True *b. False

548. Nicole’s employer pays her $150 per month towards the cost of parking near a railway station where Nicole catches the train to work. The employer also pays the cost of the rail pass, $75 per month. Nicole can exclude both of these payments from her gross income.

*a. True b. False

549. A U.S. citizen who works in France from February 1, 2013 until January 31, 2014 is eligible for the foreign earned income exclusion in 2013 and 2014.

*a. True b. False

550. Generally, a U.S. citizen is required to include in gross income the salary and wages earned while working in a foreign country even if the foreign country taxes the income.

a. True *b. False

551. Calvin miscalculated his income in 2011 and overpaid his state income tax by $10,000. In 2013, he amended his 2011 state income tax return and received a $10,000 refund and $900 interest. Calvin itemized his deductions in 2011, deducting $12,000 in state income tax and $30,000 total itemized deductions. As a result of the amended return in 2013, Calvin must recognize $10,900 of gross income.

*a. True b. False

552. A cash basis taxpayer took an itemized deduction of $5,500 for state income tax paid in 2013. His total itemized deductions in 2013 were $18,000. In 2014, he received a $900 refund of his 2013 state income tax. The taxpayer must include the $900 refund in his 2014 Federal gross income in accordance with the tax benefit rule.

*a. True b. False

553. The taxpayer incorrectly took a $5,000 deduction (e.g., incorrectly calculated depreciation) in 2013 and as a result his taxable income was reduced by $5,000. The taxpayer discovered his error in 2014. The taxpayer must add $5,000 to his 2014 gross income in accordance with the tax benefit rule to correct for the 2013 error.

a. True *b. False

554. Mother participated in a qualified state tuition program for the benefit of her son. She contributed $15,000. When the son entered college, the balance in the fund satisfied the tuition charge of $20,000. When the funds were withdrawn to pay the college tuition for her son, neither Mother nor son must include $5,000 ($20,000 – $15,000) in gross income.

*a. True b. False

555. The earnings from a qualified state tuition program account are deferred from taxation until they are used for qualified higher education expenses. At that time, the amount taken from the fund must be included in the gross income of the person who contributed to the account.

a. True *b. False

556. Benny loaned $100,000 to his controlled corporation. When it became apparent the corporation would not be able to repay the loan in the near future, Benny canceled the debt. The corporation should treat the cancellation as a nontaxable contribution to capital.

*a. True b. False

557. Zork Corporation was very profitable and had accumulated excess cash. The company decided to repurchase some of its bonds that had been issued for $1,000,000. Because of an increase in market interest rates, Zork was able to retire the bonds for $900,000. The company is not required to recognize $100,000 of income from the discharge of its indebtedness but must reduce the basis in its assets.

a. True *b. False

558. Amber Machinery Company purchased a building from Ted for $250,000 cash and a mortgage of $750,000. One year after the transaction, the mortgage had been reduced to $725,000 by principal payments by Amber, but it was apparent that Amber would not be able to continue to make the monthly payments on the mortgage. Ted reduced the amount owed by Amber to $600,000. This reduced the monthly payments to a level that Amber could pay. Amber must recognize $125,000 income from the reduction in the debt by Ted.

a. True *b. False

559. The taxpayer’s marginal tax bracket is 25%. Which would the taxpayer prefer?

a. $1.00 taxable income rather than $1.25 tax-exempt income. b. $1.00 taxable income rather than $.75 tax-exempt income. c. $1.25 taxable income rather than $1.00 tax-exempt income. *d. $1.40 taxable income rather than $1.00 tax-exempt income. e. None of the above.

560. Cash received by an individual:

a. Is not included in gross income if it was not earned. b. Is not taxable unless the payor is legally obligated to make the payment. c. Must always be included in gross income. *d. May be included in gross income although the payor is not legally obligated to make the payment. e. None of the above.

561. Sharon had some insider information about a corporate takeover. She unintentionally informed a friend, who immediately bought the stock in the target corporation. The takeover occurred and the friend made a substantial profit from buying and selling the stock. The friend told Sharon about his stock dealings, and gave her a pearl necklace because she “made it all possible.” The necklace was worth $10,000, but she already owned more jewelry than she desired.

a. The necklace is a nontaxable gift received by Sharon because the friend was not legally required to make the gift. b. The value of the necklace is not included in Sharon’s gross income unless she sells it. c. The value of the necklace is not included in Sharon’s gross income because passing the information was an illegal act and the SEC can confiscate the necklace. *d. The value of the necklace must be included in Sharon’s gross income for the tax year it was received by her. e. None of the above.

562. Carin, a widow, elected to receive the proceeds of a $150,000 life insurance policy on the life of her deceased husband in 10 installments of $17,500 each. Her husband had paid premiums of $60,000 on the policy. In the first year, Carin collected $17,500 from the insurance company. She must include in gross income:

a. $0. *b. $2,500. c. $10,000. d. $25,000. e. None of the above.

563. Iris collected $150,000 on her deceased husband’s life insurance policy. The policy was purchased by the husband’s employer under a group policy. Iris’s husband had included $5,000 in gross income from the group term life insurance premiums during the years he worked for the employer. She elected to collect the policy in 10 equal annual payments of $18,000 each.

a. None of the payments must be included in Iris’s gross income. b. The amount she receives in the first year is a nontaxable return of capital. c. For each $18,000 payment that Iris receives, she can exclude $500 ($5,000/$180,000 × $18,000) from gross income. *d. For each $18,000 payment that Iris receives, she can exclude $15,000 ($150,000/$180,000 × $18,000) from gross income. e. None of the above.

564. Turquoise Company purchased a life insurance policy on the company’s chief executive officer, Joe. After the company had paid $400,000 in premiums, Joe died and the company collected the $1.5 million face amount of the policy. The company also purchased group term life insurance on all its employees. Joe had included $16,000 in gross income for the group term life insurance premiums. Joe’s widow, Rebecca, received the $100,000 proceeds from the group term life insurance policy.

a. Rebecca can exclude the life insurance proceeds of $100,000, but Turquoise Company must include $1,100,000 ($1,500,000 – $400,000) in gross income. *b. Turquoise Company and Rebecca can exclude the life insurance proceeds of $1,500,000 and $100,000, respectively, from gross income. c. Turquoise Company can exclude $1,100,000 ($1,500,000 – $400,000) from gross income, but Rebecca must include $84,000 in gross income. d. Turquoise Company must include $1,100,000 ($1,500,000 – $400,000) in gross income and Rebecca must include $100,000 in gross income. e. None of the above.

565. Swan Finance Company, an accrual method taxpayer, requires all of its customers to carry credit life insurance. If a customer dies, the company receives from the insurance company the balance due on the customer’s loan. Ali, a customer, died owing Swan $1,500. The balance due included $200 accrued interest that Swan has included in income. When Swan collects $1,500 from the insurance company, Swan:

a. Must recognize $1,500 income from the life insurance proceeds. b. Must recognize $1,300 income from the life insurance proceeds. c. Does not recognize income because life insurance proceeds are tax-exempt. *d. Does not recognize income from the life insurance because the entire amount is a recovery of capital. e. None of the above.

566. Ben was diagnosed with a terminal illness. His physician estimated that Ben would live no more than 18 months. After he received the doctor’s diagnosis, Ben cashed in his life insurance policy and used the proceeds to take a trip to see relatives and friends before he died. Ben had paid $12,000 in premiums on the policy, and he collected $50,000, the cash surrender value of the policy. Henry enjoys excellent health, but he cashed in his life insurance policy to purchase a new home. He had paid premiums of $12,000 and collected $50,000 from the insurance company.

a. Neither Ben nor Henry is required to recognize gross income. b. Both Ben and Henry must recognize $38,000 ($50,000 – $12,000) of gross income. *c. Henry must recognize $38,000 ($50,000 – $12,000) of gross income, but Ben does not recognize any gross income. d. Ben must recognize $38,000 ($50,000 – $12,000) of gross income, but Henry does not recognize any gross income. e. None of the above.

567. Albert had a terminal illness which required almost constant nursing care for the remaining two years of his estimated life, according to his doctor. Albert had a life insurance policy with a face amount of $100,000. Albert had paid $25,000 of premiums on the policy. The insurance company has offered to pay him $80,000 to cancel the policy, although its cash surrender value was only $55,000. Albert accepted the $80,000. Albert used $15,000 to pay his medical expenses. Albert made a miraculous recovery and lived another 20 years. As a result of cashing in the policy:

a. Albert must recognize $55,000 of gross income, but he has $15,000 of deductible medical expenses. b. Albert must recognize $65,000 ($80,000 – $15,000) of gross income. c. Albert must recognize $40,000 ($80,000 – $25,000 – $15,000) of gross income. *d. Albert is not required to recognize any gross income because of his terminal illness. e. None of the above.

568. A scholarship recipient at State University may exclude from gross income the scholarship proceeds used to pay for:

a. Only tuition. *b. Tuition, books, and supplies. c. Tuition, books, supplies, meals, and lodging. d. Meals and lodging. e. None of the above.

569. Ron, age 19, is a full-time graduate student at City University. During 2013, he received the following payments:

Cash award for being the outstanding $ 1,500 resident adviser Resident adviser housing 2,500 State scholarship for ten months 6,000 (tuition and books) State scholarship (meals allowance) 2,400 Loan from college financial aid 3,000 office Cash support from parents 2,00 0 $17,400

Ron served as a resident advisor in a dormitory and, therefore, the university waived the $2,500 charge for the room he occupied. What is Ron’s adjusted gross income for 2013?

a. $1,500. *b. $3,900. c. $9,000. d. $15,400. e. None of the above.

570. Barney is a full-time graduate student at State University. He serves as a teaching assistant for which he is paid $700 per month for 9 months and his $5,000 tuition is waived. The university waives tuition for all of its employees. In addition, he receives a $1,500 research grant to pursue his own research and studies. Barney’s gross income from the above is:

a. $0. *b. $6,300. c. $11,300. d. $12,800. e. None of the above.

571. Jena is a full-time undergraduate student at State University and is claimed by her parents as a dependent. Her only source of income is a $10,000 athletic scholarship ($1,000 for books, $5,500 tuition, $500 student activity fee, and $3,000 room and board). Jena’s gross income for the year is:

a. $10,000. b. $4,000. *c. $3,000. d. $500. e. None of the above.

572. As an executive of Cherry, Inc., Ollie receives a fringe benefit in the form of annual tuition scholarships of $10,000 to each of his three children. The scholarships are paid by the company directly to each child’s educational institution and are payable only if the student maintains a B average.

a. The tuition payments of $30,000 may be excluded from Ollie’s gross income as a scholarship. b. The tuition payments of $10,000 each must be included in the child’s gross income. c. The tuition payments of $30,000 may be excluded from Ollie’s gross income because the payments are for the academic achievements of the children. *d. The tuition payments of $30,000 must be included in Ollie’s gross income. e. None of the above.

573. The taxpayer is a Ph.D. student in accounting at City University. The student is paid $1,500 per month for teaching two classes. The total amount received for the year is $13,500.

a. The $13,500 is excludible if the money is used to pay for tuition and books. *b. The $13,500 is taxable compensation. c. The $13,500 is considered a scholarship and, therefore, is excluded. d. The $13,500 is excluded because the total amount received for the year is less than her standard deduction and personal exemption. e. None of the above.

574. In 2013, Khalid was in an automobile accident and suffered physical injuries. The accident was caused by Rashad’s negligence. Khalid threatened to file a lawsuit against Amber Trucking Company, Rashad’s employer, claiming $50,000 for pain and suffering, $90,000 for loss of income, and $70,000 in punitive damages. Amber’s insurance company will not pay punitive damages; therefore, Amber has offered to settle the case for $100,000 for pain and suffering, $90,000 for loss of income, and nothing for punitive damages. Khalid is in the 35% marginal tax bracket. What is the after-tax difference to Khalid between Khalid’s original claim and Amber’s offer?

a. Amber’s offer is $20,000 less. ($50,000 + $90,000 + $70,000 – $100,000 – $90,000). b. Amber’s offer is $7,000 less. [($50,000 + $90,000 + $70,000 – $100,000 – $90,000) × .35)]. *c. Amber’s offer is $4,500 more. {$190,000 – ($50,000 + $90,000) + [$70,000 × (1 – .35)]}. d. Amber’s offer is $22,000 more. [($190,000 – $210,000) + ($120,000 × .35)]. e. None of the above.

575. Christie sued her former employer for a back injury she suffered on the job in 2013. As a result of the injury, she was partially disabled. In 2014, she received $240,000 for her loss of future income, $160,000 in punitive damages because of the employer’s flagrant disregard for the employee’s safety, and $15,000 for medical expenses. The medical expenses were deducted on her 2013 return, reducing her taxable income by $12,000. Christie’s 2014 gross income from the above is:

a. $415,000. b. $412,000. c. $255,000. d. $175,000. *e. $172,000.

576. Early in the year, Marion was in an automobile accident during the course of his employment. As a result of the physical injuries he sustained, he received the following payments during the year:

Reimbursement of medical expenses $10,000 Marion paid by a medical insurance policy he purchased Damage settlement to replace his lost 15,000 salary

What is the amount that Marion must include in gross income for the current year?

a. $25,000. b. $15,000. c. $12,500. d. $10,000. *e. $0.

577. Theresa sued her former employer for age, race, and gender discrimination. She claimed $200,000 in damages for loss of income, $300,000 for emotional harm, and $500,000 in punitive damages. She settled the claim for $700,000. As a result of the settlement, Theresa must include in gross income:

*a. $700,000. b. $500,000. c. $490,000 [($700,000/$1,000,000) × $700,000]. d. $0. e. None of the above.

578. Jack received a court award in a civil libel and slander suit against National Gossip. He received $120,000 for damages to his professional reputation, $100,000 for damages to his personal reputation, and $50,000 in punitive damages. Jack must include in his gross income as a damage award:

a. $0. b. $100,000. c. $120,000. *d. $270,000. e. None of the above.

579. Olaf was injured in an automobile accident and received $25,000 for his physical injury, $50,000 for his loss of income, and $10,000 punitive damages. As a result of the award, the amount Olaf must include in gross income is:

*a. $10,000. b. $50,000. c. $60,000. d. $85,000. e. None of the above.

580. The exclusion for health insurance premiums paid by the employer applies to:

a. Only current employees and their spouses. b. Only current employees and their spouses and dependents. c. Only current employees and their disabled spouses. *d. Present employees, retired former employees, and their spouses and dependents. e. None of the above.

581. Julie was suffering from a viral infection that caused her to miss work for 90 days. During the first 30 days of her absence, she received her regular salary of $8,000 from her employer. For the next 60 days, she received $12,000 under an accident and health insurance policy purchased by her employer. The premiums on the health insurance policy were excluded from her gross income. During the last 30 days, Julie received $6,000 on an income replacement policy she had purchased. Of the $26,000 she received, Julie must include in gross income:

a. $0. b. $6,000. c. $8,000. d. $14,000. *e. $20,000.

582. Matilda works for a company with 1,000 employees. The company has a hospitalization insurance plan that covers all employees. However, the employee must pay the first $3,000 of his or her medical expenses each year. Each year, the employer contributes $1,500 to each employee’s health savings account (HSA). Matilda’s employer made the contributions in 2012 and 2013, and the account earned $100 interest in 2013. At the end of 2013, Matilda withdrew $3,100 from the account to pay the deductible portion of her medical expenses for the year and other medical expenses not covered by the hospitalization insurance policy. As a result, Matilda must include in her 2013 gross income:

*a. $0. b. $100. c. $1,600. d. $3,100. e. None of the above.

583. All employees of United Company are covered by a group hospitalization insurance plan, but the employees must pay the premiums ($8,000 for each employee). None of the employees has sufficient medical expenses to deduct the premiums. Instead of giving raises next year, United is considering paying the employee’s hospitalization insurance premiums. If the change is made, the employee’s after-tax and insurance pay will:

a. Decrease by the same amount for all employees. b. Increase more for the lower paid employees (10% and 15% marginal tax bracket). *c. Increase more for the higher income (35% marginal tax bracket) employees. d. Increase by the same amount for all employees. e. None of the above.

584. The plant union is negotiating with the Eagle Company, which is on the verge of bankruptcy. Eagle has offered to pay for the employees’ hospitalization insurance in exchange for a wage reduction. The employees each currently pay premiums of $4,000 a year for their insurance.

a. If an employee’s wages are reduced by $5,000 and the employee is in the 28% marginal tax bracket, the employee would benefit from the offer. b. If an employee’s wages are reduced by $4,000 and the employee is in the 15% marginal tax bracket, the employee would benefit from the offer. c. If an employee’s wages are reduced by $6,000 and the employee is in the 35% marginal tax bracket, the employee would benefit from the offer. *d. a., b., and c. e. None of the above.

585. James, a cash basis taxpayer, received the following compensation and fringe benefits in 2013:

Salary $66,000 Disability income protection premiums 3,000 Long-term care insurance premiums 4,000

His actual salary was $72,000. He received only $66,000 because his salary was garnished and the employer paid $6,000 on James’s credit card debt he owed. The wage continuation insurance is available to all employees and pays the employee three-fourths of the regular salary if the employee is sick or disabled. The long-term care insurance is available to all employees and pays $150 per day towards a nursing home or similar facility. What is James’s gross income from the above?

a. $66,000. *b. $72,000. c. $73,000. d. $75,000. e. None of the above.

586. The First Chance Casino has gambling facilities, a bar, a restaurant, and a hotel. All employees are allowed to obtain food from the restaurant at no charge during working hours. In the case of the employees who operate the gambling facilities, bar, and restaurant, 60% of all of Casino’s employees, the meals are provided for the convenience of the Casino. However, the hotel workers, demanded equal treatment and therefore were also allowed to eat in the restaurant at no charge while they are at work. Which of the following is correct?

a. All the employees are required to include the value of the meals in their gross income. b. Only the restaurant employees may exclude the value of their meals from gross income. c. Only the employees who work in gambling, the bar, and the restaurant may exclude the meals from gross income. *d. All of the employees may exclude the value of the meals from gross income. e. None of the above.

587. An employee can exclude from gross income the value of meals provided by his or her employer whenever:

a. The meal is not extravagant. *b. The meals are provided on the employer’s premises for the employer’s convenience. c. There are no places to eat near the work location. d. The meals are provided for the convenience of the employee. e. None of the above.

588. Ridge is the manager of a motel. As a condition of his employment, Ridge is required to live in a room on the premises so that he would be there in case of emergencies. Ridge considered this a fringe benefit, since he would otherwise be required to pay $800 per month rent. The room that Ridge occupied normally rented for $70 per night, or $2,100 per month. On the average, 90% of the motel rooms were occupied. As a result of this rent-free use of a room, Ridge is required to include in gross income.

*a. $0. b. $800 per month. c. $2,100 per month. d. $1,890 ($2,100 × .90). e. None of the above.

589. Adam repairs power lines for the Egret Utilities Company. He is generally working on a power line during the lunch hour. He must eat when and where he can and still get his work done. He usually purchases something at a convenience store and eats in his truck. Egret reimburses Adam for the cost of his meals.

*a. Adam must include the reimbursement in his gross income. b. Adam can exclude the reimbursement from his gross income since the meals are provided for the convenience of the employer. c. Adam can exclude the reimbursement from his gross income because he eats the meals on the employer’s business premises (the truck). d. Adam may exclude from his gross income the difference between what he paid for the meals and what it would have cost him to eat at home. e. None of the above.

590. Tommy, a senior at State College, receives free room and board as full compensation for working as a resident advisor at the university dormitory. The regular housing contract is $2,000 a year in total, $1,200 for lodging and $800 for meals in the dormitory. Tommy had the option of receiving the meals or $800 in cash. Tommy accepted the meals. What must Tommy include in gross income from working as a resident advisor?

a. All items can be excluded from gross income as a scholarship. *b. The meals must be included in gross income. c. The meals may be excluded because he did not receive cash. d. The lodging must be included in gross income because it was compensation for services. e. None of the above.

591. Under the Swan Company’s cafeteria plan, all full-time employees are allowed to select any combination of the benefits below, but the total received by the employee cannot exceed $8,000 a year.

I. Group medical and hospitalization insurance for the employee, $3,600 a year. II. Group medical and hospitalization insurance for the employee’s spouse and children, $1,200 a year. III. Child-care payments, actual cost but not more than $4,800 a year. IV. Cash required to bring the total of benefits and cash to $8,000.

Which of the following statements is true?

a. Sam, a full-time employee, selects choices II and III and $2,000 cash. His gross income must include the $2,000. b. Paul, a full-time employee, elects to receive $8,000 cash because his wife’s employer provided these same insurance benefits for him. Paul is not required to include the $8,000 in gross income. c. Sue, a full-time employee, elects to receive choices I, II and $3,200 for III. Sue is required to include $3,200 in gross income. *d. All of the above. e. None of the above.

592. Heather is a full-time employee of the Drake Company and participates in the company’s flexible spending plan that is available to all employees. Which of the following is correct?

a. Heather reduced her salary by $1,200, actually spent $1,500, and received only $1,200 as reimbursement for her medical expenses. Heather’s gross income will be reduced by $1,500. *b. Heather reduced her salary by $1,200, and received only $900 as reimbursement for her actual medical expenses. She is not refunded the $300 remaining balance, but her gross income is reduced by $1,200. c. Heather reduced her salary by $1,200, and received only $800 as reimbursement for her medical expenses. She is not refunded the $400. Her gross income is reduced by $800. d. Heather reduced her salary by $1,200, and received only $900 as reimbursement for her medical expenses. She forfeits the $300. Her gross income is reduced by $300. e. None of the above.

593. Employees of the Valley Country Club are allowed to use the golf course without charge before and after working hours on Mondays, when the number of players on the course is at its lowest. Tom, an employee of the country club played 40 rounds of golf during the year at no charge when the non-employee charge was $20 per round.

a. Tom must include $800 in gross income. b. Tom is not required to include anything in gross income because it is a de minimis fringe benefit. c. Tom is not required to include the $800 in gross income because the use of the course was a gift. *d. Tom is not required to include anything in gross income because this is a “no-additional-cost service” fringe benefit. e. None of the above.

594. The Royal Motor Company manufactures automobiles. Employees of the company can buy a new automobile for Royal’s cost plus 2%. The automobiles are sold to dealers at cost plus 20%. Generally, employees of Local Dealer, Inc., are allowed to buy a new automobile from the company at the dealer’s cost. Officers of Local Dealer are allowed to use a company vehicle (for personal use) at no cost.

*a. The employees who buy automobiles at a discount are not required to recognize income from the purchase. b. None of the employees who take advantage of the fringe benefits described above are required to recognize income. c. Employees of Royal are required to recognize as gross income 18% (20% – 2%) of the cost of the automobile purchased. d. All employees must recognize gross income from their personal use of the company vehicles. e. None of the above.

595. Peggy is an executive for the Tan Furniture Manufacturing Company. Peggy purchased furniture from the company for $9,500, the price Tan ordinarily would charge a wholesaler for the same items. The retail price of the furniture was $12,500, and Tan’s cost was $9,000. The company also paid for Peggy’s parking space in a garage near the office. The parking fee was $600 for the year. All employees are allowed to buy furniture at a discounted price comparable to that charged to Peggy. However, the company does not pay other employees’ parking fees. Peggy’s gross income from the above is:

*a. $0. b. $600. c. $3,500. d. $4,100. e. None of the above.

596. The employees of Mauve Accounting Services are permitted to use the copy machine for personal purposes, provided the privilege is not abused. Ed is the president of a civic organization and uses the copier to make several copies of the organization’s agenda for its meetings. The copies made during the year would have cost $150 at a local office supply.

a. Ed must include $150 in his gross income. b. Ed may exclude the cost of the copies as a no-additional cost fringe benefit. c. Ed may exclude the cost of the copies only if the organization is a client of Mauve. *d. Ed may exclude the cost of the copies as a de minimis fringe benefit. e. None of the above.

597. The Perfection Tax Service gives employees $12.50 as “supper money” when they are required to work overtime, approximately 25 days each year. The supper money received:

a. Must be included in the employee’s gross income. b. Must be included in the employee’s gross income if the employee does not spend it for supper. c. May be excluded from the employee’s gross income as a “no- additional cost” fringe benefit. *d. May be excluded from the employee’s gross income as a de minimis fringe benefit. e. None of the above.

598. The de minimis fringe benefit:

a. Exclusion applies only to property received by the employee. *b. Can be provided on a discriminatory basis. c. Exclusion is limited to $250 per year. d. Exclusion applies to employee discounts. e. None of the above.

599. Evaluate the following statements:

I. De minimis fringe benefits are those that are so immaterial that accounting for them is impractical. II. De minimis fringe benefits are subject to strict anti-discrimination requirements. III. Generally, a fringe benefit of less than $50 is considered de minimis and can be excluded from gross income.

*a. Only I is true. b. Only III is true. c. Only I and III are true. d. I, II, and III are true. e. None of the above.

600. Kristen’s employer owns its building and provides parking space for its employees. The value of the free parking is $150 per month. Karen’s employer does not have parking facilities, but reimburses its employee for the cost of parking in a nearby garage, up to $150 per month.

a. Kristen and Karen must recognize gross income from the parking services. b. Kristen can exclude the employer provided parking from gross income, but Karen must include her reimbursement in gross income. c. Kristen must include the value of the employer provided parking from her gross income, but Karen can exclude her reimbursement from gross income. *d. Neither Kristen nor Karen is required to include the cost of parking in gross income. e. None of the above.

601. A company has a medical reimbursement plan for officers that covers all costs that the insurer will not pay. However, for all employees who are not officers, the medical reimbursement plan applies only after the employee has paid $1,000 from his or her own funds. An officer incurred $1,500 in medical expenses and was reimbursed for that amount. An hourly worker also incurred $1,500 in medical expense and was reimbursed $500.

a. Both employees must include all benefits received in gross income. b. The officer must include $500 in gross income. *c. The officer must include $1,500 in gross income. d. The hourly employee must include $1,000 in gross income. e. None of the above.

602. A U.S. citizen worked in a foreign country for the period July 1, 2012 through August 1, 2013. Her salary was $10,000 per month. Also, in 2012 she received $5,000 in dividends from foreign corporations (not qualified dividends). No dividends were received in 2013. Which of the following is correct?

a. The taxpayer cannot exclude any of the income because she was not present in the foreign country more than 330 days in either 2012 or 2013. b. The taxpayer can exclude a portion of the salary from U.S. gross income in 2012 and 2013, and all of the dividend income. c. The taxpayer can exclude from U.S. gross income $60,000 salary in 2012, but in 2013 the taxpayer will exceed the twelve month limitation and, therefore, all of the 2013 compensation must be included in gross income. All of the dividends must be included in 2012 gross income. *d. The taxpayer must include the dividend income of $5,000 in 2012 gross income, but the taxpayer can exclude a portion of the compensation income from U.S. gross income in 2012 and 2013. e. None of the above.

603. Louise works in a foreign branch of her employer’s business. She earned $5,000 per month throughout the relevant period.

*a. If Louise worked in the foreign branch from May 1, 2012 until October 31, 2013, she may exclude $40,000 from gross income in 2012 and exclude $50,000 in 2013. b. If Louise worked in the foreign branch from May 1, 2012 until October 31, 2013, she cannot exclude anything from gross income because she was not present in the country for 330 days in either year. c. If Louise began work in the foreign country on May 1, 2012, she must work through November 30, 2013 in order to exclude $55,000 from gross income in 2013 but none in 2012. d. Louise will not be allowed to exclude any foreign earned income because she made less than $97,600. e. None of the above.

604. In the case of interest income from state and Federal bonds:

a. Interest on United States government bonds received by a state resident can be subject to that state’s income tax. *b. Interest on United States government bonds is subject to Federal income tax. c. Interest on bonds issued by State A received by a resident of State B cannot be subject to income tax in State B. d. All of the above are correct. e. None of the above are correct.

605. Heather’s interest and gains on investments for 2013 were as follows:

Interest on Bland County school $600 bonds Interest on U.S. government bonds 700 Interest on a Federal income tax 200 refund Gain on the sale of Bland County 500 school bonds

Heather’s gross income from the above is:

a. $2,000. b. $1,800. *c. $1,400. d. $1,300. e. None of the above.

606. Emily is in the 35% marginal tax bracket. She can purchase a York County school bond yielding 3.5% interest and the interest is not subject to a 5% state tax. But she is interested in earning a higher return for comparable risk.

a. If she buys a corporate bond that pays 6% interest, her after- tax rate of return will be less than if she purchased the York County school bond. *b. If she buys a U.S. government bond paying 5%, her after-tax rate of return will be less than if she purchased the York County school bond. c. If she buys a common stock paying a 4% dividend, her after-tax rate of return will be higher than if she purchased the York County school bond. d. All of the above are correct. e. None of the above are correct.

607. Doug and Pattie received the following interest income in the current year:

Savings account at Greenbacks Bank $4,000 United States Treasury bonds 250 Interest on State of Virginia bonds 200 Interest on Federal tax refund 150 Interest on state income tax refund 75

Greenbacks Bank also gave Doug and Pattie a cellular phone (worth $100) for opening the savings account. What amount of interest income should they report on their joint income tax return?

a. $4,775. b. $4,675. *c. $4,575. d. $4,300. e. None of the above.

608. George, an unmarried cash basis taxpayer, received the following amounts during 2013:

Interest on savings accounts $2,000 Interest on a State tax refund 600 Interest on City of Salem school bonds 350 Interest portion of proceeds of a 5% bank certificate of deposit purchased on 250 July 1, 2012, and matured on June 30, 2013 Dividends on USG common stock 300

What amount should George report as gross income from dividends and interest for 2013?

a. $2,300. b. $2,550. *c. $3,150. d. $3,500. e. None of the above.

609. Stuart owns 300 shares of Turquoise Corporation stock and 2,000 shares of Blue Corporation stock. During the year, Stuart received 150 shares of Turquoise as a result of a 1 for 2 stock split. The value of the shares received was $4,800. Stuart also received 100 shares of Blue Corporation stock as a result of a 5% stock dividend. Stuart did not have the option of receiving cash from Blue. The additional shares he received had a value of $7,200. Stuart’s gross income from the receipt of the additional Turquoise and Blue shares is:

*a. $0. b. $4,800. c. $7,200. d. $12,000. e. None of the above.

610. Assuming a taxpayer qualifies for the exclusion treatment, the interest income on educational savings bonds:

a. Is gross income to the person who purchased the bond in the year the interest is earned. b. Is gross income to the student in the year the interest is earned. c. Is included in the student’s gross income in the year the savings bonds are sold or redeemed to pay educational expenses. *d. Is not included in anyone’s gross income if the proceeds are used to pay college tuition. e. None of the above.

611. The exclusion of interest on educational savings bonds:

a. Applies only to savings bonds owned by the child. *b. Applies to parents who purchase bonds for which the proceeds are used for their child’s education. c. Means that the child must include the interest in income if the bond is owned by the parent. d. Does apply even if used to pay for room and board. e. None of the above.

612. Martha participated in a qualified tuition program for the benefit of her son. She invested $6,000 in the fund. Four years later her son withdrew $8,000, the entire balance in the program, to pay his college tuition.

*a. Martha is not required to include the $2,000 ($8,000 – $6,000) in her gross income when the funds are used to pay the tuition. b. Martha’s son must include the $2,000 ($8,000 – $6,000) in his gross income when the funds are used to pay the tuition. c. Martha must include $8,000 in her gross income. d. Martha’s son must include $8,000 in his gross income. e. None of the above.

613. In December 2013, Todd, a cash basis taxpayer, paid $1,200 of fire insurance premiums for the calendar year 2014 on a building he held for rental income. Todd deducted the $1,200 of insurance premiums on his 2013 tax return. He had $150,000 of taxable income that year. On June 30, 2014, he sold the building and, as a result, received a $500 refund on his fire insurance premiums. As a result of the above:

a. Todd should amend his 2013 return and claim $500 less insurance expense. *b. Todd should include the $500 in 2014 gross income in accordance with the tax benefit rule. c. Todd should add the $500 to his sales proceeds from the building. d. Todd should include the $500 in 2014 gross income in accordance with the claim of right doctrine. e. None of the above.

614. Tonya is a cash basis taxpayer. In 2013, she paid state income taxes of $8,000. In early 2014, she filed her 2013 state income tax return and received a $900 refund.

a. If Tonya itemized her deductions in 2013 on her Federal income tax return, she should amend her 2013 return and reduce her itemized deductions by $900. b. If Tonya itemized her deductions in 2013 on her Federal income tax return and her itemized deductions exceeded the standard deduction by at least $900, the refund will not affect her 2014 tax return. c. If Tonya itemized her deductions in 2013 on her Federal income tax return, she must amend her 2013 Federal income tax return and use the standard deduction. *d. If Tonya itemized her deductions in 2013 on her Federal income tax return and her itemized deductions exceeded the standard deduction by more than $900, she must recognize $900 income in 2014 under the tax benefit rule. e. None of the above.

615. Harold bought land from Jewel for $150,000. Harold paid $50,000 cash and gave Jewel an 8% note for $100,000. The note was to be paid over a five-year period. When the balance on the note was $80,000, Jewel began having financial difficulties. To accelerate her cash inflows, Jewel agreed to accept $60,000 cash from Harold in final payment of the note principal.

a. Harold must recognize $20,000 ($80,000 – $60,000) of gross income. *b. Harold is not required to recognize gross income, but must reduce his cost basis in the land to $130,000. c. Harold is not required to recognize gross income, since he paid the debt before it was due. d. Jewel must recognize gross income of $20,000 ($80,000 – $60,000) from discharge of the debt. e. None of the above.

616. Hazel, a solvent individual but a recovering alcoholic, embezzled $6,000 from her employer. In the same year that she embezzled the funds, her employer discovered the theft. Her employer did not fire her and told her she did not have to repay the $6,000 if she would attend Alcoholics Anonymous. Hazel met the conditions and her employer canceled the debt.

a. Hazel did not realize any income because her employer made a gift to her. *b. Hazel must include $6,000 in gross income from discharge of indebtedness. c. Hazel must include $6,000 in gross income under the tax benefit rule. d. Hazel may exclude the $6,000 from gross income because the debt never existed. e. None of the above.

617. Gold Company was experiencing financial difficulties, but was not bankrupt or insolvent. The National Bank, which held a mortgage on other real estate owned by Gold, reduced the principal from $110,000 to $85,000. The bank had made the loan to Gold when it purchased the real estate from Silver, Inc. Pink, Inc., the holder of a mortgage on Gold’s building, agreed to accept $40,000 in full payment of the $55,000 due. Pink had sold the building to Gold for $150,000 that was to be paid in installments over 8 years. As a result of the above, Gold must:

a. Include $40,000 in gross income. b. Reduce the basis in its assets by $40,000. *c. Include $25,000 in gross income and reduce its basis in its assets by $15,000. d. Include $15,000 in gross income and reduce its basis in the building by $25,000. e. None of the above.

618. On January 1, 2003, Cardinal Corporation issued 5% 25-year bonds at par and used the $12,000,000 proceeds to finance the construction of a new plant. On January 1, 2013, the company acquired the bonds on the open market for $11,500,000. Assuming that Cardinal Corporation is neither bankrupt nor insolvent, the acquisition and retirement of the bonds results in which of the following:

*a. The company must recognize a $500,000 gain. b. The company can make an election to recognize a $500,000 gain or reduce the company’s basis in the plant by $500,000. c. The company must recognize a $500,000 gain and increase the company’s basis in the plant by $500,000. d. The company can amortize the $500,000 gain, recognizing income over the remaining life of the bonds. e. None of the above.

619. Denny was neither bankrupt nor insolvent but was short of cash and could not make the mortgage payments on his personal residence in 2013. The bank that held the mortgage agreed to reduce the principal on the debt from $100,000 to $80,000 so that Denny’s monthly mortgage payments could be reduced to a manageable amount. Denny also had a vacation home with a mortgage whose payments were beyond his means. The mortgage holder on the vacation home agreed to reduce the mortgage from $60,000 to $50,000. The value of the personal residence was $80,000 and the value of the vacation home was $45,000 at the dates of the debt reduction.

a. Denny is not required to recognize any income as a result of the reduction in the principal of the mortgages. *b. Denny is required to recognize $10,000 income from the reduction in the mortgage on the vacation home, but has no gross income from the reduction in the mortgage principal on his personal residence. c. Denny is required to recognize $5,000 income from the reduction in the mortgage on the vacation home, but nothing for the reduction in the mortgage on his personal residence. d. Denny is required to recognize $10,000 income from the reduction in the mortgage on the vacation home and $20,000 income for the reduction in the mortgage on his personal residence. e. None of the above.

620. Flora Company owed $95,000, a debt incurred to purchase land that serves as security for the debt.

a. If Flora had borrowed the funds from a bank, the bank accepts $85,000 in full payment of the debt, and Flora is solvent after the transfer, Flora does not recognize income, but the company must reduce the cost of the land by $10,000. b. If Flora had borrowed the funds from a bank, and the bank accepts $85,000 in full payment of the debt, when the value of the property is $80,000, Flora can deduct a loss. c. If Flora transfers to the bank other property, with a basis of $90,000 and a fair market value of $95,000, in full payment of the debt, Flora can recognize a $5,000 loss. *d. If the $95,000 is owed to the person who sold the property to Flora, and the creditor accepts $85,000 in full payment for the debt, Flora does not recognize gain but must reduce its basis in the land. e. None of the above.

621. Sandy is married, files a joint return, and expects to be in the 28% marginal tax bracket for the foreseeable future. All of his income is from salary and all of it is used to maintain the household. He has a paid-up life insurance policy with a cash surrender value of $100,000. He paid $60,000 of premiums on the policy. His gain from cashing in the life insurance policy would be ordinary income. If he retains the policy, the insurance company will pay him $3,000 (3%) interest each year. Sandy thinks he can earn a higher return if he cashes in the policy and invests the proceeds. a. What before-tax rate of return would Sandy be required to earn on the proceeds from cashing in the policy to equal the return earned with the insurance company? b. Assume Sandy estimates he can earn a 6% before-tax rate of return on the proceeds from cashing in the policy. Assume he can earn a 6% return for the remainder of his life and that he will reinvest all earnings at the same 6% before-tax rate of return. If Sandy expects to live 10 more years, which alternative will yield the greater amount to his beneficiaries upon Sandy’s death? (Given: The future value of an annuity in 10 years assuming a 4.32% after-tax return is 12.19. The future value of an annuity in 10 years assuming a 2.16% return is 11.03).

Correct Answer: a. If Sandy cashes in the policy, he must recognize a $40,000 gain and pay taxes of $11,200 [.28($100,000 – $60,000) = $11,200]. Therefore, he will have only $88,800 to invest ($100,000 – $11,200 = $88,800). To earn $3,000, the same as he receives from the insurance company, Sandy must earn a .03378 return on the after-tax proceeds ($88,800 × .03378 = $3,000). b. The life insurance proceeds will be exempt from income tax. Therefore, if Sandy retains the policy, the beneficiaries receive $100,000 plus the compound amount of (1 – .28)($3,000) = $2,160 interest earned each year. The interest will be reinvested at 6% before tax, or (1. – .28)(.06) = 4.32% after-tax interest. Given the compound interest factor of 12.19, the annual income will accumulate to 12.19 × $2,160 = $26,330. The after-tax policy proceeds are $100,000.

Therefore, if Sandy retains the policy, his beneficiaries would expect to receive $126,330 ($100,000 + $26,330). If Sandy cashed in the policy, his beneficiaries will receive the after-tax amount of the policy, as computed in a., above, of $88,800, plus the compound amount of the earnings on the $88,800, at a 4.32% after-tax return. The annual after-tax earnings on $88,800 is $3,836 (.0432 × $88,800). This will accumulate to $3,836 × 12.19 = $46,761 in 10 years. Therefore, his beneficiaries would receive $135,561 ($88,800 + $46,761). Cashing in the policy is the better alternative.

622. Beverly died during the current year. At the time of her death, her accrued salary and commissions totaled $3,000 and were paid to her husband. The employer also paid the husband $35,000 which represented an amount equal to Beverly’s salary for the year prior to her death. The employer had a policy of making the salary payments to “help out the family in the time of its greatest need.” Beverly’s spouse collected her interest in the employer’s qualified profit sharing plan amounting to $30,000. As beneficiary of his wife’s life insurance policy, Beverly’s spouse elected to collect the proceeds in installments. In the year of death, he collected $8,000 which included $1,500 interest income. Which of these items are subject to income tax for Beverly’s spouse?

Correct Answer: Salary and commissions $ 3,000 Profit sharing plan 30,000 Interest income 1,50 0 Included in gross income $34,500

All nonforfeitable rights to funds are includible in income (salary, commissions). This includes the accrued salary of $3,000. The collection of Beverly’s interest in the profit sharing plan of $30,000 is subject to taxation. The $35,000 payment by the employer was pursuant to a policy of charity to families of deceased employees, and there is authority for excluding this item as a gift. The IRS will probably challenge the exclusion of the $35,000. The IRS would argue that a policy of making the payment to all families of deceased employees makes the payment appear to be in the nature of compensation for prior services. Life insurance proceeds are tax-exempt. However, all interest paid on life insurance proceeds is includible in gross income

623. Barbara was injured in an automobile accident. She has threatened to file a suit against the other party involved in the accident and has proposed the following settlement:

Damages for 25% loss of the use of $200,000 her right arm Medical expenses 30,000 Loss of wages 10,000 Punitive damages 100,000 $340,000

The defendant’s insurance company is reluctant to pay punitive damages. Also, the company disputes the amount of her loss of wages amount. Instead, the company offers to pay her $300,000 for damages to her arm and $30,000 medical expenses. Assuming Barbara is in the 35% marginal tax bracket, will her after-tax proceeds from accepting the offer be equal to what she considers to be her actual damages (listed above)?

Correct Answer: Barbara’s claim for punitive damages of $100,000 is the only taxable amount. Therefore, her after-tax proceeds from receiving the $340,000 would be $305,000 [$340,000 – .35($100,000)]. None of the offer from the insurance company ($340,000) would be taxable and therefore her after-tax proceeds from the settlement would be $340,000. Thus, both the insurance company and Barbara would benefit from her accepting the insurance company’s offer.

624. George is employed by the Quality Appliance Company. All the full time employees are allowed to purchase appliances at the company’s cost plus 10%. The employee also is given, at no cost, a 1-year service contract on all the goods purchased from the company. George purchased a refrigerator for $500. The company’s normal selling price for the refrigerator is $800. George also received a service contract, at no charge, that had a value of $150. During the year, George was required to have his refrigerator serviced once. The cost of the call would have been $75 if he had not had the service contract. Is George required to recognize any income from the purchase of the refrigerator, the receipt of the service contract, and the service call?

Correct Answer: George will probably be required to recognize $120 income from the service contract. The company can sell the service contract to an employee at a 20% discount and the employee is not required to recognize income. George received a 100% discount; therefore, $120 (80% × $150) must be included in his gross income. However, George can perhaps make a convincing argument that he is merely receiving a no- additional-cost service, and thus would not be required to recognize income. George will not be required to recognize income from the bargain purchase of the refrigerator because he paid more than the employer’s cost.

625. Sonja is a United States citizen who has worked in Spain for the past 10 months. She received $8,000 a month as compensation. Her employer has offered to extend Sonja’s contract to work in Spain for another 6 months at the same rate of pay. If she rejects the offer, she can return to the United States and receive a salary of $10,000 per month. While working in Spain, she is subject to the Spain income tax, which is approximately 11% of her gross pay. The marginal tax rate on her income taxed in the United States is 25%. Compare Sonja’s after-tax income assuming she remains in Spain with her after-tax income if she returns to the United States.

Correct Answer: If Sonja returns to the United States, she will not be present in the foreign country for the requisite period (at least 330 days during any 12 consecutive months). Thus, she will not be eligible for the foreign earned income exclusion. Her income will be subject to the 25% rate in the United States. Although she will be given credit for the taxes paid in Spain, the net effect of not extending the stay is to increase her tax rate from 11% to 25% on her income for the 15-month period (the original 10 months plus the additional 5 months under consideration). This would cost her $16,800 [(.25 – .11) × (15 × $8,000)]. The higher pay in the United States would yield only additional after-tax income of $9,000 [($10,000 – $8,000)(1 – .25)(6 months)]. Since the benefits of the foreign income exclusion are greater than the loss of income, it would be beneficial to stay in Spain, from an after-tax income point of view.

626. Juan, was considering purchasing an interest in a tax-exempt bond fund for $100,000, when he discovered that the interest must be included on his state income tax return. The interest rate is 5%. His marginal Federal tax rate is 35%, and his marginal state income tax rate is 10%. Juan itemizes his deductions on his Federal income tax return. As an alternative, Juan can purchase a state bond (a “double- exempt bond”) yielding 4.9% interest that is exempt from both Federal and state income tax. Which investment would yield the greater after- tax return?

Correct Answer: Juan will receive $5,000 before-tax from the bond fund. The state income tax is $500 [(.10)($5,000)]. The state income tax will be deductible on the Federal return; thus, the state taxes will reduce Juan’s after-tax income by only $325 [(1 – .35)($500]. Therefore, the annual after-tax return is $4,675 ($5,000 – $325), or 4.675%. The double-exempt bonds will yield 4.9% after tax; therefore, it is the preferred investment, assuming equal risks.

627. Margaret is trying to decide whether to place funds in a qualified tuition program. Her son will be attending college in 4 years. She is in the 35% marginal tax bracket and she believes she can earn an 7% before tax return on alternative investments. Thus, $10,000 will accumulate to $11,948 (after-tax) in 4 years. Margaret expects tuition to increase at the rate of 5% each year to $12,155 in 4 years. Her son will be in the 15% marginal tax bracket in all relevant years. Given these assumptions, should Margaret participate in the qualified tuition program?

Correct Answer: Margaret can accumulate $11,948 by investing her funds for 4 years, but then she must pay the actual tuition. Alternatively, if she invests the $10,000 in a qualified tuition program, the tuition will be paid in 4 years, regardless of the amount. The amount of the tuition less the $10,000 will not be subject to tax. Thus, the after-tax future value of the qualified tuition fund is $12,155 ($12,155 – $0), which is greater than the alternative accumulated value. Therefore, it appears that Margaret should participate.

628. Gull Corporation was undergoing reorganization under the bankruptcy laws. The shareholders, who had made loans of $300,000 to the corporation, agreed to accept additional stock with a value of $200,000 instead of repayment on the debt. The Old Line Insurance Company, which had a $400,000 mortgage on the building, agreed to reduce the principal to $250,000. A trade creditor with a receivable of $150,000 from the company agreed to accept $70,000 in full payment for the debt incurred to purchase goods that were still on hand. Finally, the company transferred some equipment with an adjusted basis of $90,000 in satisfaction of a liability for $120,000. Compute the corporation’s gross income and other adjustments necessary as a result of the above transactions.

Correct Answer: Gull is not required to recognize income from the shareholders exchanging the debt for stock (a nontaxable contribution to capital). The $80,000 reduction in debt ($150,000 – $70,000) to trade creditors can be used to reduce the basis in the goods purchased. However, Gull is required to recognize $30,000 gain ($120,000 – $90,000) from transferring the equipment in satisfaction of the debt. However, because Gulf is in bankruptcy, the $150,000 income from discharge of indebtedness on the mortgage held by Old Line, can be used to reduce tax attributes (e.g., net operating loss carryover, basis in assets).

629. Carmen had worked for Sparrow Corporation for thirty years when she died of a heart attack at age 60. She was practically penniless at the time of her death, owed a $12,000 hospital bill, and had a disabled spouse. The company was very concerned about its public image, and rather than run the risk of embarrassment from one of its long-term employees dying and leaving her spouse with insufficient means, the Board of Directors agreed to pay Carmen’s hospital bill and to give her spouse $6,000 per year for the rest of his life. Discuss both sides of the question whether Carmen (or her estate) and her spouse realize any taxable income from the above.

Correct Answer: The argument that Carmen and her spouse realize income from the payments is as follows: the employer was compensating for the employee’s prior services. The fact that the employer had no legal obligation to make the payments is not relevant since the employer realized a benefit (prevention of embarrassment).

The argument that Carmen and her spouse do not realize income is predicated upon characterizing the payments as a gift. Conditions which indicate that a gift was intended include the following: the spouse’s dire financial condition; the decedent had been fully compensated for her past services, and any benefits the corporation received from the payments were indirect because there was no obligation to pay such amounts.

The $12,000 hospital payment is taxable because the gift is likely taxable income to her estate, as income in respect of the decedent, because the gift exclusion does not apply to payments by the employer to the employee.

630. What are the tax problems associated with payments received by a wife from her deceased husband’s employer? (Assume the wife renders no services to the employer.)

Correct Answer: An amount paid in respect of compensation owed to the employee at the time of his death is taxable to the spouse, just as the amount would have been taxable to the decedent if he had received the money prior to death. Additional noninvested amounts paid by the employer probably should be totally excluded from the spouse’s income as a gift. However, the IRS generally considers such payments to be compensation for past services rather than gifts.

Payments received from the employer’s qualified pension or profit sharing plan are subject to taxation. Also, if the employee contributed to the pension and profit sharing plan, the beneficiary is allowed to treat this amount as a nontaxable recovery of capital.

631. Bob had a terminal illness and realized that he “can’t take it with him.” Therefore, he cashed in his insurance policy and received $120,000. He had paid $50,000 in premiums on the policy. He used the money to fulfill his lifelong ambitions of going to the Super Bowl, driving an expensive sports car, and vacationing in Bermuda.

Was Bob’s behavior consistent with the Congressional intent in providing the tax exemption he was permitted to use?

Correct Answer: No. Bob was permitted to exclude from his gross income the $70,000 gain ($120,000 – $50,000) he realized from cashing in the policy. The exclusion was permitted because he was terminally ill. The rationale for the exclusion is that often the person with the terminal illness will have the need for funds to be used for his or her medical care. Bob is clearly not using the funds for this purpose to be served by the law. Nevertheless, the law does not prevent Bob from obtaining the exclusion.

632. Ben was hospitalized for back problems. While he was away from the job, he collected his regular salary from an employer-sponsored income protection insurance policy. Ben’s employer-sponsored hospitalization insurance policy also paid for 90% of his medical expenses. Ben also collected on an income protection policy that he purchased. Which of the above sources of income are taxable? Explain the basis for excluding any item or items.

Correct Answer: Only the collections on the employer-sponsored income protection policy are subject to tax. The hospitalization benefits received from the employer sponsored plan are specifically excluded. Both the premiums and the payments can be excluded. The amounts Ben collected on a policy he purchased are specifically excluded under the rationale that the payments are a recovery of Ben’s premiums.

633. The CEO of Cirtronics Inc., discovered that the company’s competitor had adopted a cafeteria plan for its employees. The CEO is concerned about retaining his talented employees and would like you to provide a brief explanation as to why a cafeteria plan may be attractive to the company’s employees.

Correct Answer: Cafeteria plans are beneficial where employees desire different types of benefits. This often occurs when employees are married and their spouses receive some benefits from their employers. For example, if the husband is covered by health insurance provided by his employer, there is no need for the wife’s employer to provide coverage for the husband also. Moreover, some employees may need child care benefits while those without children may prefer cash. The cafeteria plan provides much greater flexibility in planning benefits.

634. What Federal income tax benefits are provided for college students?

Correct Answer: The Federal income tax system provides direct benefits to college students and indirect benefits by providing tax relief for the parents of the students. College students can receive tax-exempt scholarships. The interest on educational savings bonds, which are often purchased by parents may be exempt where the proceeds are used to pay qualified educational expenses. The qualified tuition program enables parents to fund the educational expenses of their children without any income being taxed to the parents or to children.

635. The taxpayer was in the 35% marginal tax bracket in 2013 and deducted $15,000 in state income taxes as an itemized deduction that year. In 2014, he filed his 2013 state income tax return and received a $5,000 refund of state income taxes paid in 2013. His marginal tax rate in 2013 was 15%. What was the taxpayer’s Federal tax benefit from the overpayment of his 2013 state income tax?

Correct Answer: The taxpayer realized a benefit because the deduction in 2013 yielded benefits of $.35 for each dollar of deductions, while the recovery of the prior deduction was taxed at only 15%. The taxpayer’s benefit was $1,000 [(.35 – .15)($5,000)].

636. Employers can provide numerous benefits to their employees and the employees are permitted to exclude the value of these benefits from gross income. What are the effects of the exclusions on: a. The progressiveness of the tax system? b. The complexity of the tax system?

Correct Answer: a. The benefit of an exclusion varies directly with the recipient’s marginal tax rate. Thus, individuals with the highest marginal tax rate enjoy the greatest benefit from the exclusion and those taxpayers in the lowest marginal tax rate enjoy the least benefit. Also, generally, the exclusions are often available with the better paying jobs. This also causes the tax system to be less progressive than if the exclusions were not permitted. b. Any exclusion creates complexities in the system because tests must be established to determine whether the benefit is eligible (e.g., whether the benefit is provided on in a discriminatory manner) for the special treatment. Also, often limitations are often created, which requires even more testing.

637. Sally and Ed each own property with a fair market value less than the amount of the outstanding mortgage on the property and also less than the original cost basis. They each were able to convince the mortgage holder to reduce the principal amount on the mortgage. Sally’s mortgage is on her personal residence and Ed’s mortgage is on rental property he owns. a. Explain whether each of these individuals has realized income from the reduction in the debt. b. Assume that under the current system of measuring income, each of these taxpayers realized income from the reductions in the mortgages. Should either of these taxpayers be permitted to exclude any of the debt reduction income?

Correct Answer: a. Each taxpayer’s liabilities were reduced. Therefore, their net worth has increased as measured using the cost basis in the assets. Each taxpayer also experienced a loss in the value of their assets. However, the losses were not realized (because each taxpayer still owns the property). Thus, each taxpayer had income from the reduction in debt, but no recognized loss. Fortunately, recent legislation permits the taxpayer whose property is a personal residence to exclude the amount of the debt reduction from gross income. The taxpayer who owns the rental home is not eligible for the debt reduction exclusion. b. Allowing the exclusion from income for the homeowner but not for the investor can only be justified on the basis of a value system that says we should bend the otherwise equitable rules to favor home ownership.

638. If a tax-exempt bond will yield approximately .65 (1 – .35) times the yield on a taxable bond of equal risk, who benefits from the tax exemption: the Federal government, the state and local governments who issue the bonds, or the investors?

Correct Answer: The state and local governments benefit from the exemption because they are required to pay less interest. The exemption costs the Federal government, and thus the exemption shifts resources from the Federal to the state and local governments. The investors do not derive any benefit from the exemption, in this example, because the market drives the price of the exempt bonds upward so that the after-tax yields on the bonds are equal for investors in the highest marginal tax bracket (35%).

639. Deductions are allowed unless a specific provision in the tax law provides otherwise.

a. True *b. False

640. Mitch is in the 28% tax bracket. He may receive a different tax benefit for a $2,000 expenditure that is classified as a deduction from AGI than he will receive for a $1,000 expenditure that is classified as a deduction for AGI.

*a. True b. False

641. Depending on the nature of the expenditure, expenses incurred in a trade or business may be deductible for or from AGI.

a. True *b. False

642. Only some employment related expenses are classified as deductions for AGI.

*a. True b. False

643. Section 212 expenses that are related to rent and royalty income are deductions for AGI.

*a. True b. False

644. Alice incurs qualified moving expenses of $12,000. If she is reimbursed by her employer, the deduction is classified as a deduction for AGI. If not reimbursed, the deduction is classified as an itemized deduction.

a. True *b. False

645. Expenses incurred for the production or collection of income generally are deductions from adjusted gross income.

*a. True b. False

646. The Code does not specifically define what constitutes a trade or business.

*a. True b. False

647. An expense need not be recurring in order to be “ordinary.”

*a. True b. False

648. Aaron, a shareholder-employee of Pigeon, Inc., receives a $300,000 salary. The IRS classifies $100,000 of this amount as unreasonable compensation. The effect of this reclassification is to decrease Aaron’s gross income by $100,000 and increase Pigeon’s gross income by $100,000.

a. True *b. False

649. The portion of a shareholder-employee’s salary that is classified as unreasonable has no effect on the amount of a shareholder-employee’s gross income, but results in an increase in the taxable income of the corporation.

*a. True b. False

650. Generally, a closely-held family corporation is not permitted to take a deduction for a salary paid to a family member in calculating corporate taxable income.

a. True *b. False

651. Only under limited circumstances can a loss on the sale of a personal use asset be deducted.

a. True *b. False

652. The income of a sole proprietorship are reported on Schedule C (Profit or Loss from Business).

*a. True b. False

653. The cash method can be used even if inventory and cost of goods sold are an income producing factor in the business.

a. True *b. False

654. A taxpayer’s note or promise to pay satisfies the “actually paid” requirement for the cash basis method of accounting.

a. True *b. False

655. Isabella owns two business entities. She may be able to use the cash method for one and the accrual method for the other.

*a. True b. False

656. Under the “one-year rule” for the current period deduction of prepaid expenses of cash basis taxpayers, the asset must expire or be consumed by the end of the tax year following the year of payment.

*a. True b. False

657. None of the prepaid rent paid on September 1 by a calendar year cash basis taxpayer for the next 18 months is deductible in the current period.

a. True *b. False

658. The period in which an accrual basis taxpayer can deduct an expense is determined by applying the economic performance and all events tests.

*a. True b. False

659. The amount of the addition to the reserve for bad debts for an accrual method taxpayer is allowed as a deduction for tax purposes, but is not allowed for a cash method taxpayer.

a. True *b. False

660. All domestic bribes (i.e., to a U.S. official) are disallowed as deductions.

*a. True b. False

661. Fines and penalties paid for violations of the law (e.g., illegal dumping of hazardous waste) are deductible only if they relate to a trade or business.

a. True *b. False

662. Susan is a sales representative for a U.S. weapons manufacturer. She makes a $100,000 “grease” payment to a U.S. government official associated with a weapons purchase by the U.S. Army. She makes a similar payment to a Saudi Arabian government official associated with a similar sale. Neither of these payments is deductible by Susan’s employer.

a. True *b. False

663. The cost of legal advice associated with the preparation of an individual’s Federal income tax return is not deductible because it is a personal expense.

a. True *b. False

664. Two-thirds of treble damage payments under the antitrust law are not deductible.

a. True *b. False

665. The legal cost of having a will prepared is not deductible.

*a. True b. False

666. Legal expenses incurred in connection with rental property are deductions from AGI.

a. True *b. False

667. Legal fees incurred in connection with a criminal defense are not deductible even if the crime is associated with a trade or business.

a. True *b. False

668. If a taxpayer operates an illegal business, no deductions are permitted.

a. True *b. False

669. Ordinary and necessary business expenses, other than cost of goods sold, of an illegal drug trafficking business do not reduce taxable income.

*a. True b. False

670. Jacques, who is not a U.S. citizen, makes a contribution to the campaign of a candidate for governor. Cassie, a U.S. citizen, also makes a contribution to the same campaign fund. If contributions by noncitizens are illegal under state law, the contribution by Cassie is deductible, while that by Jacques is not.

a. True *b. False

671. A baseball team that pays a star player an annual salary of $25 million can deduct the entire $25 million as salary expense. If the same amount is paid to the CEO of IBM, only $1 million is deductible.

*a. True b. False

672. For a taxpayer who is engaged in a trade or business, the cost of investigating a business in the same field is deductible only if the taxpayer acquires the business.

a. True *b. False

673. Investigation of a business unrelated to one’s present business never results in a current period deduction of the entire amount if the amount of the investigation expenses exceeds $5,000.

*a. True b. False

674. In determining whether an activity should be classified as a business or as a hobby, the satisfaction of the presumption (i.e., profit in at least 3 out of 5 years) ensures treatment as a business.

a. True *b. False

675. If a taxpayer can satisfy the three-out-of-five year presumption test associated with hobby losses, then expenses from the activity can be deducted in excess of the gross income from the activity.

a. True *b. False

676. If an activity involves horses, a profit in at least two of seven consecutive years meets the presumptive rule of § 183.

*a. True b. False

677. A hobby activity can result in all of the hobby income being included in AGI and no deductions being allowed.

*a. True b. False

678. If an item such as property taxes and home mortgage interest exceed the income from a hobby, the excess amount of this item over the hobby income can be deducted if the taxpayer itemizes deductions.

*a. True b. False

679. Hobby activity expenses are deductible from AGI to the extent of hobby income. Such expenses not in excess of hobby income are not subject to the 2% of AGI floor.

a. True *b. False

680. Martha rents part of her personal residence in the summer for 3 weeks for $3,000. Anne rents all of her personal residence for one week in December for $2,500. Anne is not required to include the $2,500 in her gross income whereas Martha is required to include the $3,000 in her gross income.

*a. True b. False

681. If a vacation home is rented for less than 15 days during a year, the only expenses that can be deducted are mortgage interest, property taxes, and personal casualty losses.

*a. True b. False

682. If a vacation home is classified as primarily rental use, a deduction for all of the rental expenses is allowed.

a. True *b. False

683. If a vacation home is classified as primarily personal use (i.e., rented for fewer than 15 days), none of the related expenses can be deducted.

a. True *b. False

684. The portion of property tax on a vacation home that is attributable to personal use is an itemized deduction.

*a. True b. False

685. If a vacation home is classified as primarily personal use, part of the maintenance and utility expenses can be allocated and deducted as a rental expense.

a. True *b. False

686. A vacation home at the beach which is rented for 200 days and used personally for 16 days is classified in the personal/rental use category.

a. True *b. False

687. If a vacation home is a personal/rental residence, no maintenance and utility expenses can be claimed as a deduction.

a. True *b. False

688. Beulah’s personal residence has an adjusted basis of $450,000 and a fair market value of $390,000. Beulah converts the property to rental use on November 1, 2013. The vacation home rules that limit the amount of the deduction to the rental income will apply and the adjusted basis for depreciation is $390,000.

a. True *b. False

689. Walt wants to give his daughter $1,800 for Christmas. As an alternative, she suggests that he pay the property taxes on her residence. If Ralph pays the property taxes, he can deduct them.

a. True *b. False

690. LD Partnership, a cash basis taxpayer, purchases land and a building for $200,000 with $150,000 of the cost being allocated to the building. The gross receipts of the partnership are less than $100,000. LD must capitalize the $50,000 paid for the land, but can deduct the $150,000 paid for the building in the current tax year.

a. True *b. False

691. Purchased goodwill must be capitalized, but can be amortized over a 60-month period.

a. True *b. False

692. Marge sells land to her adult son, Jason, for its $20,000 appraised value. Her adjusted basis for the land is $25,000. Marge’s recognized loss is $5,000 and Jason’s adjusted basis for the land is $25,000 ($20,000 cost + $5,000 recognized gain of Marge).

a. True *b. False

693. For purposes of the § 267 loss disallowance provision, a taxpayer’s aunt is a related party.

a. True *b. False

694. Sammy, a calendar year cash basis taxpayer who is age 66, has the following transactions:

Salary from job $90,000 Alimony received 10,000 from ex-wife Medical expenses 8,000

Based on this information, Sammy has:

a. AGI of $90,000. b. AGI of $95,000. c. AGI of $99,500. d. Deduction for medical expenses of $0. *e. None of the above.

695. Trade and business expenses should be treated as:

a. A deduction from AGI subject to the 2%-of-AGI floor. b. A deduction from AGI not subject to the 2%-of-AGI floor. *c. Deductible for AGI. d. An itemized deduction if not reimbursed. e. None of the above.

696. Al is single, age 60, and has gross income of $140,000. His deductible expenses are as follows:

Alimony $20,000 Charitable contributions 4,000 Contribution to a traditional IRA 5,000 Expenses paid on rental property 7,500 Interest on home mortgage and property 7,200 taxes on personal residence State income tax 2,200

What is Al’s AGI?

a. $94,100. b. $103,500. *c. $107,500. d. $127,500. e. None of the above.

697. Marsha is single, had gross income of $50,000, and incurred the following expenses:

Charitable contribution $2,000 Taxes and interest on home 7,000 Legal fees incurred in a tax dispute 1,000 Medical expenses 3,000 Penalty on early withdrawal of savings 250

Her AGI is:

a. $39,750. *b. $49,750. c. $40,000. d. $39,750. e. None of the above.

698. Which of the following can be claimed as a deduction for AGI?

a. Personal casualty losses. b. Investment interest expenses. c. Medical expenses. d. Property taxes on personal use real estate. *e. None of the above.

699. Which of the following is a deduction for AGI (itemized deduction)?

*a. Contribution to a traditional IRA. b. Roof repairs to a personal use home. c. Safe deposit box rental fee in which stock certificates are stored. d. Property tax on personal residence. e. All of the above.

700. Which of the following is correct?

a. A personal casualty loss is classified as a deduction from AGI. b. Real estate taxes on a taxpayer’s personal residence are classified as deductions from AGI. c. An expense associated with rental property is classified as a deduction for AGI. d. Only a. and b. are correct. *e. a., b., and c., are correct.

701. Which of the following are deductions for AGI?

a. Mortgage interest on a personal residence. b. Property taxes on a personal residence. *c. Mortgage interest on a building used in a business. d. Fines and penalties incurred in a trade or business. e. None of the above.

702. Which of the following is incorrect?

a. Alimony is a deduction for AGI. *b. The expenses associated with royalty property are a deduction from AGI. c. Contributions to a traditional IRA are a deduction for AGI. d. Property taxes on taxpayer’s personal residence are a deduction from AGI e. All of the above are correct.

703. Which of the following is not a “trade or business” expense?

a. Interest on business indebtedness. b. Property taxes on business property. *c. Parking ticket paid on business auto. d. Depreciation on business property. e. All of the above are “trade or business” expenses.

704. Which of the following is a required test for the deduction of a business expense?

a. Ordinary. b. Necessary. c. Reasonable. *d. All of the above. e. None of the above.

705. Paula is the sole shareholder of Violet, Inc. For 2013, she receives from Violet a salary of $300,000 and dividends of $100,000. Violet’s taxable income for 2013 is $500,000. On audit, the IRS treats $100,000 of Paula’s salary as unreasonable. Which of the following statements is correct?

a. Paula’s gross income will increase by $100,000 as a result of the IRS adjustment. b. Violet’s taxable income will not be affected by the IRS adjustment. c. Paula’s gross income will decrease by $100,000 as a result of the IRS adjustment. d. Violet’s taxable income will decrease by $100,000 as a result of the IRS adjustment. *e. None of the above is correct.

706. During 2012, the first year of operations, Silver, Inc., pays salaries of $175,000. At the end of the year, employees have earned salaries of $20,000, which are not paid by Silver until early in 2013. What is the amount of the deduction for salary expense?

a. If Silver uses the cash method, $175,000 in 2012 and $0 in 2013. b. If Silver uses the cash method, $0 in 2012 and $195,000 in 2013. c. If Silver uses the accrual method, $175,000 in 2012 and $20,000 in 2013. *d. If Silver uses the accrual method, $195,000 in 2012 and $0 in 2013. e. None of the above is correct.

707. Benita incurred a business expense on December 10, 2013, which she charged on her bank credit card. She paid the credit card statement which included the charge on January 5, 2014. Which of the following is correct?

a. If Benita is a cash method taxpayer, she cannot deduct the expense until 2014. *b. If Benita is an accrual method taxpayer, she can deduct the expense in 2013. c. If Benita uses the accrual method, she can choose to deduct the expense in either 2013 or 2014. d. Only b. and c. are correct. e. a., b., and c. are correct.

708. Payments by a cash basis taxpayer of capital expenditures:

a. Must be expensed at the time of payment. b. Must be expensed by the end of the first year after the asset is acquired. *c. Must be deducted over the actual or statutory life of the asset. d. Can be deducted in the year the taxpayer chooses. e. None of the above.

709. Petal, Inc. is an accrual basis taxpayer. Petal uses the aging approach to calculate the reserve for bad debts. During 2013, the following occur associated with bad debts.

Credit sales $400,000 Collections on credit sales 250,000 Amount added to the reserve 10,000 Beginning balance in the reserve –0– Identifiable bad debts during 2013 12,000

The amount of the deduction for bad debt expense for Petal for 2013 is:

a. $10,000. *b. $12,000. c. $22,000. d. $140,000. e. None of the above.

710. Which of the following legal expenses are deductible for AGI?

a. Incurred in connection with a trade or business. b. Incurred in connection with rental or royalty property held for the production of income. c. Incurred for tax advice relative to the preparation of an individual’s income tax return. *d. Only a. and b. qualify. e. a., b., and c. qualify.

711. Rex, a cash basis calendar year taxpayer, runs a bingo operation which is illegal under state law. During 2013, a bill designated H.R. 9 is introduced into the state legislature which, if enacted, would legitimize bingo games. In 2013, Rex had the following expenses:

Operating expenses in conducting bingo $247,000 games Payoff money to state and local police 24,000 Newspaper ads supporting H.R. 9 3,000 Political contributions to legislators 8,000 who support H.R. 9

Of these expenditures, Rex may deduct:

*a. $247,000. b. $250,000. c. $258,000. d. $282,000. e. None of the above.

712. Andrew, who operates a laundry business, incurred the following expenses during the year.

• Parking ticket of $250 for one of his delivery vans that parked illegally. • Parking ticket of $75 when he parked illegally while attending a rock concert in Tulsa. • DUI ticket of $500 while returning from the rock concert. • Attorney’s fee of $600 associated with the DUI ticket.

What amount can Andrew deduct for these expenses?

*a. $0. b. $250. c. $600. d. $1,425. e. None of the above.

713. Which of the following may be deductible?

a. Bribes that relate to a U.S. business. b. Fines paid for violations of the law. *c. Interest on a loan used in a hobby. d. All of the above. e. None of the above.

714. Terry and Jim are both involved in operating illegal businesses. Terry operates a gambling business and Jim operates a drug running business. Both businesses have gross revenues of $500,000. The businesses incur the following expenses.

Terry Jim Employee salaries $200,000 $200,000 Bribes to police 25,000 25,000 Rent and utilities 50,000 50,000 Cost of goods sold –0– 125,000

Which of the following statements is correct?

a. Neither Terry nor Jim can deduct any of the above items in calculating the business profit. *b. Terry should report profit from his business of $250,000. c. Jim should report profit from his business of $500,000. d. Jim should report profit from his business of $250,000. e. None of the above.

715. Tom operates an illegal drug-running operation and incurred the following expenses:

Salaries $ 75,000 Illegal kickbacks 20,000 Bribes to border guards 25,000 Cost of goods sold 160,000 Rent 8,000 Interest 10,000 Insurance on furniture and fixtures 6,000 Utilities and telephone 20,000

Which of the above amounts reduces his taxable income?

a. $0. *b. $160,000. c. $279,000. d. $324,000. e. None of the above.

716. For a president of a publicly held corporation, which of the following are not subject to the $1 million limit on executive compensation?

a. Contribution to medical insurance plan. b. Contribution to pension plan. c. Premiums on group term life insurance of $50,000. d. Only b. and c. are not subject to the limit. *e. a., b., and c., are not subject to the limit.

717. Tommy, an automobile mechanic employed by an auto dealership, is considering opening a fast food franchise. If Tommy decides not to acquire the fast food franchise, any investigation expenses are:

a. A deduction for AGI. b. A deduction from AGI, subject to the 2 percent floor. c. A deduction from AGI, not subject to the 2 percent floor. d. Deductible up to $5,000 in the current year with the balance being amortized over a 180-month period. *e. Not deductible.

718. Iris, a calendar year cash basis taxpayer, owns and operates several TV rental outlets in , and wants to expand to other states. During 2013, she spends $14,000 to investigate TV rental stores in South Carolina and $9,000 to investigate TV rental stores in Georgia. She acquires the South Carolina operations, but not the outlets in Georgia. As to these expenses, Iris should:

a. Capitalize $14,000 and not deduct $9,000. *b. Expense $23,000 for 2013. c. Expense $9,000 for 2013 and capitalize $14,000. d. Capitalize $23,000. e. None of the above.

719. Which of the following statements is correct in connection with the investigation of a business?

a. If the taxpayer is not already engaged in the trade or business, the expenses incurred are deductible if the project is abandoned. *b. If the business is acquired, the expenses may be deducted immediately by a taxpayer engaged in a similar trade or business regardless of whether the business being investigated is acquired. c. That business must be related to the taxpayer’s present business for any expense ever to be deductible. d. Regardless of whether the taxpayer is already engaged in the trade or business, the expenses must be capitalized and amortized. e. None of the above.

720. Which of the following is not relevant in determining whether an activity is profit-seeking or a hobby?

a. Whether the activity is enjoyed by the taxpayer. b. The expertise of the taxpayers or their advisers. c. The time and effort expended. d. The relationship of profits earned and losses incurred. *e. All of the above are relevant factors.

721. For an activity classified as a hobby, the expenses are categorized as follows:

(1) Amounts that affect adjusted basis and would be deductible under other Code sections if the activity had been engaged in for profit (e.g., depreciation, amortization, and depletion).

(2) Amounts deductible under other Code sections without regard to the nature of the activity, such as property taxes and home mortgage interest.

(3) Amounts deductible under other Code sections if the activity had been engaged in for profit, but only if those amounts do not affect adjusted basis (e.g., maintenance, utilities, and supplies).

If these expenses exceed the gross income from the activity and are thus limited, the sequence in which they are deductible is:

a. (1), (2), (3). b. (1), (3), (2). *c. (2), (3), (1). d. (2), (1), (3). e. (3), (2), (1).

722. Priscella pursued a hobby of making bedspreads in her spare time. Her AGI before considering the hobby is $40,000. During the year she sold the bedspreads for $10,000. She incurred expenses as follows:

Supplies $4,000 Interest on loan to get business started 500 Advertising 6,500

Assuming that the activity is deemed a hobby, how should she report these items on her tax return?

a. Include $10,000 in income and deduct $11,000 for AGI. b. Ignore both income and expenses since hobby losses are disallowed. c. Include $10,000 in income, deduct nothing for AGI, and claim $11,000 of the expenses as itemized deductions. d. Include $10,000 in income and deduct interest of $500 for AGI. *e. None of the above.

723. Cory incurred and paid the following expenses:

Tax return preparation fee $ 600 Moving expenses 2,000 Investment expenses 500 Expenses associated with rental 1,500 property Interest expense associated with loan 400 to finance tax-exempt bonds

Calculate the amount that Cory can deduct (before any percentage limitations).

a. $5,000. *b. $4,600. c. $3,000. d. $1,500. e. None of the above.

724. Which of the following is not deductible?

a. Moving expenses in excess of reimbursement. b. Tax return preparation fees of an individual. c. Expenses incurred associated with investments in stocks and bonds. *d. Allowable hobby expenses in excess of hobby income. e. None of the above.

725. If a residence is used primarily for personal use (rented for fewer than 15 days per year), which of the following is correct?

*a. No income is included in AGI. b. No expenses are deductible. c. Expenses must be allocated between rental and personal use. d. Only a. and b. are correct. e. a., b., and c. are correct.

726. Robyn rents her beach house for 60 days and uses it for personal use for 30 days during the year. The rental income is $6,000 and the expenses are as follows:

Mortgage interest $9,000 Real estate taxes 3,000 Utilities 2,000 Maintenance 1,000 Insurance 500 Depreciation (rental part) 4,000

Using the IRS approach, total expenses that Robyn can deduct on her tax return associated with the beach house are:

a. $0. b. $6,000. c. $8,000. *d. $12,000. e. None of the above.

727. If a vacation home is determined to be a personal/rental use residence, which of the following statements is correct?

a. All rental income is included in gross income. b. All rental related expenses that are deductible are classified as deductions from AGI. c. Expenses must be allocated between rental and personal use. *d. Only a. and c. are correct. e. a., b., and c. are correct.

728. Bob and April own a house at the beach. The house was rented to unrelated parties for 8 weeks during the year. April and the children used the house 12 days for their vacation during the year. After properly dividing the expenses between rental and personal use, it was determined that a loss was incurred as follows:

Gross rental income $4,000 Less: Mortgage interest and $3,500 property taxes Other allocated 2,00 (5,50 expenses 0 0) Net rental loss ($1,500)

What is the correct treatment of the rental income and expenses on Bob and April’s joint income tax return for the current year assuming the IRS approach is used if applicable?

*a. A $1,500 loss should be reported. b. Only the mortgage interest and property taxes should be deducted. c. Since the house was used more than 10 days personally by Bob and April, the rental expenses (other than mortgage interest and property taxes) are limited to the gross rental income in excess of deductions for interest and taxes allocated to the rental use. d. Since the house was used less than 50% personally by Bob and April, all expenses allocated to personal use may be deducted. e. Bob and April should include none of the income or expenses related to the beach house in their current year income tax return.

729. Because Scott is three months delinquent on the mortgage payments for his personal residence, Jeanette (his sister) is going to cover the arrearage. Based on past experience, she does not expect to be repaid by Scott. Which of the following statements is correct?

a. If Scott receives the money from Jeanette and pays the mortgage company, Jeanette can deduct the interest part. b. If Jeanette pays the mortgage company directly, neither Scott nor Jeanette can deduct the interest part. c. If Jeanette pays the mortgage company directly, she cannot deduct the interest part. *d. Only b. and c. are correct. e. a., b., and c. are correct.

730. Melba incurred the following expenses for her dependent daughter during the current year:

Payment of principal on daughter’s $3,600 automobile loan Payment of interest on above loan 2,900 Payment of daughter’s property taxes 1,800 Payment of principal on daughter’s 2,800 personal residence loan Payment of interest on daughter’s 7,000 personal residence loan

How much may Melba deduct in computing her itemized deductions?

*a. $0. b. $8,800. c. $11,700. d. $18,100. e. None of the above.

731. Velma and Bud divorced. Velma’s attorney fee of $5,000 is allocated as follows:

General representation in obtaining the $1,500 divorce Services in obtaining custody of the 900 child Services in settlement of martial 600 property Determining the tax consequences of: Dependency deduction for child 700 Property settlement 1,300

Of the $5,000 Velma pays to her attorney, the amount she may deduct as an itemized deduction is:

a. $0. b. $700. *c. $2,000. d. $5,000. e. None of the above.

732. Which of the following must be capitalized by a business?

a. Replacement of a windshield of a business truck which was broken in an accident. b. Repair of a roof of a building used in business. *c. Amount paid for a covenant not to compete. d. Only b. and c. must be capitalized. e. a., b., and c. can be expensed rather than capitalized.

733. On January 2, 2013, Fran acquires a business from Chuck. Among the assets purchased are the following intangibles: patent with a 7-year remaining life, a covenant not to compete for 10 years, and goodwill.

Of the purchase price, $140,000 was paid for the patent and $60,000 for the covenant. The amount of the excess of the purchase price over the identifiable assets was $100,000. What is the amount of the amortization deduction for 2013?

a. $10,667. b. $16,000. *c. $20,000. d. $32,667. e. None of the above.

734. In January, Lance sold stock with a cost basis of $26,000 to his brother, James, for $24,000, the fair market value of the stock on the date of sale. Five months later, James sold the same stock through his broker for $27,000. What is the tax effect of these transactions?

a. Disallowed loss to James of $2,000; gain to Lance of $1,000. b. Disallowed loss to Lance of $2,000; gain to James of $3,000. c. Deductible loss to Lance of $2,000; gain to James of $3,000. *d. Disallowed loss to Lance of $2,000; gain to James of $1,000. e. None of the above.

735. Nikeya sells land (adjusted basis of $120,000) to her adult son, Shamed, for its appraised value of $95,000. Which of the following statements is correct?

a. Nikeya’s recognized loss is $25,000 ($95,000 amount realized – $120,000 adjusted basis). b. Shamed’s adjusted basis for the land is $120,000 ($95,000 cost + $25,000 disallowed loss for Nikeya). *c. If Shamed subsequently sells the land for $112,000, he has no recognized gain or loss. d. Only a. and b. are correct. e. a., b., and c. are correct.

736. Which of the following is not a related party for constructive ownership purposes under § 267?

*a. The taxpayer’s aunt. b. The taxpayer’s brother. c. The taxpayer’s grandmother. d. A corporation owned more than 50% by the taxpayer. e. None of the above.

737. Austin, a single individual with a salary of $100,000, incurred and paid the following expenses during the year:

Medical expenses $ 5,000 Alimony 24,000 Charitable contributions 2,000 Casualty loss (after $100 1,000 floor) Mortgage interest on personal 4,500 residence Property taxes on personal 4,200 residence Moving expenses 2,500 Contribution to a traditional 4,000 IRA Sales taxes (no state or 1,300 local income tax is imposed)

Calculate Austin’s deductions for AGI.

Correct Answer: Only the following expenses are deductible for AGI:

Alimony $24,000 Moving expenses 2,500 Contribution to IRA 4,000 Deductions for AGI $30,500

The other expenses, after applying any statutory floors, are deductions from AGI.

738. Arnold and Beth file a joint return. Use the following data to calculate their deduction for AGI.

Mortgage interest on personal residence $ 6,0 00 Property taxes on personal residence 2,500 Alimony payments 12,000 Moving expenses 7,000 Charitable contributions 1,500 State income taxes 5,000 Investment interest ($8,000 of expenses 7,500 limited to net investment income of $7,500) Unreimbursed employee expenses 2,500 Sales taxes 2,600

Correct Answer: Arnold and Beth’s deduction for AGI is $19,000 and consists of the following items:

Alimony payments $12,000 Moving expenses 7,00 0 Deduction for AGI $19,000

All of the other items are itemized deductions. Note that the taxpayer must choose between the state income taxes and the sales taxes.

739. Robin and Jeff own an unincorporated hardware store. They determine their salaries at the end of the year by using the amount required to reduce the net income of the hardware store to $0. Based on this policy, Robin and Jeff each receive a total salary of $125,000. This is paid as follows: $8,000 per month and $29,000 on December 31. Determine the amount of the salary deduction.

Correct Answer: Since the hardware store is not incorporated, the issue of the reasonableness of the salaries is not relevant. Robin and Jeff will report income of $125,000 each regardless of whether it is labeled as salary or as a distribution of the hardware store’s net income. Therefore, there is nothing wrong with the hardware store (i.e., a partnership) taking a $250,000 salary deduction.

740. Sandra owns an insurance agency. The following selected data are taken from the agency balance sheet and income statement prepared using the accrual method.

Revenue $250,000 Salaries and commissions 100,000 Rent 10,000 Insurance 5,000 Utilities 6,000 Accounts receivable, 1/1/2013 40,000 Accounts receivable, 12/31/2013 38,000 Accounts payable, 1/1/2013 12,000 Accounts payable, 12/31/2013 11,000

Calculate Sandra’s net profit using the cash method for 2013.

Correct Answer: Sandra’s accrual method net profit is calculated as follows:

Revenue $250,000 Less: Expenses Salaries and $100,000 commissions Rent 10,000 Insurance 5,000 Utilities 6,0 (121,000) 00 Net profit $129,000

To convert to cash method net profit, the following adjustments must be made.

Net profit—accrual method $129,000 Deduct: Decrease in accounts (1,000) payable ($11,000 – $12,000) Add: Decrease in accounts 2,0 receivable ($38,000 – $40,000) 00 Net profit—cash method $130,000

741. Alfred’s Enterprises, an unincorporated entity, pays employee salaries of $100,000 during the year. At the end of the year, $12,000 of additional salaries have been earned but not paid until the beginning of the next year. a. Determine the amount of the deduction for salaries if Alfred is a cash method taxpayer. b. Determine the amount of the deduction for salaries if Alfred is an accrual method taxpayer.

Correct Answer: a. The deduction for salaries is the amount paid of $100,000. b. The deduction for salaries is calculated as follows:

Salaries $100,000 Accrued salaries 12,0 00 Salary deduction $112,000

742. Taylor, a cash basis architect, rents the building in which his office is located for $5,000 per month. He commenced his practice on February 1, 2013. In order to guarantee no rent increases during an 18- month period, he signed an 18-month lease and prepaid the $90,000 on February 1, 2013. How much can Taylor deduct as rent expense for 2013?

Correct Answer: Taylor is a cash basis taxpayer. Thus, he is eligible to use the “one- year rule” on prepayments. Since his prepayments of 18-months rent does not extend beyond the end of 2014, he can deduct the $90,000 paid in 2013.

743. In order to protect against rent increases on the building in which she operates a dance studio, Mella signs an 18-month lease for $36,000. The lease commences on October 1, 2013. How much of the $36,000 payment can she deduct in 2013 and 2014? a. If Mella is an accrual basis taxpayer? b. If Mella is a cash basis taxpayer?

Correct Answer: a. As an accrual basis taxpayer, Mella can deduct the amount of the rent expenses incurred in 2013 of $6,000 ($2,000 × 3 months) for 2013 and the $24,000 ($2,000 × 12 months) incurred in 2014. b. Since Mella is a cash basis taxpayer, she can deduct the entire $36,000 prepayment in 2013 if she can satisfy the one-year rule. However, since the rental period of 18 months extends beyond the end of 2014, she fails the requirement for the one-year rule. Consequently, she can deduct only $6,000 in 2013 and $24,000 in 2014.

744. Petula’s business sells heat pumps which have a one-year warranty. Based on historical data, the warranty costs amount to 11% of sales. During 2013, heat pump sales are $400,000. Actual warranty expenses paid in 2013 are $40,000. a. Determine the amount of the warranty expense deduction for 2013 if Petula’s business uses the accrual method. b. How would your answer change if Petula used the cash method for extended warranties and the purchasers paid $25,000 for the warranties which covered the second and third years of ownership?

Correct Answer: a. Even though Petula’s business uses the accrual method, reserves for estimated warranty expenses are not permitted. Therefore, the deduction for warranty expenses is the amount paid of $40,000. b. Petula would record gross income in 2013 of $425,000 ($400,000 + $25,000). The deduction for warranty expense would still be $40,000.

745. Beige, Inc., an airline manufacturer, is conducting negotiations for the sale of military aircraft. One negotiation is with a U.S. assistant secretary of defense. She can close the deal on the purchase of 50 attack helicopters if she is paid $750,000 under the table. Another negotiation is with the minister of defense of a third world country. To complete the sale of 20 jet fighters to his government, he demands that he be paid a $1 million grease payment. Beige makes the payments and closes the deals. How much of these payments are deductible by Beige, Inc.?

Correct Answer: The $750,000 payment to the U.S. assistant secretary of defense is a bribe and is not deductible. If the grease payment of $1 million to the minister of defense of the third world country does not violate the Foreign Corrupt Practices Act of 1977, then the entire $1 million payment is deductible. However, if the grease payment does violate the Act, then none of it is deductible.

746. Albie operates an illegal drug-running business and has the following items of income and expense. What is Albie’s adjusted gross income from this operation?

Income $800,000 Expense s: Rent 24,000 Utilities 9,000 Bribes to police 55,000 Medical expense 5,000 Legal fees 25,000 Depreciation 30,000 Illegal kickbacks 30,000 Cost of goods sold 300,000

Correct Answer: Albie is allowed to reduce his AGI only by the cost of goods sold; thus, his AGI is $500,000 ($800,000 – $300,000). Note that the cost of goods sold is treated as a negative item in calculating gross income.

747. Kitty runs a brothel (illegal under state law) and has the following items of income and expense. What is the amount that she must include in taxable income from her operation?

Income $200,000 Expense s: Rent 8,000 Utilities 2,000 Bribes to police 10,000 Medical expense 5,000 Legal fees 20,000 Depreciation 14,000 Illegal kickbacks 15,000

Correct Answer: Income $200,000 Expenses: Rent $ 8,000 Utilities 2,000 Medical 5,000 Legal fees 20,000 Depreciation 14,000 (49,000) $151,000

The bribes to police of $10,000 and illegal kickbacks of $15,000 are not deductible.

748. Janet is the CEO for Silver, Inc., a closely held corporation. Her total compensation for 2013 is $5 million. Of this amount, $2 million is a salary and $3 million is a bonus. The bonus was calculated as 5% of Silver’s net income before the bonus and before taxes ($60 million X 5% = $3 million). The bonus provision has been in effect since Janet became CEO five years ago and is related to Silver’s performance. It is approved annually by the entire board of directors (1 of the 5 directors is an outside director) of Silver. How much of Janet’s compensation can Silver deduct for 2013?

Correct Answer: All of the $5 million is deductible by Silver. Since Silver is a closely held, rather than a publicly held corporation, the $1 million statutory limit on the deduction of certain executive compensation is not applicable.

749. Agnes operates a Christmas Shop in Atlantic City, NJ. She makes a weekend trip to Vero Beach, FL, for the purpose of determining the feasibility of opening another shop. Her travel expenses are $2,000 (includes $500 for meals). In addition, she pays $5,000 to a market research firm in Vero Beach to prepare a feasibility study. Determine the amount of the expenses that Agnes can deduct if: a. She opens a new shop in Vero Beach. b. She decides not to open a new shop in Vero Beach.

Correct Answer: a. Because Agnes is already in the Christmas Shop business, all of the investigation expenses ($2,000 + $5,000 = $7,000) are deductible regardless of whether or not she opens a shop in Vero Beach. Note, however, that as discussed in Chapter 9, only 50% of the cost of the meals is deductible. b. Same response as in a.

750. While she was a college student, Angel lived by a bookstore located near campus. She thinks a bookstore located on the other side of campus would be successful. She incurs expenses of $42,800 (legal fees, accounting fees, marketing survey, etc.) in exploring its business potential. Her parents have agreed to loan her the money required to start the business. What amount of these investigation costs can Angel deduct if: a. She opens the bookstore on August 1, 2013. b. She decides not to open the bookstore.

Correct Answer: a. If Angel opens the bookstore on August 1, 2013, she can deduct the following investigation expenses in 2013.

Allowed expense deduction in $5,000 first year Amortization ($37,800/180 1,050 months × 5 months) Deductible investigation $6,050 expenses

b. If Angel does not open the bookstore, she cannot deduct any of the $42,800 of expenses she incurred.

751. Calculate the net income includible in taxable income for the following hobby:

Income $23,000 Mortgage interest and property taxes 12,000 allocable to hobby Depreciation 4,000 Supplies and fees 7,000 Telephone for hobby 3,000

Correct Answer: Income (includible in $23,000 gross income) Itemized deductions : Mortgage interest and $12,000 property taxes Supplies and fees 7,000 Telephone 3,000 Depreciation (limited 1,0 (23,000) to $23,000 – $12,000 00 – $7,000 – $3,000) $ – 0–

Otherwise deductible expenses must be deducted first; only enough other expenses are allowed to offset the remaining income. Deductions affecting depreciable basis are taken last. The mortgage interest and property taxes are deductible as itemized deductions and the other hobby-related expenses are subject to the 2%-of-AGI floor. Once the taxpayer’s AGI is determined, the effect of the 2%-of-AGI floor on itemized deductions can be calculated.

752. During the year, Rita rented her vacation home for twelve days for $2,400 and she used it personally for three months. The following expenses were incurred on the home:

Property taxes $ 2,200 Mortgage interest 10,800 Utilities and maintenance 1,900 Depreciation 5,000 Insurance 900

Calculate her rental gain or loss and itemized deductions.

Correct Answer: Rita excludes the $2,400 of rental income from gross income because the home is classified as primarily personal. She can deduct the property taxes ($2,200) and mortgage interest ($10,800) as itemized deductions. No other expenses are deductible.

753. During the year, Jim rented his vacation home for 200 days and lived in it for 19 days. During the remaining days, the vacation home was available for rental use. Is the vacation home subject to the limitation on the deductions of a personal/rental vacation home?

Correct Answer: The vacation home is not subject to the limitations on the deductions of a personal/rental vacation home. It does satisfy the rental part of the classification because it is rented for greater than 14 days. However, the personal use of 19 days does not exceed the greater of (1) 14 days or (2) 10% of the rental days, since 10% of the rental days is 20 (200 rental days × 10%) days. Therefore, the appropriate classification is primarily rental use rather than personal/rental use.

754. During the year, Martin rented his vacation home for three months and spent one month there. Gross rental income from the property was $5,000. Martin incurred the following expenses: mortgage interest, $3,000; real estate taxes, $1,500; utilities, $800; maintenance, $500; and depreciation, $4,000. Compute Martin’s allowable deductions for the vacation home.

Correct Answer: Since the vacation home is rented for 15 or more days and is used for personal purposes for more than the greater of (1) 14 days or (2) 10% of the rental days, the deductions are scaled down, using the court’s approach, as follows:

Gross income $5,000 Deduct: Taxes and interest (3/12 × (1,125) $4,500) Remainder applicable to other rental $3,875 expenses Deduct: Allocable share of utilities (97 and maintenance [3/4 × ($800 + $500)] 5) Balance applicable to depreciation $2,900 Deduct: Depreciation (3/4 × $4,000 = (2,900) $3,000) but limited to above balance Net income $ – 0–

Thus, Martin may deduct $1,125 taxes and interest, $975 utilities and maintenance, and $2,900 depreciation against the gross income of $5,000. The personal portion of taxes and interest ($3,375) is deductible as an itemized deduction. Example 29

Using the IRS’s approach, though, the deductions are as follows:

Gross income $5,000 Deduct: Taxes and interest (3/4 × (3,375) $4,500) Remainder applicable to other rental $1,625 expenses Deduct: Allocable share of utilities (975) and maintenance [3/4 × ($800 + $500)] Balance applicable to depreciation $ 650

Deduct: Depreciation (3/4 × $4,000 = (650) $3,000) but limited to above balance Net income $ – 0–

Thus, Martin may deduct $3,375 taxes and interest, $975 utilities and maintenance, and $650 depreciation against the gross income of $5,000. The personal portion of taxes and interest ($1,125) is deductible as an itemized deduction.

755. Bridgett’s son, Clyde, is $12,000 in arrears on his residential mortgage payments. Of the $12,000, $7,500 represents interest and $4,500 represents principal. a. If Bridgett pays the $12,000 to the lender, how much can she deduct? How much can Clyde deduct? b. If Bridgett pays the $7,500 of interest to the lender and loans or gives $4,500 to Clyde, who pays the $4,500 of principal, how much can Bridgett deduct? How much can Clyde deduct? c. If Bridgett gives or lends the $12,000 to Clyde who pays the lender, how much can he deduct? How much can Bridgett deduct? d. Advise Bridgett and Clyde on how the payment should be made.

Correct Answer: a. A deduction cannot be taken for paying another taxpayer’s obligation. So if Bridgett pays the lender, neither Bridgett nor Clyde could deduct the $7,500 of mortgage interest expense. b. Bridgett cannot deduct the $7,500 payment identified as interest since this represents the payment of another taxpayer’s obligation. Even though Clyde has identified the $4,500 payment as relating to the principal of the mortgage, he probably can deduct the $4,500 as mortgage interest expense since Bridgett is not allowed the deduction. In any event, Bridgett is not allowed a deduction. c. Clyde could deduct the $7,500 of mortgage interest expense, and Bridgett would receive no deduction. d. Bridgett should either loan or give the funds to Clyde who then makes the mortgage payments of $12,000 ($7,500 interest + $4,500 principal).

756. Mattie and Elmer are separated and are in the process of obtaining a divorce. They incur legal fees for their respective attorneys with the expenses being itemized as follows:

For Mattie Elmer General costs of the divorce $3,500 $3,000 Determination of dependency 1,500 –0– exemptions Property settlement tax 400 1,500 consequences $5,400 $4,500

Although there is no requirement that he do so, Elmer pays Mattie’s lawyer as a gesture of the positive feelings he still has for her. a. Determine the deductions for Mattie and for Elmer. b. Classify the deductions as for AGI and from AGI.

Correct Answer: a. Only the legal fees associated with the divorce that relate solely to tax advice are deductible. Therefore, Elmer may deduct the following:

Property settlement tax consequences $1,500

If Mattie had paid her attorney, she could have deducted the following:

Determination of dependency $1,500 exemptions Property settlement tax 400 consequences $1,900

However, since Elmer paid Mattie’s lawyer, Mattie is ineligible to take the deduction. Likewise, Elmer is ineligible to take the deduction for this $1,900 since the obligation was that of Mattie.

b. Any expenses that are deductible in this situation are classified as itemized deductions (i.e., from AGI).

757. Marvin spends the following amounts on a house he owns:

Repair to roof $1,100 Carpeting for the 1,200 living room Painting of the 4,000 exterior Replacement of 800 front door

a. How much of these expenses can Marvin deduct if the house is his principal residence? b. How much of these expenses can Marvin deduct if he rents the house to a tenant? c. Classify any deductible expenses as deductions for AGI or as deductions from AGI.

Correct Answer: a. Since these expenditures are personal expenditures, no deduction is allowed. b. Since these expenditures are for rental property, Marvin can deduct $7,100 ($1,100 + $1,200 + $4,000 + $800). c. The $7,100 deduction associated with the rental property is classified as a deduction for AGI.

758. Walter sells land with an adjusted basis of $175,000 and a fair market value of $160,000 to his mother, Shirley, for $160,000. Walter reinvests the proceeds in the stock market. Shirley holds the land for one year and a day and sells it in the marketplace for $169,000. a. Determine the tax consequences to Walter. b. Determine the tax consequences to Shirley.

Correct Answer: a. Amount realized $160,000 Adjusted basis (175,000) Realized loss ($ 15,000)

Walter’s realized loss of $15,000 is disallowed because Walter and Shirley are related parties. b. Amount realized $169,000 Adjusted basis (160,000) Realized gain $ 9,000 Walter’s disallowed loss (9,000) needed to reduce Shirley’s gain to zero Recognized gain $ – 0–

Shirley may use as much of Walter’s disallowed loss as she needs to reduce her realized gain (i.e., $9,000) to $0. Thus, Shirley’s recognized gain is $0 and the $6,000 ($15,000 – $9,000) of Walter’s disallowed loss that is not used by Shirley is permanently lost.

759. Sandra sold 500 shares of Wren Corporation to Bob, her brother, for its fair market value. She had paid $26,000 for the stock. Calculate Sandra’s and Bob’s gain or loss under the following circumstances: a. Sandra sold the shares to Bob for $20,000. One year later, Bob sold them for $18,000. b. Sandra sold the shares to Bob for $30,000. One year later, Bob sold them for $27,000. c. Sandra sold the shares to Bob for $20,000. One year later, Bob sold them for $28,000.

Correct Answer: a. Sandra has no deductible loss. Bob’s recognized loss is $2,000. b. Sandra has a recognized gain of $4,000. Bob has a recognized loss of $3,000. Related party transaction rules apply only to losses. c. Sandra has no deductible loss. Bob has a recognized gain of $2,000 ($28,000 – $20,000 = $8,000 less Sandra’s disallowed loss of $6,000).

760. The stock of Eagle, Inc. is owned as follows:

Tom 23% Tom’s uncle 22% Tom’s daughter 7% Tom’s sister 15% Tom’s spouse 15% Tom’s nephew 8% Tom’s CPA, unrelated 10%

Tom sells land and a building to Eagle, Inc. for $212,000. His adjusted basis for these assets is $225,000. Calculate Tom’s realized and recognized loss associated with the sale.

Correct Answer: Tom’s realized loss is $13,000.

Amount realized $212,000 Adjusted basis (225,000 ) Realized loss ($ 13,000 )

However, his recognized loss is $0 because the loss is disallowed as a § 267 related party transaction.

A related party includes a corporation more than 50% (directly or indirectly) owned by the taxpayer. Tom’s total ownership (i.e., both direct and constructive) of Eagle, Inc. is 60%.

Tom 23% Tom’s daughter 7% Tom’s sister 15% Tom’s spouse 15% 60%

Tom’s uncle, nephew, and the CPA are not related parties for § 267 purposes.

761. Tracy invested in the following stocks and bonds during 2013.

Blue, Inc. $25,000 City of Falcon bonds 75,000

To finance the investments, she borrowed $100,000 from Swan Bank. Interest expense paid on the loan during 2013 was $5,000. During 2013, Tracy received $1,250 of dividend income from Blue, Inc. and $3,000 of interest income on the municipal bonds. a. Determine the amount of Tracy’s gross income. b. Determine the maximum amount of Tracy’s deductible interest expense.

Correct Answer: a. Tracy must include the $1,250 of dividend income in her gross income. The interest on the municipal bonds of $3,000 is tax-exempt. b. Tracy can deduct the interest paid of $1,250 ($5,000 × 1/4) on the portion of the loan that relates to the Blue, Inc. stock. The interest paid of $3,750 on the portion of the loan that relates to the municipal bonds is disallowed because the interest income from the bonds is tax-exempt.

762. Trade of business expenses are classified as deductions for AGI. Section 212 expenses, barring certain exceptions, are classified as deductions from AGI. What are these exceptions?

Correct Answer: The normal classification for § 212 expenses is as a deduction from AGI. However, expenses paid in connection with the determination, collection, or refund of taxes related to the income of sole proprietorships, rents and royalties, or farming operations are classified as deductions for AGI. In addition, other rental and royalty expenses are classified as deductions for AGI.

763. Are all personal expenses disallowed as deductions?

Correct Answer: No. Selected personal expenses can be deducted as itemized deductions. The following are examples of deductible personal expenses.

• Contributions to qualified charitable organizations (not to exceed a specified percentage of AGI).

• Medical expenses (in excess of 7.5% of AGI).

• Certain state and local taxes (e.g., real estate taxes and state and local income or sales taxes).

• Personal casualty losses (in excess of an aggregate floor of 10% of AGI and a $100 floor per casualty).

• Certain personal interest (e.g., mortgage interest on personal residence).

764. Under what circumstance can a bribe be deducted?

Correct Answer: Bribes paid to a domestic official are disallowed if the bribe is illegal under the laws of the United States. However, a bribe paid to a foreign official is disallowed only if it is unlawful under the Foreign Corrupt Practices Act of 1977.

765. Can a trade or business expense be deductible if it is necessary but not ordinary?

Correct Answer: No. To be deductible as a trade or business expense, the expense must be both ordinary and necessary.

766. Salaries are considered an ordinary and necessary expense of a trade or business if they meet what other requirement? What are the tax consequences if this requirement is not met?

Correct Answer: “Reasonableness” is an additional requirement that applies to salaries. Generally, the unreasonable expense is disallowed as a deduction to the corporation and taxable as a dividend, rather than as salary, to the shareholder.

767. If part of a shareholder/employee’s salary is classified as unreasonable, determine the effect on the: a. Shareholder/employee’s gross income. b. Corporation’s taxable income.

Correct Answer: a. The reclassification of part of a shareholder/employee’s salary as unreasonable will have no effect on the shareholder/employee’s gross income. That is, the shareholder/employee’s salary income will decrease by the same amount as his dividend income increases. Note that if the dividends are qualified dividends, they are eligible for the same preferential tax rate of 15%/0% applicable to long-term capital gains. b. Salaries are deductible in calculating corporate taxable income, whereas dividends are not. So, the taxable income of the corporation will increase due to a reduced salary deduction.

768. What losses are deductible by an individual taxpayer?

Correct Answer: Generally deductible losses of individual taxpayers are limited to (1) those incurred in a trade or business or (2) in a transaction entered into for profit. However, individuals are also allowed to deduct losses that are the result of a casualty (subject to certain statutory materiality limitations).

769. Bruce owns several sole proprietorships. Must Bruce use the same accounting method for each of these businesses?

Correct Answer: No. If a taxpayer owns multiple businesses, it may be possible to use the cash method for some and the accrual method for others.

770. Max opened his dental practice (a sole proprietorship) in March 2013. At the end of the year, he has unpaid accounts receivable of $62,000 and no unpaid accounts payable. Should Max use the accrual method or the cash method for his dental practice?

Correct Answer: A service provider generally should use the cash method. Under the cash method, Max records income from his dental practice only as he collects from his patients and/or their insurance companies. Max has income from the uncollected accounts receivable only as he receives payment. Note that since his accounts payable can only be deducted when paid under the cash method, he should continue to minimize the accounts payable balance at the end of the tax year.

771. Discuss the application of the “one-year rule” on prepayments by a cash basis taxpayer.

Correct Answer: The Regulations set forth the general rule that an expenditure that creates an asset having a useful life that extends substantially beyond the end of the tax year must be capitalized. However, under the “one- year rule” on prepayments for cash basis taxpayers, the prepayment can be expensed in the current tax year if the asset will expire or be consumed by the end of the tax year following the year of payment. Otherwise, the taxpayer must capitalize the prepayment and deduct it over the benefit period.

772. Briefly discuss the two tests that an accrual basis taxpayer must apply before an expense can be deducted.

Correct Answer: The two tests that an accrual basis taxpayer must apply before an expense can be deducted are (1) the all events test and (2) the economic performance test. The all events test provides that a deduction cannot be claimed until all the events that create the taxpayers liability have occurred and that the amount of the liability can be determined with reasonable accuracy. The economic performance test provides that the service, property, or use of property giving rise to the liability must have been performed for, provided to, or used by the taxpayer.

773. Graham, a CPA, has submitted a proposal to do the annual audit for a municipality. Owen, the city treasurer, tells Graham that for a $1,000 fee, he will use his influence to have the audit awarded to Graham. What factors are relevant in determining if Graham can deduct the $1,000 payment assuming he pays the fee to Owen?

Correct Answer: The payment from Graham to Owen appears to be a bribe. To be disallowed, the bribe must be illegal under either Federal or state law and also must subject the payer to a criminal penalty or the loss of license or privilege to engage in a trade or business. For a bribe that is illegal under state law, a deduction is denied only if the state law is generally enforced.

774. How can an individual’s consultation with a lawyer be classified as a deduction for AGI in some cases and a deduction from AGI in other instances?

Correct Answer: Legal expenses are deductible when they are directly related to a trade or business (for AGI); an income-producing activity (either for AGI or from AGI); or the determination, collection, or refund of a tax (either for AGI or from AGI). Ordinary and necessary legal expenses incurred in conjunction with a trade/business or in conjunction with rental/royalty property are deductible for AGI. All other deductible legal expenses are deductions from AGI.

775. If a taxpayer operated an illegal business (not drug trafficking), what expenses can be deducted and what expenses are disallowed?

Correct Answer: The usual expenses of operating a business are deductible. However, the following expenses are disallowed.

• Fines.

• Bribes to public officials.

• Illegal kickbacks.

• Other illegal payments.

776. Bobby operates a drug trafficking business. Because he has an accounting background, he keeps detailed financial records. What expenses can Bobby deduct on his Federal income tax return?

Correct Answer: Bobby cannot deduct any of the expenses associated with operating his illegal drug trafficking business. However, gross income for tax purposes is defined as sales minus cost of goods sold. So in calculating the net income of the business for tax purposes, cost of goods sold is treated as a negative income item rather than as an expense.

777. Abner contributes $2,000 to the campaign of the Tea Party candidate for governor, $1,000 to the campaign of the Tea Party candidate for senator, and $500 to the campaign of the Tea Party candidate for mayor. Can Abner deduct these political contributions?

Correct Answer: No. Political contributions cannot be deducted.

778. Are there any circumstances under which lobbying expenditures are deductible?

Correct Answer: Yes. Lobbying expenditures are deductible under the following three circumstances.

• Influencing local legislation.

• Activities devoted solely to monitoring legislation.

• De minimis provision for annual in-house expenditures (lobbying expenses other than those paid to professional lobbyists) if such expenditures do not exceed $2,000.

779. In applying the $1 million limit on deducting executive compensation, what corporations are subject to the deduction limit? What executives are covered?

Correct Answer: The $1 million limit on deducting the compensation of a covered executive applies to corporations that have at least one class of stock registered under the Securities Exchange Act of 1934. Covered employees include the chief executive officer and the four other most highly compensated officers.

780. Under what circumstances may a taxpayer deduct the expenses of investigating a possible business acquisition, if (1) the business is not acquired; and (2) the business is acquired?

Correct Answer: (1) The expenses of investigation may be deducted if the taxpayer is in the same or similar business to that being investigated, even if the business is not acquired. If the taxpayer is not in the same or similar trade or business to the one being investigated, the investigation expenses are nondeductible if the business is not acquired.

(2) The expenses of investigation must be capitalized by a taxpayer not in a similar business when the business is acquired. Such expenses may be immediately expensed (up to $5,000 if such expenses do not exceed $50,000) and the balance amortized over a 180-month minimum period. If the taxpayer is in the same or similar trade or business as that acquired, investigation expenses are currently deductible.

781. What are the relevant factors to be considered in determining whether an activity is profit-seeking or a hobby?

Correct Answer: The nine relevant factors detailed in Reg. § 1.183-2(b) are as follows:

(1) Whether the activity is conducted in a businesslike manner.

(2) The expertise of the taxpayers or their advisers.

(3) he time and effort expended.

(4) The expectation that the assets of the activity will appreciate in value.

(5) The previous success of the taxpayer in the conduct of similar activities.

(6) The history of income and losses from the activity.

(7) The relationship of profits earned to losses incurred.

(8) The financial status of the taxpayer.

(9) Elements of personal pleasure or recreation in the activity.

782. In distinguishing whether an activity is a hobby or a trade or business, discuss the presumptive rule.

Correct Answer: The Code provides a rebuttable presumption that an activity is profit- seeking (i.e., a trade or business) rather than a hobby if the activity shows a profit in at least three of any five (two out of seven for horses) prior consecutive years. If this test is met, the activity is presumed to be a trade or business. The burden of proof thus shifts to the IRS to show otherwise.

783. Assuming an activity is deemed to be a hobby, discuss the order and limits in which expenses must be deducted.

Correct Answer: Amounts deductible under other Code sections without regard to the nature of the activity (e.g., property taxes and mortgage interest) must be deducted first.

Amounts deductible under other Code sections had the activity been profit-seeking which do not affect adjusted basis are deducted next.

Deductions affecting adjusted basis (e.g., depreciation) are taken next. At any point where the expenses exceed income, the deduction is limited to the remaining income.

784. Describe the circumstances under which a taxpayer can receive rent income from a personal residence, but does not have to report it as gross income.

Correct Answer: If the personal residence is rented for fewer than 15 days in a year, the rent income is excluded from gross income. Only mortgage interest and real estate taxes can be deducted.

785. For a vacation home to be classified in the primarily rental use category, what attributes must be present?

Correct Answer: To be classified in the primarily rental use category, the following attributes must be present.

• The residence is rented for 15 days or more during the year. • The residence is not used for personal purposes for more than the greater of: • 14 days • 10 percent of the total days rented.

786. For a vacation home to be classified in the personal/rental use category, what attributes must be present?

Correct Answer: To be classified in the personal/rental use category, the following attributes must be present.

• The residence is rented for 15 days or more in a year.

• The residence is used for personal purposes for more than the greater of (1) 14 days or (2) 10% of the total days rented.

787. What is the appropriate tax treatment for expenditures paid by a taxpayer for another’s benefit?

Correct Answer: To be deductible, an expense must be incurred for the taxpayer’s benefit or arise from the taxpayer’s obligation. An individual cannot claim a tax deduction for the payment of the expenses of another individual. One exception to this rule is the payment of medical expenses for a dependent. Such medical expenses are deductible by the payor subject to the normal rules that govern the deductibility of medical expenses.

788. Are there any exceptions to the rule that personal expenditures cannot be deducted?

Correct Answer: Generally personal expenditures cannot be deducted. However, the Code provides that for a personal expenditure to be deductible the taxpayer must be able to identify a particular section of the Code that permits the deduction (e.g., charitable contributions, medical expenses, certain taxes, certain interest).

789. Briefly discuss the disallowance of deductions for capital expenditures.

Correct Answer: Any expenditures that add to the value or prolong the life of property or adapt the property to a new or different use are capital expenditures which must be capitalized and depreciated or amortized.

790. Why are there restrictions on the recognition of gains and losses resulting from transactions between related parties?

Correct Answer: Sham transactions can be structured between related parties such that no real economic change occurs in the status of the parties, but a tax savings results. This is an abuse of the tax law which has resulted in restrictions on the recognition of such transactions.

791. In a related party transaction where realized loss is disallowed, when can the disallowed loss be used by the buyer on the subsequent sale of the property? In the case of a related party disallowed loss transaction, can the related party seller’s disallowed loss be used by a taxpayer other than the related party buyer?

Correct Answer: The related party buyer is permitted to use as much of the disallowed loss of the seller as is needed to reduce any realized gain on the subsequent sale of the property. If the property in the hands of the buyer appreciates to at least the amount of the seller’s adjusted basis at the date of the original sale, all of the disallowed loss can be used by the buyer on the subsequent sale. The related party seller’s disallowed loss can be used only by the related party buyer.

792. Olive, Inc., an accrual method taxpayer, is a corporation that is equally owned by Maurice and Alex, who are brothers. The corporation uses the accrual method of accounting and the shareholders use the cash method. To provide Olive with funds to acquire additional working capital, the shareholders each loan Olive $100,000 with a 6% interest rate. At the end of the tax year, there is unpaid accrued interest of $3,000 due to each shareholder. From a timing perspective, when should Olive deduct this $6,000 and when should Maurice and Alex include the $3,000 in gross income? Olive pays the $3,000 to each shareholder early next year.

Correct Answer: Maurice and Alex are related parties with Olive. So Olive (accrual method) must claim the deduction of $6,000 in the same tax year that the cash method shareholders include the $3,000 each in gross income (next year). Note that this matching provision applies only if the payor uses the accrual method and the payee uses the cash method.

793. Briefly explain why interest on money borrowed to buy tax-exempt municipal bonds is disallowed as a deduction.

Correct Answer: Because the interest income on municipal bonds is excludible from gross income, the related expense should not be deductible. Otherwise, a taxpayer could borrow money, at say 10%, invest the funds in tax-exempt securities, at say 8%, and realize a profit if the interest expense were deductible. The entire profit would be derived from the tax treatment.

794. James is in the business of debt collection. He purchased a $20,000 account receivable from Green Corporation for $15,000. During the year, James collected $17,000 in final settlement of the account. James can take a $2,000 bad debt deduction in the current year.

a. True *b. False

795. If a business debt previously deducted as partially worthless becomes totally worthless this year, only the amount not previously deducted can be deducted this year.

*a. True b. False

796. Last year, taxpayer had a $10,000 nonbusiness bad debt. Taxpayer also had an $8,000 short-term capital gain and taxable income of $35,000. If taxpayer collects the entire $10,000 during the current year, $8,000 needs to be included in gross income.

a. True *b. False

797. A cash basis taxpayer must include as income the proceeds from the sale of an account receivable to a collection agency.

*a. True b. False

798. If an account receivable written off during a prior year is subsequently collected during the current year, the amount collected must be included in the gross income of the current year to the extent it created a tax benefit in the prior year.

*a. True b. False

799. A nonbusiness bad debt deduction can be taken any year after the debt becomes totally worthless.

a. True *b. False

800. A business bad debt is a debt unrelated to the taxpayer’s trade or business either when it was created or when it became worthless.

*a. True b. False

801. In determining whether a debt is a business or nonbusiness bad debt, the debtor’s use of the borrowed funds is important.

a. True *b. False

802. A corporation which makes a loan to a shareholder can have a nonbusiness bad debt deduction.

a. True *b. False

803. A nonbusiness bad debt can offset an unlimited amount of long-term capital gain.

*a. True b. False

804. The amount of partial worthlessness on a nonbusiness bad debt is deducted in the year partial worthlessness is determined.

a. True *b. False

805. A bona fide debt cannot arise on a loan between father and son.

a. True *b. False

806. A bond held by an investor that is uncollectible will be treated as a worthless security and hence, produce a capital loss.

*a. True b. False

807. A loss from a worthless security is always treated as a short-term capital loss.

a. True *b. False

808. A loss is not allowed for a security that declines in value.

*a. True b. False

809. Several years ago, John purchased 2,000 shares of Red Corporation § 1244 stock from Mark for $40,000. Last year, John sold one-half of his Red Corporation stock to Mike for $12,000. During the current year, John sold the remaining Red Corporation stock for $3,000. John has a $17,000 ($3,000 – $20,000) ordinary loss for the current year.

a. True *b. False

810. If a taxpayer sells their § 1244 stock at a loss, all of the loss will be ordinary loss.

a. True *b. False

811. Al, who is single, has a gain of $40,000 on the sale of § 1244 stock (small business stock) and a loss of $80,000 on the sale of § 1244 stock. As a result, Al has a $40,000 ordinary loss.

a. True *b. False

812. An individual may deduct a loss on rental property even if it does not meet the definition of a casualty loss.

*a. True b. False

813. “Other casualty” means casualties similar to those associated with fires, storms, or shipwrecks.

*a. True b. False

814. A father cannot claim a loss on his daughter’s rental use property.

*a. True b. False

815. A personal casualty loss deduction may be allowed for losses resulting from termites.

*a. True b. False

816. If the amount of the insurance recovery for a theft of business property is greater than the asset’s fair market value but less than it’s adjusted basis, a gain is recognized.

a. True *b. False

817. A theft loss is taken in the year of the theft.

a. True *b. False

818. Last year, Amos had AGI of $50,000. Amos also had a diamond ring stolen which cost $20,000 and was worth $17,000 at the time of the theft. He itemized deductions on last year’s tax return. In the current year, Amos recovered $17,000 from the insurance company. Therefore, he must include $11,900 in gross income on the tax return for the current year.

*a. True b. False

819. If investment property is stolen, the amount of the loss is the adjusted basis of the property at the time of the theft reduced by $100 and 10% of AGI.

a. True *b. False

820. The cost of repairs to damaged property is not an acceptable measure of the loss in value of the property.

a. True *b. False

821. Taxpayer’s home was destroyed by a storm in the current year and the area was declared a disaster area. If the taxpayer elects to treat the loss as having occurred in the prior year, it will be subject to the 10%-of-AGI reduction based on the AGI of the current year.

a. True *b. False

822. The amount of loss for partial destruction of business property is the decline in fair market value of the business property.

a. True *b. False

823. If personal casualty gains exceed personal casualty losses (after deducting the $100 floor), there is no itemized deduction.

*a. True b. False

824. The amount of a loss on insured personal use property is reduced by the insurance coverage if no claim is made against the insurer.

*a. True b. False

825. Losses on rental property are classified as deductions for AGI.

*a. True b. False

826. When a nonbusiness casualty loss is spread between two taxable years, the loss in the second year is reduced by 10% of adjusted gross income for the first year.

a. True *b. False

827. A theft loss of investment property is an itemized deduction not subject to the 2%-of-AGI floor.

*a. True b. False

828. Research and experimental expenditures do not include the cost of consumer surveys.

*a. True b. False

829. The cost of depreciable property is not a research and experimental expenditure.

*a. True b. False

830. If an election is made to defer deduction of research expenditures, the amortization period is based on the expected life of the research project if less than 60 months.

a. True *b. False

831. For tax years beginning in 2013, the domestic production activities deduction (DPAD) for a sole proprietor is calculated by multiplying 9% times adjusted gross income.

a. True *b. False

832. If qualified production activities income (QPAI) cannot be used in the calculation of the domestic production activities deduction in 2013 because of the taxable income limitation, the product of the amount not allowed multiplied by 9% can be carried over for 5 years.

a. True *b. False

833. The limit for the domestic production activities deduction (DPAD) uses all W-2 wages paid to employees by the taxpayer during the tax year.

a. True *b. False

834. A reimbursed employee business expense cannot create an NOL for an individual.

*a. True b. False

835. A taxpayer can carry any NOL incurred forward 20 years.

*a. True b. False

836. A farming NOL may be carried back 2 years.

*a. True b. False

837. The amount of a farming loss cannot exceed the amount of the taxpayer’s NOL for the taxable year.

*a. True b. False

838. Nonbusiness income for net operating loss purposes includes dividends received.

*a. True b. False

839. A theft of investment property can create or increase a net operating loss for an individual.

*a. True b. False

840. An NOL carryforward is used in determining the current year’s charitable contribution deduction.

*a. True b. False

841. The excess of nonbusiness capital gains over nonbusiness capital losses must be added to taxable income to compute the net operating loss of an individual.

a. True *b. False

842. An individual taxpayer who does not itemize deductions uses the standard deduction to compute the excess of nonbusiness deductions over the sum of nonbusiness income and net nonbusiness capital gains for purposes of computing net operating loss.

*a. True b. False

843. When a net operating loss is carried back to a non-loss year, the net operating loss can affect the medical expense deduction of the carryback year.

*a. True b. False

844. Peggy is in the business of factoring accounts receivable. Last year, she purchased a $30,000 account receivable for $25,000. This year, the account was settled for $25,000. How much loss can Peggy deduct and in which year?

a. $5,000 for the current year. b. $5,000 for the prior year and $5,000 for the current year. c. $5,000 for the prior year. d. $10,000 for the current year. *e. None of the above.

845. Jed is an electrician. Jed and his wife are accrual basis taxpayers and file a joint return. Jed wired a new house for Alison and billed her $15,000. Alison paid Jed $10,000 and refused to pay the remainder of the bill, claiming the fee to be exorbitant. Jed took Alison to Small Claims Court for the unpaid amount and was awarded a $2,000 judgement. Jed was able to collect the judgement but not the remainder of the bill from Alison. What amount of loss may Jed deduct in the current year?

a. $0. b. $2,000. *c. $3,000. d. $5,000. e. None of the above.

846. On June 2, 2012, Fred’s TV Sales sold Mark a large HD TV, on account, for $12,000. Fred’s TV Sales uses the accrual method. In 2013, when the balance on the account was $8,000, Mark filed for bankruptcy. Fred was notified that he could not expect to receive any of the amount owed to him. In 2014, final settlement was made and Fred received $1,000. How much bad debt loss can Fred deduct in 2014?

*a. $0. b. $7,000. c. $8,000. d. $12,000. e. None of the above.

847. Mary incurred a $20,000 nonbusiness bad debt last year. She also had an $8,000 long-term capital gain last year. Her taxable income for last year was an NOL of $15,000. During the current year, she unexpectedly collected $12,000 on the debt. How should Mary account for the collection?

a. $0 income. *b. $8,000 income. c. $11,000 income. d. $12,000 income. e. None of the above.

848. Last year, Lucy purchased a $100,000 account receivable for $90,000. During the current year, Lucy collected $97,000 on the account. What are the tax consequences to Lucy associated with the collection of the account receivable? No subsequent collections are expected.

a. $0. b. $2,000 gain. c. $3,000 loss. d. $13,000 loss. *e. None of the above.

849. Two years ago, Gina loaned Tom $50,000. Tom signed a note the terms of which called for monthly payments of $2,000 plus 6% interest on the outstanding balance. Last year, when the balance owing on the loan was $18,000, Tom defaulted on the note. As of the end of last year, there appeared to be no reasonable prospect of Gina recovering the $18,000. As a consequence, Gina claimed the $18,000 as a nonbusiness bad debt. Last year, Gina had AGI of a negative $6,000 which included $5,000 net long-term capital gains and $4,000 of qualified dividends. Gina did not itemize her deductions. During the current year, Tom paid Gina $13,000 in final settlement of the loan. How should Gina account for the payment in the current year?

a. File an amended tax return for last year. b. Report no income for the current year. c. Report $8,000 of income for the current year. d. Report $12,000 of income for the current year. *e. None of the above.

850. Five years ago, Tom loaned his son John $20,000 to start a business. A note was executed with an interest rate of 8%, which is the Federal rate. The note required monthly payments of the interest with the $20,000 due at the end of ten years. John always made the interest payments until last year. During the current year, John notified his father that he was bankrupt and would not be able to repay the $20,000 or the accrued interest of $1,800. Tom is an accrual basis taxpayer whose only income is salary and interest income. The proper treatment for the nonpayment of the note is:

a. No deduction. *b. $3,000 deduction. c. $20,000 deduction. d. $21,800 deduction. e. None of the above.

851. Three years ago, Sharon loaned her sister $30,000 to buy a car. A note was issued for the loan with the provision for monthly payments of principal and interest. Last year, Sharon purchased a car from the same dealer, Hank’s Auto. As partial payment for the car, the dealer accepted the note from Sharon’s sister. At the time Sharon purchased the car, the note had a balance of $18,000. During the current year, Sharon’s sister died. Hank’s Auto was notified that no further payments on the note would be received. At the time of the notification, the note had a balance due of $15,500. What is the amount of loss, with respect to the note, that Hank’s Auto may claim on the current year tax return?

a. $0. b. $3,000. *c. $15,500. d. $18,000. e. None of the above.

852. On September 3, 2012, Able, a single individual, purchased § 1244 stock in Red Corporation from his friend Al for $60,000. On December 31, 2012, the stock was worth $85,000. On August 15, 2013, Able was notified that the stock was worthless. How should Able report this item on his 2013 tax return?

a. $85,000 capital loss. b. $85,000 ordinary loss. c. $50,000 ordinary loss and $35,000 capital loss. d. $60,000 ordinary loss. *e. None of the above.

853. On February 20, 2012, Bill purchased stock in Pink Corporation (the stock is not small business stock) for $1,000. On May 1, 2013, the stock became worthless. During 2013, Bill also had an $8,000 loss on § 1244 small business stock purchased two years ago, a $9,000 loss on a nonbusiness bad debt, and a $5,000 long-term capital gain. How should Bill treat these items on his 2013 tax return?

a. $4,000 long-term capital loss and $9,000 short-term capital loss. b. $4,000 long-term capital loss and $3,000 short-term capital loss. *c. $8,000 ordinary loss and $3,000 short-term capital loss. d. $8,000 ordinary loss and $5,000 short-term capital loss. e. $8,000 long-term capital loss and $6,000 short-term capital loss.

854. John files a return as a single taxpayer. In 2013, he had the following items:

• Salary of $40,000. • Loss of $65,000 on the sale of § 1244 stock acquired two years ago. • Interest income of $6,000.

Determine John’s AGI for 2013.

a. ($5,000). *b. $0. c. $45,000. d. $51,000. e. None of the above.

855. Bruce, who is single, had the following items for the current year:

• Salary of $80,000. • Gain of $20,000 on the sale of § 1244 stock acquired two years earlier. • Loss of $75,000 on the sale of § 1244 stock acquired three years earlier. • Worthless stock of $15,000. The stock was acquired on February 1 of the prior year and became worthless on January 15 of the current year.

Determine Bruce’s AGI for the current year.

*a. $27,000. b. $38,000. c. $42,000. d. $47,000. e. None of the above.

856. On July 20, 2011, Matt (who files a joint return) purchased 3,000 shares of Orange Corporation stock (the stock is § 1244 small business stock) for $24,000. On November 10, 2012, Matt purchased an additional 1,000 shares of Orange Corporation stock from a friend for $150,000. On September 15, 2013, Matt sold the 4,000 shares of stock for $120,000. How should Matt treat the sale of the stock on his 2013 return?

a. $54,000 ordinary loss. b. $100,000 ordinary loss; $46,000 net capital gain. c. $100,000 ordinary loss; $20,000 STCL. d. $130,000 ordinary loss; $66,000 LTCG. *e. None of the above.

857. Which of the following events would produce a deductible loss?

a. Erosion of personal use land due to rain or wind. b. Termite infestation of a personal residence over a several year period. c. Damages to personal automobile resulting from a taxpayer’s willful negligence. *d. A misplaced diamond ring. e. None of the above.

858. In 2013, Wally had the following insured personal casualty losses (arising from one casualty). Wally also had $42,000 AGI for the year before considering the casualty.

Fair Market Insurance Value Asset Adjusted Before After Recovery Basis A $9,200 $8,000 $1,000 $2,000 B 3,000 4,000 4,000 –0– C 3,700 1,700 9 –0– 00

Wally’s casualty loss deduction is:

a. $1,500. b. $1,600. c. $4,800. d. $58,000. *e. None of the above.

859. Jim had a car accident in 2013 in which his car was completely destroyed. At the time of the accident, the car had a fair market value of $30,000 and an adjusted basis of $40,000. Jim used the car 100% of the time for business use. Jim received an insurance recovery of 70% of the value of the car at the time of the accident. If Jim’s AGI for the year is $60,000, determine his deductible loss on the car.

a. $900. b. $2,900. c. $3,000. d. $9,000. *e. None of the above.

860. Norm’s car, which he uses 100% for personal purposes, was completely destroyed in an accident in 2013. The car’s adjusted basis at the time of the accident was $13,000. Its fair market value was $10,000. The car was covered by a $2,000 deductible insurance policy. Norm did not file a claim against the insurance policy because of a fear that reporting the accident would result in a substantial increase in his insurance rates. His adjusted gross income was $14,000 (before considering the loss). What is Norm’s deductible loss?

a. $0. b. $100. *c. $500. d. $9,500. e. None of the above.

861. In 2013, Grant’s personal residence was completely destroyed by fire. Grant was insured for 100% of his actual loss, and he received the insurance settlement. Grant had adjusted gross income, before considering the casualty item, of $30,000. Pertinent data with respect to the residence follows:

Cost basis $280,000 Value before casualty 250,000 Value after casualty –0–

What is Grant’s allowable casualty loss deduction?

*a. $0. b. $6,500. c. $6,900. d. $10,000. e. $80,000.

862. John had adjusted gross income of $60,000. During the year his personal use summer home was damaged by a fire. Pertinent data with respect to the home follows:

Cost basis $260,000 Value before the fire 400,000 Value after the fire 100,000 Insurance recovery 270,000

John had an accident with his personal use car. As a result of the accident, John was cited with reckless driving and willful negligence. Pertinent data with respect to the car follows:

Cost basis $80,000 Value before the accident 56,000 Value after the accident 20,000 Insurance recovery 18,000

What is John’s itemized casualty loss deduction?

*a. $0. b. $2,000. c. $17,000. d. $18,000. e. None of the above.

863. In 2013, Mary had the following items:

Salary $30,000 Personal use casualty gain 10,000 Personal use casualty loss (after 17,000 $100 floor) Other itemized deductions 4,000

Assuming that Mary files as head of household (has one dependent child), determine her taxable income for the current year.

*a. $13,250. b. $14,100. c. $14,300. d. $24,300. e. None of the above.

864. In 2013, Morley, a single taxpayer, had an AGI of $30,000 before considering the following items:

Loss from damage to rental property ($6,000) Loss from theft of bonds (3,000) Personal casualty gain 4,000 Personal casualty loss (after $100 (9,000) floor)

Determine the amount of Morley’s itemized deduction from the losses.

a. $0. b. $2,900. c. $5,120. *d. $5,600. e. None of the above.

865. In 2013, Theo, an employee, had a salary of $30,000 and experienced the following losses:

Loss from damage to rental property ($10,000) Unreimbursed loss from theft of (5,000) business computer Personal casualty gain 4,000 Personal casualty loss (after $100 (9,000) floor)

Determine the amount of Theo’s itemized deduction from these losses.

a. $0. b. $2,800. c. $2,900. d. $4,580. *e. None of the above.

866. Alicia was involved in an automobile accident in 2013. Her car was used 60% for business and 40% for personal use. The car had originally cost $40,000. At the time of the accident, the car was worth $20,000 and Alicia had taken $8,000 of depreciation. The car was totally destroyed and Alicia had let her car insurance expire. If Alicia’s AGI is $50,000 (before considering the loss), determine her itemized deduction for the casualty loss.

*a. $4,500. b. $6,100. c. $8,000. d. $24,000. e. None of the above.

867. Last year, Sarah (who files as single) had silverware worth $10,000 (basis $6,000) stolen from her home. Sarah’s insurance company told her that her policy did not cover the theft. Sarah’s other itemized deductions last year were $2,000. She had AGI of $30,000 last year. In August of the current year, Sarah’s insurance company decided that Sarah’s policy did cover the theft of the silverware and they paid Sarah $5,000. Determine the tax treatment of the $5,000 received by Sarah during the current year.

*a. None of the $5,000 should be included in gross income. b. $2,900 should be included in gross income. c. $5,000 should be included in gross income. d. Last year’s return should be amended to include the $5,000. e. None of the above.

868. Alma is in the business of dairy farming. During the year, one of her barns was completely destroyed by fire. The adjusted basis of the barn was $90,000. The fair market value of the barn before the fire was $75,000. The barn was insured for 95% of its fair market value, and Alma recovered this amount under the insurance policy. Alma has adjusted gross income for the year of $40,000 (before considering the casualty). Determine the amount of loss she can deduct on her tax return for the current year.

a. $3,750. b. $14,650. c. $14,750. *d. $18,750. e. None of the above.

869. In 2013, Juan’s home was burglarized. Juan had the following items stolen:

• Securities worth $25,000. Juan purchased the securities four years ago for $20,000. • New tools which Juan had purchased two weeks earlier for $8,000. Juan uses the tools in making repairs at an apartment house that he owns and manages. • An antique worth $15,000. Juan inherited the antique (a family keepsake) when the property was worth $11,000.

Juan’s homeowner’s policy had a $50,000 deductible clause for thefts. If Juan’s salary for the year is $50,000, determine the amount of his itemized deductions as a result of the theft.

a. $3,100. b. $6,000. c. $26,100. d. $26,500. *e. None of the above.

870. Regarding research and experimental expenditures, which of the following are not qualified expenditures?

*a. Costs of ordinary testing of materials. b. Costs to develop a plant process. c. Costs of developing a formula. d. Depreciation on a building used for research. e. All of the above are qualified expenditures.

871. Blue Corporation incurred the following expenses in connection with the development of a new product:

Salaries $100,000 Utilities 18,000 Materials 25,000 Advertising 5,000 Market survey 3,000 Depreciation on machine 9,000

Blue expects to begin selling the product next year. If Blue elects to amortize research and experimental expenditures over 60 months, determine the amount of the deduction for research and experimental expenditures for the current year.

*a. $0. b. $118,000. c. $143,000. d. $152,000. e. $160,000.

872. Last year, Green Corporation incurred the following expenditures in the development of a new plant process:

Salaries $250,000 Materials 90,000 Utilities 20,000 Quality control testing costs 40,000 Management study costs 5,000 Depreciation of equipment 15,000

During the current year, benefits from the project began being realized in May. If Green Corporation elects a 60 month deferral and amortization period, determine the amount of the deduction for the current year.

a. $48,000. *b. $50,400. c. $54,667. d. $57,067. e. None of the above.

873. Ivory, Inc., has taxable income of $600,000 and qualified production activities income (QPAI) of $700,000 in 2013. Ivory’s domestic production activities deduction is:

a. $36,000. b. $42,000. *c. $54,000. d. $63,000. e. None of the above.

874. For the year 2013, Amber Corporation has taxable income of $880,000, alternative minimum taxable income of $600,000, and qualified production activities income (QPAI) of $640,000. The total W-2 wages paid to employees engaged in qualified domestic production activities are $116,000. Amber’s DPAD for 2013 is:

*a. $54,000. b. $57,600. c. $58,000. d. $79,200. e. None of the above.

875. Cream, Inc.’s taxable income for 2013 before any deduction for an NOL carryforward of $30,000 is $70,000. Cream’s qualified production activities income (QPAI) is $60,000. What is the amount of Cream’s domestic production activities deduction (DPAD) for 2013?

a. $1,200. b. $1,800. c. $2,400. *d. $3,600. e. None of the above.

876. In the computation of a net operating loss, which of the following items is not added to the negative taxable income?

a. Losses incurred in a transaction entered into for profit. *b. Deductible alimony payments. c. Personal theft loss. d. Losses from theft of securities. e. None of the above.

877. If a taxpayer has an NOL in 2013 of $20,000, of which $8,000 is attributable to a theft of rental use property, the taxpayer may:

a. Carry all of the NOL of $20,000 back 5 years. b. Carry all of the NOL of $20,000 back 3 years. *c. Carry $8,000 of the NOL back 3 years and the remainder of the NOL of $12,000 back 2 years. d. All of the above. e. None of the above.

878. Wu, who is single, has the following items for 2013:

Salary $25,000 Interest income 8,000 Itemized deductions ($27,000 (32,000) attributable to casualty loss)

What is Wu’s NOL for 2013?

*a. $0. b. $1,000. c. $2,000. d. $25,000. e. None of the above.

879. Khalid, who is single, had the following items for 2013:

Salary $40,000 Interest income on U.S. Treasure 8,000 bonds Loss on theft of securities (60,000) Interest income on New York state 12,000 bonds

What is Khalid’s NOL for 2013?

a. $10,000. *b. $12,000. c. $15,000. d. $25,100. e. None of the above.

880. Janice, single, had the following items for the year 2013:

Salary $30,000 Dividend income 8,000 Loss on § 1244 small business stock (45,000) held for three years Total itemized deductions (5,000)

Determine Janice’s net operating loss for the year 2013.

a. $0. b. $5,000. c. $15,000. d. $20,000. *e. None of the above.

881. Bill, age 40, is married with two dependents. Bill had the following items for the year 2013:

Bill’s business loss ($40,000) Bill’s salary 50,000 Personal casualty gains 5,000 Personal casualty loss (after $100 (23,000) floor) Other itemized deductions (15,000)

Based on the above information, what is the net operating loss for Bill and his spouse for the year 2013?

a. $0. b. $5,000. *c. $7,000. d. $12,000. e. None of the above.

882. Jack, age 30 and married with no dependents, is a self-employed individual. For 2013, his self-employed business sustained a net loss from operations of $10,000. The following additional information was obtained from his personal records for the year:

Nonbusiness long-term capital gain $ 2,0 00 Interest income 6,000 Itemized deductions—consisting of (12,000 taxes and interest )

Based on the above information, what is Jack’s net operating loss for the current year if he and his spouse file a joint return?

a. $2,000. b. $8,000. *c. $10,000. d. $11,000. e. $16,400.

883. Stella, age 38, is single with no dependents. The following information was obtained from her personal records for the current year.

Salary $30,000 Interest income 7,000 Alimony received 12,000 Individual retirement account 2,000 contribution Home mortgage interest expense 4,000 Property taxes 2,000 Personal casualty loss (after the 38,000 $100 floor) Stolen investment property 16,000 Unreimbursed employee business loss 3,000

Based on the above information, what is Stella’s net operating loss for the current year?

a. $0. b. ($9,000). *c. ($10,360). d. ($11,200). e. None of the above.

884. Ralph is single and has the following items for the current year:

Nonbusiness capital gains $ 9,000 Nonbusiness capital losses (3,000) Interest income 6,000 Itemized deductions (none of the (10,000) amount resulted from a casualty loss)

In calculating Ralph’s net operating loss, and with respect to the above amounts only, what amount must be added back to taxable income (loss)?

*a. $0. b. $2,000. c. $3,000. d. $4,000. e. None of the above.

885. Elizabeth has the following items for the current year:

Nonbusiness capital gains $ 5,000 Nonbusiness capital losses (3,000) Interest income 3,000 Itemized deductions (including a (27,000) $20,000 casualty loss)

In calculating Elizabeth’s net operating loss, and with respect to the above amounts only, what amount must be added back to taxable income (loss)?

a. $0. b. $1,000. *c. $2,000. d. $20,000. e. None of the above.

886. Steve and Holly have the following items for 2013:

Dividend income $8,000 Interest income 7,000 Itemized deductions (none of the (13,000) amount resulted from a casualty loss) Business capital gains 1,000 Business capital losses (5,000)

In calculating their net operating loss, and with respect to the above amounts only, what amount must be added back to taxable income (loss)?

a. $0. b. $1,300. *c. $2,000. d. $3,000. e. None of the above.

887. Tonya had the following items for last year:

Salary $40,000 Short-term capital gain 12,000 Nonbusiness bad debt (25,000) Long-term capital gain 8,000

For the current year, Tonya had the following items:

Salary $45,000 Collection of last year’s bad debt 25,000

Determine Tonya’s adjusted gross income for the current year.

Correct Answer: Salary $45,000 Income under tax benefit rule 23,000 Long-term capital loss carryover (2,000 ) AGI $66,000

Income on collection of nonbusiness $23,000 bad debt (classified as STCL) to the extent of tax benefit in the prior year ($20,000 offset against capital gain and $3,000 offset against ordinary income).

888. Maria, who is single, had the following items for 2013:

Salary $80,000 Loss on sale of § 1244 small (60,000) business stock acquired 3 years ago Stock acquired 2 years ago became (5,000) worthless during the year Long-term capital gain 25,000 Nonbusiness bad debt (15,000) Casualty loss on property held 6 (6,000) months Casualty gain on property held 4 4,000 years

Determine Maria’s adjusted gross income for 2013.

Correct Answer: Salary $80,000 Ordinary loss from § (50,000) 1244 stock Capital gains and losses Long-term capital gain $29,000 ($25,000 + $4,000) Less: Long-term capital loss [($60,000 – (15,000) $50,000) + $5,000] Net long-term capital gain $14,000 Less: Short-term capital loss (19,000) ($15,000 + $4,000*) Net capital loss (limited to (5,000) (3,000) $3,000) Adjusted gross $27,000 income

*Casualty losses to the extent of casualty gains.

889. Mike, single, age 31, had the following items for 2013:

Salary $50,000 Nonbusiness bad debt (6,000) Casualties—independent events Asset A (personal use property 3,000 held for two years)—gain Securities (stolen)—loss (8,000) Dividends 2,000 Interest expense on personal 10,000 residence

Compute Mike’s taxable income for 2013.

Correct Answer: Salary $50,000 Dividends 2,000 Casualty gain (long-term capital $3,000 gain) Nonbusiness bad debt (short-term (6,000) capital loss) Net short-term capital (3,00 loss 0) Adjusted gross income $49,000 Less: Interest expense on personal (10,000) residence Miscellaneous itemized deduction: (8,000) casualty loss Personal exemption (3,90 0) Taxable income $27,100

890. Jose, single, had the following items for 2013:

Salary $44,000 § 1244 loss on stock acquired (70,000) 3 years ago § 1244 gain on stock acquired 26,000 10 months ago Worthless security purchased (4,000) in June of last year Nonbusiness bad debt (7,000) Interest income 8,000

Compute Jose’s adjusted gross income for 2013.

Correct Answer: Salary $44,000 Ordinary loss from § (50,000) 1244 stock Interest income 8,000 Short-term capital $26,000 gain Short-term capital (7,000) loss Net short-term capital $19,000 gain Long-term loss from § 1244 stock ($70,000 – $50,000) ($20,000) Worthless security (4,00 (24,000) 0) Net long-term capital loss ($ 5,000) Limit (2,000) Adjusted gross income $ – 0–

891. Julie, who is single, has the following items for 2013:

· Salary—$100,000.

· A hurricane completely destroyed Julie’s duplex during the current year. Julie lived in one-half of the duplex and rented out the other half. Julie paid $400,000 for the duplex and has taken $80,000 of cost recovery on the rental portion of the duplex. The duplex was worth $420,000 at the time of the destruction. Julie’s insurance policy paid her 90% of the fair market value of the duplex.

· Household items destroyed in the hurricane had a basis of $15,000 and a fair market value of $8,500. There was no insurance recovery on the household items.

· Julie purchased a painting three years ago for $4,000. At the time of the hurricane, the painting was worth $10,000. Julie purchased the painting as an investment with the intent that she would sell it when its value exceeded $12,000. There was no insurance recovery on the painting.

· Julie had an automobile accident in the current year. Julie used the car 100% for personal purposes. The car cost $37,000 and had a decline in FMV as a result of the accident of $5,000. The car was insured, but the policy had a $2,000 deductible clause. Julie chose not to file a claim for the damage.

· Julie owned a computer that she used 100% for business. The computer was also completely destroyed in the hurricane. It had a basis of $6,000 and a FMV of $4,000 at the time it was destroyed. Julie was not reimbursed by her employer for the loss on the computer.

· Home mortgage interest—$10,000.

Determine the amount of Julie’s taxable income for 2013.

Correct Answer: Salary $100,000 Plus: Gain on rental duplex Recovery [($420,000 × 90%) × 50%] $189,000 Cost (50% × 400,000) $200,000 Less: cost recovery (80,000) Adjusted basis (120,000) Casualty gain 69,000 AGI $169,000 Less: Itemized deductions Casualty loss Dwelling Basis ($400,000 × $200,000 50%) Recovery (189,000) Loss $ 11,000 Household items 8,50 0 Total loss $ 19,500 Less: $100 floor (1 $ 19,400 00) Automobile Loss $ 5,000 Recovery if claim (3,000) filed Loss $ 2,000 Less: $100 floor (1 1,900 00) Total loss $ 21,300 Less: 10% × $169,000 (AGI) (16,900) Deductible casualty loss $ 4,400 Home mortgage interest 10,000 Other miscellaneous itemized deduction— 4,000 painting Computer loss [$6,000 – (2% × $169,000)] 2, 620 Total itemized deductions (21,020) Personal exemption (3,900 ) Taxable income $144,080

892. Juanita, single and age 43, had the following items for 2013:

Salary $60,000 Interest income 6,000 Casualty loss on business property (15,000) Casualty loss on rental property (5,000) Loss on theft of securities (8,000) Personal casualty gains 9,000 Personal casualty loss (after $100 (13,000) floor) Other itemized deductions (9,000)

Compute Juanita’s taxable income for 2013.

Correct Answer: Salary $60,000 Interest income 6,000 Casualty loss on (15,000) business property Casualty loss on (5,000) rental property Personal casualty $9,000 gains Personal casualty (9,000) –0– loss AGI $46,000 Less: Itemized deductions Casualty loss (–0–) [($13,000 – $9,000) – (10% × $46,000)] Theft of (8,000) securities Other itemized (9,000) deductions Personal exemption (3,900) Taxable income $25,100

893. While Susan was on vacation during the current year, someone broke into her home and stole the following items:

· A computer used 60% in connection with Susan’s employment as an employee and 40% for her personal use. The cost of the computer was $8,000. Depreciation of $3,000 had been taken on the computer and it had a fair market value of $4,000 at the time of the theft.

· A painting, which Susan purchased as an investment for $10,000, had a fair market value of $17,000.

· Silverware purchased for $3,000 had a fair market value of $5,000.

· Cash of $30,000.

Susan’s adjusted gross income, before considering any of the above items, is $60,000. Determine the total amount of Susan’s itemized deductions resulting from the theft.

Correct Answer: Painting loss (investment property) $10,000 Casualty loss Computer ($4,000 × 40%) $ 1,60 0 Silverware 3,000 Cash 30,00 0 Total $34,600 Less: $100 floor (100) 10% × $60,000 (AGI) (6,00 28 0) Loss on nonreimbursed employee business expense—computer

Lesser of: (1) Cost ($8,000 × 60%) $4,800 Depreciation (3,000) Adjusted basis $1,800

(2) Fair market value ($4,000 × $2,400 60%) Loss $1,800 Less: 2% × $60,000 (AGI) (1,200)

Total itemized deductions $3

894. Neal, single and age 37, has the following items for 2013:

Salary $50,000 Casualty loss on business (8,000) property Casualty loss on rental property (5,000) Personal casualty gains 3,000 Personal casualty losses (after (12,000) $100 floor) Interest expense on personal (7,000) residence

Determine Neal’s taxable income for 2013.

Correct Answer: Salary $50,000 Casualty loss on (8,000) business property Casualty loss on (5,000) rental property Personal casualty $3,000 gains Personal casualty (3,000) – losses 0– AGI $37,000 Less: Itemized deductions Casualty loss $9,000 ($12,000 – $3,000) Less: 10% × (3,700) $37,000 (AGI) Casualty loss $5,300 deduction Interest 7,000 (12,300) expense Personal (3,900) exemption Taxable income $20,800

895. Gary, who is an employee of Red Corporation, has the following items for 2013:

Salary $80,000 Personal casualty gain 7,000 Personal casualty loss from one 15,000 event (before the $100 floor) Loss on rental property 6,000 Theft of bearer bonds 18,000 Unreimbursed loss from theft of a 4,000 computer used 100% for business

Determine Gary’s AGI and total amount of itemized deductions for 2013.

Correct Answer: Salary $80,000 Loss on rental property (6,000) Personal casualty gain 7,000 Personal casualty loss (7,000) Adjusted gross income $74,000

Personal casualty loss ($15,000 – $ 8,000 $7,000) Less: $100 floor (100) 10% × $74,000 (7,400) (AGI) $ 500

Theft of bearer bonds 18,000 Theft of computer [$4,000 – (.02 × 2,52 $74,000)] 0 Total itemized deductions $21,020

896. Susan has the following items for 2013: · Loss on rental property caused by termites—$150,000. Insurance covered 80% of the loss.

· Loss on personal use automobile—$10,000. The insurance policy does not cover the first $3,000 of loss. Susan decided not to file a claim for the loss.

· Loss on a painting stolen from Susan’s house. Susan purchased the painting three years ago as an investment. She paid $40,000 for the painting and it was worth $35,000 at the time of the theft. The painting was insured for the fair market value.

· Salary—$40,000.

Determine Susan’s AGI and total amount of itemized deductions for 2013.

Correct Answer: Salary $40,000 Loss on rental property [(80% × (30,000) $150,000) – $150,000] Adjusted gross income $10,000

Casualty loss (limited to $3,000 not $3,000 subject to insurance) Less: $100 floor (100) 10% × $10,000 (AGI) (1,000) Casualty loss deduction $1,900 Loss on stolen painting ($40,000 – 5,000 $35,000) Total itemized deductions $6,900

897. Roger, an individual, owns a proprietorship called Green Thing. For the year 2013, Roger has the following items:

· Business income—$200,000. · Business expense—$150,000. · Loss on a completely destroyed business machine. The machine had an adjusted basis of $25,000 and a fair market value of $20,000.

· Loss on a business truck. The truck had an adjusted basis of $8,000. The repairs to fix the truck cost $10,000.

Determine Roger’s adjusted gross income for 2013.

Correct Answer: Business income $200,000 Business expense (150,000) Loss on business machine (25,000) Loss on business truck (8,0 00) Adjusted gross income $ 17,000

898. In 2012, Robin Corporation incurred the following expenditures in connection with the development of a new product:

Salaries $100,000 Supplies 40,000 Market survey 10,000 Depreciation 25,000

In 2013, Robin incurred the following additional expenditures in connection with the development of the product:

Salaries $125,000 Supplies 50,000 Depreciation 30,000 Advertising 10,000

In October 2013, Robin began receiving benefits from the project. If Robin elects to expense research and experimental expenditures, determine the amount and year of the deduction.

Correct Answer: Deductibility of research and experimental expenditures is permitted in the year of incurrence.

2012 Salaries $100,000 Supplies 40,000 Depreciation 25,00 0 Deductible expenses $165,000

The market survey is not a research and experimental expenditure.

2013 Salaries $125,000 Supplies 50,000 Depreciation 30,0 00 Deductible expenses $205,000

The advertising is not a research and experimental expenditure

899. In 2013, Tan Corporation incurred the following expenditures in connection with the development of a new product:

Salaries $ 60,000 Supplies 20,000 Depreciation on research equipment 10,000 Testing for quality control 5,000 Advertising 8,000 Overhead allocated to research 2,000

Tan began selling the product in November, 2013. If Tan elects to amortize research and experimental expenditures, determine Tan’s deduction for 2013.

Correct Answer: Salaries $60,000 Supplies 20,000 Depreciation 10,000 Overhead allocated to research 2,00 0 Total qualifying research $92,000 expenditures

[($92,000/60 months) × 2 months] = $3,067

900. Green, Inc., manufactures and sells widgets. During 2013, an examination of the company records showed the following items:

Domestic production gross $3,000,00 receipts 0 Cost of goods sold for domestic 750,000 products Expenses directly related to domestic production gross 300,000 receipts (other than wages) W-2 wages paid to employees engaged in qualified domestic 300,000 production activities Ratable portion of other 100,000 expenses Total W-2 wages 325,000 Taxable income 1,600,000

Determine Green’s domestic production activities deduction for 2013.

Correct Answer: Domestic production gross receipts $3,000,00 0 Less: Cost of goods sold (750,000) Direct expenses (300,000) W-2 wages directly (300,000) related Allocated expenses (100 ,000) Qualified production activities $1,550,00 income (QPAI) 0

Domestic production activities deduction Lessor of: QPAI × 9% ($1,550,000 × 9%) $ 139 Taxable income × 9% ,500 ($1,600,000 × 9%)

Limited to 50% of related W-2 wages $ 150 (50% × $300,000) [no limit] ,000

901. Red Company is a proprietorship owned by Sally, a single individual. Red manufactures and sells widgets. During 2013, an examination of Red’s records shows the following items:

Domestic production gross $2,500,00 receipts 0 Cost of goods sold for domestic 600,000 products Expenses directly related to 280,000 domestic production gross receipts (other than wages) Ratable portion of other 100,000 expenses Other expenses not allocated to 30,000 domestic production gross receipts W-2 wages paid to employees 270,000 engaged in qualified domestic production activities Total W-2 wages 320,000

Sally also had the following additional items: Dividends received $ 20,00 0 Interest income 10,000

Determine Sally’s domestic production activities deduction for 2013.

Correct Answer: Gross receipts $2,500,000 Less: Cost of goods sold (600,000) Expenses directly (280,000) related Other allocated (130,000) and nonallocated expenses ($100,000 + $30,000) Wages (320,000) Dividends received 20,000 Interest received 10,000 Modified adjusted gross income $1,200,000

Domestic production gross receipts $2,500,000 Less: Cost of goods sold (600,000) Direct expenses (280,000) Allocated (100,000) expenses W-2 wages (270, directly related 000) Qualified production activities $1,250,000 income (QPAI)

Lesser of: QPAI × 9% ($1,250,000) = $112,500 Mod ified AGI × 9% ($1,200,000) = $108,000 Not to exceed 50% × $270,000 = $135,000 Sally’s QPAD = $108,000

902. Nora, single, had the following income and deductions for 2013:

Sales $ 50,00 0 Business expenses (100,000 ) Alimony received 30,000 Interest income 1,000 Dividends 2,000 Nonbusiness capital gains 4,000 § 1244 stock loss (18,000) Itemized deductions (4,000) Business capital loss (2,000) Business capital gain 1,000

Compute Nora’s net operating loss for 2013.

Correct Answer: Sales $ 50,000 Business (100,000) expenses Alimony received 30,000 Interest income 1,000 Dividends 2,000 § 1244 stock (18,000) (ordinary loss) Capital gains $5,000 ($4,000 + $1,000) Less: Capital (2,000) losses Net capital 3,000 gains Adjusted gross ($ 32,000) income Standard (6,100) deduction (single) Personal (3,900) exemption (1 × $3,900) Taxable income ($ 42,000)

Taxable income ($ 42,000) Personal 3,900 exemption Excess of nonbusiness –0– capital losses over nonbusiness capital gains Excess of nonbusiness deductions over sum of nonbusiness income and net nonbusiness capital gains Standard $6,100 deduction Interest (1,000) Dividends (2,000) Alimony received (30,000) Nonbusiness (4,000) –0– capital gains Excess of business capital losses over the sum of business capital gains and – the excess of 0– nonbusiness capital gains over nonbusiness deductions {$2,000 – [$1,000 + ($1,000 + $2,000 + $30,000 + $4,000 – $6,100)]} Net operating ($ 38,100) loss

903. Juan, married and filing jointly, had the following income and deductions for 2013:

Sales $600,000 Business expenses (650,000) Interest income 3,000 Dividends 4,000 Personal casualty loss (after (25,000) deducting the $100 floor) Taxes paid on personal residence (7,000) Interest paid on personal (9,000) residence Alimony paid (18,000)

Juan has three dependent children. Calculate the net operating loss for 2013.

Correct Answer: Sales $600,000 Business expense (650,000) Interest income 3,000 Dividends 4,000 Alimony paid (18,000) Adjusted gross ($ 61,000) income Less: Itemized deductions Casualty loss $25,000 [$25,000 – (10% × $0)] Taxes on personal 7,000 residence Interest on 9,000 (41,000) personal residence Personal and (19,500) dependency exemptions (5 × $3,900) Taxable income ($121,500)

Taxable income ($121,500) Personal and dependency 19,500 exemptions Excess of nonbusiness deductions over nonbusiness income: Itemized deductions $16,000 ($41,000 – $25,000) Alimony paid 18,000 Total nonbusiness $34,000 deductions Interest income (3,000) Dividends (4,000) 27,0 00 Net operating loss ($ 75,000)

904. Jason, married and filing jointly, had the following income for 2013:

Salary $ 70,00 0 Loss on the sale of § 1244 stock (110,000 held for 5 years ) Dividends 25,000 Interest income 10,000 Itemized deductions (no casualty (12,000) losses)

Jason has four dependent children. Calculate the net operating loss for 2013.

Correct Answer: Salary $ 70,000 Ordinary loss (§ 1244 stock) (100,000) Long-term capital loss (3,000) [($110,000 – $100,000) = $10,000] limited to Dividends 25,000 Interest income 10,000 AGI $ 2,000 Itemized deductions (12,000) Personal exemptions (6 × (23,400) $3,900) Taxable income ($ 33,400)

Taxable income ($ 33,400) Excess of nonbusiness deductions over nonbusiness income [$12,000 – ($25,000 + –0– $10,000 – $3,000)] Personal exemptions (6 × 23,40 $3,900) 0 Net operating loss ($ 10,000)

905. Ruth, age 66, sustains a net operating loss (NOL) of $15,000 for 2013. Because Ruth had no taxable income in 2011, the loss is carried back to 2012. For 2012, the joint income tax return of Ruth and her husband was as follows:

Salary $20,000 Interest income 5,000 Net short-term capital loss (2,000 ) AGI $23,000 Itemized deductions Charitable contributions $4,000 Medical expenses [$2,000 275 – ($23,000 ´ .075)] Interest 5,000 Taxes 2,500 (11,775) Personal exemptions (2 ´ $3,800) (7,600 ) Taxable income $ 3,625

Calculate Ruth’s remaining 2013 NOL to be carried to 2014.

Correct Answer: Salary $20,000 Interest income 5,000 Net short-term capital loss (not (– permitted) 0–) Adjusted gross income $25,000 Itemized deductions Charitable contributions $4,000 Medical expenses [$2,000 – 125 ($25,000 × .075)] Interest 5,000 Taxes 2,500 (11,625) Personal exemptions (– 0–) Modified taxable income $13,375

NOL for 2013 ($15,000) Modified taxable income 13,375 NOL to carry forward ($ 1,625)

906. Milt, married and filing jointly, had the following items for 2013:

Sales $200,000 Business expenses 210,000 Interest income 3,000 Dividends 5,000 Salary 20,000 Alimony received 8,000 Nonbusiness long-term capital 5,000 gains Nonbusiness short-term capital 7,000 losses Business short-term capital 4,000 losses Business long-term capital 2,000 gains IRA contributions 5,000 Charitable contributions 9,000 Medical expenses 8,450 Property taxes 7,000 Casualty loss on personal 35,000 property (after the $100 floor) Loss on stolen bonds 5,000 Unreimbursed employee business 8,000 expenses

Milt has two dependent children. If Milt and his wife file a joint return, compute their net operating loss for 2013.

Correct Answer: Sales $200,000 Less: business expenses (210,000) Net business loss (10,000) Interest income 3, 000 Dividends 5, 000 Salary 20,0 00 Alimony received 8, 000 Long-term capital gains ($5,000 + $2,000) $ 7,000 Short-term capital losses ($7,000 + $4,000) (11,000) Net short-term capital losses ($ 4,000) Limit (3,000) IRA contributions (5 ,000) Adjusted gross income (AGI) $ 18,00 0 Less: Itemized deductions Charitable contributions $ 9,000 Medical expenses [$8,450 – ($18,000 6,650 × .10)] Property taxes 7,000 Casualty loss [$35,000 – ($18,000 33,200 × .10)] Theft loss on stolen bonds 5,000 Nonreimbursed business expenses [$8,000 – ($18,000 × .02)] 7,640 (68,490) Personal and dependency exemption deduction (4 (15,600) × $3,900) Taxable income ($66,090) Personal exemptions and dependency 15,600 deductions Nonbusiness capital losses in excess of nonbusiness 2,000 capital gains ($7,000 – $5,000) Nonbusiness deductions in excess of nonbusiness income IRA $ 5,000 Itemized deductions $68,490 Less: Casualty loss (33,200) Bonds theft loss (5,000) Employee business 22,650 expenses (7,640) Total nonbusiness $27,650 deductions Nonbusiness income Interest $3,000 Dividends 5,000 Alimony received 8,000 (16,000) 11,650 Business capital losses in excess of business capital 1,000 gains ($4,000 – $2,000) limited to $1,000 Net operating loss (NOL) ($35,840)

907. Identify the factors that should be considered in determining whether a transaction is a business bad debt or a nonbusiness bad debt.

Correct Answer: Factors to be considered in determining whether a transaction is a business bad debt or a nonbusiness bad debt are as follows:

• Was the debt related to the taxpayer’s business when it was created?

• Was the debt related to the taxpayer’s business when it became worthless?

• Was the lender engaged in the business of lending money?

• Was there a proximate relationship between the creation of the debt and the lender’s business?

908. Discuss the tax treatment of non-reimbursed losses of an employee in connection with a trade or business.

Correct Answer: The loss is deductible from adjusted gross income as a miscellaneous itemized deduction subject to the 2 percent-of-AGI limitation.

909. A taxpayer who sustains a casualty loss in an area designated by the President of the United States as a disaster area may take the loss in the year in which the loss occurred or elect to take the loss in the previous year. Identify factors that should be considered in deciding in which year to take the loss.

Correct Answer: Factors that should be considered include:

• The marginal tax rates of the two different years.

• The adjusted gross incomes of the two different years.

• Other casualty losses in the two different years.

• The benefits of a faster refund (or reduction of tax).

910. Discuss the treatment, including the carryback and carryforward periods, of casualty losses incurred with personal use property.

Correct Answer: These losses are subject to the $100 per event floor and the 10% of AGI limitations. The losses are itemized deductions. Casualty losses on personal use property can have a three-year carryback. The carryover period is 20 years.

911. What are the three methods of handling research and experimental expenditures incurred in a trade or business? Under what circumstances would you choose each?

Correct Answer: The following methods are permitted:

• The expense method, where the expenditures are written off immediately, is attractive where the taxpayer is currently in a high tax bracket and has sufficient other income to offset the deductions.

• Deferral and amortization of expenditures over a period of not less than 60 months is generally chosen when the total deduction is not wanted immediately because future income is expected to be available to offset the deduction.

• The capitalization method allows no deduction until the project is abandoned or becomes worthless. Usually taxpayers do not choose this method, since the tax benefit is deferred for an indefinite period.

912. Why was the domestic production activities deduction (DPAD) enacted by Congress?

Correct Answer: The American Jobs Creation Act of 2004 provision creating DPADs was enacted to replace certain tax provisions that our world trading partners regarded as allowing unfair advantage to U.S. exports. Note, however, in no way is the DPAD limited to exports.

913. How is qualified production activities income (QPAI) calculated?

Correct Answer: QPAI is calculated as follows:

The excess of domestic production gross receipts (DPGR) over the sum of:

• Cost of goods sold allocated to such receipts.

• Other deductions, expenses, or losses directly allocated to such receipts.

• The ratable portion of deductions, expenses, and losses not directly allocable to such receipts or another class of income.

914. Sally is an employee of Blue Corporation. Last year, she purchased a very expensive computer with her own funds. She used the computer 100% for business purposes. During the current year, the computer was completely destroyed in a fire. Blue Corporation did not reimburse her for her loss. Discuss whether Sally’s loss will create or increase Sally’s net operating loss.

Correct Answer: The loss is incurred in connection with Sally’s trade or business. However, because Sally is an employee and the loss is not reimbursed, it is an itemized deduction which is a deduction from adjusted gross income. The loss is not incurred in a transaction entered into for profit nor is the loss a casualty loss on personal use property. However, the loss is connected with Sally’s trade or business and therefore, even though it is an itemized deduction and subject to the 2%-of-AGI floor, it can create an NOL.

915. Discuss the treatment of alimony paid and deductible individual retirement account contributions in computing an individual’s net operating loss.

Correct Answer: Alimony paid and deductible individual retirement account contributions are treated as nonbusiness deductions in computing a net operating loss. Hence, neither item can create or increase a net operating loss.

916. Discuss the computation of NOL remaining to be carried forward after the NOL has been applied in a carryback year.

Correct Answer: The NOL amount to be carried forward is the excess of the NOL over the taxable income of the year to which the NOL is being applied. However, the taxable income of the year to which the NOL is being applied must be computed with the following modifications:

• No deduction is allowed for the excess of capital losses over capital gains.

• No deduction is allowed for the NOL that is being carried back. However, deductions are allowed for NOLs occurring before the loss year.

• Any deductions claimed that are based on or limited by AGI must be determined after making the preceding adjustments. However, charitable contributions do not take into account any NOL carryback.

• No deduction is allowed for personal or dependency exemptions.

917. Discuss the effect of alimony in computing a net operating loss.

Correct Answer: Alimony received is treated as nonbusiness income. Alimony paid is treated as a nonbusiness deduction.

918. Property which is classified as personalty may be depreciated.

*a. True b. False

919. The basis of cost recovery property must be reduced by at least the cost recovery allowable.

*a. True b. False

920. Antiques may be eligible for cost recovery if they are used in a trade or business.

a. True *b. False

921. The key date for calculating cost recovery is the date the asset is placed in service.

*a. True b. False

922. Land improvements are generally not eligible for cost recovery.

a. True *b. False

923. The cost recovery basis for property converted from personal use to business use may be the fair market value of the property at the time of the conversion.

*a. True b. False

924. The maximum cost recovery method for all personal property under MACRS is 150% declining balance.

a. True *b. False

925. The cost recovery period for 3-year class property is 4 years.

*a. True b. False

926. All personal property placed in service in 2013 and used in a trade or business qualifies for additional first-year depreciation.

a. True *b. False

927. If more than 40% of the value of property, other than real property, is placed in service during the last quarter, all of the property placed in service in the second quarter will be allowed 7.5 months of cost recovery.

*a. True b. False

928. Under MACRS, if the mid-quarter convention is applicable, all property sold is treated as being sold at the mid-point of the quarter in which it is placed in service.

a. True *b. False

929. The factor for determining the cost recovery for eligible real estate under MACRS, in the year of disposition, is taken from the month of the disposition.

a. True *b. False

930. Residential rental real estate includes property where 80% or more of the net rental revenues are from nontransient dwelling units.

a. True *b. False

931. Motel buildings have a cost recovery period of 27.5 years.

a. True *b. False

932. Taxpayers may elect to use the straight-line method under MACRS for personalty.

*a. True b. False

933. Under the MACRS straight-line election for personalty, only the half-year convention is applicable.

a. True *b. False

934. The cost recovery method for new farm equipment placed in service during 2013 is 200% declining balance.

a. True *b. False

935. In a farming business, MACRS straight-line cost recovery is required for all fruit bearing trees.

*a. True b. False

936. In a farming business, if the uniform capitalization rules are not used, cost is recovered using the ADS straight-line method.

*a. True b. False

937. When lessor owned leasehold improvements are abandoned because of the termination of the lease, a loss can be taken for the unrecovered basis.

*a. True b. False

938. The costs of qualified leasehold improvements qualify for additional first-year depreciation.

*a. True b. False

939. For personal property placed in service in 2013, the § 179 maximum deduction is limited to $500,000.

*a. True b. False

940. The § 179 deduction can exceed $500,000 in 2013 if the taxpayer had a § 179 amount which exceeded the taxable income limitation in the prior year.

a. True *b. False

941. Any § 179 expense amount that is carried forward is subject to the business income limitation in the carryforward year.

*a. True b. False

942. Taxable income for purposes of § 179 limited expensing is computed by including the MACRS deduction.

*a. True b. False

943. The basis of an asset on which $500,000 has been expensed under § 179 will be reduced by $500,000, even if $500,000 cannot be expensed in the current year because of the taxable income limitation.

*a. True b. False

944. Property used for the production of income is not eligible for § 179 expensing.

*a. True b. False

945. The statutory dollar cost recovery limits under § 280F does apply to all automobiles.

a. True *b. False

946. The § 179 limit for a sports utility vehicle with a GVW of 7,000 pounds will not apply if the sports utility vehicle is used as a taxi.

a. True *b. False

947. Once the more-than-50% business usage test is passed for listed property, it does matter if the business usage for the property drops to 50% or less during the recovery period.

*a. True b. False

948. If a new car that is used predominantly in business is placed in service in 2013, the statutory dollar cost recovery limit under § 280F will depend on whether the taxpayer takes MACRS or straight-line depreciation.

a. True *b. False

949. If an automobile is placed in service in 2013, the limitation for cost recovery in 2015 will be based on the cost recovery limits for the year 2013.

*a. True b. False

950. The statutory dollar cost recovery limits under § 280F for passenger automobiles are changed if mid-quarter cost recovery is used.

a. True *b. False

951. If a used $35,000 automobile used 100% for business in the first year (2013) fails the 50% business usage test in the second year, no cost recovery will be recaptured.

*a. True b. False

952. The inclusion amount for a leased automobile is adjusted by a business usage percentage.

*a. True b. False

953. All listed property is subject to the substantiation requirements of § 274.

*a. True b. False

954. If a taxpayer uses regular MACRS for all property, an adjustment is made with respect to the depreciation on all property, regardless of the class life.

a. True *b. False

955. MACRS depreciation is used to compute earnings and profits.

a. True *b. False

956. Under the alternative depreciation system (ADS), the half-year convention must be used for personalty.

a. True *b. False

957. A taxpayer may elect to use the alternative depreciation system (ADS) to compute depreciation for earnings and profits.

*a. True b. False

958. An election to use straight-line under ADS is made on an asset-by- asset basis for property other than eligible real estate.

a. True *b. False

959. For real property, the ADS convention is the mid-month convention.

*a. True b. False

960. The cost of a covenant not to complete for 10 years incurred in connection with the acquisition of a business is amortized over 10 years.

a. True *b. False

961. Goodwill associated with the acquisition of a business cannot be amortized.

a. True *b. False

962. A purchased trademark is a § 197 intangible.

*a. True b. False

963. If startup expenses total $53,000 in 2013, $51,000 is amortized over 180 months.

*a. True b. False

964. The amortization period in 2013 for $58,000 of startup expenses is 180 months.

*a. True b. False

965. Cost depletion is determined by multiplying the depletion cost per unit by the number of units sold.

*a. True b. False

966. Percentage depletion enables the taxpayer to recover more than the cost of an asset.

*a. True b. False

967. Intangible drilling costs must be capitalized and written off through depletion.

a. True *b. False

968. Grape Corporation purchased a machine in December of the current year. This was the only asset purchased during the current year. The machine was placed in service in January of the following year. No assets were purchased in the following year. Grape Corporation’s cost recovery would begin:

a. In the current year using a mid-quarter convention. b. In the current year using a half-year convention. c. In the following year using a mid-quarter convention. *d. In the following year using a half-year convention. e. None of the above.

969. Which of the following assets would be subject to cost recovery?

a. A painting by Picasso hanging on a doctor’s office wall. b. An antique vase in a doctor’s waiting room. *c. Landscaping around the doctor’s office. d. a., b., and c. e. None of the above.

970. On June 1 of the current year, Tab converted a machine from personal use to rental property. At the time of the conversion, the machine was worth $90,000. Five years ago Tab purchased the machine for $120,000. The machine is still encumbered by a $50,000 mortgage. What is the basis of the machine for cost recovery?

a. $70,000. *b. $90,000. c. $120,000. d. $140,000. e. None of the above.

971. Tara purchased a machine for $40,000 to be used in her business. The cost recovery allowed and allowable for the three years the machine was used are as follows:

Cost Recovery Cost Recovery Allowed Allowable Year 1 $16,000 $ 8,000 Year 2 9,600 12,800 Year 3 5,760 7,680

If Tara sells the machine after three years for $15,000, how much gain should she recognize?

a. $3,480. b. $6,360. c. $9,240. *d. $11,480. e. None of the above.

972. Hazel purchased a new business asset (five-year asset) on September 30, 2013, at a cost of $100,000. On October 4, 2013, Hazel placed the asset in service. This was the only asset Hazel placed in service in 2013. The only election with respect to the asset was not to take § 179. On August 20, 2014, Hazel sold the asset. Determine the cost recovery for 2014 for the asset.

a. $9,600. *b. $11,875. c. $23,750. d. $38,000. e. None of the above.

973. Tan Company acquires a new machine (ten-year property) on January 15, 2013, at a cost of $200,000. Tan also acquires another new machine (seven-year property) on November 5, 2013, at a cost of $40,000. No election is made to use the straight-line method. The company does not make the § 179 election. Tan elects to not take additional first-year depreciation. Determine the total deductions in calculating taxable income related to the machines for 2013.

a. $24,000. *b. $25,716. c. $102,000. d. $132,858. e. None of the above.

974. James purchased a new business asset (three-year personalty) on July 23, 2013, at a cost of $40,000. James takes additional first-year depreciation Determine the cost recovery deduction for 2013.

a. $8,333. *b. $26,666. c. $33,333. d. $41,665. e. None of the above.

975. Alice purchased office furniture on September 20, 2012, for $100,000. On October 10, 2012, she purchased business computers for $80,000. Alice placed all of the assets in service on January 15, 2013. Alice did not elect to expense any of the assets under § 179, nor did she elect straight-line cost recovery. She did not take additional first-year depreciation. Determine the cost recovery deduction for the business assets for 2013.

a. $6,426. b. $14,710. c. $25,722. *d. $30,290. e. None of the above.

976. Barry purchased a used business asset (seven-year property) on September 30, 2013, at a cost of $200,000. This is the only asset he purchased during the year. Barry did not elect to expense any of the asset under § 179, nor did he elect straight-line cost recovery. Barry sold the asset on July 17, 2014. Determine the cost recovery deduction for 2014.

a. $19,133. *b. $24,490. c. $34,438. d. $55,100. e. None of the above.

977. Bonnie purchased a new business asset (five-year property) on March 10, 2013, at a cost of $30,000. She also purchased a new business asset (seven-year property) on November 20, 2013, at a cost of $13,000. Bonnie did not elect to expense either of the assets under § 179, nor did she elect straight-line cost recovery. Bonnie takes additional first-year depreciation. Determine the cost recovery deduction for 2013 for these assets.

a. $5,858. b. $7,464. c. $9,586. d. $19,429. *e. None of the above.

978. Doug purchased a new factory building on January 15, 1988, for $400,000. On March 1, 2013, the building was sold. Determine the cost recovery deduction for the year of the sale assuming he did not use the MACRS straight-line method.

a. $0. b. $1,587. *c. $2,645. d. $12,696. e. None of the above.

979. Cora purchased a hotel building on May 17, 2013, for $3,000,000. Determine the cost recovery deduction for 2014.

a. $48,150. b. $59,520. c. $69,000. *d. $76,920. e. None of the above.

980. Carlos purchased an apartment building on November 16, 2013, for $3,000,000. Determine the cost recovery for 2013.

a. $9,630. b. $11,910. c. $13,950. d. $22,740. *e. None of the above.

981. Diane purchased a factory building on April 15, 1993, for $5,000,000. She sells the factory building on February 2, 2013. Determine the cost recovery deduction for the year of the sale.

a. $16,025. *b. $19,838. c. $26,458. d. $158,750. e. None of the above.

982. Howard’s business is raising and harvesting peaches. On March 10, 2013, Howard purchased 10,000 new peach trees at a cost of $60,000. Howard does not elect to expense assets under § 179. If eligible, Howard takes additional first-year depreciation. Determine the cost recovery deduction for 2013.

a. $0. b. $3,000. *c. $31,500. d. $60,000. e. None of the above.

983. On May 15, 2013, Brent purchased new farm equipment for $200,000. Brent used the equipment in connection with his farming business. Brent does not elect to expense assets under § 179. Brent does not take additional first-year depreciation. Determine the cost recovery deduction for 2013.

a. $12,852. *b. $21,420. c. $30,000. d. $36,000. e. None of the above.

984. On June 1, 2013, Sam purchased used farm machinery for $150,000. Sam used the machinery in connection with his farming business. Sam does not elect to expense assets under § 179. Sam has, however, made an election to not have the uniform capitalization rules apply to the farming business. Sam takes additional first-year depreciation when available. Determine the cost recovery deduction for 2013.

a. $5,000. *b. $7,500. c. $10,000. d. $78,750. e. None of the above.

985. On May 30, 2013, Jane signed a 20-year lease on a factory building to use for her business. The lease begins on June 1, 2013. In August 2013, Jane paid $300,000 for qualified leasehold improvements to the building. Jane takes additional first-year depreciation. Determine Jane’s total deduction with respect to the leasehold improvements for 2013.

a. $2,890. b. $150,000. *c. $154,995. d. $300,000. e. None of the above.

986. On February 20, 2013, Susan paid $200,000 for a leasehold improvement to an office building that she is going to lease to John. The leasehold improvement is not a qualified leasehold improvement. The lease will begin on June 1, 2013, and terminate on May 31, 2023. At the termination of the lease, the improvement will be worthless. Determine Susan’s deductible loss as a result of the termination of the lease.

a. $0. b. $123,503. c. $127,990. d. $128,631. *e. None of the above.

987. White Company acquires a new machine (seven-year property) on January 10, 2013, at a cost of $600,000. White makes the election to expense the maximum amount under § 179. No election is made to use the straight-line method. White does take additional first-year depreciation. Determine the total deductions in calculating taxable income related to the machine for 2013 assuming White has taxable income of $800,000.

a. $71,593. b. $128,610. c. $385,296. d. $390,868. *e. None of the above.

988. Augie purchased one new asset during the year (five-year property) on November 10, 2013, at a cost of $650,000. She made the § 179 election. The income from the business before the cost recovery deduction and the § 179 deduction was $600,000. She takes additional first-year depreciation. Determine the total cost recovery deduction with respect to the asset for 2013.

a. $22,500. b. $154,550. c. $500,000. d. $600,700. *e. None of the above.

989. In 2012, Gail had a § 179 deduction carryover of $30,000. In 2013, she elected § 179 for an asset acquired at a cost of $115,000. Gail’s § 179 business income limitation for 2013 is $140,000. Determine Gail’s § 179 deduction for 2013.

a. $25,000. b. $115,000. c. $130,000. *d. $140,000. e. None of the above.

990. The only asset Bill purchased during 2013 was a new seven-year class asset. The asset, which was listed property, was acquired on June 17 at a cost of $50,000. The asset was used 40% for business, 30% for the production of income, and the rest of the time for personal use. Bill always elects to expense the maximum amount under § 179 whenever it is applicable. The net income from the business before the § 179 deduction is $100,000. Determine Bill’s maximum deduction with respect to the property for 2013.

a. $1,428. *b. $2,499. c. $26,749. d. $33,375. e. None of the above.

991. Mary purchased a new five-year class asset on March 7, 2013. The asset was listed property (not an automobile). It was used 60% for business and the rest of the time for personal use. The asset cost $900,000. Mary made the § 179 election. The income from the business before the § 179 deduction was $600,000. Mary does take additional first-year depreciation. Determine the total deductions with respect to the asset for 2013.

a. $72,000. b. $271,600. *c. $524,000. d. $600,000. e. None of the above.

992. Hans purchased a new passenger automobile on August 17, 2013, for $30,000. During the year the car was used 40% for business and 60% for personal use. Determine his cost recovery deduction for the car for 2013.

a. $500. b. $1,000. c. $1,224. d. $1,500. *e. None of the above.

993. On June 1, 2013, Irene places in service a new automobile that cost $21,000. The car is used 70% for business and 30% for personal use. (Assume this percentage is maintained for the life of the car.) She does not take additional first-year depreciation. Determine the cost recovery deduction for 2014.

a. $3,160. b. $3,290. *c. $3,570. d. $6,720. e. None of the above.

994. On June 1, 2013, James places in service a new automobile that cost $40,000. The car is used 60% for business and 40% for personal use. (Assume this percentage is maintained for the life of the car.) James does take additional first-year depreciation. Determine the cost recovery deduction for 2013.

a. $1,776. b. $1,836. *c. $6,696. d. $11,160. e. None of the above.

995. On May 2, 2013, Karen placed in service a new sports utility vehicle that cost $60,000 and has a gross vehicle weight of 6,300 lbs. The vehicle is used 60% for business and 40% for personal use. Determine the cost recovery for 2013. Karen wants to maximize her deductions.

a. $2,200. b. $3,060. c. $25,000. d. $27,200. *e. None of the above.

996. On July 17, 2013, Kevin places in service a used automobile that cost $25,000. The car is used 80% for business and 20% for personal use. In 2014, he used the automobile 40% for business and 60% for personal use. Determine the cost recovery recapture for 2014.

a. $0. *b. $528. c. $2,000. d. $2,500. e. None of the above.

997. Janet purchased a new car on June 5, 2013, at a cost of $20,000. She used the car 80% for business and 20% for personal use in 2013. She used the automobile 40% for business and 60% for personal use in 2014. Janet takes additional first-year depreciation. Determine Janet’s cost recovery recapture for 2014.

a. $0. b. $928. c. $1,008. *d. $7,328. e. None of the above.

998. On July 10, 2013, Ariff places in service a new sports utility vehicle that cost $70,000 and weighed 6,300 pounds. The SUV is used 100% for business. Determine Ariff’s maximum deduction for 2013, assuming Ariff’s § 179 business income is $110,000. Ariff does not take additional first-year depreciation.

a. $2,960. b. $25,000. *c. $34,000. d. $70,000. e. None of the above.

999. On March 1, 2013, Lana leases and places in service a passenger automobile. The lease will run for five years and the payments are $500 per month. During 2013, she uses her car 60% for business and 40% for personal activities. Assuming the dollar amount from the IRS table is $20, determine Lana’s inclusion as a result of the lease.

a. $0. *b. $10. c. $17. d. $20. e. None of the above.

1000. On June 1, 2013, Norm leases a taxi and places it in service. The lease payments are $1,000 per month. Assuming the dollar amount from the IRS table is $241, determine Norm’s inclusion amount.

*a. $0. b. $241. c. $907. d. $1,687. e. None of the above.

1001. Bhaskar purchased a new factory building on September 10, 2013, for $3,700,000. Five hundred thousand of the purchase price was allocated to the land. He elected the alternative depreciation system (ADS). Determine the cost recovery deduction for 2014.

a. $23,328. *b. $80,000. c. $82,048. d. $92,500. e. None of the above.

1002. Pat purchased a used five-year class asset on March 15, 2013, for $60,000. He did not elect § 179 expensing. Determine the cost recovery deduction for 2013 for earnings and profits purposes.

a. $2,000. b. $3,000. *c. $6,000. d. $12,000. e. None of the above.

1003. George purchases used seven-year class property at a cost of $200,000 on April 20, 2013. Determine George’s cost recovery deduction for 2013 for alternative minimum tax purposes, assuming George does not elect § 179.

a. $2,500. b. $10,000. c. $14,280. d. $28,580. *e. None of the above.

1004. During the past two years, through extensive advertising and improved customer relations, Orange Corporation estimated that it had developed customer goodwill worth $500,000. For the current year, determine the amount of goodwill Orange Corporation may amortize.

a. $16,667. b. $26,667. c. $33,333. d. $100,000. *e. None of the above.

1005. On June 1, 2013, Red Corporation purchased an existing business. With respect to the acquired assets of the business, Red allocated $300,000 of the purchase price to a patent. The patent will expire in 20 years. Determine the total amount that Red may amortize for 2013 for the patent.

a. $0. b. $1,667. *c. $11,667. d. $35,000. e. None of the above.

1006. Orange Corporation begins business on April 2, 2013. The corporation has startup expenditures of $64,000 which it incurred last year. If Orange Corporation elects § 195, determine the total amount that Orange may deduct in 2013.

a. $0. *b. $3,200. c. $4,267. d. $7,950. e. None of the above.

1007. On January 15, 2013, Vern purchased the rights to a mineral interest for $3,500,000. At that time it was estimated that the recoverable units would be 500,000. During the year, 40,000 units were mined and 25,000 units were sold for $800,000. Vern incurred expenses during 2013 of $500,000. The percentage depletion rate is 22%. Determine Vern’s depletion deduction for 2013.

a. $150,000. *b. $175,000. c. $176,000. d. $200,000. e. $250,000.

1008. Tom purchased and placed in service used office furniture on January 3, 2013, for $40,000. Tom’s accountant depreciated the furniture using straight-line depreciation over 10 years for financial reporting purposes. The accountant also used the same depreciation amounts when filing Tom’s income tax returns. On January 10, 2018, Tom sold the furniture. Determine the tax basis of the furniture at the time of the sale.

Correct Answer: The cost of the asset must be reduced by the greater of the cost recovery allowed or allowable in calculating the tax basis.

Cost $40,000 2013 allowable ($40,000 (5,716) × .1429) 2014 allowable ($40,000 (9,796) × .2449) 2015 allowable ($40,000 (6,996) × .1749) 2016 allowable ($40,000 (4,996) × .1249) 2017 allowed ($40,000 (3,572) × .0893) 2018 allowable ($40,000 (1,784) × .0892 × .50) Tax basis $ 7,140

1009. Jim acquires a new seven-year class asset on September 20, 2013, for $80,000. He placed the asset in service on October 5, 2013. He does not elect to expense any of the asset under § 179 or elect straight-line, cost recovery. He takes additional first-year depreciation. He sells the asset on August 25, 2014. This is the only asset he acquires in 2013. Determine Jim’s cost recovery in 2013 and 2014.

Correct Answer: The mid-quarter convention applies.

2013 Additional first-year depreciation $40,000 ($80,000 × .50) MACRS cost recovery ($40,000 × .0357) 1,42 8 Total for 2013 $41,428

2014 MACRS cost recovery [$40,000 × .2755 × $ 6,888 (2.5/4)]

1010. Rod paid $1,950,000 for a new warehouse on April 14, 2013. He sold the warehouse on September 29, 2018. Determine the cost recovery deduction for 2013 and 2018.

Correct Answer: 2013: $1,950,000 × .01819 = $35,471.

2018: $1,950,000 × .02564 × 8.5/12 = $35,415.

1011. On March 3, 2013, Sally purchased and placed in service a building costing $12,000,000. The building has 10 floors. The bottom three floors are rented out to businesses. The top seven floors are residential apartments. The gross rents from the businesses are $60,000 and the gross rents from the apartments are $110,000. Determine Sally’s cost recovery for the building in 2013.

Correct Answer: The gross rents from the apartments are not 80% or more of the total gross rents and hence, the whole building cannot be treated as residential rental real estate.

Residential [(70% × $12,000,000) $241,836 × .02879] Nonresidential [(30% × $12,000,000) 73,1 × .02033] 88 Total cost recovery $315,024

1012. Sid bought a new $1,210,000 seven-year class asset on August 2, 2013. On December 2, 2013, he purchased $860,000 of used five-year class assets. Sid does take additional first-year depreciation if available. If Sid elects § 179, what is the maximum write-off for these purchases for 2013?

Correct Answer: § 179 expense [$500,000 – $430,000 ($2,070,000 – $2,000,000)]

Taking § 179 expense on 7-year property:

7-year property § 179 expense $ 430,000 Additional first-year 390,000 depreciation [($1,210,000 – $430,000) × .50] MACRS cost recovery ($390,000 55,731 × .1429)

5-year property MACRS cost recovery ($860,000 172,000 × .20) Total deduction $1,047,731

Taking § 179 expense on 5-year property:

7-year property Additional first-year $ 605,000 depreciation ($1,210,000 × .50) MACRS cost recovery ($605,000 86,455 × .1429)

5-year property § 179 expense 430,000 MACRS cost recovery [($860,000 86, – $430,000) × .20] 000 Total deduction $1,207,455

Using § 179 on the used 5-year asset produces the greater total deduction in 2013.

1013. Polly purchased a new hotel on July 20, 2013, for $6,000,000. On January 20, 2020, the building was sold. Determine the cost recovery deduction for the year of the sale.

Correct Answer: $6,000,000 × .02564 × .5/12 = $6,410.

1014. Rustin bought used 7-year class property on May 15, 2013, for $728,000. Rustin elects § 179 and straight-line cost recovery. Rustin’s taxable income would not create a limitation for purposes of the § 179 deduction. Determine the maximum write-off Rustin can take in 2013.

Correct Answer: § 179 expense election $500,000 Cost recovery [($728,000 – 16,27 $500,000) × .0714 (Table 8.3)] 9 Total deduction $516,279

1015. Audra acquires the following new five-year class property in 2013:

Asset Acquisition Cost Date A January $ 106,000 10 B July 5 70,000 C Novembe 1,950,000 r 15 Total $2,126,000

Audra elects § 179 for Asset C. Audra’s taxable income from her business would not create a limitation for purposes of the § 179 deduction. Audra takes additional first-year depreciation. Determine her total cost recovery deduction (including the § 179 deduction) for the year.

Correct Answer: $1,950,000/$2,126,000 = 91.7%. Therefore, Audra must use the mid- quarter convention.

Asset A: Additional first-year $ 53,000 depreciation ($106,000 × .50) MACRS cost recovery ($53,000 18,550 × .35)

Asset B: Additional first-year 35,000 depreciation ($70,000 × .50) MACRS cost recovery ($35,000 5,250 × .15)

Asset C: § 179 expense [$500,000 – 374,000 ($2,126,000 – $2,000,000)] Additional first-year 788,000 depreciation [($1,950,000 – $374,000) × .50] MACRS cost recovery ($788,000 39 × .05) ,400 Total deduction $1,313,200

1016. On April 5, 2013, Orange Corporation purchased, and placed in service, seven-year class assets costing $540,000 and five-year class assets costing $140,000. Orange elects to expense the maximum amount under § 179. Orange does not take additional first-year depreciation. Assume taxable income is not a limitation. Determine Orange Corporation’s cost recovery with respect to the assets for 2013.

Correct Answer: § 179 limit $500,000

Seven-year assets § 179 expense $500,000 Regular MACRS [($540,000 – 5,716 $500,000) × .1429] Five-year assets Regular MACRS ($140,000 × .20) 28,00 0 Total cost recovery $533,716

1017. Martin is a sole proprietor of a business. On March 4, 2013, Martin purchased and placed in service new seven-year class assets costing $560,000. Martin’s business has income for the year, before any deductions associated with the purchased assets, of $160,000. Martin also has $30,000 of interest income for the year which is not related to the business. Martin wants his adjusted gross income for the year to be as low as possible. With this objective in mind, determine how Martin should recover the cost of the acquired assets.

Correct Answer: Electing § 179:

§ 179 expense $500,000

Business income before MACRS $160,000 deductions Additional first-year depreciation (30,000) [($560,000 – $500,000) × 50%] MACRS cost recovery ($30,000 (4,287) × .1429) Business income limitation $125,713

§ 179 limit $125,713

Business income $ – 0– Interest income 30,000 Adjusted gross income $ 30,000

Not electing § 179:

Business income before MACRS $160,000 deductions Additional first-year (280,000) depreciation ($560,000 × 50%) MACRS cost recovery ($280,000 (40,012) × .1429) Business income ($160,012) Interest income 30,000

Adjusted gross income ($130,012)

Not electing § 179 will produce the lowest adjusted gross income because the § 179 expense cannot create a business loss.

1018. On February 21, 2013, Joe purchased new farm equipment for $600,000. Joe has made an election to not have the uniform capitalization rules apply to his farming business. He does not take additional first-year depreciation. If Joe elects § 179, what is the maximum write-off for this purchase for 2013?

Correct Answer: § 179 expense $500,000 ADS straight-line [($600,000 – 5,0 $500,000) × .05] 00 Total deduction $505,000

1019. On April 15, 2013, Sam placed in service a storage facility (a single-purpose agricultural structure) costing $80,000. Sam also purchased and planted fruit trees costing $40,000. Sam does not elect to expense any of the acquisitions under § 179. Sam elected not to take additional first-year depreciation. Determine Sam’s cost recovery from these two items for 2013.

Correct Answer: Storage facility ($80,000 $6,000 × .075) (Table 8.4) Trees ($40,000 × .05) (Table 2,000 8.3) Total cost recovery $8,000

1020. On August 20, 2012, May signed a 10-year lease on a building for her business. On November 28, 2013, May paid $80,000 for a qualified leasehold improvement to the building. She takes additional first-year depreciation, but does not elect § 179 expensing. What is May’s cost recovery deduction for the improvement in 2013?

Correct Answer: Additional first-year depreciation $40,000 ($80,000 × .50) MACRS cost recovery ($40,000 × .0333) 1,33 2 Total deduction $41,332

1021. On July 15, 2013, Mavis paid $275,000 for qualified leasehold improvements on a commercial building she was leasing. Determine the maximum total cost recovery from the improvements in 2013.

Correct Answer: § 179 expense $250,000 Additional first-year 12,500 depreciation [($275,000 – $250,000) × 50%] Regular MACRS ($12,500 416 × .0333) $262,916

1022. Joe purchased a new five-year class asset on June 1, 2013. The asset is listed property (not an automobile). It was used 55% for business and 45% for the production of income. The asset cost $1,000,000. Joe made the § 179 election. Joe’s taxable income would not create a limitation for purposes of the § 179 deduction. Joe does not take additional first-year depreciation. Determine Joe’s total cost recovery (including the § 179 deduction) for the year.

Correct Answer: Business use: $550,000 ($1,000,000 × 55%)

§ 179 expense $500,000 Regular MACRS [($550,000 – $500,000) 10,000 × .20]

Production of income use: $450,000 ($1,000,000 × 45%) Regular MACRS ($450,000 × .20) 90,0 00 Total deduction $600,000

*Property used for the production of income is not eligible for § 179 expensing.

1023. Nora purchased a new automobile on July 20, 2013, for $29,000. The car was used 60% for business and 40% for personal use. In 2014, the car was used 30% for business and 70% for personal use. Nora elects not to take additional first-year depreciation. Determine the cost recovery recapture and the cost recovery deduction for 2014.

Correct Answer: Cost recovery in 2013: MACRS ($29,000 × .20) = $5,800 (limited $1,896 to $3,160*); $3,160 × 60% Straight-line ($29,000 × .10) = $2,900 (1,740) (limited to $3,160*); $2,900 × 60% Cost recovery recapture in 2014 $ 156

Cost recovery in 2014: Straight-line ($29,000 × .20) = $5,800 $1,530 (limited to $5,100*); $5,100 × .30

*These depreciation limits are indexed annually.

1024. Norm purchases a new sports utility vehicle (SUV) on October 12, 2013, for $50,000. The SUV has a gross vehicle weight of 6,200 lbs. It is used 100% of the time for business and it is the only business asset acquired by Norm during 2013. Compute the maximum deduction with respect to the SUV for 2013. Norm does take additional first-year depreciation.

Correct Answer: The SUV is not classified as a passenger automobile because of its GVW exceeding 6,000 lbs. Therefore, it is not subject to the cost recovery limits of § 280F.

Section 179 expense (limited to $25,000 $25,000) Additional first-year 12,500 depreciation [($50,000 – $25,000) × .50] MACRS cost recovery ($12,500 625 × .05) Total deduction $38,125

1025. On June 1, 2013, Gabriella purchased a computer and peripheral equipment (five-year property) for $25,000. She used the assets 40% for business, 50% for the production of income, and 10% for personal use. These are the only assets Gabriella purchased during the current year. Determine her total cost recovery deduction for the current year.

Correct Answer: A computer and peripheral equipment are listed property. Since the more-than-50% business use test is not satisfied, Gabriella cannot elect § 179 expensing, she must use straight-line cost recovery, and is not eligible for additional first-year depreciation. The 10% of personal usage does not qualify for cost recovery.

$25,000 × .10 × 90% = $2,250.

1026. In 2013, Marci is considering starting a new business. Marci had the following costs associated with this venture:

Advertising $ 5,000 Travel 10,000 Market surveys 8,000 Professional services 30,000 Interest expense 2,000 Taxes 1,000

Marci started the new business on January 5, 2014. Determine the deduction for Marci’s startup costs for 2013.

Correct Answer: Marci is not allowed to deduct any startup costs in 2013 because the business was not started until 2014.

1027. Rick purchased a uranium interest for $10,000,000 on January 3, 2013, when recoverable reserves were estimated at 200,000 units. A total of 10,000 units were extracted in 2013 and 7,000 units were sold in 2013. Gross income from the property was $2,800,000 and taxable income without the allowance for depletion was $1,000,000. Determine the depletion deduction for 2013.

Correct Answer: Cost depletion

Percentage depletion

Lesser of: 22% × $2,800,000 = $616,000 50% × $1,000,000 = $500,000

Therefore the depletion deduction would be $500,000.

1028. Discuss the difference between the half-year convention and the mid-quarter convention.

Correct Answer: The half-year convention assumes property is placed in service at mid- year and thus provides for a half-year’s cost recovery for that year. The mid-quarter convention assumes property placed in service during the year is placed in service at the middle of the quarter in which it is actually placed in service.

1029. Discuss the criteria used to determine whether a building is residential or nonresidential realty. Also explain the tax consequences resulting from this determination if the property is placed in service in 2013.

Correct Answer: Residential realty is property for which 80% or more of the gross rental revenues are from nontransient dwelling units. Residential realty has a recovery period of 27.5 years. Nonresidential realty has a recovery period of 39 years.

1030. Discuss the effect on the cost recovery method of a taxpayer election if the uniform capitalization rules apply to a farming business.

Correct Answer: The cost recovery method is generally MACRS using the 150% declining- balance method.

1031. Discuss the tax consequences of listed property being used for the production of income compared to being used in a trade or business.

Correct Answer: Section 179 expensing cannot be taken on property used for the production of income. However, additional first-year depreciation can be taken.

1032. Discuss the beneficial tax consequences of an SUV not being classified as a passenger automobile.

Correct Answer: If an automobile is not classified as a passenger automobile, it is not subject to the statutory dollar cost recovery limits under § 280F. In addition to a larger cost recovery deduction each year, it also results in the total recovery of the cost over a six-year period. While the automobile is still listed property, if it passes the more-than-50% business use test, MACRS cost recovery can be used as well as an election under § 179. However, the § 179 limit for SUVs is $25,000 rather than $500,000 in 2013. The automobile also is eligible for additional first-year depreciation.

1033. Discuss the reason for the inclusion amount with respect to leased automobiles.

Correct Answer: The purpose of the inclusion amount is to prevent taxpayers from circumventing the cost recovery dollar limitations by leasing, instead of purchasing, an automobile.

1034. Discuss the requirements in order for startup expenditures to be amortized under § 195.

Correct Answer: The expenditures must meet two requirements.

• The expenditures must be paid or incurred in connection with:

• Creating an active trade or business;

• Investigating the creation or acquisition of an active trade or business; or

• Any activity engaged in for profit in anticipation of such activity becoming an active trade or business.

• Such costs must be the kinds of costs that would be currently deductible if paid or incurred in connection with the operation of an existing trade or business in the same field as that entered into by the taxpayer.

1035. Discuss the tax implications of a seller allocating the selling price to goodwill or a covenant not to compete.

Correct Answer: Goodwill is a capital asset and any gain or loss recognized on the sale of the goodwill will be capital gain or loss. A covenant not to compete is an ordinary asset and any gain or loss will be ordinary gain or loss.

1036. One indicia of independent contractor (rather than employee) status is when the individual performing the services is paid based on time spent (rather than on tasks performed).

a. True *b. False

1037. In some cases it may be appropriate for a taxpayer to report work-related expenses by using both Form 2106 and Schedule C.

*a. True b. False

1038. The IRS will issue advanced rulings as to whether a worker’s status is that of an employee or an independent contractor.

*a. True b. False

1039. Jake performs services for Maude. If Maude provides the helper and tools, this is indicative of independent contractor (rather than employee) status.

a. True *b. False

1040. A statutory employee is not a common law employee but is subject to Social Security tax.

*a. True b. False

1041. For tax purposes, a statutory employee is treated the same as a common law employee.

a. True *b. False

1042. If an individual is subject to the direction or control of another only to the extent of the end result but not as to the means of accomplishment, an employer-employee relationship does not exist.

*a. True b. False

1043. The work-related expenses of an independent contractor will be subject to the 2%-of-AGI floor.

a. True *b. False

1044. After she finishes working at her main job, Ann returns home, has dinner, then drives to her second job. Ann may deduct the mileage between her first and second job.

*a. True b. False

1045. A taxpayer who maintains an office in the home to conduct his only business will not have nondeductible commuting expense.

*a. True b. False

1046. After the automatic mileage rate has been set by the IRS for a year, it cannot later be changed by the IRS.

a. True *b. False

1047. In choosing between the actual expense method and the automatic mileage method, a taxpayer should consider the cost of insurance on the automobile.

*a. True b. False

1048. A taxpayer who uses the automatic mileage method to compute auto expenses can also deduct the business portion of tolls and parking.

*a. True b. False

1049. A deduction for parking and other traffic violations incurred during business use of the automobile is allowed under the actual cost method but not the automatic mileage method.

a. True *b. False

1050. A taxpayer who uses the automatic mileage method for the business use of an automobile can change to the actual cost method in a later year.

*a. True b. False

1051. Once the actual cost method is used, a taxpayer cannot change to the automatic mileage method in a later year.

a. True *b. False

1052. For tax purposes, “travel” is a broader classification than “transportation.”

*a. True b. False

1053. Amy lives and works in St. Louis. In the morning she flies to Boston, has a three-hour business meeting, and returns to St. Louis that evening. For tax purposes, Amy was away from home.

a. True *b. False

1054. Janet, who lives and works in Newark, travels to Atlanta for a Thursday-Friday business conference. She stays over after the conference and visits relatives and friends on Saturday. Under certain circumstances, the meals and lodging expenses for Saturday can be considered as business related.

*a. True b. False

1055. Marvin lives with his family in Alabama. He has two jobs: one in Alabama and one in North Carolina. Marvin’s tax home is where he lives (Alabama).

a. True *b. False

1056. A taxpayer who lives and works in Kansas City is sent to Chicago on an eight-day business trip. While in Chicago, taxpayer uses the hotel valet service to have some laundry done. The valet charge is a nondeductible personal travel expense.

a. True *b. False

1057. The tax law specifically provides that a taxpayer cannot be temporarily away from home for any period of employment that exceeds one year.

*a. True b. False

1058. A taxpayer who lives and works in Tulsa travels to Buffalo for five days. If three days are spent on business and two days are spent on visiting relatives, only 60% of the airfare is deductible.

a. True *b. False

1059. Bob lives and works in Newark, NJ. He travels to London for a three-day business meeting, after which he spends three days touring Scotland. All of his air fare is deductible.

*a. True b. False

1060. Eileen lives and works in Mobile. She travels to Rome for an eight-day business meeting, after which she spends two days touring Italy. All of Eileen’s airfare is deductible.

*a. True b. False

1061. Liam just graduated from college. Because it is his first job, the cost of moving his personal belongings from his parents’ home to the job site does not qualify for the moving expense deduction.

a. True *b. False

1062. Sick of her 65 mile daily commute, Edna purchases a condo that is only four miles from her job. Edna’s moving expenses to her new condo are not allowed and cannot be claimed by her as a deduction.

*a. True b. False

1063. In November 2013, Katie incurs unreimbursed moving expenses to accept a new job. Katie cannot deduct any of these expenses when she timely files her 2013 income tax return since she has not yet satisfied the 39-week time test.

a. True *b. False

1064. In May 2013, after 11 months on a new job, Ken is fired after he assaults a customer. Ken must include in his gross income for 2013 any deduction for moving expenses he may have claimed on his 2012 tax return.

a. True *b. False

1065. A moving expense deduction is allowed even if at the time of the move the taxpayer did not have a job at the new location.

*a. True b. False

1066. Kelly, an unemployed architect, moves from Boston to Phoenix to accept a job as a chef at a restaurant. Kelly’s moving expenses are not deductible because her new job is in a different trade or business.

a. True *b. False

1067. Alexis (a CPA) sold her public accounting practice in Des Moines and accepted a job with the Seattle office of a national accounting firm. Her moving expenses are not deductible because she has changed employment status (i.e., went from self-employed to employee).

a. True *b. False

1068. Qualified moving expenses include the cost of lodging but not meals during the move.

*a. True b. False

1069. Qualified moving expenses of an employee that are not reimbursed are a deduction for AGI.

*a. True b. False

1070. At age 65, Camilla retires from her job in Boston and moves to Florida. As a retiree, she is not subject to the time test in deducting her moving expenses.

a. True *b. False

1071. After completing an overseas assignment in Oslo (Norway), Daniel retires from Pelican Corporation and moves to a retirement community in Key West (Florida). Daniel’s moving expenses from Oslo to Key West are deductible as they are exempt from the time test.

*a. True b. False

1072. An education expense deduction may be allowed even if the education results in a promotion or pay raise for the employee.

*a. True b. False

1073. Lloyd, a practicing CPA, pays tuition to attend law school. Since a law degree involves education leading to a new trade or business, the tuition is not deductible.

a. True *b. False

1074. Under the right circumstances, a taxpayer’s meals and lodging expense can qualify as a deductible education expense.

*a. True b. False

1075. There is no cutback adjustment for meals and entertainment as to employees who are subject to regulation by the U.S. Department of Transportation.

a. True *b. False

1076. Mallard Corporation pays for a trip to Aruba for its two top salespersons. This expense is subject to the cutback adjustment.

a. True *b. False

1077. Flamingo Corporation furnishes meals at cost to its employees by means of a cafeteria it maintains. The cost of operating the cafeteria is not subject to the cutback adjustment.

*a. True b. False

1078. A taxpayer takes six clients to an NBA playoff game. If all of the tickets (list price of $120 each) are purchased on the Internet for $1,800 ($300 each), only $60 ($120 ´ 50% cutback adjustment) per ticket is deductible.

*a. True b. False

1079. Ethan, a bachelor with no immediate family, uses the Pine Shadows Country Club exclusively for his business entertaining. All of Ethan’s annual dues for his club membership are deductible.

a. True *b. False

1080. Jackson gives his supervisor and her husband each a $30 box of chocolates at Christmas. Jackson may claim only $25 as a deduction.

a. True *b. False

1081. On their birthdays, Lily sends gift certificates (each valued at $25) to Caden (a key client) and to each of Caden’s two minor children. Lily can deduct only $25 as to these gifts.

*a. True b. False

1082. Tickets to a theater performance or sporting event can be treated as either business entertainment or a business gift if the taxpayer does not accompany the client to the event.

*a. True b. False

1083. In the case of an office in the home deduction, the exclusive business use test does not apply when the home is used as a daycare center.

*a. True b. False

1084. If a taxpayer does not own a home but rents an apartment, the office in the home deduction is not available.

a. True *b. False

1085. A taxpayer who claims the standard deduction will not be able to claim an office in the home deduction.

a. True *b. False

1086. The portion of the office in the home deduction that exceeds the income from the business can be carried over to future years.

*a. True b. False

1087. If the cost of uniforms is deductible, their maintenance cost (e.g., laundry, dry cleaning, alterations) also is deductible.

*a. True b. False

1088. Tired of renting, Dr. Smith buys the academic robes she will wear at her college’s graduation procession. The cost of this attire does not qualify as a uniform expense.

a. True *b. False

1089. Frank, a recently retired FBI agent, pays job search expenses to obtain a position with a city police department. Frank’s job search expenses do qualify as deductions.

*a. True b. False

1090. After graduating from college with a degree in chemistry, Alberto obtains a job as a chemist with DuPont. Alberto’s job search expenses qualify as deductions.

a. True *b. False

1091. Qualifying job search expenses are deductible even if the taxpayer does not change jobs.

*a. True b. False

1092. Madison is an instructor of fine arts at a local community college. If she spends $600 (not reimbursed) on art supplies for her classes, $250 of this amount can be claimed as a deduction for AGI.

a. True *b. False

1093. Both traditional and Roth IRAs possess the advantage of tax-free accumulation of income within the plan.

*a. True b. False

1094. When contributions are made to a traditional IRA, they are deductible by the participant. Later distributions from the IRA upon retirement are fully taxed.

*a. True b. False

1095. By itself, credit card receipts will not constitute adequate substantiation for travel expenses.

*a. True b. False

1096. The Federal per diem rates that can be used for “deemed substantiated” purposes are the same for all locations in the country.

a. True *b. False

1097. For self-employed taxpayers, travel expenses are not subject to the 2%-of-AGI floor.

*a. True b. False

1098. A taxpayer who claims the standard deduction will not avoid the 2% floor on unreimbursed employee expenses.

a. True *b. False

1099. Employees who render an adequate accounting to the employer and are fully reimbursed will shift the 50% cutback adjustment to their employer.

*a. True b. False

1100. Aiden performs services for Lucas. Which, if any, of the following factors indicate that Aiden is an employee, rather than an independent contractor?

a. Aiden provides his own support services (e.g., work assistants). b. Aiden obtained his training (i.e., job skills) from his father. *c. Aiden is paid based on hours worked. d. Aiden makes his services available to others. e. None of the above.

1101. Jordan performs services for Ryan. Which, if any, of the following factors indicate that Jordan is an independent contractor, rather than an employee?

a. Ryan sets the work schedule. b. Ryan provides the tools used. c. Jordan files a Form 2106 with his Form 1040. *d. Jordan is paid based on tasks performed. e. None of the above.

1102. Which, if any, of the following factors is not a characteristic of independent contractor status?

*a. Work-related expenses are reported on Form 2106. b. Receipt of a Form 1099 reporting payments received. c. Workplace fringe benefits are not available. d. Services are performed for more than one party. e. None of the above.

1103. A worker may prefer to be treated as an independent contractor (rather than an employee) for which of the following reasons:

a. Avoids the cutback adjustment as to business meals. b. All of the self-employment tax is deductible for income tax purposes. *c. Work-related expenses are not subject to the 2%-of-AGI floor. d. A Schedule C does not have to be filed. e. None of the above.

1104. A worker may prefer to be classified as an employee (rather than an independent contractor) for which of the following reasons:

a. To claim unreimbursed work-related expenses as a deduction for AGI. *b. To avoid the self-employment tax. c. To avoid the cutback adjustment on unreimbursed business entertainment expenses. d. To avoid the 2%-of-AGI floor on unreimbursed work-related expenses. e. None of the above.

1105. Statutory employees:

a. Report their expenses on Form 2106. b. Include common law employees. c. Are subject to income tax withholdings. *d. Claim their expenses as deductions for AGI. e. None of the above.

1106. Corey is the city sales manager for “RIBS,” a national fast food franchise. Every working day, Corey drives his car as follows:

Miles Home to office 20 Office to RIBS No. 1 15 RIBS No. 1 to No. 2 18 RIBS No. 2 to No. 3 13 RIBS No. 3 to home 30

Corey’s deductible mileage is:

a. 0 miles. b. 50 miles. c. 66 miles. d. 76 miles. *e. None of the above.

1107. Amy works as an auditor for a large major CPA firm. During the months of September through November of each year, she is permanently assigned to the team auditing Garnet Corporation. As a result, every day she drives from her home to Garnet and returns home after work. Mileage is as follows:

Miles Home to office 10 Home to Garnet 30 Office to Garnet 35

For these three months, Amy’s deductible mileage for each workday is:

a. 0. b. 30. c. 35. *d. 60. e. None of the above.

1108. Aaron is a self-employed practical nurse who works out of his home. He provides nursing care for disabled persons living in their residences. During the day he drives his car as follows.

Miles Aaron’s home to 12 patient Louise Patient Louise to 4 patient Carl Patient Carl to 6 patient Betty Patient Betty to 10 Aaron’s home

Aaron’s deductible mileage for each workday is:

a. 10 miles. b. 12 miles. c. 20 miles. d. 22 miles. *e. 32 miles.

1109. When using the automatic mileage method, which, if any, of the following expenses also can be claimed?

a. Engine tune-up. *b. Parking. c. Interest on automobile loan. d. MACRS depreciation. e. None of the above.

1110. In which, if any, of the following situations is the automatic mileage available?

a. A limousine to be rented by the owner for special occasions (e.g., weddings, high school proms). b. The auto belongs to taxpayer’s mother. c. One of seven cars used to deliver pizzas. d. MACRS statutory percentage method has been claimed on the automobile. *e. None of the above.

1111. Under the actual cost method, which, if any, of the following expenses will not be allowed?

a. Car registration fees. b. Auto insurance. *c. Interest expense on a car loan (taxpayer is an employee). d. Dues to auto clubs. e. All of the above will be allowed.

1112. Dave is the regional manager for a national chain of auto-parts stores and is based in Salt Lake City. When the company opens new stores in Boise, Dave is given the task of supervising their initial operation. For three months, he works weekdays in Boise and returns home on weekends. He spends $350 returning to Salt Lake City but would have spent $410 had he stayed in Boise for the weekend. As to the weekend trips, how much, if any, qualifies as a deduction?

a. $0, since the trips are personal and not work related. b. $0, since Dave’s tax home has changed from Salt Lake City to Boise. c. $60. *d. $350. e. $410.

1113. Allowing for the cutback adjustment (50% reduction for meals and entertainment), which of the following trips, if any, will qualify for the travel expense deduction?

*a. Dr. Jones, a general dentist, attends a two-day seminar on developing a dental practice. b. Dr. Brown, a surgeon, attends a two-day seminar on financial planning. c. Paul, a romance language high school teacher, spends summer break in France, Portugal, and Spain improving his language skills. d. Myrna went on a two-week vacation in Boston. While there, she visited her employer’s home office to have lunch with former co- workers. e. All of the above.

1114. During the year, John went from Milwaukee to Alaska on business. Preceding a five-day business meeting, he spent four days vacationing at the beach. Excluding the vacation costs, his expenses for the trip are:

Air fare $3,200 Lodging 900 Meals 800 Entertainment 600

Presuming no reimbursement, deductible expenses are:

a. $3,200. b. $3,900. *c. $4,800. d. $5,500. e. None of the above.

1115. During the year, Sophie went from Omaha to Lima (Peru) on business. She spent four days on business, two days on travel, and four days on vacation. Disregarding the vacation costs, Sophie’s unreimbursed expenses are:

Air fare $3,000 Lodging 800 Meals 600 Entertainment 400

Sophie’s deductible expenses are:

a. $4,300. *b. $3,100. c. $2,800. d. $2,500. e. None of the above.

1116. During the year, Walt travels from Seattle to Tokyo (Japan) on business. His time was spent as follows: 2 days travel (one day each way), 2 days business, and 2 days personal. His expenses for the trip were as follows (meals and lodging reflect only the business portion):

Air fare $3,000 Lodging 2,000 Meals and entertainment 1,000

Presuming no reimbursement, Walt’s deductible expenses are:

a. $3,500. b. $4,500. *c. $5,500. d. $6,000. e. None of the above.

1117. In terms of meeting the distance test for purposes of deducting moving expenses, which of the following statements is correct?

a. The taxpayer’s new job location must be at least 50 miles away from the old job. b. The taxpayer’s new residence must be at least 50 miles away from the new job. c. The taxpayer’s new residence must be at least 50 miles away from the old residence. *d. The taxpayer’s new job location must be at least 50 miles farther from the old residence than the old residence was to the old job. e. None of the above.

1118. As to meeting the time test for purposes of deducting moving expenses, which of the following statements is correct?

*a. Work at the new location must involve a full-time job—part- time job will not suffice. b. The taxpayer has two years in which to satisfy the 39-weeks or 78-weeks requirement. c. The time test is waived for persons whose move follows retirement. d. The moving expense deduction cannot be claimed if the taxpayer has not yet met the time test. e. None of the above.

1119. Due to a merger, Allison transfers from Miami to Chicago. Under a new job description, she is reclassified from employee to independent contractor status. Her moving expenses, which are not reimbursed, are as follows:

Transportation $1,400 Meals 400 Lodging 500 Cost of moving household 4,000 goods Penalty for breaking 3,000 lease on Miami apartment

Allison’s deductible moving expense is:

a. $0. *b. $5,900. c. $6,100. d. $8,900. e. $9,300.

1120. Rachel is single and has a college degree in finance. She is employed as a loan officer at a bank; her yearly AGI approximates $50,000. During the year, she enrolled in a weekend MBA program and incurred the following nonreimbursed expenses: $4,100 (tuition), $300 (books), $200 (other school supplies), and $200 (transportation to and from campus). Disregarding the 2%-of-AGI limitation, as to the MBA program, Rachel has a:

a. Deduction for and deduction from AGI of $0. *b. Deduction for AGI of $4,000 and deduction from AGI of $800. c. Deduction for AGI of $4,000 and deduction from AGI of $700. d. Deduction for AGI of $4,100 and deduction from AGI of $700. e. None of the above.

1121. The § 222 deduction for tuition and related expenses is available:

a. Only if the taxpayer itemizes deductions from AGI. b. To deduct that portion of the tuition in excess of that allowed under the lifetime learning credit. c. To cover the tuition of a son who does not qualify as taxpayer’s dependent. d. Only if job related. *e. None of the above.

1122. The § 222 deduction for tuition and related expenses is available:

a. Regardless of the amount of a taxpayer’s MAGI. b. To cover room and board expenses to attend college. c. To a married taxpayer filing a separate return. *d. Even if a taxpayer does claim the standard deduction. e. None of the above.

1123. Which, if any, of the following is subject to a cutback adjustment?

*a. An airline pilot for an executive jet rental company who pays his own travel expenses. b. Meals provided at cost to employees by a cafeteria funded by the employer. c. Fourth of July company picnic for employees. d. A trip to Bermuda awarded to the company’s top salesperson. e. None of the above.

1124. Robert entertains several of his key clients on January 1 of the current year. Expenses paid by Robert are as follows:

Cab fare $ 60 Cover charge at supper 200 club Dinner at club 800 Tips to waiter 160

Presuming proper substantiation, Robert’s deduction is:

a. $610. *b. $640. c. $740. d. $1,220. e. None of the above.

1125. Tracy, the regional sales director for a manufacturer of exercise equipment, pays $2,500 to rent a skybox for a visiting performance of the Harlem Globetrotters. The skybox holds 10 seats, and Tracy invites 7 clients to the event. Nonluxury seats range in price from $80 to $120. The refreshments provided during the event cost $600. If Tracy meets all of the requirements for deductibility (i.e., business discussion, substantiation), she may deduct:

*a. $900. b. $1,100. c. $1,260. d. $1,500. e. $1,550.

1126. Ralph made the following business gifts during the year.

To Robert (a key client) at Christmas $50 To Angel (Robert’s 8-year old 20 daughter) on her birthday To Art (Ralph’s secretary) on his 30 birthday ($3 was for gift wrapping) To Paige (Ralph’s boss) at Christmas 40

Presuming proper substantiation, Ralph’s deduction is:

a. $0. *b. $53. c. $73. d. $78. e. $98.

1127. Which of the following expenses, if any, qualify as deductible?

a. Contributions to a Coverdell Education Savings Account (CESA). b. Contributions to a qualified tuition program (§ 529 plan). c. Job hunting expense of FBI agent who applies for the job of city manager of Beaumont (TX). *d. Contribution to a traditional IRA. e. None of the above.

1128. Which, if any, of the following expenses is subject to the 2%-of- AGI floor?

a. Qualified tuition expenses under § 222. b. Contribution to traditional IRA. c. Cost of a CPA exam review course—taxpayer just began employment with an accounting firm. d. Office in the home deduction for a self-employed taxpayer. *e. None of the above.

1129. Which, if any, of the following expenses is subject to the 2%-of- AGI floor?

a. Gambling losses (to the extent of gambling gains). b. Moving expenses (not reimbursed by employer). *c. Teaching supplies (in excess of $250) purchased by a fifth grade teacher. d. Union dues of self-employed machinist. e. None of the above.

1130. Which, if any, of the following expenses are not subject to the 2%-of-AGI floor?

a. Safety shoes purchased by an employed plumber. *b. Reimbursed employee expenses. Taxpayer-employee renders an adequate accounting to the employer. c. Unreimbursed employee expenses. d. Tax return preparation fee paid by a non-employed retiree. e. None of the above.

1131. In contrasting the reporting procedures of employees and self- employed persons regarding job-related transactions, which of the following items involve self-employed?

*a. Schedule C of Form 1040. b. Form 2106. c. Form W-2. d. Schedule A of Form 1040. e. None of the above.

1132. One of the tax advantages of being self-employed (rather than being an employee) is:

a. The self-employment tax is lower than the Social Security tax. b. The cutback adjustment does not apply. c. The actual cost method for deducting the business use of an automobile can be selected. *d. Job-related expenses are deductions for AGI. e. A deduction for an office in the home is available.

1133. Sue performs services for Lynn. Regarding this arrangement, use the legend provided to classify each statement.The services are performed at Sue’s premises.Sue does not work for other parties.Lynn determines when the services are to be performed.Sue has unreimbursed expenses.Sue was trained by Lynn.Sue uses her own helpers.Sue charges by the hour for her work.Sue files a Form 2106 with her Form 1040.Sue files a Schedule SE with her Form 1040.Sue uses her own tools.Indicates independent contractor status. Indicates employee status. Indicates employee status. Indicates independent contractor status. Indicates employee status. Indicates independent contractor status. Indicates employee status. Indicates employee status. Indicates independent contractor status. Indicates independent contractor status.

[a] 1. The services are performed at Sue’s premises. [b] 2. Sue does not work for other parties. [c] 3. Lynn determines when the services are to be performed. [d] 4. Sue has unreimbursed expenses. [e] 5. Sue was trained by Lynn. [f] 6. Sue uses her own helpers. [g] 7. Sue charges by the hour for her work. [h] 8. Sue files a Form 2106 with her Form 1040. [i] 9. Sue files a Schedule SE with her Form 1040. [j] 10. Sue uses her own tools.

a. Indicates independent contractor status. b. Indicates employee status. c. Indicates employee status. d. Indicates independent contractor status. e. Indicates employee status. f. Indicates independent contractor status. g. Indicates employee status. h. Indicates employee status. i. Indicates independent contractor status. j. Indicates independent contractor status.

1134. Match the statements that relate to each other. (Note: Choice L may be used more than once.)Characteristic of a taxpayer who has the status of an employeeCharacteristic of a taxpayer who is self- employedActual cost method of determining auto expenseAutomatic mileage methodDeductible moving expenseNondeductible moving expenseMixed use (both business and pleasure) foreign travelEducation expense that is not deductible to maintain or improve existing skillsTax home has changedDeductible job-hunting expensesDeduction by an employee of unreimbursed office-in-the-home expensesMixed use (both business and pleasure) domestic travel.Taxpayer has tools and helper provided for him Payment for services rendered based on tasks performed Can include actual cost of parking Excludes use of MACRS depreciation Lodging while in route Meals while in route Transportation must be allocated if taxpayer spends two weeks on business and one week sightseeing Paralegal obtains a law degree Out-of-town job assignment lasts for more than one year Must involve the same trade or business Must be for the convenience of the employer Correct match not provided

[a] 1. Characteristic of a taxpayer who has the status of an employee [b] 2. Characteristic of a taxpayer who is self-employed [c] 3. Actual cost method of determining auto expense [d] 4. Automatic mileage method [e] 5. Deductible moving expense [f] 6. Nondeductible moving expense [g] 7. Mixed use (both business and pleasure) foreign travel [h] 8. Education expense that is not deductible to maintain or improve existing skills [i] 9. Tax home has changed [j] 10. Deductible job-hunting expenses [k] 11. Deduction by an employee of unreimbursed office-in- the-home expenses [l] 12. Mixed use (both business and pleasure) domestic travel.

a. Taxpayer has tools and helper provided for him b. Payment for services rendered based on tasks performed c. Can include actual cost of parking d. Excludes use of MACRS depreciation e. Lodging while in route f. Meals while in route g. Transportation must be allocated if taxpayer spends two weeks on business and one week sightseeing h. Paralegal obtains a law degree i. Out-of-town job assignment lasts for more than one year j. Must involve the same trade or business k. Must be for the convenience of the employer l. Correct match not provided

1135. Match the statements that relate to each other. (Note: Choice L may be used more than once.)Actual cost method of determining car expenseDistance test (for moving expenses) not satisfiedTime test (for moving expenses) waivedRoth IRAsKeogh (H.R. 10) plansCutback adjustment appliesCutback adjustment does not applyDeemed substantiationJob hunting expensesQualified tuition and related expenses (under § 222)Club dues deductibleClub dues not deductibleCan include cost of car insurance and automobile club dues Taxpayer moves to a new residence 55 miles closer to his present job Expatriate (U.S. person who is employed overseas) returns home to retire Distribution from plan is not taxable Distribution from plan is taxable Cover charge paid to entertain client at a night club. Company picnic sponsored by employer Use of Federal per diem allowance to substantiate meals while in travel status Deductible even if taxpayer does not take the new job Does not have to be job related Correct match not provided Country club membership fee

[a] 1. Actual cost method of determining car expense [b] 2. Distance test (for moving expenses) not satisfied [c] 3. Time test (for moving expenses) waived [d] 4. Roth IRAs [e] 5. Keogh (H.R. 10) plans [f] 6. Cutback adjustment applies [g] 7. Cutback adjustment does not apply [h] 8. Deemed substantiation [i] 9. Job hunting expenses [j] 10. Qualified tuition and related expenses (under § 222) [k] 11. Club dues deductible [l] 12. Club dues not deductible

a. Can include cost of car insurance and automobile club dues b. Taxpayer moves to a new residence 55 miles closer to his present job c. Expatriate (U.S. person who is employed overseas) returns home to retire d. Distribution from plan is not taxable e. Distribution from plan is taxable f. Cover charge paid to entertain client at a night club. g. Company picnic sponsored by employer h. Use of Federal per diem allowance to substantiate meals while in travel status i. Deductible even if taxpayer does not take the new job j. Does not have to be job related k. Correct match not provided l. Country club membership fee

1136. Meredith holds two jobs and attends graduate school on weekends. The education improves her skills, but does not qualify her for a new trade of business. Before going to the second job, she returns home for dinner. Relevant mileage is as follows?

Home to first job 9 miles First job to second job 13 miles Home to second job 15 miles Home to university 7 miles

How much of the mileage qualifies for deduction purposes?

Correct Answer: 27 miles. The mileage from the first job to the second job, 13 miles, qualifies. Also, round trip mileage to the university (14 miles) can be claimed if the education Meredith is obtaining is deductible.

1137. Paul is employed as an auditor by a CPA firm. On most days, he commutes by auto from his home to the office. During one month, however, he has an extensive audit assignment closer to home. For this engagement, Paul drives directly from home to the client’s premises and back. Mileage information is summarized below:

Home to 12 miles office Office to 15 miles audit client Audit client 10 miles to home

If Paul spends 20 days on the audit, what is his deductible mileage?

Correct Answer: 400 miles [20 miles (each day) × 20 days].

1138. During 2013, Eva used her car as follows: 12,000 miles (business), 1,400 miles (commuting), and 4,000 miles (personal). In addition, she spent $440 for tolls (business) and $620 for parking (business). If Eva uses the automatic mileage method, what is the amount of her deduction?

Correct Answer: $7,840. (12,000 miles × $0.565) + $440 + $620 = $7,840.

1139. Rod uses his automobile for both business and personal use and claims the automatic mileage rate for all purposes. During 2013, his mileage was as follows:

Miles Driven Personal 4,000 Business 8,000 Qualifying moving 3,500 expense Medical 1,800 Charitable 1,500 Qualifying education 800 (MBA program)

How much can Rod claim for mileage?

Correct Answer: $6,454 [(8,800 miles × $0.565, business and education) + (3,500 miles × $0.24, moving) + (1,800 miles × $0.24, medical) + (1,500 miles × $0.14, charitable)].

1140. Elsie lives and works in Detroit. She is the regional sales manager for a national fast-food chain. Due to unusual developments, she is compelled to work six straight weeks in the Cleveland area. Instead of spending the weekend there, she flies home every Friday night and returns early Monday morning. The cost of coming home for the weekend approximates $600. Had she stayed in Cleveland, deductible meals and lodging would have been $700. How much, if any, may Elsie deduct as to each weekend?

Correct Answer: $600 for each weekend, the lesser of the $600 cost of returning home or the $700 cost of staying in Cleveland.

1141. Alfredo, a self-employed patent attorney, flew from his home in Chicago to Miami, had lunch alone at the airport, conducted business in the afternoon, and returned to Chicago in the evening. His expenses were as follows:

Airfare $900 Airport parking 60 (Chicago) Lunch 30 Taxis (Miami) 42

What is Alfredo’s deductible expense for the trip?

Correct Answer: $1,002 ($900 + $60 + $42). As Alfredo was not away from home, his lunch is not deductible.

1142. Lily went from her office in Portland to Lisbon on business. While there, she spent part of the time on vacation. How much of the air fare of $5,000 can she deduct based on the following assumptions: a. Lily was gone five days (i.e., three business and two personal). b. Lily was gone five weeks (i.e., four business and one personal). c. Lily was gone five weeks (i.e., three business and two personal).

Correct Answer: a. $5,000. b. $5,000. c. $3,000. (60% × $5,000).

Transportation costs for mixed use (i.e., both business and personal) need not be allocated as long as domestic trips are involved. Such allocation is necessary, however, for foreign trips unless one of two exceptions applies. One exception deals with trips lasting seven days or less and covers part a. (but not part b. or part c.) above. The second exception, less than 25% of the time was for personal use, applies to part b. but not to part c. Thus, in part b. no allocation is necessary, while in part c. the air fare must be allocated.

1143. Noah moved from Delaware to Arizona to accept a better job. He incurred the following unreimbursed moving expenses:

Transportation $3,000 Lodging 400 Meals in route 500 Penalty for breaking 1,000 apartment lease Forfeiture of athletic 500 club membership Moving company cost 7,000

What is Noah’s moving expense deduction?

Correct Answer: $10,400 ($3,000 + $400 + $7,000). Meals ($500), lease penalty ($1,000), and membership forfeiture ($500) are not qualifying moving expenses.

1144. After graduating from college, Clint obtained employment in Omaha. In moving from his parents’ home in Baltimore to Omaha, Clint incurred the following expenses:

Transportation $ 700 Meals 380 Lodging 400 Cost of moving household goods 3,500

a. How much may Clint deduct as moving expense? b. Would any deduction be allowed if Clint claimed the standard deduction for the year of the move? Explain.

Correct Answer: a. $4,600 ($700 + $400 + $3,500). No deduction is allowed for meals. b. Yes. Moving expenses are deductions for AGI and can be claimed regardless of whether a taxpayer takes the standard deduction or chooses to itemize.

1145. Arnold is employed as an assistant manager in the furniture division of a national chain of department stores. He is a recent college graduate with a degree in marketing. During 2013, he enrolls in the evening MBA program of a local university and incurs the following expenses: tuition, $4,200; books and computer supplies, $800; transportation expense to and from the university, $450; and meals while on campus, $400. Arnold is single and his annual AGI is $64,000. As to these expenses, what are Arnold’s: a. Deductions for AGI? b. Deductions from AGI?

Correct Answer: a. $4,000. Although the tuition was $4,200, § 222 imposes a limitation of $4,000. b. $1,450. $200 (excess tuition not allowed under § 222) + $800 (books and computer supplies) + $450 (transportation) = $1,450. No deduction is allowed for meals, since Arnold is not in travel status. Further adjustment will be required due to the 2%-of-AGI limitation on certain itemized deductions.

1146. Cathy takes five key clients to a nightclub and incurs the following costs: $320 limousine rental, $460 cover charge, $920 drinks and dinner, and $200 tips. Several days after the function, Cathy mails each client a pen costing $25. To the cost must be added $4 for engraving of the client’s name. Assuming adequate substantiation and a business justification, what is Cathy’s deduction?

Correct Answer: $1,255. $320 + [($460 + $920 + $200) × 50%] = $1,110. Also allowed are the gifts of $145 (5 × $29). The cost of engraving ($4) can be added to the maximum amount of gift allowed ($25).

1147. For the current football season, Tern Corporation pays $40,000 for a skybox (containing 12 seats) at Veterans Stadium for eight home games. Regular nonskybox seats at each game range from $120 to $150 a seat. In November, a Tern employee and ten clients use the skybox to attend a game. The event is preceded by a bona fide business discussion, and Tern spent $1,000 for food and drinks during the game. What is Tern’s deduction?

Correct Answer: $1,400. (12 seats × $150) + $1,000 = $2,800 × 50% (cutback adjustment) = $1,400. Even though only ten clients attended, all available sets are considered. The highest cost of a regular seat ($150) is used in the computation.

1148. Brian makes gifts as follows:

Recipient Cost of Gift Mr. Brown (a client) $27* Mrs. Brown (Mr. Brown’s wife) 15 Ms. Smith (Brian’s 30 receptionist) Mr. Jones (Brian’s boss) 40

* Includes $2 for engraving

Presuming adequate substantiation and no reimbursement, how much may Brian deduct?

Correct Answer: $52 ($27 + $25). The deduction for Mr. Brown’s gift can include nominal costs for gift-wrapping and engraving. No deduction is allowed for the gift to Mrs. Brown unless she is in a separate business. Otherwise, she falls under the $25 limit applicable to Mr. Brown. The deduction for Ms. Smith’s gift is limited to $25. No deduction is allowed for Brian’s gift to his boss.

1149. Rocky has a full-time job as an electrical engineer for the city utility. In his spare time, Rocky repairs TV sets in the basement of his personal residence. Most of his business comes from friends and referrals from former customers, although occasionally he runs an ad in the local suburbia newspaper. Typically, the sets are dropped off at Rocky’s house and later picked up by the owner when notified that the repairs have been made.

The floor space of Rocky’s residence is 2,500 square feet, and he estimates that 20% of this is devoted exclusively to the repair business. Gross income from the business is $13,000, while expenses (other than home office) are $5,000. Expenses relating to the residence are as follows:

Real property taxes $4,500 Interest on home mortgage 8,000 Operating expenses of 3,000 residence Depreciation (based on 20% 1,000 business use)

What is Rocky’s net income from the repair business?

Correct Answer: Business income $13,000 Less: Expenses (5,000 ) Net business income before $ 8,000 home office expenses Less: Real property taxes ($4,500 (900) × 20%) Mortgage interest ($8,000 × (1,600) 20%) Operating expenses ($3,000 (600) × 20%) Depreciation (1,000 ) Net income from business $ 3,900

1150. In the current year, Bo accepted employment with a Kansas City law firm after graduating from law school. Her expenses for the year are listed below:

Cost of bar review course $1,500 Qualified moving expenses 2,100 Job hunting expenses 1,800 Subscriptions to legal 450 journals Membership dues in bar 620 associations State bar dues 300

Since Bo worked just part of the year, her salary was only $32,100. In terms of deductions from AGI, how much does Bo have?

Correct Answer: $770. AGI is $30,000 [$32,100 (salary) – $2,100 (moving expenses)]. Thus, the 2%-of-AGI floor is $600 ($30,000 × 2%). Qualifying itemized deductions are $1,370 ($450 + $620 + $300). Thus, $1,370 – $600 = $770 is the allowable itemized deduction. No deduction is allowed for the bar review course and the job hunting expenses. In this case, it is apparent that Bo should claim the standard deduction and not itemize.

1151. For the current year, Wilbur is employed as a deputy sheriff of a county. He has AGI of $50,000 and the following unreimbursed employee expenses:

Attendance at a law enforcement $1,020 conference ($800 for travel and $220 for meals) Uniforms (does not include $300 for 1,000 laundry and dry cleaning) Professional dues and journals 300 Contribution to sheriff’s reelection 400 campaign (highly recommended by Wilbur’s boss)

How much of these expenses are allowed as deductions from AGI?

Correct Answer: $1,510. $910 + $1,300 + $300 = $2,510 – (2% × $50,000) = $1,510. A cutback adjustment of $110 ($220 × 50%) is required for the meals obtained during the conference. No deduction is available for the campaign contribution.

1152. If a business retains someone to provide services, that person may either be an employee or be self-employed (i.e., independent contractor). a. What are the tax advantages to the business of having the service provider classified as self-employed? b. What are the advantages and disadvantages to the service provider of self-employed status?

Correct Answer: a. Self-employed persons do not have to be included in various fringe benefits programs (e.g., group-term life insurance, accident and health plans) and retirement plans. Because they are not covered by FICA and FUTA, these payroll costs are avoided. b. The advantages are that expenses qualify as deductions for AGI. Consequently, deductibility is not affected by the choice of the standard deduction, and no reduction is required for the 2%-of-AGI floor. In terms of disadvantages, being self-employed can lead to other state and local taxes (e.g., license fees, franchise taxes, occupation taxes, gross receipts levy). The most significant, however, is the self-employment tax that materializes.

1153. In terms of income tax treatment, what is the difference between common law and statutory employees?

Correct Answer: Although subject to Social Security tax, statutory employees are not subject to Income tax withholding. Statutory employees may claim all of their unreimbursed expenses as deductions for AGI. For common law employees, such expenses are deductions from AGI.

1154. Myra’s classification of those who work for her as independent contractors is being questioned by the IRS. It is the position of the IRS that these workers are really employees. What type of factors can Myra utilize to justify her classification?

Correct Answer: Myra needs to show that she has a reasonable basis for not treating her workers as employees. In this regard, can she prove reliance on any of the following?

• Judicial precedent, published ruling, or technical advice.

• A past IRS audit that resulted in no employment tax assessment.

• A longstanding practice of independent contractor status in the same industry.

Has Myra consistently treated these workers as independent contractors? Has she reported their earnings by filing Form 1099-MISC?

1155. Taylor performs services for Jonathan on a regular basis. There exists considerable doubt as to whether Taylor is an employee or an independent contractor. a. What can Jonathan do to clarify the matter? b. Suppose Jonathan treats Taylor as an independent contractor but Taylor thinks she is an employee. What is Taylor’s recourse, if any?

Correct Answer: a. Jonathan can obtain a ruling from the IRS by filing Form SS–8 (Determination of Worker Status for Purposes of Federal Employment Taxes and Income Tax Withholding). b. Taylor should file a Form 8919 (Uncollected Social Security and Medicare Tax on Wages) with the IRS.

1156. Jacob is a landscape architect who works out of his home. He wonders whether or not he will have nondeductible commuting expenses when he drives to the locations of his clients. Please comment.

Correct Answer: Jacob has no commuting expenses since his tax home is his residence. Thus, his mileage to and from the premises of his clients should be fully deductible.

1157. Under the automatic mileage method, one rate does not cover every type of expense. For 2013, what are the rates for business use, education, moving, charitable, and medical?

Correct Answer: $0.565 (business use); $0.565 (education); $0.24 (moving); $0.14 (charitable); and $0.24 (medical).

1158. Christopher just purchased an automobile for $40,000 which he plans to claim 100% as being for business use. In order to take advantage of MACRS and § 179, he plans to use the actual cost method for determining his deduction in the first year. In subsequent years, he will switch to the automatic mileage method. Comment on Christopher’s proposed approach.

Correct Answer: First, limitations are imposed on how much depreciation can be claimed over each year of the five-year write-off period. In most cases, such limitations significantly reduce the depreciation normally allowed. Second, the use of MACRS and/or § 179 precludes later converting to the automatic mileage method. Lastly, how many automobiles does Christopher own? If just one, 100% business use could be almost impossible to justify since it negates any personal use.

1159. Once set for a year, when might the IRS change the rate for the automatic mileage method?

Correct Answer: Changes in the past have been justified by significant increases in fuel prices. Such a change places a premium on keeping track of mileage on a monthly basis.

1160. Travel status requires that the taxpayer be away from home overnight. a. What does “away from home overnight” mean? b. What tax advantages result from being in travel status?

Correct Answer: a. “Home” for this purpose is the place of taxpayer’s principal employment. “Overnight” need not be a 24-hour period, but it must be a time duration substantially longer than an ordinary day’s work such as to require rest or sleep. b. If “travel status” exists, many otherwise nondeductible expenses (e.g., meals, lodging, transportation) become deductible.

1161. When is a taxpayer’s work assignment in a new locale temporary? Permanent? What difference does it make?

Correct Answer: Temporary indicates that the assignment’s termination is expected within a reasonably short period of time. In no event can the period of absence exceed one year. If the taxpayer’s assignment is indefinite, and not temporary, his or her tax home changes and travel status ends.

1162. Nick Lee is a linebacker for the Baltimore Ravens (a professional football club). During the football season he rents an apartment in a Baltimore suburb. The rest of the time he lives with his family in Ann Arbor (MI) and works at a local bank as a vice president in charge of public relations. Can Nick deduct his expenses while away from Ann Arbor? Explain.

Correct Answer: Probably not. Although his personal residence is in Ann Arbor, Nick’s tax home is likely to be the Baltimore area. Considering the salaries of professional football players, Nick’s income from the Ravens must heavily outweigh that received from an Ann Arbor bank. Furthermore, his principal activity and the time spent (e.g., conditioning, training, playing) is that of a player and not a banker.

1163. How are combined business/pleasure trips treated for travel within the United States as opposed to foreign travel?

Correct Answer: The major difference is that transportation charges are fully deductible if the trip is primarily for business and within the North American area. If the foreign trip is primarily business, transportation expenses must be allocated between business and personal unless (1) the taxpayer was away from home for seven days or less or (2) if less than 25% of the time was spent on personal pursuits.

1164. Concerning the deduction for moving expenses, what circumstances, if any, will excuse a taxpayer from meeting the time test of 39 or 78 weeks?

Correct Answer: The time test will be disregarded if the taxpayer dies, becomes disabled, or is discharged (other than for willful misconduct) or transferred by the new employer. Under certain conditions, expatriates who retire while overseas are exempt from the time test.

1165. A taxpayer just changed jobs and incurred unreimbursed moving expenses. a. What are his (or her) options if an income tax return has to be filed before the time test is met? b. What happens if the moving expense deduction has been claimed and the time test is not later satisfied?

Correct Answer: a. The taxpayer may claim the deduction right away or wait until the time test is met and file an amended return for the year of the move. b. If the moving expense is claimed and the time test is not later met, a taxpayer has two options. First, he can increase the income for the following year by the amount of the deduction claimed. Second, he can file an amended return for the year of the move in which the deduction is not claimed.

1166. Nicole just retired as a partner in a Philadelphia law firm. She moved to San Francisco where she took a job as an adjunct professor at a local law school. Can Nicole deduct her moving expenses? Explain

Correct Answer: She can deduct her moving expenses if her status as an adjunct professor is full-time employment. She cannot if her status is regarded as part-time employment.

1167. In terms of IRS attitude, what do the following expenses have in common? a. Cost of a CPA exam review course. b. Cost of a review course for the bar exam. c. Cost of a law degree by a taxpayer who does not intend to practice law.

Correct Answer: a. It is the position of the IRS that the costs associated with becoming a CPA are incurred in order to acquire a basic skill and thus are not deductible as education expenses. b. The same reasoning as noted in part a. above applies to review course taken to pass the bar exam. c. Regardless of a taxpayer’s career goals, the IRS considers a law degree to be the acquisition of a basic skill.

1168. Madison and Christopher are staff accountants at a major public accounting firm. Both are pursuing a law degree in a night program at local law school. Madison contends that the cost of the program is a deduction for income tax purposes, while Christopher maintains that it is not. Who is correct and why?

Correct Answer: Both are partly correct and incorrect. The IRS takes the position that pursuing a law degree is not maintaining or improving existing skills but results in acquiring a new job skill. Thus, a legal education is not job related to accounting but constitutes a new trade or business. However, § 222 allows a deduction for tuition (up to $4,000 or less) even if the education is not job related. This provision allows some or all of the law school tuition to be deductible but not other associated expenses (e.g., textbooks, transportation). In summary, some costs can be claimed and some cannot.

1169. Logan, Caden, and Olivia are three unrelated parties who claim the standard deduction. All are married and attend Citron University and each pays tuition of $6,100. Of this payment, Logan can claim a deduction of $4,000; Caden a deduction of $2,000; and Olivia no deduction at all. Explain.

Correct Answer: The most apparent explanation is the limitation imposed on the § 222 tuition deduction. Logan’s MAGI must be less than $130,000, while Caden’s MAGI is more than $130,000 but not more than $160,000. Lastly, Olivia’s MAGI must be in excess of $160,000.

1170. Meg teaches the fifth grade at a local school. During the year, she spends $1,200 for school supplies for use in her classroom. On her income tax return, some of this expense is not reported and the balance is deducted in two different places. Explain what has probably happened.

Correct Answer: Meg probably has been reimbursed for some of these expenses. If she renders an adequate accounting to her employer, the reimbursement is not reported in her income nor is the expense involved deducted. As to the remaining expenses, she has claimed $250 as a deduction for AGI and the balance as a miscellaneous itemized deduction.

1171. In connection with the office in the home deduction, comment on the following: a. Mixed use (i.e., business and personal) of the portion of the home allocated to business. b. The difference between direct and indirect expenses for deduction purposes. c. The classification of the expense (i.e., dfor or dfrom AGI).

Correct Answer: a. In most cases, mixed use of the portion allocated to business is not allowed. Consequently, the office must be exclusively used for business. An exception is made, however, when the home is used as a daycare center. Here, mixed use is allowed. b. Direct expenses (e.g., painting the office) are deductible in full because they only benefit the office area. Indirect expenses (e.g., homeowners casualty insurance) must be allocated since they relate to both the business and personal portions of the home. c. Like most job-related expenses, the taxpayer’s status (i.e., employed, self- employed) governs the classification of the expense. Thus, an office in the home expense to an employee is a deduction from AGI, while that of a self-employed taxpayer is a deduction for AGI.

1172. Faith just graduated from college and she needs advice on the tax treatment of the costs she incurs in connection with her first job (a sales person for a pharmaceutical company). Specifically, she wants to know about the following items: a. Job search costs. b. Business wardrobe cost. c. Moving expenses. d. Deduction for office in the home.

Correct Answer: a. Not deductible since it involves Faith’s first job. b. Not deductible since the clothes are suitable for street wear. c. As long as the time and distance requirements are met, the moving expenses are deductible. d. Unless Faith’s employer has an office available for her in the locale where she works, she can qualify for an office in the home deduction. This result is consistent with the convenience of the employer requirement applicable in the case of employees.

1173. Regarding tax favored retirement plans for employees and self- employed persons, comment on the following: a. The exclusion versus deduction approaches as to contributions by participants. b. Tax-free accumulation of earnings. c. The deferral of income tax consequences. d. Employee versus self-employed status.

Correct Answer: a. Contributions by employees to retirement plans that are qualified provide a tax benefit either in the form of an exclusion from gross income or as a deduction for AGI. The traditional IRA takes the deduction approach. This is also the case of contributions by a self-employed person to a Keogh (H.R. 10) plan. b. A characteristic of all tax-favored retirement plans is that income accumulates free of tax. This allows for a greater accumulation to take place up to the point of distribution. c. Except in the Roth IRA, the main objective of retirement plans is to defer taxation until distributions are made. d. Keogh (H.R. 10) plans are, in effect, the self-employed version of employee retirement plans.

1174. Ashley and Matthew are husband and wife and both are practicing CPAs. On a joint return, Ashley gets to deduct her professional dues but Matthew does not. Explain.

Correct Answer: Most likely Ashley is self-employed, while Matthew is employed. Thus, Ashley’s expenses are deductions for AGI and Matthew’s are itemized deduction subject to the 2% floor. If they do not itemize (or if they do and cannot exceed the 2% floor), Matthew loses the deduction for his professional dues.

1175. Isabella is a dental hygienist who works for five different dentists. She spends one day a week (i.e., Monday through Friday) with each. All of the dentists except Dr. Stanki (the Wednesday assignment) treat her as an employee. Dr. Stanki, however, classifies her as being self-employed. Comment on this discrepancy in treatment.

Correct Answer: Unless Isabella’s Wednesday duties are significantly different than the rest of the week, Dr. Stanki has placed himself in a vulnerable position. In terms of the proper work classification of a service provider, one key consideration is how they are treated by other comparable businesses. In all likelihood, therefore, Dr. Stanki will have a tough time convincing the IRS that Isabella is an independent contractor when four of his colleagues consider her to be an employee. More likely, Dr. Stanki desires to save on payroll taxes and avoid covering Isabella in any fringe benefits offered to other employees.

1176. Felicia, a recent college graduate, is employed as an accountant by an oil company. She would like to continue her education and obtain a law degree. Discuss Felicia’s tax status if she attends a local law school on a: a. Part-time basis. b. Full-time basis.

Correct Answer: a. Regarding the usual education expense justification of maintaining and improving on existing skills, Felicia will not succeed. The IRS has successfully maintained that obtaining a law degree leads to a new trade or business. Reliance on § 222, however, is appropriate since the education need not be job related. Unfortunately, § 222 only covers tuition, is limited to $4,000, and may be reduced (or eliminated) by an AGI limitation. b. If Felicia quits her job and returns to college on a full-time basis, she no longer is in a trade or business. A temporary break in employment status (e.g., leave of absence), however, will cure this problem if properly structured. However, Felicia will still encounter the new trade or business provision.

1177. Discuss the 2%-of-AGI floor and the 50% cutback limitation in connection with various employee expenses under the following arrangements: a. The employee is not reimbursed by the employer. b. The employee is fully reimbursed under a nonaccountable plan. c. The employee is partially reimbursed under an accountable plan. d. The employee is fully reimbursed under an accountable plan.

Correct Answer: a. Any cutback adjustment and the 2%-of-AGI floor are imposed on the employee. The employee has the burden of being able to substantiate the expenses involved. Furthermore, no deduction is available if the employee does not itemize deductions from AGI. b. The reimbursement must be reported as income. Deductions are handled as in part a. above. c. Unless the reimbursement identifies the expenses covered, it must be allocated between the items that are and are not subject to the cutback adjustment. d. A washout results. In effect the expenses are treated as deductions for AGI and fully offset the reimbursement. As far as the employee is concerned, the expenses are not subject to the cutback adjustment or the 2%-of-AGI floor.

1178. Ramon and Ingrid work in the field of public relations and incur sizable entertainment expenses. Ramon is employed by a consumer products company, while Ingrid is a self-employed consultant. Regarding the tax treatment of the entertainment expenses, when would: a. Ramon be better off than Ingrid? b. Ingrid be better off than Ramon?

Correct Answer: a. If Ramon renders an adequate accounting to his employer and is fully reimbursed. In such a case, the employee reports no income or deduction as to these expenses. The cutback adjustment is suffered by the employer. b. If Ramon does not render an adequate accounting to his employer, he will have to recognize any reimbursement as income and substantiate any deduction. Even worse, he will be subject to the cutback adjustment and the deduction will be a miscellaneous itemized deduction upon which the 2%-of-AGI limit is imposed. Although Ingrid also will be subject to the cutback adjustment, she has a deduction for AGI upon which the 2% rule does not apply.

1179. Personal expenditures that are deductible as itemized deductions include medical expenses, Federal income taxes, state income taxes, property taxes on a personal residence, mortgage interest, and charitable contributions.

a. True *b. False

1180. The election to itemize is appropriate when total itemized deductions are less than the standard deduction based on the taxpayer’s filing status.

a. True *b. False

1181. Adrienne sustained serious facial injuries in a motorcycle accident. To restore her physical appearance, Adrienne had cosmetic surgery. She cannot deduct the cost of this procedure as a medical expense.

a. True *b. False

1182. A physician recommends a private school for Ellen’s dependent child. Because of the physician’s recommendation, the cost of the private school will qualify as a medical expense deduction (subject to percentage limitations).

a. True *b. False

1183. Mindy paid an appraiser to determine how much a capital improvement made for medical reasons increased the value of her personal residence. The appraisal fee qualifies as a deductible medical expense.

a. True *b. False

1184. Upon the recommendation of a physician, Ed has a swimming pool installed at his residence because of a heart condition. If he is allowed to deduct all or part of the cost of the pool, Ed’s increase in utility bills due to the operation of the pool qualifies as a medical expense.

*a. True b. False

1185. Mason, age 70, a physically handicapped individual, pays $10,000 in 2013 for the installation of wheelchair ramps, support bars, and railings in his personal residence. These improvements increase the value of his personal residence by $2,000. Only $8,000 of the expenditure qualifies as a medical deduction (subject to the AGI floor).

a. True *b. False

1186. Chad pays the medical expenses of his son, James. James would qualify as Chad’s dependent except that he earns $7,500 during the year. Chad may claim James’ medical expenses even if he is not a dependent.

*a. True b. False

1187. Bill paid $2,500 of medical expenses for his daughter, Marie. Marie is married to John and they file a joint return. Bill can include the $2,500 of expenses when calculating his medical expense deduction.

*a. True b. False

1188. In 2013, Dena traveled 600 miles for specialized medical treatment that was not available in her hometown. She paid $90 for meals during the trip, $145 for a hotel room on Tuesday night, and $15 in parking fees. She did not keep records of other out-of-pocket costs for transportation. Dena can include $209 in computing her medical expenses.

*a. True b. False

1189. Maria traveled to Rochester, Minnesota, with her son, who was operated on at the Mayo Clinic. Her son stayed at the clinic for the duration of his treatment. She paid airfare of $300 and $50 per night for lodging. The cost of Maria’s airfare and lodging cannot be included in determining her medical expense deduction.

a. True *b. False

1190. In 2013, Brandon, age 72, paid $3,000 for long-term care insurance premiums. He may include the $3,000 in computing his medical expense deduction for the year.

*a. True b. False

1191. Jim’s employer pays half of the premiums on a group medical insurance plan covering all employees, and employees pay the other half. Jim can exclude the half of the premium paid by his employer from his gross income and may include the half he pays in determining his medical expense deduction.

*a. True b. False

1192. Matt, a calendar year taxpayer, pays $11,000 in medical expenses in 2013. He expects $5,000 of these expenses to be reimbursed by an insurance company in 2014. In determining his medical expense deduction for 2013, Matt must reduce his 2013 medical expenses by the amount of the reimbursement he expects in 2014.

a. True *b. False

1193. In 2014, Rhonda received an insurance reimbursement for medical expenses incurred in 2013. She is not required to include the reimbursement in gross income in 2014 if she claimed the standard deduction in 2013.

*a. True b. False

1194. Georgia contributed $2,000 to a qualifying Health Savings Account in 2013. The entire amount qualifies as an expense deductible for AGI.

*a. True b. False

1195. Shirley pays FICA (employer’s share) on the wages she pays her maid to clean and maintain Shirley’s personal residence. The FICA payment is not deductible as an itemized deduction.

*a. True b. False

1196. Fees for automobile inspections, automobile titles and registration, bridge and highway tolls, parking meter deposits, and postage are not deductible if incurred for personal reasons, but they are deductible as deductions for AGI if incurred as a business expense by a self-employed taxpayer.

*a. True b. False

1197. A taxpayer may not deduct the cost of new curbing (relative to a personal residence), even if the construction is required by the city and the curbing provides an incidental benefit to the public welfare.

*a. True b. False

1198. Sergio was required by the city to pay $2,000 for the cost of new curbing installed by the city in front of his personal residence. The new curbing was installed throughout Sergio’s neighborhood as part of a street upgrade project. Sergio may not deduct $2,000 as a tax, but he may add the $2,000 to the basis of his property.

*a. True b. False

1199. Trent sells his personal residence to Chester on July 1, 2013. He had paid $7,000 in real property taxes on March 1, 2013, the due date for property taxes for 2013. Trent may not deduct the portion of the taxes he paid for the period the property was owned by Chester.

*a. True b. False

1200. Herbert is the sole proprietor of a furniture store. He can deduct real property taxes on his store building but he cannot deduct state income taxes related to his net income from the furniture store as a business deduction.

*a. True b. False

1201. Grace’s sole source of income is from a restaurant that she owns and operates as a proprietorship. Any state income tax Grace pays on the business net income must be deducted as a business expense rather than as an itemized deduction.

a. True *b. False

1202. In April 2013, Bertie, a calendar year cash basis taxpayer, had to pay the state of Michigan additional income tax for 2012. Even though it relates to 2012, for Federal income tax purposes the payment qualifies as a tax deduction for tax year 2013.

*a. True b. False

1203. In January 2014, Pam, a calendar year cash basis taxpayer, made an estimated state income tax payment for 2013. The payment is deductible in 2013.

a. True *b. False

1204. Phyllis, a calendar year cash basis taxpayer who itemized deductions, overpaid her 2012 state income tax and is entitled to a refund of $400. Phyllis chooses to apply the $400 overpayment toward her state income taxes for 2013. She is required to recognize that amount as income in 2013.

*a. True b. False

1205. Tom, whose MAGI is $40,000, paid $3,500 of interest on a qualified student loan in 2013. Tom is single. He may deduct the $3,500 interest as an itemized deduction.

a. True *b. False

1206. For purposes of computing the deduction for qualified residence interest, a qualified residence includes only the taxpayer’s principal residence.

a. True *b. False

1207. For purposes of computing the deduction for qualified residence interest, a qualified residence includes the taxpayer’s principal residence and two other residences of the taxpayer or spouse.

a. True *b. False

1208. Interest paid or accrued during the tax year on aggregate acquisition indebtedness of $2 million or less ($1 million or less for married persons filing separate returns) is deductible as qualified residence interest.

a. True *b. False

1209. A taxpayer pays points to obtain financing to purchase a second residence. At the election of the taxpayer, the points can be deducted as interest expense for the year paid.

a. True *b. False

1210. Points paid by the owner of a personal residence to refinance an existing mortgage must be capitalized and amortized over the life of the new mortgage.

*a. True b. False

1211. Jack sold a personal residence to Steven and paid points of $3,500 on the loan to help Steven finance the purchase. Jack can deduct the points as interest.

a. True *b. False

1212. Letha incurred a $1,600 prepayment penalty to a lending institution because she paid off the mortgage on her home early. The $1,600 is deductible as interest expense.

*a. True b. False

1213. Leona borrows $100,000 from First National Bank and uses the proceeds to purchase City of Houston bonds. The interest Leona pays on this loan is deductible as investment interest subject to the investment interest limits.

a. True *b. False

1214. Joe, a cash basis taxpayer, took out a 12-month business loan on December 1, 2013. He prepaid all $3,600 of the interest on the loan on December 1, 2013. Joe can deduct only $300 of the prepaid interest in 2013.

*a. True b. False

1215. Sadie mailed a check for $2,200 to a qualified charitable organization on December 31, 2013. The $2,200 contribution is deductible on Sadie’s 2013 tax return.

*a. True b. False

1216. On December 31, 2013, Lynette used her credit card to make a $500 contribution to the United Way, a qualified charitable organization. She will pay her credit card balance in January 2014. If Lynette itemizes, she can deduct the $500 in 2013.

*a. True b. False

1217. Judy paid $40 for Girl Scout cookies and $40 for Boy Scout popcorn. Judy may claim an $80 charitable contribution deduction.

a. True *b. False

1218. For all of the current year, Randy (a calendar year taxpayer) allowed the Salvation Army to use a building he owns rent-free. The building normally rents for $24,000 a year. Randy will be allowed a charitable contribution deduction this year of $24,000.

a. True *b. False

1219. Al contributed a painting to the Metropolitan Art Museum of St. Louis, Missouri. The painting, purchased six years ago, was worth $40,000 when donated, and Al’s basis was $20,000. If this painting is immediately sold by the museum and the proceeds are placed in the general fund, Al’s charitable contribution deduction is $20,000 (subject to percentage limitations).

*a. True b. False

1220. During the year, Victor spent $300 on bingo games sponsored by his church. If all profits went to the church, Victor has a charitable contribution deduction of $300.

a. True *b. False

1221. In 2013, Allison drove 800 miles to volunteer in a project sponsored by a qualified charitable organization in Utah. In addition, she spent $250 for meals while away from home. In total, Allison may take a charitable contribution deduction of $112 (800 miles ´ $.14).

a. True *b. False

1222. During the year, Eve (a resident of Billings, Montana) spends three consecutive weeks in Louisville, Kentucky. One week is spent representing the Billings First Christian Church at the national convention, and two weeks are spent vacationing with relatives. One third of Eve’s travel expenses will qualify as a charitable deduction.

a. True *b. False

1223. In order to dissuade his pastor from resigning and taking a position with a larger church, Michael, an ardent leader of the congregation, gives the pastor a new car. The cost of the car is deductible by Michael as a charitable contribution.

a. True *b. False

1224. Dan contributed stock worth $16,000 to his college alma mater, a qualified charity. He acquired the stock eleven months ago for $4,000. He may deduct $16,000 as a charitable contribution deduction (subject to percentage limitations).

a. True *b. False

1225. Ronaldo contributed stock worth $12,000 to the Children’s Protective Agency, a qualified charity. He acquired the stock twenty months ago for $6,000. He may deduct $6,000 as a charitable contribution deduction (subject to percentage limitations).

a. True *b. False

1226. Any capital asset donated to a public charity that would result in long-term capital gain if sold, is subject to the 30%-of-AGI ceiling limitation on charitable contributions for individuals.

*a. True b. False

1227. John gave $1,000 to a family whose house was destroyed by fire. John may claim a charitable deduction of $1,000 on his tax return for the current year.

a. True *b. False

1228. In the year of her death, Maria made significant charitable contributions of capital gain property. In fact, the amount of the contributions exceeds 30% of her AGI. Maria’s executor can elect to deduct charitable contributions of up to 50% of Maria’s AGI on Maria’s final income tax return.

*a. True b. False

1229. The reduced deduction election enables a taxpayer to move from the 30%-of-AGI limitation to the 50%-of-AGI limitation.

*a. True b. False

1230. Excess charitable contributions that come under the 30%-of-AGI ceiling are always subject to the 30%-of-AGI ceiling in the carryover year.

*a. True b. False

1231. Contributions to public charities in excess of 50% of AGI may be carried back 3 years or forward for up to 5 years.

a. True *b. False

1232. Employee business expenses for travel qualify as itemized deductions subject to the 2% floor if they are not reimbursed.

*a. True b. False

1233. Gambling losses may be deducted to the extent of the taxpayer’s gambling winnings. Such losses are subject to the 2% floor for miscellaneous itemized deductions.

a. True *b. False

1234. The phaseout of certain itemized deductions has been reinstated for years beginning in 2013.

*a. True b. False

1235. Edna had an accident while competing in a rodeo. She sustained facial injuries that required cosmetic surgery. While having the surgery done to restore her appearance, she had additional surgery done to reshape her chin, which was not injured in the accident. The surgery to restore her appearance cost $9,000 and the surgery to reshape her chin cost $6,000. How much of Edna’s surgical fees will qualify as a deductible medical expense (before application of the AGI limitation)?

a. $0. b. $6,000. *c. $9,000. d. $15,000. e. None of the above.

1236. Fred and Lucy are married, ages 33 and 32, and together have AGI of $120,000 in 2013. They have four dependents and file a joint return. They pay $5,000 for a high deductible health insurance policy and contribute $2,600 to a qualified Health Savings Account. During the year, they paid the following amounts for medical care: $9,200 in doctor and dentist bills and hospital expenses, and $3,000 for prescribed medicine and drugs. In October 2013, they received an insurance reimbursement of $4,400 for the hospitalization. They expect to receive an additional reimbursement of $1,000 in January 2014. Determine the maximum deduction allowable for medical expenses in 2013.

*a. $800. b. $3,400. c. $9,200. d. $12,800. e. None of the above.

1237. Richard, age 50, is employed as an actuary. For calendar year 2013, he had AGI of $130,000 and paid the following medical expenses:

Medical insurance premiums $5,300 Doctor and dentist bills for 7,900 Derrick and Jane (Richard’s parents) Doctor and dentist bills for 5,100 Richard Prescribed medicines for Richard 830 Nonprescribed insulin for Richard 960

Derrick and Jane would qualify as Richard’s dependents except that they file a joint return. Richard’s medical insurance policy does not cover them. Richard filed a claim for $4,800 of his own expenses with his insurance company in November 2013 and received the reimbursement in January 2014. What is Richard’s maximum allowable medical expense deduction for 2013?

a. $0. *b. $7,090. c. $13,000. d. $20,090. e. None of the above.

1238. Sandra is single and does a lot of business entertaining at home. Because Arthur, Sandra’s 80-year old dependent grandfather who lived with Sandra, needs medical and nursing care, he moved to Twilight Nursing Home. During the year, Sandra made the following payments on behalf of Arthur:

Room at Twilight $4,500 Meals for Arthur at 850 Twilight Doctor and nurse fees 700 Cable TV service for 107 Arthur’s room Total $6,157

Twilight has medical staff in residence. Disregarding the AGI floor, how much, if any, of these expenses qualify for a medical deduction by Sandra?

a. $6,157. *b. $6,050. c. $5,200. d. $1,550. e. None of the above.

1239. Phillip, age 66, developed hip problems and was unable to climb the stairs to reach his second-floor bedroom. His physician advised him to add a first-floor bedroom to his home. The cost of constructing the room was $32,000. The increase in the value of the residence as a result of the room addition was determined to be $17,000. In addition, Phillip paid the contractor $5,500 to construct an entrance ramp to his home and $8,500 to widen the hallways to accommodate his wheelchair. Phillip’s AGI for 2013 was $100,000. How much of these expenditures can Phillip deduct as a medical expense in 2013?

a. $14,000. b. $15,000. *c. $21,500. d. $29,000. e. None of the above.

1240. Quinn, who is single and lives alone, is physically handicapped as a result of a diving accident. In order to live independently, he modifies his personal residence at a cost of $30,000. The modifications included widening halls and doorways for a wheelchair, installing support bars in the bathroom and kitchen, installing a stairway lift, and rewiring so he could reach electrical outlets and appliances. Quinn pays $200 for an appraisal that places the value of the residence at $129,000 before the improvements and $140,000 after. As a result of the operation of the stairway lift, Quinn experienced an increase of $680 in his utility bills for the current year. Disregarding the percentage of AGI limitation, how much of the above expenditures qualify as medical expense deductions?

a. $11,680. *b. $30,680. c. $30,880. d. $34,880. e. None of the above.

1241. Brad, who would otherwise qualify as Faye’s dependent, had gross income of $9,000 during the year. Faye, who had AGI of $120,000, paid the following medical expenses in 2013:

Cataract operation for Brad $ 5,400 Brad’s prescribed contact 1,800 lenses Faye’s doctor and dentist 12,600 bills Prescribed drugs for Faye 2,55 0 Total $22,350

Assuming Faye is age 45, she has a medical expense deduction of:

a. $3,150. b. $4,950. *c. $10,350. d. $13,350. e. None of the above.

1242. Tom, age 48, is advised by his family physician that he needs back surgery to correct a problem from his last back surgery. Since Tom is in a wheel chair, he needs his wife, Jean, to accompany him on his trip to Rochester, Minnesota, for in-patient treatment at the Mayo Clinic, which specializes in this type of surgery. Tom incurred the following costs in 2013:

Round-trip airfare ($350 $ 700 each) Jean’s hotel in Rochester 380 for four nights ($95 per night) Jean’s meals while in 105 Rochester Tom’s medical treatment 3,500 Tom’s prescription medicine 600

Compute Tom’s medical expenses for the trip (subject to the 10% floor).

a. $4,000. *b. $5,000. c. $5,180. d. $5,285. e. None of the above.

1243. Your friend Scotty informs you that he received a “tax-free” reimbursement in 2013 of some medical expenses he paid in 2012. Which of the following statements best explains why Scotty is not required to report the reimbursement in gross income?

a. Scotty itemized deductions in 2012. *b. Scotty did not itemize deductions in 2012. c. Scotty itemized deductions in 2013. d. Scotty did not itemize deductions in 2013. e. Scotty itemized deductions in 2013 but not in 2012.

1244. In 2013, Boris pays a $3,800 premium for high-deductible medical insurance for himself and his family. In addition, he contributes $3,400 to a Health Savings Account. Which of the following statements is true?

*a. If Boris is self-employed, he may deduct $7,200 as a deduction for AGI. b. If Boris is self-employed, he may deduct $3,400 as a deduction for AGI and may include the $3,800 premium when calculating his itemized medical expense deduction. c. If Boris is an employee, he may deduct $7,200 as a deduction for AGI. d. If Boris is an employee, he may include $7,200 when calculating his itemized medical expense deduction. e. None of the above.

1245. During 2013, Hugh, a self-employed individual, paid the following amounts:

Real estate tax on Iowa residence $3,800 State income tax 1,700 Real estate taxes on land in 1,100 Puerto Rico (held as an investment) Gift tax paid on gift to daughter 1,200 State sales taxes 1,750 State occupational license fee 300 Property tax on value of his 475 automobile (used 100% for business)

What is the maximum amount Hugh can claim as taxes in itemizing deductions from AGI?

a. $6,600. *b. $6,650. c. $7,850. d. $8,625. e. None of the above.

1246. During 2013, Nancy paid the following taxes:

Tax on residence (for the period from $5,250 March 1 through August 31, 2013) State motor vehicle tax (based on the 430 value of the personal use automobile) State sales tax 3,500 State income tax 3,050

Nancy sold her personal residence on June 30, 2013, under an agreement in which the real estate taxes were not prorated between the buyer and the seller. What amount qualifies as a deduction from AGI for 2013 for Nancy?

a. $9,180. b. $9,130. *c. $7,382. d. $5,382. e. None of the above.

1247. In Lawrence County, the real property tax year is the calendar year. The real property tax becomes a personal liability of the owner of real property on January 1 in the current real property tax year (assume this year is not a leap year). The tax is payable on June 1. On May 1, Reggie sells his house to Dana for $350,000. On June 1, Dana pays the entire real estate tax of $7,950 for the year ending December 31. How much of the property taxes may Reggie deduct?

a. $0. *b. $2,614. c. $2,625. d. $7,950. e. None of the above.

1248. Brad, who uses the cash method of accounting, lives in a state that imposes an income tax (including withholding from wages). On April 14, 2013, he files his state return for 2012, paying an additional $600 in state income taxes. During 2013, his withholdings for state income tax purposes amount to $3,550. On April 13, 2014, he files his state return for 2013 claiming a refund of $800. Brad receives the refund on June 3, 2014. If he itemizes deductions, how much may Brad claim as a deduction for state income taxes on his Federal income tax return for calendar year 2013 (filed in April 2014)?

a. $3,350. b. $3,550. *c. $4,150. d. $5,150. e. None of the above.

1249. Barry and Larry, who are brothers, are equal owners in Chickadee Corporation. On July 1, 2013, each loans the corporation $10,000 at an annual interest rate of 10%. Both shareholders are on the cash method of accounting, while Chickadee Corporation is on the accrual method. All parties use the calendar year for tax purposes. On June 30, 2014, Chickadee repays the loans of $20,000 together with the specified interest of $2,000. How much of the interest can Chickadee Corporation deduct in 2013?

*a. $0. b. $500. c. $1,000. d. $2,000. e. None of the above.

1250. Rick and Carol Ryan, married taxpayers, took out a mortgage of $160,000 when purchasing their home ten years ago. In October of the current year, when the home had a fair market value of $200,000 and they owed $125,000 on the mortgage, the Ryans took out a home equity loan for $110,000. They used the funds to purchase a sailboat to be used for recreational purposes. The sailboat does not qualify as a residence. What is the maximum amount of debt on which the Ryans can deduct home equity interest?

*a. $75,000. b. $90,000. c. $110,000. d. $125,000. e. None of the above.

1251. Joseph and Sandra, married taxpayers, took out a mortgage on their home for $350,000 in 1991. In May of this year, when the home had a fair market value of $450,000 and they owed $250,000 on the mortgage, they took out a home equity loan for $220,000. They used the funds to purchase a single engine airplane to be used for recreational travel purposes. What is the maximum amount of debt on which they can deduct home equity interest?

a. $50,000. *b. $100,000. c. $220,000. d. $230,000. e. None of the above.

1252. Pedro’s child attends a school operated by the church the family attends. Pedro made a donation of $1,000 to the church in lieu of the normal registration fee of $200. In addition, Pedro paid the regular tuition of $6,000 to the school. Based on this information, what is Pedro’s charitable contribution?

a. $0. *b. $800. c. $1,000. d. $6,800. e. $7,000.

1253. In 2013, Jerry pays $8,000 to become a charter member of Mammoth University’s Athletic Council. The membership ensures that Jerry will receive choice seating at all of Mammoth’s home basketball games. Also in 2013, Jerry pays $2,200 (the regular retail price) for season tickets for himself and his wife. For these items, how much qualifies as a charitable contribution?

a. $6,200. *b. $6,400. c. $8,000. d. $10,200. e. None of the above.

1254. Emily, who lives in Indiana, volunteered to travel to Louisiana in March to work on a home-building project for Habitat for Humanity (a qualified charitable organization). She was in Louisiana for three weeks. She normally makes $500 per week as a carpenter’s assistant and plans to deduct $1,500 as a charitable contribution. In addition, she incurred the following costs in connection with the trip: $600 for transportation, $1,200 for lodging, and $400 for meals. What is Emily’s deduction associated with this charitable activity?

a. $600. b. $1,200. c. $1,800. *d. $2,200. e. $3,700.

1255. Hannah makes the following charitable donations in the current year:

Basis Fair Market Value Inventory held for resale in Hannah’s business $8,000 $ 7,200 (a sole proprietorship) Stock in HBM, Inc., held as an investment (acquired 16,000 40,000 four years ago) Baseball card collection held as an investment 4,000 20,000 (acquired six years ago)

The HBM stock and the inventory were given to Hannah’s church, and the baseball card collection was given to the United Way. Both donees promptly sold the property for the stated fair market value. Disregarding percentage limitations, Hannah’s current charitable contribution deduction is:

a. $28,000. *b. $51,200. c. $52,000. d. $67,200. e. None of the above.

1256. Byron owned stock in Blossom Corporation that he donated to a museum (a qualified charitable organization) on June 8 this year. What is the amount of Byron’s deduction assuming that he had purchased the stock for $10,500 last year on August 7, and the stock had a fair market value of $13,800 when he made the donation?

a. $3,300. *b. $10,500. c. $12,150. d. $13,800. e. None of the above.

1257. Zeke made the following donations to qualified charitable organizations during 2013:

Basis Fair Market Value Used clothing (all acquired before 2012) of $ 1,350 $ 375 taxpayer and his family Stock in ABC, Inc., held as an investment for 12,000 10,875 fifteen months Stock in MNO, Inc., held as an investment for 15,000 18,000 eleven months Real estate held as an 15,000 30,000 investment for two years

The used clothing was donated to the Salvation Army; the other items of property were donated to Eastern State University. Both are qualified charitable organizations. Disregarding percentage limitations, Zeke’s charitable contribution deduction for 2013 is:

a. $43,350. *b. $56,250. c. $59,250. d. $60,375. e. None of the above.

1258. Karen, a calendar year taxpayer, made the following donations to qualified charitable organizations in 2013:

Basis Fair Market Value Cash donation to State $30,000 $ 30,000 University Unimproved land to the 70,000 210,000 City of Terre Haute, Indiana

The land had been held as an investment and was acquired 4 years ago. Shortly after receipt, the City of Terre Haute sold the land for $210,000. Karen’s AGI is $450,000. The allowable charitable contribution deduction is:

a. $84,000 if the reduced deduction election is not made. b. $100,000 if the reduced deduction election is not made. *c. $165,000 if the reduced deduction election is not made. d. $170,000 if the reduced deduction election is made. e. None of the above.

1259. During 2013, Ralph made the following contributions to the University of Oregon (a qualified charitable organization):

Cash $63,000 Stock in Raptor, Inc. (a 94,500 publicly traded corporation)

Ralph acquired the stock in Raptor, Inc., as an investment fourteen months ago at a cost of $42,000. Ralph’s AGI for 2013 is $189,000. What is Ralph’s charitable contribution deduction for 2013?

a. $56,700. b. $63,000. *c. $94,500. d. $157,500. e. None of the above.

1260. Pat died this year. Before she died, Pat gave 5,000 shares of stock in Coyote Corporation (a publicly traded corporation) to her church (a qualified charitable organization). The stock was worth $180,000 and she had acquired it as an investment four years ago at a cost of $150,000. In the year of her death, Pat had AGI of $300,000. In completing her final income tax return, how much of the charitable contribution should Pat’s executor deduct?

a. $90,000. *b. $150,000. c. $180,000. d. $210,000. e. None of the above.

1261. Which of the following items would be an itemized deduction on Schedule A of Form 1040 not subject to the 2%-of-AGI floor?

a. Professional dues paid by an accountant (employed by Ford Motor Co.) to the National Association of Accountants. *b. Gambling losses to the extent of gambling winnings. c. Job hunting costs. d. Appraisal fee paid to a valuation expert to determine the fair market value of art work donated to a qualified museum. e. None of the above.

1262. Paul, a calendar year married taxpayer, files a joint return for 2013. Information for 2013 includes the following:

AGI $175,000 State income taxes 13,500 State sales tax 3,000 Real estate taxes 18,900 Gambling losses (gambling 6,800 gains were $12,000)

Paul’s allowable itemized deductions for 2013 are:

a. $13,500. b. $32,400. *c. $39,200. d. $42,200. e. None of the above.

1263. Marilyn, age 38, is employed as an architect. For calendar year 2013, she had AGI of $204,000 and paid the following medical expenses:

Medical insurance premiums $ 7,800 Doctor bills for Peter and 7,300 Esther (Marilyn’s parents) Doctor and dentist bills for 11,100 Marilyn Prescription medicines for 750 Marilyn Nonprescription insulin for 950 Marilyn

Peter and Esther would qualify as Marilyn’s dependents except that they file a joint return. Marilyn’s medical insurance policy does not cover them. Marilyn filed a claim for reimbursement of $6,000 of her own expenses with her insurance company in December 2013 and received the reimbursement in January 2014. What is Marilyn’s maximum allowable medical expense deduction for 2013?

Correct Answer: Marilyn’s medical expense deduction is $7,500, determined as follows:

Medical insurance $ 7,800 premiums Doctor and dentist bills 7,300 for Peter and Esther Doctor and dentist bills 11,100 for Marilyn Prescription medicines 750 for Marilyn Nonprescription insulin 9 for Marilyn 50 Total medical expenses $27,900 Less: 10% of $204,000 (20,400) (AGI) Deductible portion of $ 7,500 medical expenses

Although Peter and Esther cannot be claimed as Marilyn’s dependents, they could have been had they not filed a joint return. Therefore, they qualify for the medical expense deduction. Insulin is an exception to the rule that nonprescribed drugs do not qualify as medical expenses. The insurance recovery was not received until 2014. Therefore, it has no effect on the medical expense deduction for 2013.

1264. Aaron, age 45, had AGI of $40,000 for 2013. He was injured in a skiing accident and paid $3,600 for hospital expenses and $1,400 for doctor bills. Aaron also incurred medical expenses of $1,200 for his child, who lives with his former wife and is claimed as a dependent by her. In 2014, Aaron was reimbursed $1,300 by his insurance company for the medical expenses attributable to the skiing accident. a. Compute Aaron’s deduction for medical expenses in 2013. b. Assume that Aaron would have elected to itemize his deductions even if he had no medical expenses in 2013. How much, if any, of the $1,300 reimbursement must be included in gross income in 2014? c. Assume that Aaron’s other itemized deductions in 2013 were $7,000 and that he filed as a head of household. How much of the $1,300 reimbursement must he include in gross income in 2014?

Correct Answer: a. Aaron can claim medical expenses he paid for his child, even though his former wife is the custodial parent. His deduction for medical expenses in 2013 is computed as follows:

Hospitalization $3,600 Bills for doctor’s services 1,400 Medical expenses for child 1,200 Total $6,200 Less: 10% of $40,000 AGI (4,000) Medical expense deduction (assuming $2,200 Aaron itemizes his deductions) b. If the reimbursement for medical care had occurred in 2013, the medical expense deduction would have been only $900 [$6,200 (total medical expenses) – $1,300 (reimbursement) – $4,000 (floor)], and Aaron would have paid more income tax. Since the reimbursement was made in a subsequent year, Aaron would include $1,300 in gross income for 2014. If Aaron had not itemized in 2013, he would not include the $1,300 reimbursement in 2014 gross income because he would have received no tax benefit in 2013. c. Aaron’s deduction for medical expenses in 2013 would have been $2,200 (see computation in a. above). He would include the reimbursement in gross income to the extent of his $250 tax benefit, as computed below:

Other itemized deductions $7,000 Medical expenses in excess of 10% 2,200 floor Total itemized deductions $9,200 Standard deduction for head of (8,950) household in 2013 Tax benefit from medical expense $ 250 deduction

1265. During 2013, Kathy, who is self-employed, paid $650 per month for an HSA contract that provides medical insurance coverage with a $3,000 deductible. The plan covers Kathy, her husband, and their three children. Of the $650 monthly fee, $300 was for the high-deductible policy, and $350 was deposited into an HSA. How much of the amount paid for the high-deductible policy can Kathy deduct as a deduction for AGI?

Correct Answer: Because Kathy is self-employed, she can deduct $3,600 ($300 per month × 12 months) of the amount paid for the high-deductible policy as a deduction for AGI (refer to Example 10). In addition, she may deduct the $4,200 ($350 per month × 12 months) paid to the HSA as a deduction for AGI. Thus, Kathy may deduct $7,800 ($3,600 + $4,200) for AGI.

1266. In 2013, Shirley sold her personal residence to Mike for $400,000. Before the sale, Shirley paid the real estate taxes of $7,030 for the calendar year. For income tax purposes, the deduction is apportioned as follows: $4,000 to Shirley and $3,030 to Mike. a. What is Mike’s basis in the residence? b. What is Shirley’s amount realized from the sale of the residence? c. What amount of real estate taxes can Mike deduct? d. What amount of real estate taxes can Shirley deduct?

Correct Answer: General discussion. For Federal income tax purposes, real estate taxes must be apportioned between the buyer and the seller. The taxes paid by the seller that are apportioned to the buyer affect both the basis of the buyer’s property and the amount realized by the seller. a. Mike’s basis in the residence is $396,970 [$400,000 (purchase price) – $3,030 (property taxes allocated to Mike but paid by Shirley)]. b. Shirley’s amount realized is $396,970 [$400,000 (sales price) – $3,030 (property taxes allocated to Mike but paid by Shirley)]. c. Mike can deduct the $3,030 apportioned to him, even though the tax was paid by Shirley. d. Shirley can deduct only the tax apportioned to her of $4,000, even though she paid the entire amount of $7,030.

1267. Brian, a self-employed individual, pays state income tax payments of:

$900 on January 15, 2013 (4th estimated tax payment for 2012) $1,000 on April 15, 2013 (1st estimated tax payment in 2013) $1,000 on June 17, 2013 (2nd estimated tax payment in 2013) $1,000 on September 16, 2013 (3rd estimated tax payment in 2013) $800 on January 15, 2014 (4th estimated tax payment of 2013)

Brian had a tax overpayment of $500 on his 2012 state income tax return and applied this to his 2013 state income taxes. What is the amount of Brian’s state income tax itemized deduction for his 2013 Federal income tax return?

Correct Answer: $4,400 is the itemized deduction. $900 + $1,000 + $1,000 + $1,000 + $500 (overpayment).

1268. In Piatt County, the real property tax year is the calendar year. The real property tax becomes a personal liability of the owner of real property on January 1 and is payable on July 1 in the real property tax year. On June 30 of this year (assume not a leap year), Harry sells his house to Judy for $110,000 and on July 1, Judy pays the entire real estate tax of $4,380 for the current year ending December 31. a. How much of the property taxes may Harry deduct? b. How much of the property taxes may Judy deduct?

Correct Answer: General discussion. Real property taxes must be apportioned, regardless of whether the buyer or seller pays the taxes. In making the apportionment, the date of sale counts as a day the property is owned by the buyer. a. Rate of tax per day $ 12 ($4,380/365) Days Harry owned property 180 (January 1 - June 29) Taxes apportioned to Harry $2,160 ($12 × 180) b. Rate of tax per day $ 12 ($4,380/365) Days Judy owned property 185 (June 30 - December 31) Taxes apportioned to Judy $2,220 ($12 × 185)

1269. In 2006, Ross, who is single, purchased a personal residence for $170,000 and took out a mortgage of $100,000 on the property. In May of the current year, when the residence had a fair market value of $220,000 and Ross owed $70,000 on the mortgage, he took out a home equity loan for $110,000. He used the funds to purchase a BMW for himself and a Lexus SUV for his wife. For both vehicles, 100% of the use is for personal activities. What is the maximum amount on which Ross can deduct home equity interest?

Correct Answer: Interest is deductible only on the portion of a home equity loan that does not exceed the lesser of:

• The fair market value of the residence, reduced by the acquisition indebtedness ($220,000 FMV – $70,000 acquisition indebtedness = $150,000).

• $100,000 ($50,000 for married persons filing separate returns).

Ross can deduct all of the interest on the first mortgage since it is acquisition indebtedness. Of the $110,000 home equity loan, interest on $100,000 is deductible as home equity interest.

1270. Georgia had AGI of $100,000 in 2013. She donated Heron Corporation stock with a basis of $8,500 to a qualified charitable organization on July 5, 2013. a. What is the amount of Georgia’s deduction, assuming that she purchased the stock on December 4, 2012, and the stock had a fair market value of $15,000 when she made the donation? b. Assume the same facts as in a., except that Georgia purchased the stock on July 1, 2005. c. Assume the same facts as in a., except that the stock had a fair market value of $6,000 (rather than $15,000) when Georgia donated it to the charity.

Correct Answer: General discussion. The deduction for a contribution of capital gain property is based on the fair market value, while the deduction for a contribution of ordinary income property is equal to the lesser of the adjusted basis or the fair market value. a. Because Georgia did not hold the stock for the long-term holding period (December 4, 2012 - July 5, 2013), it is short-term capital gain property that is subject to the rules for ordinary income property. Therefore, her deduction is limited to $8,500. b. Georgia held the stock for the long-term holding period (July 1, 2005 - July 5, 2013); so it is capital gain property. Therefore, her deduction is equal to the fair market value of the stock, $15,000. c. The deduction for a contribution of loss property (FMV is less than adjusted basis) is limited to the fair market value. Therefore, Georgia’s deduction is $6,000.

1271. Linda, who has AGI of $120,000 in 2013, contributes stock in Mauve Corporation (a publicly traded corporation) to the Salvation Army, a qualified charitable organization. The stock is worth $65,000, and Linda acquired it as an investment four years ago at a cost of $50,000. a. What is the total amount that Linda can deduct as a charitable contribution, assuming she carries over any disallowed contribution from 2013 to future years? b. What is the maximum amount that Linda can deduct as a charitable contribution in 2013? c. What factors should Linda consider in deciding how to treat the contribution for Federal income tax purposes? d. Assume Linda dies in December 2013. What advice would you give the executor of her estate with regard to possible elections that can be made relative to the contribution?

Correct Answer: General discussion. The stock is appreciated long-term capital gain property. The general rule limits the deduction for the contribution of such property to 30% of AGI. However, under the reduced deduction election, a taxpayer may choose to forgo a deduction of the appreciation on capital gain property. This enables the taxpayer to move from the 30% limitation to a 50% limitation. a. Linda can deduct a total of $65,000, the fair market value of the stock. The deduction for 2013 is limited to $36,000 (30% of $120,000 AGI). The remaining $29,000 can be carried forward and deducted in the future, subject to the same percentage limitations. b. If Linda makes the reduced deduction election, she can deduct $50,000 in 2013, but she will forgo a deduction for the $15,000 appreciation ($65,000 FMV – $50,000 adjusted basis). c. Although the reduced deduction election appears attractive, it should be considered carefully. The election sacrifices a deduction for the appreciation on capital gain property that might eventually be allowed. Linda should do a present value analysis to compare the value of a deduction of $50,000 in 2013 versus the value of a $36,000 deduction in 2013 plus $29,000 of deductions to be carried over to future years. d. If Linda dies in December 2013, her executor should make the reduced deduction election, which would yield a charitable contribution deduction of $50,000. If the election is not made, the deduction will be $36,000 (30% of $120,000) and the $29,000 carryover will be lost because the 2013 return will be the final return for Linda.

1272. George is single and age 56, has AGI of $255,000, and incurs the following expenditures in 2013.

Medical expenses (before 10% $27,000 floor) Interest on home mortgage 15,500 State income tax 7,500 State sales tax 4,500 Real estate tax 8,600 Charitable contribution 6,500

What is the amount of itemized deductions George may claim?

Correct Answer: George is subject to the overall limitation on certain itemized deductions because his AGI exceeds the $250,000 threshold for single filers. His itemized deductions in 2013 after application of the overall limitation are computed below:

Itemized deductions subject to overall limitation: Interest on home $15,500 mortgage State income tax 7,500 Real estate tax 8,600 Charitable 6,50 contributions 0 Total $38,100 Reduction equals the lesser of the following: [3% × ($255,000 AGI – $ $250,000)] 150 80% of itemized deductions subject to limitation ($38,100 × 80%) 30,480 Amount of reduction (150) Deductible itemized $37,950 deductions subject to overall limitation Itemized deductions not subject to overall limitation: Medical expenses 1,50 [$27,000 – (10% × 0 $255,000 AGI)] Total itemized $39,450 deductions

The state sales tax is not deductible because George deducts the (higher) state income tax expense.

1273. For calendar year 2013, Jon and Betty Hansen (ages 59 and 60) file a joint return reflecting AGI of $280,000. They incur the following expenditures:

Medical expenses before AGI $30,000 floor Casualty loss (not covered 30,000 by insurance) before statutory floors Interest on home mortgage 10,000 Interest on credit cards 800 Property taxes on home 13,000 Charitable contributions 17,000 State income tax 15,000 Tax return preparation fees 1,20 0

What is the amount of itemized deductions the Hansens may claim?

Correct Answer: For the medical expenses, the taxpayers are allowed $2,000 [$30,000 – (10% × $280,000 AGI)]. The casualty loss must first be reduced by $100 and then by $28,000 (10% × $280,000 AGI). Thus, only $1,900 [$30,000 – $28,100 ($100 + $28,000)] can be deducted. Also, note that the tax return preparation fees are miscellaneous itemized deductions subject to the 2% floor. The floor of $5,600 (2% × $280,000 AGI) reduces the $1,200 to $0.

The Hansens are not subject to the overall limitation on certain itemized deductions because their AGI does not exceed the $300,000 threshold for joint filers.

The itemized deductions total $58,900 ($10,000 mortgage interest + $13,000 property tax + $17,000 contributions + $15,000 state income tax + $2,000 medical + $1,900 casualty).

1274. Charles, who is single and age 61, had AGI of $400,000 during 2013. He incurred the following expenses and losses during the year.

Medical expenses before 10%- $39,500 of-AGI limitation State and local income taxes 5,200 Real estate taxes 4,400 Home mortgage interest 5,400 Charitable contributions 4,800 Casualty loss before 10% 47,000 limitation (after $100 floor) Unreimbursed employee expenses 8,900 subject to 2%-of-AGI limitation Gambling losses (Charles had 9,800 $7,400 of gambling income)

Compute Charles’s total itemized deductions for the year.

Correct Answer: Charles’s itemized deductions are computed below:

Medical expenses [$39,500 – $ (10% × $400,000 AGI)] –0– State and local income taxes 5,200 Real estate taxes 4,400 Home mortgage interest 5,400 Charitable contributions 4,800 Casualty loss [$47,000 – (10% 7,000 × $400,000 AGI)] Unreimbursed employee expenses 900 [$8,900 – (2% × $400,000 AGI)] Gambling losses ($9,800 loss 7, limited to $7,400 of gambling 400 income) Total itemized deductions $35,100

Charles’s itemized deductions in 2013 after application of the overall limitation are computed below:

Itemized deductions subject to overall limitation: State and local income $ 5,200 taxes Real estate taxes 4,400 Home mortgage interest 5,400 Charitable contributions 4,800 Unreimbursed employee 9 expenses 00 Total itemized deductions $20,700

Reduction equals the lesser of the following: [3% × ($400,000 AGI – $ 4 $250,000)] ,500 80% of itemized deductions subject to limitation ($20,700 × 80%) 16 ,560 Amount of reduction (4, 500) Deductible itemized $16,200 deductions subject to overall limitation Itemized deductions not subject to overall limitation: Casualty loss 7,000 Gambling loss 7 ,400 Total itemized deductions $30,600

1275. Helen pays nursing home expenses of $3,000 per month for her mother. The monthly charge covers the following items: $1,400 for medical care, $900 for lodging, and $700 for food. Under what circumstances can Helen include the $3,000 per month payment when computing her medical expense deduction for the year? If Helen is not allowed to include the entire payment, how much can she include?

Correct Answer: Helen may include the entire amount paid to the nursing home ($3,000 per month) if the primary reason for being in the nursing home is to get medical care. If the primary reason for being in the nursing home is personal, Helen may include only the $1,400 cost of medical care in calculating her medical expense deduction.

1276. Manny, age 57, developed a severe heart condition, and his physician advised him to install an elevator in his home. The cost of installing the elevator was $15,000, and the increase in the value of the residence was determined to be $5,800. Manny’s AGI for the year was $52,000. a. How much of the expenditure can Manny deduct as a medical expense? b. Assume the same facts as in part a., except that Manny was paralyzed in an automobile accident and the expenditures were incurred to build entrance and exit ramps and widen the hallways in his home to accommodate his wheelchair. How much of the expenditure can Manny deduct as a medical expense?

Correct Answer: a. A capital improvement that ordinarily would not have a medical purpose qualifies as a medical expense if it is directly related to prescribed medical care and is deductible to the extent that the expenditure exceeds the increase in value of the related property. The medical expense deduction is $4,000 [($15,000 – $5,800) – ($52,000 × 10%)]. b. The full cost of certain home-related capital expenditures incurred to enable a physically handicapped individual to live independently and productively qualifies as a medical expense. Qualifying costs include expenditures for constructing entrance and exit ramps to the residence, widening hallways and doorways to accommodate wheelchairs, installing support bars and railings in bathrooms and other rooms, and adjusting electrical outlets and fixtures. These expenditures are subject to the 10% floor only, and the increase in the home’s value is deemed to be zero. Manny’s deduction is $9,800 [$15,000 – ($52,000 ×10%)].

1277. Samuel, a 36 year old individual who has been physically handicapped for a year, paid $15,000 for the installation of wheelchair ramps, support bars, railings, and widening doorways in his personal residence. These improvements increased the value of his personal residence by $4,000. How much of Samuel’s expenditures qualify as a medical expense deduction (subject to the 10% floor)? Explain.

Correct Answer: The full cost of certain home-related capital expenditures incurred to enable a physically handicapped individual to live independently and productively qualifies as a medical expense. Qualifying costs include expenditures for constructing entrance and exit ramps to the residence, widening hallways and doorways to accommodate wheelchairs, installing support bars and railings in bathrooms and other rooms, and adjusting electrical outlets and fixtures. These expenditures are subject to the 10% floor only, and the increase in the home’s value is deemed to be zero. So even though the improvements increased the value of Samuel’s personal residence by $4,000, the entire $15,000 of the expenditure qualifies as a medical expense deduction (subject to the 10% floor).

1278. Paul and Patty Black (both are age 66) are married and together have AGI of $140,000 in 2013. They have two dependents and file a joint return. During the year, they paid $8,000 for medical insurance, $15,000 in doctor bills and hospital expenses, and $1,000 for prescribed medicine and drugs. a. In December 2013, the Blacks received an insurance reimbursement of $3,500 for hospitalization expenses. Determine the deduction allowable for medical expenses paid during the year. b. Assume instead that the Blacks received the $3,500 insurance reimbursement in February 2014. Determine the deduction allowable for medical expenses incurred in 2013. c. Assume that the Blacks received the $3,500 insurance reimbursement in February 2014. Discuss whether the reimbursement will be included in their gross income for 2014.

Correct Answer: General discussion. All of the following expenses are deductible, subject to the 7.5% floor: $8,000 for medical insurance, $15,000 in doctor bills and hospital expenses, and $1,000 for prescribed medicine and drugs. a. Assuming Paul and Patty received the insurance reimbursement in December 2013, their medical expense deduction would be $6,000, computed as follows:

Medical insurance $ 8,000

Doctor bills and hospital 15,000 expenses Prescribed medicine and 1,0 drugs 00 Total medical expenses $24,000 incurred Minus: December 2013 (3,500) reimbursement Total medical expenses $20,500 after reimbursement Minus: 7.5% × $140,000 AGI (10,500) Medical expense deduction $10,000

b. Assuming Paul and Patty received the insurance reimbursement in February 2014, they could ignore the reimbursement in computing their 2013 medical expense deduction. Their medical expense deduction would be $13,500, computed as follows:

Medical insurance $ 8,000 Doctor bills and hospital 15,000 expenses Prescribed medicine and 1,0 drugs 00 Total medical expenses $24,000 incurred Minus: 7.5% × $140,000 AGI (10,500) Medical expense deduction $13,500

c. If Paul and Patty itemized in 2013, they would report the reimbursement as gross income in 2014, to the extent they received a tax benefit from itemizing in 2013. If they did not itemize in 2013 (i.e., took the standard deduction), they would not be required to report the reimbursement as gross income in 2014.

1279. Harry and Sally were divorced three years ago. In July of the current year, their son, Joe, broke his arm falling out of a tree. Joe lives with Sally and Sally claims him as a dependent on her tax return. Harry paid for the medical expenses related to Joe’s injury. Can Harry claim the medical expenses he paid for Joe on his tax return?

Correct Answer: Harry may be able to include the payments related to Joe’s injury with his own medical expenses. For divorced parents with children, the noncustodial parent may claim any medical expenses he or she pays even though the custodial parent claims the children as dependents. This rule applies if the dependency exemption could have been shifted to the noncustodial parent by the custodial parent’s waiver (see Chapter 3).

1280. For the past several years, Jeanne and her two sisters have taken turns claiming a dependency exemption deduction for their mother under a multiple support agreement. This year Jeanne will be entitled to the exemption, and her mother needs money for surgery and new eyeglasses. Should Jeanne pay for the medical expenses as her share of her mother’s expenses? How would this benefit Jeanne?

Correct Answer: Since Jeanne is claiming her mother as a dependent on her tax return this year, she is allowed to deduct any medical expenses she pays for her. Jeanne can claim the medical expenses as part of her portion of her mother’s support.

1281. Linda is planning to buy Vicki’s home. They want to keep the transaction simple, so the sales agreement will not apportion the property taxes that Vicki has already paid on the home. Comment on the tax implications for Linda and Vicki.

Correct Answer: Real estate taxes are apportioned between buyer and seller based upon the number of days the property is owned by each. Linda and Vicki will each be able to deduct taxes based on the number of days they each own the home. When the seller pays the entire real estate tax bill, adjustments are made to the amount realized by the seller and the basis of the home in the hands of the buyer. Vicki will reduce the amount realized on the sale by the amount of taxes apportioned to Linda, and Linda will reduce her basis in the home by the same amount.

1282. Linda borrowed $60,000 from her parents for a down payment on a condominium. She paid interest of $5,500 in 2011, $0 in 2012, and $9,000 in 2013. The IRS disallowed the deduction. Can you offer any explanation for the disallowance?

Correct Answer: Because of the irregular patterns of Linda’s payments, it does not appear that this is a bona fide loan. Thus, the amounts paid could not be interest. Additionally, the interest would not represent deductible qualified residence interest unless the loan was secured by the condominium.

1283. Diane contributed a parcel of land to the United Way. In addition, she contributed bibles and song books from her proprietorship’s book store inventory to First Church, a qualified charitable organization. Should Diane’s charitable contribution deduction for these contributions be determined by the basis or fair market value of the contributed items?

Correct Answer: The land is a capital asset. Diane’s deduction for the land contribution will be determined by the fair market value if she has held the land for the long-term holding period. Otherwise, it will be determined by the basis (i.e., assuming the FMV is at least equal to the basis). Inventory is ordinary income property, so Diane’s deduction will be determined by her basis in the bibles and song books.

1284. Joe, who is in the 33% tax bracket in 2013, expects to retire in 2014 and be in the 25% tax bracket. He plans to donate $50,000 to his church. Because he will not have the cash available until 2014, Joe donates land (long-term capital gain property) with a basis of $10,000 and fair market value of $50,000 to the church in December 2013. He reacquires the land for $50,000 in February 2014. Discuss Joe’s tax objectives and all tax issues related to his actions.

Correct Answer: Joe is attempting to accelerate his charitable contribution deduction into 2013. There are several potential advantages to accelerating the deduction by donating the land in 2013.

• His contribution will be deducted in a tax year when his marginal tax rate is 33% rather than 25%.

• He might avoid disallowance of part of the deduction due to AGI percentage limitations because his contribution base will be higher in 2013 than in 2014.

• He can deduct the fair market value of the land without recognizing the $40,000 appreciation as income.

• He can step up his basis in the land from $10,000 to $50,000 when he reacquires it in 2014.

Joe’s plan will generate many favorable outcomes if he does not run afoul of the IRS. While it does not appear that Joe has done anything that does not comply with the tax law, the IRS might collapse the transaction; that is, focus on the outcome and ignore the steps involved. The outcome is that Joe has transferred $50,000 cash to his church. The IRS might disallow the deduction for the land contribution in 2013 and treat the transaction as a cash contribution in 2014. In this case, Joe’s basis for the purchased land would be $10,000 and his deduction would be at the lower 2014 marginal tax rate.

1285. Sherri owns an interest in a business that is not a passive activity and in which she has $20,000 at risk. If the business incurs a loss from operations during the year and her share of the loss is $32,000, this loss will be fully deductible.

a. True *b. False

1286. Jack owns a 10% interest in a partnership (not real estate) in which his at-risk amount is $42,000 at the beginning of the year. During the year, the partnership borrows $80,000 on a nonrecourse note and incurs a loss of $60,000 from operations. Jack’s at-risk amount at the end of the year is $44,000.

a. True *b. False

1287. In the current year, Don has a $55,000 loss from a business he owns. His at-risk amount at the end of the year, prior to considering the current year loss, is $36,000. He will be allowed to deduct the $55,000 loss this year if he is a material participant in the business.

a. True *b. False

1288. Judy owns a 20% interest in a partnership (not real estate) in which her at-risk amount was $35,000 at the beginning of the year. The partnership borrowed $50,000 on a recourse note and made a $40,000 profit during the year. Her at-risk amount at the end of the year is $43,000.

a. True *b. False

1289. Tonya owns an interest in an activity (not real estate) that converted recourse financing to nonrecourse financing. Recapture of previously allowed losses is required if Tonya’s at-risk amount is reduced below zero as a result of the debt restructuring.

*a. True b. False

1290. Kelly, who earns a yearly salary of $120,000, sold an activity with a suspended passive loss of $44,000. The activity was sold at a loss and Kelly has no other passive activities. The suspended loss is not deductible.

a. True *b. False

1291. All of a taxpayer’s tax credits relating to a passive activity can be utilized when the activity is sold at a loss.

a. True *b. False

1292. Jackson Company incurs a $50,000 loss on a passive activity during the year. The company has active income of $34,000 and portfolio income of $24,000. If Jackson is a personal service corporation, it may deduct $34,000 of the passive loss.

a. True *b. False

1293. Oriole Corporation has active income of $45,000 and a passive loss of $23,000 in the current year. Under an exception, Oriole can deduct the $23,000 loss if it is a personal service corporation.

a. True *b. False

1294. Gray Company, a closely held C corporation, incurs a $50,000 loss on a passive activity during the year. The company has active income of $34,000 and portfolio income of $24,000. If Gray is not a personal service corporation, it may deduct $34,000 of the passive loss.

*a. True b. False

1295. Wolf Corporation has active income of $55,000 and a passive loss of $33,000 in the current year. Wolf cannot deduct the $33,000 loss if it is a closely held C corporation that is not a personal service corporation.

a. True *b. False

1296. Linda owns investments that produce portfolio income and Activity A that produces losses. From a tax perspective, Linda will be better off if Activity A is not passive.

*a. True b. False

1297. Nathan owns Activity A, which produces income, and Activity B, which produces passive losses. From a tax planning perspective, Nathan will be better off if Activity A is passive.

*a. True b. False

1298. A taxpayer is considered to be a material participant if he or she spends more than 500 hours in the activity.

*a. True b. False

1299. Dick participates in an activity for 90 hours during the year. He has no employees and there are no other participants. Dick is a material participant.

*a. True b. False

1300. Mary Jane participates for 100 hours during the year in an activity she owns. She has no employees and is the only participant in the activity. The activity is a significant participation activity.

a. True *b. False

1301. A taxpayer is considered to be a material participant in a significant participation activity if he or she spends at least 400 hours in the activity.

a. True *b. False

1302. Tom participates for 300 hours in Activity A and 250 hours in Activity B, both of which are nonrental businesses. Both activities are active.

*a. True b. False

1303. Tom participates for 100 hours in Activity A and 450 hours in Activity B, both of which are nonrental businesses. Both activities are active.

a. True *b. False

1304. From January through November, Vern participated for 420 hours as a salesman in a partnership in which he owns a 50% interest. The partnership has four full-time employees. During December, Vern spends 110 hours cleaning the store and painting the walls in order to meet the material participation standards. Vern qualifies as a material participant.

a. True *b. False

1305. Joyce owns an activity (not real estate) in which she participates for 100 hours a year; her husband participates for 450 hours. Joyce qualifies as a material participant.

*a. True b. False

1306. When determining whether an individual is a material participant, participation by an owner’s spouse generally counts.

*a. True b. False

1307. If an owner participates for more than 500 hours in a DVD rental activity, any loss from that activity is treated as an active loss that can offset active income.

*a. True b. False

1308. Bruce owns a small apartment building that produces a $25,000 loss during the year. His AGI before considering the rental loss is $85,000. Bruce must be a material participant with respect to the rental activity in order to deduct the $25,000 loss under the real estate rental exception.

a. True *b. False

1309. Wayne owns a small apartment building that produces a $45,000 loss during the year. His AGI before considering the rental loss is $85,000. Because Wayne is an active participant with respect to the rental activity, he may deduct the $45,000 loss.

a. True *b. False

1310. Services performed by an employee are treated as being related to a real estate trade or business if the employee performing the services has more than a 5% ownership interest in the employer.

*a. True b. False

1311. In the current year, Kelly had a $35,000 loss from a real estate rental activity in which she is a 10% owner. If she is an active participant and if her modified AGI is $100,000, she can deduct $25,000 of the loss.

*a. True b. False

1312. Individuals can deduct from active or portfolio income losses of up to $25,000 from real estate rental activities in which they actively participate.

*a. True b. False

1313. Individuals with modified AGI of $100,000 can deduct against active or portfolio income losses of up to $25,000 from real estate rental activities in which they actively participate.

*a. True b. False

1314. Roger owns and actively participates in the operations of an apartment building which produces a $40,000 loss during the year. He has AGI of $150,000 from an active business. He may deduct $25,000 of the loss.

a. True *b. False

1315. Lucy owns and actively participates in the operations of an apartment complex that produces a $50,000 loss during the year. Her modified AGI is $125,000 from an active business. Disregarding any at- risk amount limitation, she may deduct $25,000 of the loss, and the remaining $25,000 is a suspended passive loss.

a. True *b. False

1316. Kim dies owning a passive activity with a basis of $75,000, a fair market value of $140,000, and suspended losses of $80,000. All of the $80,000 passive loss can be deducted on Kim’s final income tax return.

a. True *b. False

1317. Chris receives a gift of a passive activity from his father whose basis was $60,000. Suspended losses related to the activity are $18,000. Chris will be allowed to offset the $18,000 suspended losses against future passive income.

a. True *b. False

1318. Eric makes an installment sale of a passive activity having suspended losses of $40,000. He collects 25% of the sales price in the current year, and will collect 25% in each of the next three years. Eric can deduct $10,000 of the passive loss this year.

*a. True b. False

1319. Gail exchanges passive Activity A, which has suspended losses of $15,000, for passive Activity B in a nontaxable exchange. The new owner of passive Activity A can offset the $15,000 suspended losses against passive income in the future.

a. True *b. False

1320. David earned investment income of $20,000, incurred investment interest expense of $12,000, and other investment expenses of $9,000 during the current year. David can deduct $12,000 of investment interest for this year.

a. True *b. False

1321. Investment income can include gross income from interest, dividends, annuities, and royalties not derived in the ordinary course of a trade or business; income from a passive activity; and income from a real estate activity in which the taxpayer actively participates.

a. True *b. False

1322. Bob realized a long-term capital gain of $8,000. In calculating his net investment income, Bob may elect to include the gain in investment income.

*a. True b. False

1323. Harry earned investment income of $18,500, incurred investment interest expense of $15,500, and other investment expenses of $9,000 during the current year. Harry may deduct $9,500 of investment interest expense this year and carry forward $6,000 to future years.

*a. True b. False

1324. In 2013, Arnold invests $80,000 for a 20% interest in a partnership in which he is a material participant. The partnership incurs a loss with $100,000 being Arnold’s share. Which of the following statements is incorrect?

a. Since Arnold has only $80,000 of capital at risk, he cannot deduct any more than this amount against his other income. b. Arnold’s nondeductible loss of $20,000 can be carried over and used in future years (subject to the at-risk provisions). c. If Arnold has taxable income of $40,000 from the partnership in 2014 and there are no other transactions that affect his at- risk amount, he can use all of the $20,000 loss carried over from 2013. *d. Arnold’s $100,000 loss is nondeductible in 2013 and 2014 under the passive loss provisions. e. All of the statements are correct.

1325. Last year, Ted invested $100,000 for a 50% interest in a partnership in which he was a material participant. The partnership incurred a loss, and Ted’s share was $150,000. Which of the following statements is incorrect?

a. Ted’s nondeductible loss of $50,000 can be carried over and used in the future (subject to the at-risk provisions). b. If Ted has taxable income of $50,000 from the partnership in the current year and no other transactions that affect his at- risk amount, he can use all of the $50,000 loss carried over. c. Since Ted has only $100,000 of capital at risk, he cannot deduct more than $100,000 against his other income. *d. None of the above is incorrect.

1326. In 2013, Joanne invested $90,000 for a 20% interest in a limited liability company (LLC) in which she is a material participant. The LLC reported losses of $340,000 in 2013 and $180,000 in 2014. Joanne’s share of the LLC’s losses was $68,000 in 2013 and $36,000 in 2014. How much of these losses can Joanne deduct?

a. $68,000 in 2013; $36,000 in 2014. *b. $68,000 in 2013; $22,000 in 2014. c. $0 in 2013; $0 in 2014. d. $68,000 in 2013; $0 in 2014. e. None of the above.

1327. In 2013, Kipp invested $65,000 for a 30% interest in a partnership conducting a passive activity. The partnership reported losses of $200,000 in 2013 and $100,000 in 2014, Kipp’s share being $60,000 in 2013 and $30,000 in 2014. How much of the losses from the partnership can Kipp deduct assuming he owns no other investments and does not participate in the partnership’s operations?

a. $0 in 2013; $30,000 in 2014. b. $60,000 in 2013; $30,000 in 2014. c. $60,000 in 2013; $5,000 in 2014. d. $60,000 in 2013; $0 in 2014. *e. None of the above.

1328. Josh has investments in two passive activities. Activity A (acquired three years ago) produces income of $30,000 this year, while Activity B (acquired two years ago) produces a loss of $50,000. What is the amount of Josh’s suspended loss for the year?

a. $0. b. $18,000. *c. $20,000. d. $50,000. e. None of the above.

1329. Carl, a physician, earns $200,000 from his medical practice in the current year. He receives $45,000 in dividends and interest during the year as well as $5,000 of income from a passive activity. In addition, he incurs a loss of $50,000 from an investment in a passive activity. What is Carl’s AGI for the current year after considering the passive investment?

a. $195,000. b. $200,000. c. $240,000. *d. $245,000. e. None of the above.

1330. Nell sells a passive activity with an adjusted basis of $45,000 for $105,000. Suspended losses attributable to this property total $45,000. The total gain and the taxable gain are:

a. $60,000 total gain; $105,000 taxable gain. b. $10,000 total gain; $15,000 taxable gain. c. $60,000 total gain; $0 taxable gain. *d. $60,000 total gain; $15,000 taxable gain. e. None of the above.

1331. Matt has three passive activities and has at-risk amounts in excess of $100,000 for each. During the year, the activities produced the following income (losses).

Activity A ($60,000) Activity B (40,000) Activity C 75 ,000 Net passive loss ($25,000)

Matt’s suspended losses are as follows:

a. $25,000 is allocated to C; $0 to A and B. b. $12,500 is allocated to A; $12,500 to B. *c. $15,000 is allocated to A; $10,000 to B. d. $8,333 is allocated to A, B, and C. e. None of the above.

1332. Green Corporation earns active income of $50,000 and receives $40,000 in dividends during the year. In addition, Green incurs a loss of $70,000 from an investment in a passive activity acquired several years ago. Consider the following two statements:

(1) Green’s current deduction for passive losses is $50,000 if it is a closely held C corporation that is not a personal service corporation.

(2) Green’s current deduction for passive losses is $0 if it is a personal service corporation.

Which of the following answers is correct?

a. Only statement 1. b. Only statement 2. *c. Both statements 1 and 2. d. Neither statement 1 or 2. e. None of the above.

1333. White Corporation, a closely held personal service corporation, has $150,000 of passive losses, $120,000 of active business income, and $30,000 of portfolio income. How much of the passive loss can White Corporation deduct?

*a. $0. b. $30,000. c. $120,000. d. $150,000. e. None of the above.

1334. Charles owns a business with two separate departments. Department A produces $100,000 of income and Department B incurs a $60,000 loss. Charles participates for 550 hours in Department A and 100 hours in Department B. He has full-time employees in both departments.

a. If Charles elects to treat both departments as a single activity, he cannot offset the $60,000 loss against the $100,000 income. b. Charles may not treat Department A and Department B as separate activities because they are parts of one business. c. If Charles elects to treat the two departments as separate activities, he can offset the $60,000 loss against the $100,000 income. *d. If Charles elects to treat both departments as a single activity, he can offset the $60,000 loss against the $100,000 income. e. None of the above.

1335. Tara owns a shoe store and a bookstore. Both businesses are operated in a mall. She also owns a restaurant across the street and a jewelry store several blocks away.

*a. All four businesses can be treated as a single activity if Tara elects to do so. b. Only the shoe store and bookstore can be treated as a single activity, the restaurant must be treated as a separate activity, and the jewelry store must be treated as a separate activity. c. The shoe store, bookstore, and restaurant can be treated as a single activity, and the jewelry store must be treated as a separate activity. d. All four businesses must be treated as separate activities. e. None of the above.

1336. Which of the following factors should be considered in determining whether an activity is treated as an appropriate economic unit?

a. The similarities and differences in types of business. b. The extent of common control. c. The extent of common ownership. d. The geographic location. *e. All of the above.

1337. Which of the following is not a factor that should be considered in determining whether an activity is treated as an appropriate economic unit?

a. The interdependencies between the activities. b. The extent of common control. c. The extent of common ownership. d. The geographical location. *e. All of the above are relevant factors.

1338. Tess owns a building in which she rents apartments to tenants and operates a restaurant. Which of the following statements is incorrect?

a. If 60% of Tess’s gross income is from apartment rentals and 40% is from the restaurant, the rental operation and the restaurant business must be treated as separate activities. b. If 95% of Tess’s gross income is from apartment rentals and 5% is from the restaurant, she may treat the rental operation and the restaurant business as a single activity that is a rental activity. c. If 5% of Tess’s gross income is from apartment rentals and 95% is from the restaurant, she may treat the rental operation and the restaurant business as a single activity that is not a rental activity. *d. If 98% of Tess’s gross income is from apartment rentals and 2% is from the restaurant, the rental operation and the restaurant business must be treated as a single activity that is not a rental activity. e. None of the above.

1339. Rick, a computer consultant, owns a separate business (not real estate) in which he participates. He has one employee who works part- time in the business.

a. If Rick participates for 500 hours and the employee participates for 620 hours during the year, Rick qualifies as a material participant. *b. If Rick participates for 550 hours and the employee participates for 2,000 hours during the year, Rick qualifies as a material participant. c. If Rick participates for 120 hours and the employee participates for 120 hours during the year, Rick does not qualify as a material participant. d. If Rick participates for 95 hours and the employee participates for 5 hours during the year, Rick probably does not qualify as a material participant. e. None of the above.

1340. Ned, a college professor, owns a separate business (not real estate) in which he participates in the current year. He has one employee who works part-time in the business.

a. If Ned participates for 120 hours and the employee participates for 120 hours during the year, Ned does not qualify as a material participant. b. If Ned participates for 95 hours and the employee participates for 5 hours during the year, Ned probably does not qualify as material participant. c. If Ned participates for 500 hours and the employee participates for 520 hours during the year, Ned qualifies as material participant. *d. If Ned participates for 600 hours and the employee participates for 2,000 hours during the year, Ned qualifies as a material participant. e. None of the above.

1341. Ahmad owns four activities. He participated for 120 hours in Activity A, 150 hours in Activity B, 140 hours in Activity C, and 100 hours in Activity D. Which of the following statements is correct?

a. Activities A, B, C, and D are all significant participation activities. *b. Activities A, B, and C are significant participation activities. c. Ahmad is a material participant with respect to Activities A, B, and C. d. Ahmad is a material participant with respect to Activities A, B, C, and D. e. None of the above.

1342. Paula owns four separate activities. She elects not to group them together as a single activity under the “appropriate economic unit” standard. Paula participates for 130 hours in Activity A, 115 hours in Activity B, 260 hours in Activity C, and 100 hours in Activity D. She has one employee, who works 125 hours in Activity D. Which of the following statements is correct?

a. Activities A, B, C, and D are all significant participation activities. b. Paula is a material participant with respect to Activities A, B, C, and D. c. Paula is not a material participant with respect to Activities A, B, C, and D. d. Losses from all of the activities can be used to offset Paula’s active income. *e. None of the above.

1343. Dena owns interests in five businesses and has full-time employees in each business. She participates for 100 hours in Activity A, 120 hours in Activity B, 130 hours in Activity C, 140 hours in Activity D, and 125 hours in Activity E.

a. All five of Dena’s activities are significant participation activities. b. Dena is a material participant with respect to all five activities. c. Dena is not a material participant in any of the activities. *d. Dena is a material participant with respect to Activities B, C, D, and E. e. None of the above.

1344. Maria, who owns a 50% interest in a restaurant, has been a material participant in the restaurant activity for the last 20 years. She retired from the restaurant at the end of last year and will not participate in the restaurant activity in the future. However, she continues to be a material participant in a retail store in which she is a 50% partner. The restaurant operations produce a loss for the current year, and Maria’s share of the loss is $80,000. Her share of the income from the retail store is $150,000. She does not own interests in any other activities.

a. Maria cannot deduct the $80,000 loss from the restaurant because she is not a material participant. *b. Maria can offset the $80,000 loss against the $150,000 of income from the retail store. c. Maria will not be able to deduct any losses from the restaurant until she has been retired for at least three years. d. Assuming Maria continues to hold the interest in the restaurant, she will always treat the losses as active. e. None of the above.

1345. Leigh, who owns a 50% interest in a sporting goods store, was a material participant in the activity for the last fifteen years. She retired from the sporting goods store at the end of last year and will not participate in the activity in the future. However, she continues to be a material participant in an office supply store in which she is a 50% partner. The operations of the sporting goods store resulted in a loss for the current year and Leigh’s share of the loss is $40,000. Leigh’s share of the income from the office supply store is $75,000. She does not own interests in any other activities.

a. Leigh cannot deduct the $40,000 loss from the sporting goods store because she is not a material participant. *b. Leigh can offset the $40,000 loss from the sporting goods store against the $75,000 of income from the office supply store. c. Leigh will not be able to deduct any losses from the sporting goods store until future years. d. Leigh will not be able to deduct any losses from the sporting goods store until she has been retired for at least four years. e. None of the above.

1346. Jed spends 32 hours a week, 50 weeks a year, operating a DVD rental store that he owns. He also owns a music store in another city that is operated by a full-time employee. He elects not to group them together as a single activity under the “appropriate economic unit” standard. Jed spends 40 hours per year working at the music store.

a. Neither store is a passive activity. b. Both stores are passive activities. c. Only the DVD rental store is a passive activity. *d. Only the music store is a passive activity. e. None of the above.

1347. Jenny spends 32 hours a week, 50 weeks a year, operating a DVD rental store that she owns. She also owns a music store in another city that is operated by a full-time employee. Jenny spends 140 hours per year working at the music store. She elects not to group them together as a single activity under the “appropriate economic unit” standard.

*a. Neither store is a passive activity. b. Both stores are passive activities. c. Only the DVD rental store is a passive activity. d. Only the music store is a passive activity. e. None of the above.

1348. Skeeter invests in vacant land for the purpose of realizing a profit on its appreciation. He leases the land during the period he holds it. The unadjusted basis of the property is $75,000 and its fair market value is $105,000. The lease payments are $1,200 per year.

a. The leasing activity will be treated as a rental activity and will be treated as a passive activity regardless of how many hours Skeeter participates. b. The leasing activity will be treated as a rental activity and will not be treated as a passive activity if Skeeter qualifies as a real estate professional. *c. The leasing activity will not be treated as a rental activity. d. The leasing activity will be treated as a rental activity and will not be treated as a passive activity if Skeeter devotes more than 500 hours to the activity. e. None of the above.

1349. Josh has investments in two passive activities. Activity A, acquired three years ago, produces income in the current year of $60,000. Activity B, acquired last year, produces a loss of $100,000 in the current year. At the beginning of this year, Josh’s at-risk amounts in Activities A and B are $10,000 and $100,000, respectively. What is the amount of Josh’s suspended passive loss with respect to these activities at the end of the current year?

a. $0. b. $36,000. *c. $40,000. d. $100,000. e. None of the above.

1350. Sandra acquired a passive activity three years ago. Until last year, the activity was profitable and her at-risk amount was $300,000. Last year, the activity produced a loss of $100,000, and in the current year, the loss is $50,000. Assuming Sandra has received no passive income in the current or prior years, her suspended passive loss from the activity is:

a. $90,000 from last year and $50,000 from the current year. *b. $100,000 from last year and $50,000 from the current year. c. $0 from last year and $0 from the current year. d. $50,000 from the current year. e. None of the above.

1351. Rita earns a salary of $150,000, and invests $40,000 for a 20% interest in a passive activity. Operations of the activity result in a loss of $250,000, of which Rita’s share is $50,000. How is her loss characterized?

*a. $40,000 is suspended under the passive loss rules and $10,000 is suspended under the at-risk rules. b. $40,000 is suspended under the at-risk rules and $10,000 is suspended under the passive loss rules. c. $50,000 is suspended under the passive loss rules. d. $50,000 is suspended under the at-risk rules. e. None of the above.

1352. Art’s at-risk amount in a passive activity was $60,000 at the beginning of 2012. His loss from the activity in 2012 is $80,000, and he had no passive activity income during the year. Art had $20,000 of passive income from the activity in 2013. Under the passive loss rules, Art’s suspended loss at the end of 2013 is:

a. $15,000. b. $20,000. c. $45,000. *d. $60,000. e. None of the above.

1353. Vic’s at-risk amount in a passive activity is $200,000 at the beginning of the current year. His current loss from the activity is $80,000. Vic had no passive activity income during the year. At the end of the current year:

*a. Vic has an at-risk amount in the activity of $120,000 and a suspended passive loss of $80,000. b. Vic has an at-risk amount in the activity of $200,000 and a suspended passive loss of $80,000. c. Vic has an at-risk amount in the activity of $120,000 and no suspended passive loss. d. Vic has an at-risk amount in the activity of $200,000 and no suspended passive loss. e. None of the above.

1354. Wes’s at-risk amount in a passive activity is $25,000 at the beginning of the current year. His current loss from the activity is $35,000 and he has no passive activity income. At the end of the current year, which of the following statements is incorrect?

a. Wes has a loss of $25,000 suspended under the passive loss rules. b. Wes has an at-risk amount in the activity of $0. c. Wes has a loss of $10,000 suspended under the at-risk rules. *d. Wes has a loss of $35,000 suspended under the passive loss rules. e. None of the above is incorrect.

1355. Jon owns an apartment building in which he is a material participant and a computer consulting business. Of the 2,000 hours he spends on these activities during the year, 55% of the time is spent operating the apartment building and 45% of the time is spent in the computer consulting business.

a. The computer consulting business is a passive activity but the apartment building is not. b. The apartment building is a passive activity but the computer consulting business is not. c. Both the apartment building and the computer consulting business are passive activities. *d. Neither the apartment building nor the computer consulting business is a passive activity. e. None of the above.

1356. Consider the following three statements:

(1) Tad invests in vacant land for the purpose of realizing a profit on its appreciation. He leases the land during the period he holds it. The unadjusted basis of the property is $25,000 and its fair market value is $35,000. The lease payments are $400 per year.

(2) A farmer owns land with an unadjusted basis of $25,000 and a fair market value of $35,000. He used it for farming purposes in the two prior years. In the current year, he leases the land to another farmer for $400.

(3) At City Hospital, each inpatient is provided a private room while medical care is provided.

In which of the three cases above could the rental activity automatically be considered a passive activity?

a. Case 1 only. b. Case 2 only. c. Case 3 only. d. Cases 1, 2, and 3. *e. None of the above.

1357. Pablo, who is single, has $95,000 of salary, $10,000 of income from a limited partnership, and a $27,000 passive loss from a real estate rental activity in which he actively participates. His modified adjusted gross income is $95,000. Of the $27,000 loss, how much is deductible?

a. $0. b. $10,000. c. $25,000. *d. $27,000. e. None of the above.

1358. Josie, an unmarried taxpayer, has $155,000 in salary, $10,000 in income from a limited partnership, and a $26,000 passive loss from a real estate rental activity in which she actively participates. If her modified adjusted gross income is $155,000, how much of the $26,000 loss is deductible?

a. $0. *b. $10,000. c. $25,000. d. $26,000. e. None of the above.

1359. Kate dies owning a passive activity with an adjusted basis of $100,000. Its fair market value at that date is $130,000. Suspended losses relating to the property were $45,000.

*a. The heir’s adjusted basis is $130,000, and Kate’s final deduction is $15,000. b. The heir’s adjusted basis is $130,000, and Kate’s final deduction is $45,000. c. The heir’s adjusted basis is $100,000, and Kate’s final deduction is $45,000. d. The heir’s adjusted basis is $175,000, and Kate has no final deduction. e. None of the above.

1360. Caroyl made a gift to Tim of a passive activity (adjusted basis of $50,000, suspended losses of $20,000, and a fair market value of $80,000). No gift tax resulted from the transfer.

a. Tim’s adjusted basis is $80,000, and Tim can deduct the $20,000 of suspended losses in the future. b. Tim’s adjusted basis is $80,000. c. Tim’s adjusted basis is $50,000, and the suspended losses are lost. d. Tim’s adjusted basis is $50,000, and Tim can deduct the $20,000 of suspended losses in the future. *e. None of the above.

1361. Identify from the list below the type of disposition of a passive activity where the taxpayer keeps the suspended losses of the disposed activity and utilizes them on a subsequent taxable disposition.

a. Disposition of a passive activity by gift. *b. Nontaxable exchange of a passive activity. c. Disposition of a passive activity at death. d. Installment sale of a passive activity. e. None of the above.

1362. Raul is married and files a joint tax return. His current investment interest expense of $95,000 is related to a loan used to purchase a parcel of unimproved land. Income from investments [dividends (not qualified) and interest] total $18,000 and miscellaneous itemized deductions (after adjustment for the 2%-of-AGI floor) amount to $2,800. In addition to the $1,400 of investment expenses included in miscellaneous itemized deductions, Raul paid $3,600 of real estate taxes on the unimproved land. He also has a $4,500 net long-term capital gain from the sale of another parcel of unimproved land. Raul’s maximum investment interest deduction for the year is:

a. $95,000. b. $18,000. *c. $17,500. d. $13,000. e. None of the above.

1363. Judy incurred $58,500 of interest expense this year related to her investments. Her investment income includes $15,000 of interest, $9,000 of qualified dividends, and a $22,500 net capital gain on the sale of securities. The maximum amount of Judy’s investment interest expense deduction for the year is:

a. $15,000. b. $24,000. c. $37,500. *d. $46,500. e. None of the above.

1364. Match the term with the correct response. More than one response may be correct.Significant participation activity.At-risk amount.Material participation.Extraordinary personal services.Active participation.One in which the individual’s participation equals more than 100 hours during the year. No correct choice is given. Taxpayer devotes time aggregating more than 500 hours in all significant participation activities during the year. The use of property is incidental to the receipt of services. Participates in making management decisions in a significant and bonafide sense. Taxpayer devotes time in the activity which constitutes substantially all of the participation in the activity of all individuals.

[a] 1. Significant participation activity. [b] 2. At-risk amount. [c] 3. Material participation. [d] 4. Extraordinary personal services. [e] 5. Active participation.

a. One in which the individual’s participation equals more than 100 hours during the year. b. No correct choice is given. c. Taxpayer devotes time aggregating more than 500 hours in all significant participation activities during the year. d. The use of property is incidental to the receipt of services. e. Participates in making management decisions in a significant and bonafide sense. f. Taxpayer devotes time in the activity which constitutes substantially all of the participation in the activity of all individuals.

1365. Match the treatment for the following types of transactions.Treatment of a disposition of a passive activity by gift.Treatment of a disposition of a passive activity at death.Treatment of an installment sale of a passive activity.Treatment of a nontaxable exchange of a passive activity.Treatment of a sale of a passive activity where all of the realized gain or loss is recognized currently.Treatment of suspended credits when passive activity is sold at a loss.The suspended losses are added to the basis of the property. Suspended losses are allowed to the taxpayer to the extent they exceed the amount, if any, of the step-up in basis allowed. The losses are allowed in the years in which gain is recognized. The taxpayer keeps the suspended losses. Any suspended losses may be used in the current year. No correct choice is given. Suspended losses are allowed to offset the income from the activity, other passive activities, or active income.

[a] 1. Treatment of a disposition of a passive activity by gift. [b] 2. Treatment of a disposition of a passive activity at death. [c] 3. Treatment of an installment sale of a passive activity. [d] 4. Treatment of a nontaxable exchange of a passive activity. [e] 5. Treatment of a sale of a passive activity where all of the realized gain or loss is recognized currently. [f] 6. Treatment of suspended credits when passive activity is sold at a loss.

a. The suspended losses are added to the basis of the property. b. Suspended losses are allowed to the taxpayer to the extent they exceed the amount, if any, of the step-up in basis allowed. c. The losses are allowed in the years in which gain is recognized. d. The taxpayer keeps the suspended losses. e. Any suspended losses may be used in the current year. f. No correct choice is given. g. Suspended losses are allowed to offset the income from the activity, other passive activities, or active income.

1366. Sarah purchased for $100,000 a 10% interest in a business venture that is not subject to the passive activity rules. During the first year, her share of the entity’s loss was $120,000. At the beginning of the second year, the entity obtained $800,000 of recourse financing. During the second year, Sarah withdrew cash of $20,000, and her share of the entity’s loss was $25,000. Calculate the amount of loss that Sarah may claim in each of the two years and determine her at-risk amount at the end of each year.

Correct Answer: Initial at-risk $100,000 amount Subtract: Deductibl e first year loss of (100,000) $120,000, limited to at-risk amount of $100,000 At-risk amount at the $ – end of first year 0–

Suspended loss at the $ 20,000 end of first year

At-risk amount at the $ – beginning of the 0– second year Add: Share of 80,000 recourse debt Subtract: Withdrawa (20,000) l Deductible $25,000 second year loss + (45,000) $20,000 loss suspended from prior year At-risk amount at the $ 15,000 end of second year

Suspended loss at the $ – end of second year 0–

1367. In 2013, Emily invests $120,000 in a limited partnership that is not a passive activity. During 2013, her share of the partnership loss is $90,000. In 2014, her share of the partnership loss is $50,000. How much can Emily deduct in 2013 and 2014?

Correct Answer: Although the passive loss rules do not apply, the at-risk rules limit Emily’s deductions. She can deduct $90,000 in 2013 and her at-risk amount will be reduced to $30,000 ($120,000 – $90,000 deducted). She will be limited to a $30,000 deduction in 2014 unless she increases her amount at risk. For example, if Emily invests an additional $20,000 in 2014, her at-risk amount would be $50,000 ($30,000 balance + $20,000 additional investment), and she would be able to deduct the entire $50,000 loss in 2014.

1368. In 2012, Kelly earns a salary of $200,000 and invests $40,000 for a 20% interest in a partnership not subject to the passive loss rules. Through the use of $800,000 of nonrecourse financing, the partnership acquires assets worth $1 million. The activity produces a loss of $150,000, of which Kelly’s share is $30,000. In 2013, Kelly’s share of the loss from the partnership is $15,000. How much of the loss from the partnership can Kelly deduct?

Correct Answer: Kelly has $40,000 at risk at the end of 2012 and can deduct the $30,000 loss in that year. This decreases his at-risk amount to $10,000. Consequently, at the end of 2013, he can deduct only $10,000 of the $15,000 loss.

1369. Lindsey, an attorney, earns $125,000 from her law practice in the current year. In addition, she receives $50,000 in dividends and interest during the year. Further, she incurs a loss of $40,000 from an investment in a passive activity. What is Lindsey’s AGI for the year after considering the passive investment?

Correct Answer: Lindsey cannot deduct the passive loss against active or portfolio income. Therefore, her AGI after considering the passive investment is $175,000 ($125,000 active income + $50,000 portfolio income).

1370. Anne sells a rental house for $100,000 (adjusted basis of $55,000). During her ownership, $60,000 of losses have been suspended under the passive activity loss rules. Determine the tax treatment to Anne on the disposition of the property.

Correct Answer: Because Anne disposes of her entire interest in the passive activity, she is able to fully recognize the suspended losses. By utilization of the $60,000 suspended loss, a deductible loss of $15,000 results.

Net sales price $100,000 Less: Adjusted basis (55,00 0) Total gain $ 45,000

Less: Suspended losses (60,00 0) Deductible (nonpassive) loss ($ 15,000)

1371. Hugh has four passive activities which generate the following income and losses in the current year.

Activity Gain (Loss) A ($60,000 ) B (20,00 0) C (10,00 0) D 10,00 0 Total ($80,000 )

How much of the $80,000 net passive loss can Hugh deduct this year? Calculate the suspended losses (by activity).

Correct Answer: None. The suspended losses of $80,000 are allocated as follows:

Activity Suspended Loss A $60,000/$90,00 $53,333 0 × $80,000 B $20,000/$90,00 17,778 0 × $80,000 C $10,000/$90,00 8,889 0 × $80,000 Total suspended loss $80,000

1372. Pat sells a passive activity for $100,000 that has an adjusted basis of $55,000. During the years of her ownership, $60,000 of losses have been incurred that were suspended under the passive activity loss rules. In addition, the passive activity generated tax credits of $10,000 that were not utilized and suspended. Determine the tax treatment to Pat on the disposition of the property.

Correct Answer: Because Pat disposes of her entire interest in the passive activity, she is able to recognize fully the losses that had been suspended during the years of her ownership. With the current utilization of the $60,000 suspended loss, a net deductible loss of $15,000 results, which is treated as a loss that is not from a passive activity. However, the suspended credits are lost and may not be used. The tax credits are allowed on dispositions only when there is sufficient tax on the disposition (i.e., due to a gain) to absorb them.

Net sales price $100,000 Less: Adjusted basis (55,0 00) Total gain $ 45,000 Less: Suspended losses (60,0 00) Deductible loss ($ 15,000)

1373. Purple Corporation, a personal service corporation, earns active income of $600,000. The corporation receives $60,000 in dividends and incurs a loss of $100,000 from an investment in a passive activity acquired three years ago. What is Purple’s income after considering the passive investment?

Correct Answer: A personal service corporation cannot offset passive losses against active or portfolio income. Purple’s income is $660,000 ($600,000 active income + $60,000 dividend income). The $100,000 passive loss is suspended.

1374. Orange Corporation, a closely held (non-personal service) C corporation, earns active income of $300,000 in the current year. The corporation also receives $35,000 in dividends and incurs a loss of $50,000 from an investment in a passive activity. What is Orange’s income for the year after considering the passive investment?

Correct Answer: A closely held (non-personal service) C corporation can offset passive losses against active, but not portfolio income. Therefore, Orange’s income is $285,000 [($300,000 active income – $50,000 passive loss) + $35,000 portfolio income].

1375. Lloyd, a life insurance salesman, earns a $400,000 salary in the current year. As he works only 30 hours per week in this job, he has time to participate in several other businesses. He owns an ice cream parlor and a car repair shop in Tampa. He also owns an ice cream parlor and a car repair shop in Portland and a car repair shop in St. Louis. A preliminary analysis on December 1 of the current year shows projected income and losses for the various businesses as follows: Income (Loss) Tampa ice cream parlor (95 $56,000 hours participation) Tampa car repair shop (140 (89,000) hours participation) Portland ice cream parlor 34,000 (90 hours participation) Portland car repair shop (41,000) (170 hours participation) St. Louis car repair shop (15,000) (180 hours participation)

Lloyd has full-time employees at each of the five businesses listed above. Review all possible groupings for Lloyd’s activities. Which grouping method and other strategies should Lloyd consider that will provide the greatest tax advantage?

Correct Answer: The basic issue relates to how the car repair shops and ice cream parlors should be grouped under the passive activity rules so as to maximize the tax benefit to Lloyd. The $400,000 salary is active income. If the participation levels stay the same in the ice cream parlor and car repair shop businesses, all profits and losses will be passive, assuming each location is a separate activity. As a result, a net passive loss of $55,000 ($89,000 loss + $41,000 loss + $15,000 loss – $56,000 profit – $34,000 profit) would be suspended and not be available to offset his salary. To mitigate this result, three options should be considered. Option 1 is based on the significant participation activity rule. If all of the businesses are treated as separate activities, Lloyd would not be considered a material participant, even under the significant participation activity rule. Under the significant participation activity rule, the car repair shops would be considered significant activities, but the ice cream parlors would not. But even with the car repair shops, the total participation is not expected to exceed the more-than-500 hour threshold (140 + 170 + 180 = 490). If Lloyd could participate 11 more hours in any of the car repair shop businesses, they would be treated as active and the net loss from the car repair shops of $145,000 ($89,000 + $41,000 + $15,000) could be offset against his salary. Further, if Lloyd does not participate any more in the other ice cream parlor businesses, their combined $90,000 of income will be reported as passive income. This characterization as passive could be helpful if Lloyd were to acquire additional businesses in the future that produce passive losses. Under option 2, both the ice cream parlor and car repair shop businesses could be combined as a “single activity” based on common ownership. Because Lloyd has participated more than 500 hours in the five businesses, the net loss of $55,000 would be considered active and could be used to offset his salary. Option 3 would combine the car repair shops as one activity based on product while the ice cream parlors would be treated as a separate activity based on product. As with option 1, if Lloyd could participate 11 more hours in any of the car repair shop businesses, they would be treated as active, and the net loss of $145,000 ($89,000 + $41,000 + $15,000) could be offset against his salary. Also, he could treat the ice cream parlors as a single business and the net income would be passive, which could be helpful in the future if other passive ventures would be acquired.

1376. Vail owns interests in a beauty salon, a natural foods store, and a tanning salon. Several full-time employees work at each of the enterprises. As of the end of November of the current year, Vail has worked 180 hours in the beauty salon, 220 hours at the natural foods store, and 80 hours at the tanning salon. These three ventures collectively will produce income. Vail also owns one other passive activity that is producing a loss (a limited partnership in which she has reported no participation). How should Vail plan her activities for the remainder of the year?

Correct Answer: If Vail spends an additional 21 hours in the tanning salon activity, she has participated more than 500 hours in all of her significant participation activities. Consequently, she is considered a material participant in the ventures, and the resulting active income is not available to absorb the passive loss generated by the limited partnership. Therefore, Vail should avoid devoting additional time to the ventures. Instead, she should plan to fail the material participation standard to keep the income classified as passive.

1377. Ken has a $40,000 loss from an investment in a partnership in which he does not materially participate. He paid $30,000 for his interest. How much of the loss is disallowed by the at-risk rules? How much is disallowed by the passive loss rules?

Correct Answer: The at-risk limits disallow $10,000 of the deduction ($40,000 loss – $30,000 at risk). Ken is not a material participant, so the remaining $30,000 is disallowed by the passive loss rules.

1378. During the current year, Ryan performs personal services as follows: 700 hours in his management consulting practice, 650 hours in a real estate development business, and 550 hours in an apartment leasing operation. He expects that losses will be realized from the two real estate ventures while his consulting practice will show a profit. Ryan files a joint return with his wife whose salary is $125,000. Discuss the character and treatment of the income and losses generated by these activities.

Correct Answer: Ryan is considered a material participant in all three ventures, so the income and loss from these operations will be fully reflected on his income tax return. The losses from the real estate activities will not be subject to the passive loss rules because more than 50% of his personal services were devoted to real property trades or businesses in which he is a material participant, and this participation exceeded 750 hours. Thus, the losses from these activities can offset the income from his management consulting practice and his wife’s salary.

1379. In the current year, Lucile, who has AGI of $70,000 before considering rental activities, is active in three separate real estate rental activities and is in the 28% tax bracket. She had $15,000 of losses from Activity A, $25,000 of losses from Activity B, and income of $20,000 from Activity C. She also had $3,100 of tax credits from Activity A. Calculate her deductions and credits currently allowed and the suspended losses and credits.

Correct Answer: Lucile can utilize $20,000 of losses and $1,400 of credits under the real estate rental activities exception as follows:

Income Activity A ($15,000) (Loss): Activity B (25,000) Activity C 20,000 Net loss ($20,000) Utilized 20,000 loss Suspended $ loss –0–

Utilized $ 1,400 credit Suspended $ 1,700 credit

After deducting the $20,000 loss, Lucile has an available deduction equivalent of $5,000 [$25,000 (maximum loss allowed) – $20,000 (utilized loss)]. Then the maximum amount of credits Lucile may claim is $1,400 [$5,000 deduction equivalent × .28 (marginal tax bracket)] that is allocated to Activity A.

1380. Last year, Wanda gave her daughter a passive activity (adjusted basis of $80,000; fair market value of $160,000) with suspended losses of $20,000. In the current year, her daughter realizes income of $10,000 from the activity. What are the tax effects to Wanda and her daughter?

Correct Answer: Wanda loses the suspended losses of $20,000 but they are added to the basis of the gifted property. She may or may not have to pay gift taxes, depending on the value of other gifts she has given during her lifetime. Wanda’s daughter can add the suspended losses to the basis in the property but she cannot apply them against the income received.

1381. Barb borrowed $100,000 to acquire a parcel of land to be held for investment purposes and paid interest of $11,000 on the loan. She has AGI of $75,000 for the year. Other items related to Barb’s investments include the following:

Investment income $10,00 0 Long-term capital gain 7,500 on sale of stock Investment counsel 2,000 fees

Barb is unmarried and elects to itemize her deductions. She has no miscellaneous itemized deductions other than the investment counsel fees. a. Determine Barb’s current investment interest deduction, assuming she does not make any special election regarding the computation of investment income. b. Discuss the treatment of Barb’s investment interest that is disallowed in the current year. c. What election could Barb make to increase the amount of her current investment interest deduction?

Correct Answer: a. Barb’s investment interest deduction is limited to net investment income, which is computed as follows:

Income from $10,000 investments Less: Investment expenses* (500) Net investment $ 9,500 income

*Because Barb has no other miscellaneous itemized deductions, the deductible investment expenses are the smaller of (1) $2,000, the amount of investment expenses included in the total of miscellaneous itemized deductions subject to the 2%-of-AGI floor, or (2) $500, the amount of miscellaneous expenses deductible after the 2%-of-AGI floor is applied [$2,000 – $1,500 (2% of $75,000 AGI)].

Barb’s deduction for investment interest is limited to $9,500, the amount of net investment income. The balance of $1,500 is disallowed in the current year.

Total investment $11,000 interest expense Less: Net investment (9 income ,500) Investment interest $ 1,5 disallowed in the 00 current year

b. The $1,500 of investment interest disallowed may be carried over and becomes investment interest expense in the subsequent year subject to the net investment income limitation in that year. c. Barb could increase her investment interest deduction by electing to treat the LTCG as investment income. This would increase her investment income for purposes of calculating her investment interest deduction. She would be able to deduct the full $11,000 of investment interest expense. If she makes the election, the amount so elected could not be taxed using the beneficial capital gains tax rate.

1382. Describe the types of activities and taxpayers that are subject to the at-risk rules.

Correct Answer: The at-risk provisions limit the deductibility of losses from business and income-producing activities. The provisions apply to individuals and closely held corporations. In the case of an S corporation or a partnership, the at-risk limits apply at the owner level.

1383. Identify how the passive loss rules broadly classify various types of income and losses. Provide examples of each category.

Correct Answer: The passive loss rules require income and losses to be classified into one of three categories: active, passive, or portfolio. Active income includes salary and wages, profit from a trade or business in which the taxpayer is a material participant, and gain on the sale of assets used in an active trade or business. Portfolio income includes interest, dividends, annuities, and royalties not derived in the ordinary course of a trade or business. The final category, passive income or loss, is generated by a passive activity. The following activities are treated as passive: (1) any trade or business or income-producing activity in which the taxpayer does not materially participate, and (2) subject to exceptions, all rental activities, whether the taxpayer materially participates or not.

1384. Discuss the treatment given to suspended passive activity losses and credits. What happens to an activity’s unused losses and credits when the activity is sold?

Correct Answer: In general, passive losses are deductible to the extent of passive income from all of the taxpayer’s current-year passive activities. Passive credits can be utilized only against regular tax attributable to passive income. If passive losses or credits are not used in the current year, they are carried over indefinitely for potential use in the succeeding years to offset passive income (or regular tax attributable to passive income) in those years.

An activity’s unused (or suspended) passive losses that exist when a taxpayer sells the passive activity may be used to reduce the gain from the sale, or increase the recognized loss. Thus, the suspended passive losses are fully utilized in the year of disposition. In contrast, passive credits are allowed on dispositions only when there is sufficient tax on passive income to absorb them.

1385. List the taxpayers that are subject to the passive loss rules and summarize the general impact of these rules on these taxpayers.

Correct Answer: The passive loss rules apply to individuals, estates, trusts, personal service corporations, and closely held C corporations. Passive income or loss from investments held by S corporations or partnerships flows through to the owners and the passive loss rules apply at the owner level.

For individuals, estates, trusts, and personal service corporations, losses or expenses generated by passive activities can be deducted only to the extent of income from all of the taxpayer’s passive activities. The application of the passive loss rules to closely held (non-personal service) C corporations is slightly different: these taxpayers may use passive losses to offset active income, but not portfolio income. Any unused passive losses are suspended and carried forward to future years to offset passive income generated in those years. Otherwise, suspended losses may be used when a taxpayer disposes of his or her entire interest in an activity.

1386. What special passive loss treatment is available to real estate activities?

Correct Answer: The special passive loss rules available to real estate activities allow the deduction of all or part of real estate rental losses against active or portfolio income, even though the activity otherwise is defined as a passive activity. The special rules are available in two situations:

• Losses from real estate rental activities are not treated as passive losses for certain qualifying real estate professionals.

• Qualifying individuals may deduct up to $25,000 of losses from real estate rental activities against active and portfolio income. The potential annual $25,000 deduction is reduced by 50 percent of the taxpayer’s AGI in excess of $100,000.

1387. When a taxpayer disposes of a passive activity by gift, what happens to any unused passive losses?

Correct Answer: In a disposition of a taxpayer’s interest in a passive activity by gift, the suspended losses are added to the basis of the property.

1388. Describe the general rules that limit the deduction of investment interest expense.

Correct Answer: The deduction of investment interest expense is limited to net investment income for the year. Any investment interest expense not deducted in the current year is carried over for potential use in succeeding years.

Investment interest expense is interest incurred on borrowed funds used to acquire or continue to hold investment assets. Net investment income is investment income (e.g., interest, annuities, royalties) reduced by investment expenses (i.e., deductible expenses directly connected to the production of investment income).

1389. Identify the types of income that are classified as investment income. Discuss the flexibility that a taxpayer has with respect to certain types of income that may potentially be considered investment income.

Correct Answer: Investment income is gross income from interest, dividends (in certain cases), annuities, and royalties not derived in the ordinary course of a trade or business. Passive activity income and income from a real estate activity in which the taxpayer actively participates are not included as investment income.

The taxpayer may elect to treat net capital gain and qualified dividends as investment income. Net capital gain includes gain attributable to the dispositions of property producing investment income or held for investment purposes. Qualified dividends are dividends that are taxed at the same marginal rate that is applicable to a net capital gain. If the taxpayer elects to treat net capital gain and qualified dividend income as investment income, they may not be taxed using the preferential capital gains rates.

1390. In deciding to enact the alternative minimum tax, Congress was concerned about the inequity that resulted when taxpayers with substantial economic incomes could avoid paying regular income tax.

*a. True b. False

1391. The AMT calculated using the indirect method will produce a different amount than the AMT calculated using the direct method.

a. True *b. False

1392. AMT adjustments can be positive or negative, whereas AMT preferences are always positive.

*a. True b. False

1393. Since most tax preferences are merely timing differences, they eventually will reverse and net to zero.

a. True *b. False

1394. Unless circulation expenditures are amortized over a three-year period for regular income tax purposes, there will be an AMT adjustment.

*a. True b. False

1395. A taxpayer who expenses circulation expenditures in the year incurred for regular income tax purposes will have a positive AMT adjustment in the following year.

a. True *b. False

1396. Assuming no phaseout, the AMT exemption amount for a married taxpayer filing separately for 2013 is more than the AMT exemption amount for C corporations.

*a. True b. False

1397. If the AMT base is greater than $175,000, the AMT rate for an individual taxpayer is the same as the AMT rate for a C corporation.

a. True *b. False

1398. The net capital gain included in an individual taxpayer’s AMT base is eligible for the beneficial alternative tax rate on net capital gain. This favorable alternative rate applies both in calculating the regular income tax and the AMT.

*a. True b. False

1399. Business tax credits reduce the AMT and the regular income tax in the same way.

a. True *b. False

1400. Prior to the effect of tax credits, Clarence’s regular income tax liability is $200,000 and his tentative AMT is $180,000. Clarence has nonrefundable business tax credits of $35,000. His tax liability is $165,000.

a. True *b. False

1401. If Abby’s alternative minimum taxable income exceeds her regular taxable income, she will have an alternative minimum tax.

a. True *b. False

1402. Madge’s tentative AMT is $112,000. Her regular income tax liability is $99,000. Madge’s AMT is $13,000.

*a. True b. False

1403. The phaseout of the AMT exemption amount for a taxpayer filing as a head of household both begins and ends at a higher income level than it does for a single taxpayer.

a. True *b. False

1404. Negative AMT adjustments for the current year caused by timing differences are offset by the positive AMT adjustments for prior tax years also caused by timing differences.

*a. True b. False

1405. Paul incurred circulation expenditures of $180,000 in 2013 and deducted that amount for regular income tax purposes. Paul has a $60,000 negative AMT adjustment for 2014, 2015, and for 2016.

a. True *b. False

1406. Keosha acquires 10-year personal property to use in her business in 2013 and takes the maximum cost recovery deduction for regular income tax purposes. As a result of this, Keosha will have a positive AMT adjustment in 2013.

*a. True b. False

1407. Joel placed real property in service in 2013 that cost $900,000 and used MACRS for regular income tax purposes. He is required to make a positive adjustment for AMT purposes in 2013 for the excess of depreciation calculated for regular income tax purposes over the depreciation calculated for AMT purposes.

a. True *b. False

1408. After personal property is fully depreciated for both regular income tax purposes and AMT purposes, the positive and negative adjustments that have been made for AMT purposes will net to zero.

*a. True b. False

1409. The required adjustment for AMT purposes for pollution control facilities placed in service in 2013 is equal to the difference between the amortization deduction allowed for regular income tax purposes and the depreciation deduction computed under ADS.

a. True *b. False

1410. The AMT adjustment for mining exploration and development costs can be avoided if the taxpayer elects to write off the expenditures in the year incurred for regular income tax purposes, rather than writing off the expenditures over a 10-year period for regular income tax purposes.

a. True *b. False

1411. Benita expensed mining exploration and development costs of $500,000 incurred in 2013. She will be required to make negative AMT adjustments for each of the next ten years and a positive AMT adjustment in the current tax year.

a. True *b. False

1412. The AMT adjustment for research and experimental expenditures can be avoided if the taxpayer capitalizes the expenditures and amortizes them over a 10-year period.

*a. True b. False

1413. Income from some long-term contracts can be reported using the completed contract method for regular income tax purposes, but the percentage of completion method is required for AMT purposes for all long-term contracts.

*a. True b. False

1414. Evan is a contractor who constructs both commercial and residential buildings. Even though some of the contracts could qualify for the use of the completed contract method, Evan decides to use the percentage of the completion method for all of his contracts. Unfortunately, this will have the effect of increasing Evan’s AMT adjustment associated with long-term contracts for the current year.

a. True *b. False

1415. Elmer exercises an incentive stock option (ISO) in 2013 for $6,000 (fair market value of the stock on the exercise date is $7,600). If Elmer sells the stock later in 2013 for $8,000, the AMT positive adjustment is $1,600 and the AMT negative adjustment is $2,000.

a. True *b. False

1416. In 2013, Ben exercised an incentive stock option (ISO), acquiring stock with a fair market value of $190,000 for $170,000. His AMT basis for the stock is $170,000, his regular income tax basis for the stock is $170,000, and his AMT adjustment is $0 ($170,000 – $170,000).

a. True *b. False

1417. The recognized gain for regular income tax purposes and the recognized gain for AMT purposes on the sale of stock acquired under an incentive stock option (ISO) program are always the same because the adjusted basis is the same.

a. True *b. False

1418. The sale of business property might result in an AMT adjustment.

*a. True b. False

1419. Cher sold undeveloped land that originally cost $150,000 for $225,000. There is a positive AMT adjustment of $75,000 associated with the sale of the land.

a. True *b. False

1420. Because passive losses are not deductible in computing either taxable income or AMTI, no adjustment for passive losses is required for AMT purposes.

a. True *b. False

1421. The deduction for charitable contributions in calculating the regular income tax can differ from that in calculating the AMT because the percentage limitations (20%, 30%, and 50%) may be applied to a different base amount.

a. True *b. False

1422. AGI is used as the base for application of percentage limitations (i.e., 20%, 30%, 50%) that apply to the charitable contribution deduction for regular income tax purposes. Modified AGI is used as the base for application of percentage limitations that apply to the charitable contribution deduction for AMT purposes.

a. True *b. False

1423. If a gambling loss itemized deduction is permitted for regular income tax purposes, there will be no AMT adjustment associated with the gambling loss.

*a. True b. False

1424. Nell has a personal casualty loss deduction of $14,500 for regular income tax purposes. The deduction would have been $26,600, but it had to be reduced by $100 and by $12,000 (10% ´ $120,000 AGI). For AMT purposes, the casualty loss deduction also is $14,500.

*a. True b. False

1425. If the regular income tax deduction for medical expenses is $0, under certain circumstances the AMT deduction for medical expenses can be greater than $0.

a. True *b. False

1426. The amount of the deduction for medical expenses under the regular income tax may be different than for AMT purposes if the taxpayer is at least age 65.

*a. True b. False

1427. Kerri, who had AGI of $120,000, itemized her deductions in the current year. She incurred unreimbursed employee business expenses of $8,500. Kerri must make a positive AMT adjustment of $2,400 in computing AMT.

a. True *b. False

1428. Interest on a home equity loan cannot be deducted for AMT purposes.

a. True *b. False

1429. The deduction for personal and dependency exemptions is allowed for regular income tax purposes, but is disallowed for AMT purposes. This results in a positive AMT adjustment.

*a. True b. False

1430. Kay had percentage depletion of $119,000 for the current year for regular income tax purposes. Cost depletion was $60,000. Her basis in the property was $90,000 at the beginning of the current year. Kay must treat the percentage depletion deducted in excess of cost depletion, or $59,000, as a tax preference in computing AMTI.

a. True *b. False

1431. If the taxpayer elects to capitalize intangible drilling costs and to amortize them over a 3-year period for regular income tax purposes, there is no adjustment or preference for AMT purposes.

a. True *b. False

1432. Interest income on private activity bonds issued before 2009 and after 2010, reduced by expenses incurred in carrying the bonds, is a tax preference item that is included in computing AMTI.

*a. True b. False

1433. Jackson sells qualifying small business stock for $125,000 (adjusted basis of $105,000) in 2013. In calculating gross income for regular income tax purposes, he excludes all of his realized gain of $20,000. The $20,000 exclusion is a tax preference in calculating Jackson’s AMTI.

a. True *b. False

1434. For individual taxpayers, the AMT credit is applicable for the AMT that results from timing differences, but it is not available for the AMT that results from the adjustment for itemized deductions or exclusion preferences.

*a. True b. False

1435. The corporate AMT no longer applies.

a. True *b. False

1436. The AMT exemption for a C corporation is $50,000 reduced by 25% of the amount by which AMTI exceeds $150,000.

a. True *b. False

1437. The AMT exemption for a corporation with $225,000 of AMTI is $18,750.

a. True *b. False

1438. Certain adjustments apply in calculating the corporate AMT that do not apply in calculating the noncorporate AMT and certain adjustments apply in calculating the noncorporate AMT that do not apply in calculating the corporate AMT.

*a. True b. False

1439. C corporations are subject to a positive AMT adjustment equal to 75% of the excess of ACE over AMTI before the ACE adjustment.

*a. True b. False

1440. All of a C corporation’s AMT is available for carryover as a minimum tax credit regardless of whether the adjustments and preferences originate from timing differences or AMT exclusions.

*a. True b. False

1441. Which of the following statements is incorrect?

a. The AMT calculated under the direct and indirect methods produces the same amount. b. The AMT calculated under the direct and the indirect methods produces different amounts. c. The tax forms use the direct method to calculate the AMT. *d. Only b. and c. are incorrect. e. a., b., and c. are incorrect.

1442. Ashby, who is single and age 30, provides you with the following information from his financial records for 2013.

Regular income tax $ 47,22 liability 8 AMT positive 40,000 adjustments AMT preferences 20,000 Taxable income 195,000

Calculate his AMT exemption for 2013.

a. $0. b. $23,450. *c. $17,000. d. $51,900. e. None of the above.

1443. Beula, who is a head of household and age 40, provides you with the following information from her financial records for 2013.

Regular income tax liability $ 35,77 6 AMT positive adjustments 33,000 AMT preferences 25,000 Taxable income 170,000

Calculate her AMTI for 2013.

a. $0. b. $171,300. c. $195,925. *d. $218,000. e. None of the above.

1444. Vicki owns and operates a news agency (as a sole proprietorship). During 2013, she incurred expenses of $24,000 to increase circulation of newspapers and magazines that her agency distributes. For regular income tax purposes, she elected to expense the $24,000 in 2013. In addition, Vicki incurred $15,000 in circulation expenditures in 2014 and again elected expense treatment. What AMT adjustments will be required in 2013 and 2014 as a result of the circulation expenditures?

*a. $16,000 positive in 2013, $2,000 positive in 2014. b. $16,000 negative in 2013, $2,000 positive in 2014. c. $16,000 negative in 2013, $10,000 positive in 2014. d. $16,000 positive in 2013, $10,000 positive in 2014. e. None of the above.

1445. Which of the following statements is correct?

a. If the tentative AMT is $10,000 and the regular income tax liability is $12,000, the AMT is $2,000. b. If the tentative AMT is $12,000 and the regular income tax liability is $10,000, the AMT is $12,000. c. If the tentative AMT is $10,000 and the regular income tax liability is $12,000, the AMT is a negative $2,000. *d. If the tentative AMT is $12,000, and the regular income tax liability is $10,000, the AMT is $2,000. e. None of the above.

1446. Which of the following statements is correct?

a. If the tentative minimum tax exceeds the regular income tax liability, the AMT is $0. *b. The exemption amount decreases as AMTI increases. c. The AMT tax rate for an individual taxpayer can be as high as 26%. d. Only a. and c. are correct. e. a., b., and c. are correct.

1447. For regular income tax purposes, Yolanda, who is single, is in the 35% tax bracket. Her AMT base is $220,000. Her tentative AMT is:

a. $57,200. *b. $58,100. c. $61,600. d. $77,000. e. None of the above.

1448. Ashly is able to reduce her regular income tax liability from $47,000 to $43,500 as the result of the alternative tax on net capital gain. Ashly’s tentative AMT is $51,000.

*a. Ashly’s tax liability is reduced by $3,500 as the result of the alternative tax calculation on net capital gain. b. Ashly’s AMT is increased by $3,500 as the result of the alternative tax calculation on net capital gain. c. Ashly’s tax liability is $43,500. d. Ashly’s tax liability is $47,000. e. None of the above.

1449. Prior to the effect of the tax credits, Justin’s regular income tax liability is $200,000 and his tentative AMT is $195,000. Justin has the following credits:

Child tax credit $1,000 Adoption expenses credit 5,000

Calculate Justin’s tax liability after credits.

a. $190,000. *b. $194,000. c. $195,000. d. $200,000. e. None of the above.

1450. Prior to the effect of tax credits, Eunice’s regular income tax liability is $325,000 and her tentative AMT is $312,000. Eunice has general business credits available of $20,000. Calculate Eunice’s tax liability after tax credits.

a. $0. b. $305,000. *c. $312,000. d. $325,000. e. None of the above.

1451. Which of the following normally produces positive AMT adjustments?

a. Real property taxes deduction. b. Personal exemption deduction. c. Charitable contribution deduction. *d. Only a. and b. are correct. e. a., b., and c. are correct.

1452. Dale owns and operates Dale’s Emporium as a sole proprietorship. On January 30, 1998, Dale’s Emporium acquired a warehouse for $100,000. For regular income tax purposes in 2013, depreciation was deducted under MACRS using a rate of 2.564%. Determine the AMT adjustment for depreciation and indicate whether it is positive or negative.

a. $64 negative adjustment. *b. $64 positive adjustment. c. No adjustment is required because Dale’s Emporium used the Alternative Depreciation System (ADS) to compute depreciation on the property for AMT purposes. d. No adjustment is required because Dale’s Emporium used MACRS to compute the depreciation of the property for regular income tax purposes. e. None of the above.

1453. Omar acquires used 7-year personal property for $100,000 to use in his business in February 2013. Omar does not elect § 179 expensing, but does take the maximum regular cost recovery deduction. He elects not to take additional first-year depreciation. As a result, Omar will have a positive AMT adjustment in 2013 of what amount?

a. $0. *b. $3,580. c. $10,710. d. $14,290. e. None of the above.

1454. In 2013, Blake incurs $270,000 of mining exploration expenditures, and deducts the entire amount for regular income tax purposes. Which of the following statements is correct?

a. For AMT purposes, Blake will have a positive adjustment of $243,000 in 2013. b. Blake will have a negative AMT adjustment of $27,000 in 2018. c. Over a 10-year period, positive and negative adjustments will net to zero. d. Only a. and c. are correct. *e. a., b., and c. are correct.

1455. Wallace owns a construction company that builds both commercial and residential buildings. He contracts to build a residential building for $800,000 for which he is eligible to use the completed contract method of accounting. In the current year for regular income tax purposes, Wallace does not recognize any income on the contract. Under the percentage of complete method, the income recognized under the contract would have been $60,000. Wallace’s AMT adjustment is:

a. $0. b. $60,000 negative adjustment. *c. $60,000 positive adjustment. d. $800,000 positive adjustment. e. None of the above.

1456. In 2005, Collies exercised an incentive stock option (ISO), acquiring 150 shares of stock at an option price of $75 per share (fair market value at the date of exercise was $130 per share). In 2013, the rights in the stock become freely transferable (fair market value is still $130 per share). Which of the following statements is incorrect?

a. Collis has no AMT adjustment from the ISO in 2011. b. Collis has no taxable income from the ISO in 2011. c. Collis has an AMT basis of $19,500 in the stock. d. Collis has an income tax basis of $11,250 in the stock. *e. All of the above are correct.

1457. Marvin, the vice president of Lavender, Inc., exercises stock options for 100 shares of stock in March 2013. The stock options are incentive stock options (ISOs). Their exercise price is $20 and the fair market value on the date of exercise is $28. The options were granted in March 2010 and all restrictions on the free transferability had lapsed by the exercise date.

a. If Marvin sells the stock in December 2013 for $3,000, his AMT adjustment in 2013 is a positive adjustment of $800. b. If Marvin sells the stock in December 2014 for $3,000, his AMT adjustment in 2014 is $0. c. If Marvin sells the stock in December 2013 for $3,000, his AMT adjustment in 2013 is a negative adjustment of $800. d. If Marvin sells the stock in December 2014 for $3,000, his AMT adjustment in 2014 is a negative adjustment of $1,000. *e. None of the above.

1458. Factors that can cause the adjusted basis for AMT purposes to be different from the adjusted basis for regular income tax purposes include the following:

a. A different amount of depreciation (cost recovery) has been deducted for AMT purposes and regular income tax purposes. b. The spread on an incentive stock option (ISO) is recognized for AMT purposes, but is not recognized for regular income tax purposes. c. A different amount has been deducted for circulation expenditures for AMT purposes and for regular income tax purposes. d. Only a. and b. *e. a., b., and c.

1459. In 2013, Glenn had a $108,000 loss on a passive activity. None of the loss is attributable to AMT adjustments or preferences. She has no other passive activities. Which of the following statements is correct?

a. In 2013, Glenn can deduct $108,000 for regular income tax purposes and for AMT purposes. b. Glenn will have a $108,000 tax preference in 2013 as a result of the passive activity. *c. For regular income tax purposes, none of the loss is allowed in 2013. d. In 2013, Glenn will have a positive adjustment of $25,000 as a result of the passive loss. e. None of the above.

1460. Akeem, who does not itemize, incurred a net operating loss (NOL) of $50,000 in 2013. His deductions in 2013 included AMT tax preference items of $20,000, and he had no AMT adjustments. Assuming the NOL is not carried back, what is Akeem’s ATNOLD carryover to 2014?

a. $50,000. *b. $30,000. c. $20,000. d. $40,000. e. None of the above.

1461. Vinny’s AGI is $250,000. He contributed $200,000 in cash to the Boy Scouts, a public charity. What is Vinny’s charitable contribution deduction for AMT purposes?

a. $0. b. $50,000. c. $75,000. *d. $125,000. e. None of the above.

1462. Mitch, who is single and age 66 and has no dependents, had AGI of $100,000 in 2013. His potential itemized deductions were as follows:

Medical expenses (before percentage $15,000 limitation) State income taxes 3,000 Real estate taxes 7,000 Mortgage (qualified housing and 9,000 residence) interest Cash contributions to various 4,000 charities Unreimbursed employee expenses 4,300 (before percentage limitation)

What is the amount of Mitch’s AMT adjustment for itemized deductions for 2013?

*a. $14,800. b. $16,800. c. $19,300. d. $25,800. e. None of the above.

1463. Which of the following itemized deductions definitely will be the same amount for the regular income tax and the AMT and thus result in no AMT adjustment in 2013?

a. Real property taxes. b. Casualty losses. c. Charitable contributions. *d. Only b. and c. e. a., b., and c.

1464. Ted, who is single, owns a personal residence in the city. He also owns a condo near the ocean. He uses the condo as a vacation home. In March 2013, he borrowed $50,000 on a home equity loan and used the proceeds to acquire a luxury automobile. During 2013, he paid the following amounts of interest:

• on his personal residence $15,500 • on the condo 6,200 • on the home equity loan 4,800 • on credit card obligations 1,700

What amount, if any, must Ted recognize as an AMT adjustment in 2013?

a. $0. *b. $4,800. c. $6,200. d. $11,000. e. None of the above.

1465. Which of the following statements is correct?

*a. The deduction for personal and dependency exemptions is not permitted in calculating the AMT. Therefore, in converting regular taxable income to AMTI, a positive adjustment is required. b. To the extent that itemized deductions exceed the standard deduction for regular income tax purposes, a positive AMT adjustment is required in converting regular taxable income to AMTI. c. The charitable contribution deduction for AMT purposes and for regular income tax purposes can be different. If this occurs, a positive AMT adjustment is required for the amount of the difference. d. Only a. and b. are correct. e. a., b., and c. are correct.

1466. Eula owns a mineral property that had a basis of $23,000 at the beginning of the year. Cost depletion is $19,000. The property qualifies for a 15% depletion rate. Gross income from the property was $200,000 and net income before the percentage depletion deduction was $50,000. What is Eula’s tax preference for excess depletion?

a. $15,000. b. $23,000. c. $25,000. d. $0. *e. None of the above.

1467. Which of the following can produce an AMT preference rather than an AMT adjustment?

a. Interest on private activity bonds issued in 2013. b. Percentage depletion. c. Incentive stock options (ISOs). *d. Only a. and b. e. a., b., and c.

1468. Celia and Amos, who are married filing jointly, have one dependent and do not itemize deductions. They have taxable income of $82,000 and tax preferences of $53,000 in 2013. What is their AMT base for 2013?

a. $0. *b. $79,350. c. $94,450. d. $158,900. e. None of the above.

1469. Robin, who is a head of household and age 42, provides you with the following information from his financial records for 2013.

Regular income tax liability $ 42,53 9 AMT positive adjustments 30,000 AMT preferences 20,000 Taxable income 185,000

Calculate his AMT for 2013.

*a. $13,601. b. $17,825. c. $42,539. d. $62,300. e. None of the above.

1470. Kay, who is single, had taxable income of $0 in 2013. She has positive timing adjustments of $206,300 and exclusion items of $100,000 for the year. What is the amount of her alternative minimum tax credit for carryover to 2014?

a. $80,500. *b. $68,861. c. $20,930. d. $13,403. e. None of the above.

1471. Sand Corporation, a calendar year taxpayer, has alternative minimum taxable income [before adjustment for adjusted current earnings (ACE)] of $900,000 for 2013. If Sand’s (ACE) is $975,000, its tentative minimum tax for 2013 is:

a. $0. b. $56,250. c. $180,000. *d. $191,250. e. None of the above.

1472. Mauve, Inc., has the following for 2012, 2013, and 2014 and no prior ACE adjustments.

2012 2013 2014 Pre-adjusted AMTI $12,000 $15,000 $8,000 Adjusted current 10,00 17,0 5,00 earnings 0 00 0

What is the ACE adjustment for each of the three years?

2012 2013 2014

*a. $0 $1,500 ($1,500) b. ($2,000) $2,000 ($3,000) c. $2,000 ($2,000) $3,000 d. ($1,500) $1,500 $2,250 e. $1,500 ($1,500) ($2,250)

1473. Which of the following statements is correct?

a. A C corporation classified as a small corporation is eligible to have the AMT calculated using the AMT provisions for individuals. *b. A C corporation classified as a small corporation is not subject to the AMT. c. A C corporation classified as a small corporation is subject to the AMT only on its adjusted current earnings. d. A C corporation classified as a small corporation is eligible to use the 20%/15%/0% tax rate on net capital gain and qualified dividends rather than the regular tax rates applicable to other C corporations. e. a., b., and c. are correct.

1474. Use the following data to calculate Jolene’s AMTI.

Taxable income $190,000 AMT adjustments Positive 70,000 Negative (14,000) Tax preferences 25,000

Correct Answer: Jolene’s AMTI is calculated as follows:

Taxable income $190,000 Plus: Positive AMT 70,000 adjustments Minus: Negative AMT (14,000) adjustments Plus: Tax preferences 25,000 AMTI $271,000

1475. Use the following selected data to calculate Devon’s taxable income.

Tax preferences $ 45,00 0 Positive AMT adjustments 52,000 Negative AMT adjustments 15,000 AMTI 290,000

Correct Answer: AMTI $290,000 Plus: Negative AMT 15,000 adjustments Minus: Positive AMT (52,000) adjustments Minus: Tax preferences (45,000) Equals: Taxable income $208,000

1476. Arlene, who is single, has taxable income for 2013 of $112,000. Calculate her alternative minimum tax, if any, given the following additional information.

AMT adjustments Positive $22,000 Negative (25,000) Tax preferences 46,000

Correct Answer: Arlene’s AMT is calculated as follows:

Taxable income $112,000 Plus: Positive AMT 22,000 adjustments Minus: Negative AMT (25,000) adjustments Plus: Tax preferences 46,000 AMTI $155,000 Minus: Exemption [$51,900 (42,000) – 25%($155,000 – $115,400)] Alternative minimum tax base $113,000 Times: 26% rate × 26% Tentative AMT $ 29,380 Minus: Regular income tax (24,653) liability (Note 1) AMT $ 4,72 7

Note 1

Tax on $87,850 $17,891 28% × $24,150 ($112,000 – 6,76 $87,850) 2 $24,653

1477. Caroline and Clint are married, have no dependents, and file a joint return in 2013. Use the following selected data to calculate their Federal income tax liability.

AMTI $285,000 Regular income tax liability 42,066 AMT tax preferences 90,000

Correct Answer: The AMT is calculated as follows:

AMTI $285,000 AMT exemption [$80,800 – (48,0 25%($285,000 – $153,900)] 25) AMT base $236,975

$175,000 × 26% = $ 45,500 $61,975 × 28% = 17,3 53 Tentative AMT $ 62,853 Regular income tax liability (42,06 6) AMT $ 20,787

Note that the AMT preferences of $90,000 are already included in AMTI. The Federal income tax liability is the summation of the regular income tax liability and the AMT ($42,066 + $20,787 = $62,853).

1478. Bianca and Barney have the following for 2013:

Regular income tax before $32,000 credits Tentative AMT before 45,000 credits

a. Calculate Bianca and Barney’s AMT if they qualify for the adoption expense credit of $11,000. b. Calculate Bianca and Barney’s AMT if they qualify for the adoption expense credit of $12,970.

Correct Answer: a. In addition to the foreign tax credit, nonrefundable personal credits (e.g., child tax credit, adoption expense credit) in 2013 may offset both the regular income tax and the AMT. Thus, Bianca and Barney’s AMT is calculated as follows:

Tentative AMT before credits $45,000 Less: Regular income tax (32,500) before credits AMT before adoption expense $12,500 credit Less: Adoption expense (11,000) credit AMT $ 1,500

b. Tentative AMT before $45,000 credits Less: Regular income tax (32,500) before credits AMT before adoption $12,500 expense credit Less: Adoption expense (12,500) credit AMT $ – 0–

Only $12,500 of the adoption expense credit of $12,970 is needed to reduce the AMT to $0.

1479. Gunter, who is divorced, has the following items for 2013.

Salary $60,000 Gain on sale of land held 10,000 nine months (regular income tax basis is $4,000 less than AMT basis) Traditional IRA contribution 5,000 Alimony paid 9,000 Itemized deductions: Charitable contributions $3,000 Home mortgage interest on 4,000 his principal residence State income taxes 2,000 9,000 Tax preferences 15,000

Calculate Gunter’s AMTI for 2013.

Correct Answer: Gunter’s regular income tax taxable income is calculated as follows:

Gross income: Salary $60,000 Gain on sale of land 10,000 $70,000

Less: Deductions for AGI Traditional IRA $ 5,000 contribution Alimony 9,00 (14,000) 0 AGI $56,000

Less: Deductions from AGI Itemized deductions Charitable $ 3,000 contributions Home mortgage 4,000 interest State income taxes 2,00 (9,000) 0 Personal exemption (3,9 deduction (1 × 00) $3,900) Taxable income $43,100

His AMTI is calculated as follows:

Taxable income $43,100 Plus: Positive AMT adjustments: Disallowed state $ 2,000 income taxes Disallowed 3,90 5,900 personal exemption 0 deduction Minus: Negative AMT adjustment: Smaller gain on (4,000) sale of land Plus: Tax preferences 15,000 AMTI $60,000

1480. Tad and Audria, who are married filing a joint return, have AMTI of $256,000 for 2013. Calculate their AMT exemption.

Correct Answer: Statutory amount $80,800 Less phaseout: 25%($256,000 (25,525) – $153,900) AMTI exemption $55,275

1481. In 2013, Linda incurs circulation expenses of $240,000 which she deducts in calculating taxable income. a. Calculate Linda’s AMT adjustment for circulation expenses for 2013, 2014, 2015, and 2016. b. Advise Linda on how she could reduce or eliminate the AMT adjustment in 2013.

Correct Answer: a. Linda deducted the $240,000 circulation expenses in 2013 for regular income tax purposes. For AMT purposes, the $240,000 must be amortized over a 3-year period ($80,000 per year). So Linda has a positive AMT adjustment of $160,000 ($240,000 – $80,000) in 2013. In 2014 and 2015, she has a negative AMT adjustment of $80,000 each year. There is no regular income tax or AMT effect in 2016. b. Louise could elect to amortize the circulation expenses of $240,000 over a 3- year period ($80,000 each year). In this case, the deduction for regular income tax and AMT purposes would be the same and no AMT adjustment is necessary.

1482. In June, Della purchases a building for $800,000 to use in her business as an office building. Della uses the depreciation method which will provide her with the greatest deduction for regular income tax purposes. a. Calculate the AMT adjustment for depreciation in 2013 if Della purchased the building in 2013. b. Calculate the AMT preference for depreciation in 2013 if Della purchased the building in 1986.

Correct Answer: a. $0. For real property placed in service after 1998, the MACRS recovery periods are used in calculating the AMT depreciation. Thus, the depreciation deduction [$11,128 ($800,000 × 1.391%)] is the same for both regular income tax and AMT purposes. b. For real property placed in service before 1987, the AMT preference is the excess of accelerated over straight-line depreciation. However, since the building is fully depreciated at the end of the 20th year for both regular income tax and AMT purposes, the AMT preference is $0. The 28th year of the building’s life is 2013.

1483. Lavender, Inc., incurs research and experimental expenditures of $210,000 in 2013. Determine the amount of the AMT adjustment for 2013 and for 2014 if for regular income tax purposes: a. Lavender expenses the research and experimental expenses. b. Lavender capitalizes the research and experimental expenses and elects to amortize them over a 10-year period.

Correct Answer: a. The expense for regular income tax purposes is $210,000 in 2013 and $0 in 2014. For AMT purposes, the expense is $21,000 ($210,000/10) for 2013 and 2014. The AMT adjustment is: 2013 2014 Expense for regular $210,000 $ –0– income tax purposes Expense for AMT (21,000) (21,000) purposes AMT adjustment $189,000 ($21,000)

b. The amount of the AMT adjustment is $0 for both 2012 and 2013. No adjustment is needed as the amount of the expense for regular income tax and for AMT purposes in both 2013 and 2014 is the same.

1484. In May 2012, Swallow, Inc., issues options to Karrie, a corporate officer, to purchase 100 shares of Swallow stock under an ISO plan. At the date the stock options are issued, the fair market value of the stock is $1,000 per share and the option price is $1,200 per share. The stock becomes freely transferable in 2013. Karrie exercises the options in November 2012 when the stock is selling for $1,500 per share. She sells the stock in December 2014 for $1,800 per share. a. Determine the amount of the AMT adjustment for 2012. b. Determine the amount of the AMT adjustment for 2013. c. Determine Karrie’s recognized gain for regular income tax purposes and for AMT purposes in 2014 on the sale of the stock. d. Determine the amount of the AMT adjustment for 2014.

Correct Answer: a. For regular income tax purposes, Karrie does not have income upon the exercise of the ISO in November 2012. However, the $30,000 excess of the fair market value of the stock over the exercise price ($150,000 – $120,000 = $30,000) is a positive adjustment for AMT purposes in the year the stock becomes freely transferable (2013). Therefore, there is no AMT adjustment in 2012. b. A positive AMT adjustment of $30,000 ($150,000 – $120,000) occurs in 2013 when the stock becomes freely transferable.

c. Regular Income Tax AMT Amount realized $180,000 $180,000 Adjusted basis (120,000)(150,000 ) Realized and recognized $ 60,000 $ 30,0 gain 00

d. The AMT adjustment in 2014 is the excess of the recognized gain for regular income tax purposes over the recognized gain for AMT purposes ($60,000 – $30,000 = $30,000). The adjustment is a negative adjustment.

1485. Frederick sells land and building whose adjusted basis for regular income tax purposes is $345,000 and for AMT purposes is $380,000. The sales proceeds are $850,000. Determine the effect on: a. Taxable income. b. AMTI.

Correct Answer: a. Amount realized $850,000 Adjusted basis (345,000) Recognized gain $505,000

The recognized gain increases taxable income by $505,000.

b. Amount realized $850,000 Adjusted basis (380,000) Recognized gain $470,000

Using the direct method to calculate AMTI, the recognized gain increases AMTI by $470,000. Using the indirect method, the recognized gain using the regular income tax adjusted basis increases taxable income by $505,000 which would in turn increase AMTI by the same amount. However, a negative AMT adjustment of $35,000 is made in calculating AMTI for the difference between the recognized gain for regular income tax purposes of $505,000 and the recognized gain for AMT purposes of $470,000.

1486. Cindy, who is single and age 66, has no dependents and has adjusted gross income of $50,000 in 2013. Her potential itemized deductions are as follows:

Medical expenses (before $10,000 percentage limitation) State income taxes 2,500 Real estate taxes 4,000 Mortgage (qualified housing and 5,500 residence) interest Unreimbursed employee expenses 2,600 (before percentage limitation)

What is the amount of Cindy’s AMT adjustment for itemized deductions for 2013?

Correct Answer: Cindy’s adjustments for itemized deductions are as follows:

Medical expenses (Note 1) $1,250 State income taxes 2,500 Real estate taxes 4,000 Unreimbursed employee 1,600 expenses (Note 2) Mortgage interest (Note 3) – 0– Total $9,350

Notes

(1) Medical expenses: For income tax [$10,000 – $6,250 (7.5% × $50,000 AGI)] For AMT [$10,000 – (10% × (5,000) $50,000 AGI)] Positive adjustment $1,250

(2) Unreimbursed employee $1,600 expenses: $2,600 – (2% × $50,000 AGI)

(3) The mortgage interest of $5,500 is the same (i.e., qualified housing and residence interest) in this case for AMT purposes and regular income tax purposes.

1487. In calculating her 2013 taxable income, Rhonda, who is age 45, deducts the following itemized deductions.

Medical expenses [$6,500 $ 500 – 10%($60,000)] State income taxes 2,500 Property taxes 3,000 Home mortgage interest on 5,000 principal residence Charitable contributions 3,500 Miscellaneous itemized 800 deductions [$2,000 – 2%($60,000)] Total itemized deductions $15,300 for regular income tax

Calculate Rhonda’s AMT adjustment for itemized deductions.

Correct Answer: Rhonda’s allowed itemized deductions for AMT purposes are as follows:

Medical expenses [$6,500 – $ 500 (10% × $60,000)] Qualified residence interest 5,000 Charitable contributions 3,500 Total itemized deductions for $9,000 AMT

Therefore, Rhonda’s positive adjustment in computing AMTI is $6,300 ($15,300 – $9,000).

1488. Darin’s, who is age 30, has itemized deductions in calculating 2013 taxable income as follows:

Medical expenses [$15,000 – $ 5,000 10%($100,000)] Mortgage interest on personal 7,000 residence State income taxes 4,000 Real property taxes 4,500 Personal property taxes 2,500 Casualty loss [$14,000 – (10% 4,000 ´ $100,000)] Charitable contributions 3,500 Gambling losses ($3,200 1,000 limited to gambling income of $1,000) Miscellaneous itemized 500 deductions [$2,500 – 2%($100,000)] Itemized deductions $32,000

a. Calculate Darin’s itemized deductions for AMT purposes using the direct method. b. Calculate Darin’s itemized deductions for AMT purposes using the indirect method.

Correct Answer: a. For AMT purposes, the allowable itemized deductions are as follows:

Medical expenses [$15,000 – (10% $ 5,000 × $100,000)] Mortgage interest on personal 7,000 residence Casualty loss [$14,000 – (10% × 4,000 $100,000)] Charitable contributions 3,500 Gambling losses ($3,200 limited to 1, gambling income of $1,000) 000 Itemized deductions $20,500

b. Using the indirect method, the effect of itemized deductions on AMTI is as follows:

Itemized deductions $32,000 included in taxable income Positive adjustments –0– Negative adjustments: Medical expenses [$5,000 – (–0–) (10% × $100,000)] State income taxes (4,000) Real property taxes (4,500) Personal property taxes (2,500) Miscellaneous itemized (500) deductions $20,500

1489. Abbygail, who is single, had taxable income of $115,000 for 2013. She had positive AMT adjustments of $30,000; negative AMT adjustments of $12,000; and tax preference items of $50,000. a. Compute her alternative minimum taxable income (AMTI). b. Assume the same facts as in part a. Compute Abbygail’s tentative minimum tax.

Correct Answer: a. Taxable income $115,000 Plus: Positive AMT 30,000 adjustments Less: Negative AMT (12,000) adjustments Plus: Tax preference items 50,000 AMTI $183,000

b. AMTI $183,000 Less: AMT exemption [$51,900 (35,000) – .25($183,000 – $115,400)] AMT base $148,000

Tentative minimum tax $148,000 × .26 $ 38,480

1490. Lilly is single and has no taxable income for 2013. She has positive timing adjustments of $600,000 and AMT exclusions of $200,000. a. Calculate Lilly’s tentative AMT. b. Calculate Lilly’s AMT credit carryover to 2014.

Correct Answer: a. Taxable income $ – 0– AMT adjustments and preferences 800,000 ($600,000 + $200,000) AMTI $800,000 AMT exemption [$51,900 – .25($800,000 – $115,400)] –0– AMT base $800,000

Tentative minimum tax $175,000 $ 45,500 × .26 = $625,000 175,000 × .28 = Tentative AMT $220,500

b. Tentative AMT without positive timing adjustments AMT exclusions $200,000 AMT exemption [$51,900 – (30,750) .25($200,000 – $115,400)] AMT base $169,250

Tentative minimum tax $169,250 × .26 = $44,005

Tentative AMT $220,500 Portion related to (44,005) AMT exclusions AMT credit carryover $176,495

1491. Sage, Inc., has the following gross receipts and taxable income:

Gross Taxable receipts income 2001 $5,600,000 $120,000 2002 4,300,000 190,000 2003 7,200,000 162,000 2004 9,000,000 180,000 2005 6,200,000 150,000 2006 6,500,000 160,000 2007 6,800,000 162,000 2008 7,000,000 154,000 2009 7,200,000 190,000 2010 7,100,000 200,000 2011 7,300,000 185,000 2012 7,200,000 201,000 2013 7,000,000 225,000

Is Sage, Inc., subject to the AMT in 2013?

Correct Answer: Sage, Inc. is not subject to the AMT in 2013. It qualifies for the small corporation exemption (i.e., had average annual gross receipts of less than $5 million for the 3-year period beginning after 1993 and had average annual gross receipts of less than $7.5 million for the 3-year period preceding each subsequent current tax year). Even if these requirements were not satisfied for 2001, the small corporation exemption would apply for the first year of existence of the corporation.

For 2002 (the $7.5 million requirement) [($0 + $0 + $5,600,000)/1] = $5.6 million

For 2003 (the $7.5 million requirement) [($0 + $5,600,000 + $4,300,000)/2] = $4.95 million

For 2004 (the $7.5 million requirement) [($5,600,000 + $4,300,000 + $7,200,000)/3] = $5.7 million

For 2005 (the $7.5 million requirement) [($4,300,000 + $7,200,000 + $9,000,000)/3] = $6.83 million

For 2006 (the $7.5 million requirement) [($7,200,000 + $9,000,000 + $6,200,000)/3] = $7.47 million

For 2007 (the $7.5 million requirement) [($9,000,000 + $6,200,000 + $6,500,000)/3] = $7.23 million

For 2008 (the $7.5 million requirement) [($6,200,000 + $6,500,000 + $6,800,000)/3] = $6.5 million

For 2009 (the $7.5 million requirement) [($6,500,000 + $6,800,000 + $7,000,000)/3]= $6,766,667

For 2010 (the $7.5 million requirement) [($6,800,000 + $7,000,000 + $7,200,000)/3] = $7 million

For 2011 (the $7.5 million requirement) [($7,000,000 + $7,200,000 + $7,100,000)/3] = $7.1 million

For 2012 (the $7.5 million requirement) [($7,200,000 + $7,100,000 + $7,300,000)/3] = $7.2 million For 2013 (the $7.5 million requirement) [($7,100,000 + $7,300,000 + $7,200,000)/3)] = $7.2 million

1492. Smoke, Inc., provides you with the following information:

Regular corporate tax liability $ 22,25 0 AMT adjustments and preferences 80,000 (excluding ACE adjustment) ACE adjustment (prior positive (11,000) adjustments are $17,000) Taxable income 100,000

Calculate Smoke’s AMT for 2013.

Correct Answer: Smoke’s AMT is calculated as follows:

Taxable income $100,000 + AMT adjustments and preferences 80,000 (excluding ACE adjustment) – AMT adjustment for ACE (11,000 ) AMTI $169,000 – AMT exemption [$40,000 – (40,000 25%($150,000 – $150,000)] ) = AMT base $129,000 × Rate × 2 0% = Tentative AMT $ 25,800 – Regular income tax liability (22,250 ) AMT $ 3,55 0

1493. Sand, Inc., has AMTI of $200,000. Calculate the amount of the AMT exemption if: a. Sand is a small corporation for AMT purposes. b. Sand is not a small corporation for AMT purposes.

Correct Answer: a. If Sand is a small corporation for AMT purposes, the AMT does not apply to Sand. b. If Sand is not a small corporation for AMT purposes, the AMT exemption is calculated as follows:

AMTI $200,000 – AMT exemption threshold (150,000) Excess ($ 50,000) × 25% = Phaseout of exemption amount $ 12,500

Statutory exemption amount $ 40,000 – AMT exemption phaseout (12,500) AMT exemption $ 27,500

1494. Why is there a need for a second tax system called the alternative minimum tax?

Correct Answer: The AMT was enacted as a backup to the regular income tax. The purpose is to ensure that no taxpayer with significant economic income can avoid significant tax liability by using the exclusions, deductions, and credits contained in the regular income tax system.

1495. What is the relationship between taxable income and AMTI?

Correct Answer: The indirect method, rather than the direct method, is normally used to calculate the AMT (i.e., the starting point is taxable income).

If the direct method were used, AMTI could be calculated as follows:

Gross income computed by applying the AMT rules Minus: Deductions computed by applying the AMT rules Equals: AMTI before tax preferences Plus: Tax preferences Equals: Alternative minimum taxable income

Since the indirect method is normally used to calculate AMTI, the model is as follows:

Taxable income Plus: Positive AMT adjustments Minus: Negative AMT adjustments Equals: Taxable income after AMT adjustments Plus: Tax preferences Equals: Alternative minimum taxable income (AMTI)

The above model illustrates the relationship between taxable income and AMTI.

1496. What is the relationship between the regular income tax liability and the tentative AMT?

Correct Answer: If the tentative AMT exceeds the regular income tax liability, the AMT is the amount of the excess. If the regular income tax liability is equal to or greater than the tentative AMT, the AMT is zero.

1497. In calculating the AMT using the indirect method, do AMT adjustments and AMT tax preferences increase or decrease AMTI?

Correct Answer: AMT adjustments can be both positive and negative. AMT preferences can only be positive so AMT adjustments can increase or decrease AMTI whereas AMT tax preferences can only increase AMTI.

1498. Will all AMT adjustments reverse? That is, do they relate to timing differences?

Correct Answer: Most, but not all, AMT adjustments do relate to timing differences and thus will reverse. However, since not all AMT adjustments relate to timing differences, not all will reverse. For example, some of the AMT adjustments relating to itemized deductions will not reverse.

1499. What is the purpose of the AMT exemption amount? What is the maximum amount for each filing status for an individual taxpayer and for a corporate taxpayer?

Correct Answer: The AMT exemption amount can be thought of as a materiality provision. As such, it enables a taxpayer with a small amount of positive AMT adjustments and tax preferences to avoid being subject to the AMT.

For individual taxpayers, the maximum exemption amounts for 2013 are as follows:

• Married filing $80,800 jointly • Single 51,9 00 • Married filing 40,4 separately 00

For corporate taxpayers, the maximum exemption amount is $40,000.

1500. Melinda is in the 35% marginal tax bracket. She has a net capital gain of $150,000 on the sale of land which is eligible for the alternative tax on net capital gain in calculating the regular income tax. Discuss the tax rate that applies to the $150,000 net capital gain in calculating the tentative AMT for Melinda.

Correct Answer: The alternative tax rate on net capital gain is available in calculating the tentative AMT. Since the 15% rate is lower than the regular AMT rates (i.e., 26% and 28%), the 15% rate should be used by Melinda.

1501. What tax rates apply in calculating the tentative AMT for an individual taxpayer?

Correct Answer: For an individual taxpayer, a 26% rate applies to the first $175,000 of AMT base and a 28% rate applies to the AMT base in excess of $175,000.

1502. How can an AMT adjustment be avoided for a taxpayer who incurs circulation expenditures in the current tax year?

Correct Answer: For regular income tax purposes, the taxpayer can capitalize the expenditures and amortize them over a three-year period beginning with the tax year in which they were made. For AMT purposes, only the three- year amortization is allowed. Thus, no adjustment will be necessary.

1503. In determining the amount of the AMT adjustments, discuss the difference in the treatment of a building placed in service after 1986 and before January 1, 1999 and a building placed in service after December 31, 1998.

Correct Answer: For a building placed in service after December 31, 1998, the same MACRS recovery periods and methods are used in calculating depreciation for both regular income tax purposes and AMT purposes. Therefore, there is no AMT adjustment necessary.

For a building placed in service after 1986 and before January 1, 1999, depreciation is computed for AMT purposes under the alternative depreciation system (ADS) using the straight-line method over a 40-year life. For a building placed in service after 1986 and before January 1, 1999, depreciation is computed for regular income tax purposes under MACRS using the straight-line method over a 27.5, 31.5, or 39-year life. The difference between the depreciation amounts is the adjustment in computing the AMT. However, if the taxpayer elects to use ADS in calculating deprecation for regular income tax purposes, there is no AMT adjustment.

1504. How can the positive AMT adjustment for research and experimental expenditures be avoided?

Correct Answer: For regular income tax purposes, research and experimental expenses can be deducted in the year incurred. For AMT purposes, such expenses must be amortized over a 10-year period. So there will be a positive AMT adjustment in year 1 and negative AMT adjustments in years 2 through 10.

If the taxpayer elects to amortize the research and experimental expenses over a 10-year period for regular income tax purposes, the deductions for regular income tax and AMT will be the same and no AMT adjustments are necessary.

1505. Durell owns a construction company that builds residential housing. The company is eligible to use the completed contract method for regular income tax purposes. What can Durell do to minimize his AMT?

Correct Answer: The use of the completed contract method for regular income tax purposes by Durell will result in the excess of the income calculated under the percentage of completion method over that reported under the completed contract method being treated as a positive adjustment for AMT purposes. Likewise, if for a particular tax year, the income calculated under the percentage of completion method is less than that under the completed contract method, this difference is a negative adjustment for AMT purposes. Durell can eliminate this AMT positive or negative adjustment by using the percentage of completion method for regular income tax purposes. Then, the same method of accounting will be used for both regular income tax purposes and AMT purposes.

1506. Discuss the tax year in which an AMT adjustment is first required for an ISO.

Correct Answer: For regular income tax purposes, no income is recognized due to the exercise of an ISO. For AMT purposes, the exercise of an ISO triggers the recognition of income (to the extent of the spread) in the tax year in which the rights in the stock are freely transferable or are not subject to a substantial risk of forfeiture.

1507. If Jessica exercises an ISO and disposes of the option in the same tax year, are any AMT adjustments required?

Correct Answer: In this case, no AMT adjustments are required for Jessica because the positive AMT adjustment and the negative AMT adjustment would offset.

1508. Danica owned a car that she used exclusively for business. The car was purchased in 2010 and sold in 2013 for a recognized gain of $9,000. However, the sale resulted in no AMT. Why?

Correct Answer: The car is classified as personalty used in a business. Since it was placed in service by Danica after December 31, 1998, the depreciation for AMT and regular income tax purposes can be the same (150% declining-balance method). Under this assumption, there was no annual AMT adjustment for depreciation and no AMT adjustment is necessary on the sale of the car by Danica (the adjusted basis for AMT and regular income tax purposes is the same).

1509. A taxpayer has a passive activity loss for the current tax year for regular income tax purposes and for AMT purposes. Is it possible that the passive activity losses will be the same amount?

Correct Answer: The passive activity loss for regular income tax purposes and AMT purposes will differ only if the rules relating to the calculation of each differ.

1510. What itemized deductions are allowed for both regular income tax purposes and for AMT purposes?

Correct Answer: The following itemized deductions are allowed for both regular income tax purposes and for AMT purposes.

• Casualty losses.

• Gambling losses.

• Charitable contributions.

• Medical expenses in excess of 10% of AGI for AMT purposes. For regular income tax purposes, medical expenses in excess of 10% if the taxpayer is not at least age 65 and in excess of 7.5% of AGI if the taxpayer is at least age 65.

• Estate tax on income in respect of a decedent.

• Qualified interest.

1511. What effect do deductible gambling losses for regular income tax purposes have in calculating AMTI?

Correct Answer: The amount allowable in calculating the gambling loss deduction under the regular income tax is the same as that for AMTI purposes. So no adjustment is required in converting regular taxable income to AMTI.

1512. Why is there no AMT adjustment for charitable contributions?

Correct Answer: The charitable contribution deduction for AMT purposes is the same as the charitable contribution deduction for regular income tax purposes.

1513. Is it possible that no AMT adjustment is necessary for medical expenses in calculating AMTI for a taxpayer who is at least age 65 even though the floor limitation is different (7.5% of AGI for regular income tax compared to 10% for AMT purposes)?

Correct Answer: Yes, this occurs when medical expenses are not large enough to exceed either floor. Since the medical expense deduction for both purposes is $0, there is no AMT adjustment.

1514. When qualified residence interest exceeds qualified housing interest, the positive adjustment required in calculating AMT is a timing adjustment. That is, in the future, there will be an offsetting negative adjustment. Comment on the validity of this statement.

Correct Answer: This AMT adjustment is not a timing adjustment. So it will not reverse.

1515. Andrea, who is single, has a personal exemption deduction in calculating her 2013 taxable income. She has no dependency deductions. What is the amount of the AMT adjustment in calculating AMTI?

Correct Answer: In calculating Andrea’s 2013 taxable income, she deducts $3,900 for her personal exemption deduction. So in calculating her AMTI, she has a positive adjustment of $3,900.

1516. How can interest on a private activity bond issued in 2008 result in both an AMT adjustment that decreases AMTI and an AMT preference that increases AMTI?

Correct Answer: The interest on a private activity bond issued in 2008 is not included in taxable income, but the interest (net of related expenses) is a tax preference that increases AMTI.

For regular income tax purposes, the interest on a private activity bond is not included in net investment income in calculating the deduction for investment interest. However, for AMT purposes such interest is included in net investment income in calculating the deduction for investment interest. To the extent that such private activity bond interest increases the amount of the investment interest deduction for AMT purposes, therefore, it is a negative AMT adjustment. If the bond had been issued in 2009 or 2010, the related private activity bond interest would not be a tax preference.

1517. If a taxpayer deducts the standard deduction in calculating regular taxable income, what effect does this have in calculating AMTI?

Correct Answer: In converting regular taxable income to AMTI, the standard deduction is a positive adjustment.

1518. Identify an AMT adjustment that applies for the individual taxpayer that does not apply for the corporate taxpayer and identify an AMT adjustment that applies for the corporate taxpayer that does not apply for the individual taxpayer.

Correct Answer: Among the AMT adjustments that apply for the individual taxpayer that do not apply for the corporate taxpayer are the following:

• Adjustment for ISOs.

• Adjustments for certain itemized deductions.

An adjustment that applies only to corporate taxpayers is the ACE adjustment.

1519. Are the AMT rates for the individual taxpayer the same as those for a corporate taxpayer?

Correct Answer: No, the rates are different. The corporate AMT rate is 20%. The individual taxpayer has two rates: 26% for the first $175,000 of AMT base and 28% for the AMT basis that is greater than $175,000.

1520. Under what circumstances are corporations exempt from the AMT?

Correct Answer: Small corporations are exempt from the AMT. A corporation is automatically classified as a small corporation in the first tax year of existence. For subsequent tax years, a corporation is a small corporation if it had average annual gross receipts of not more than $5 million for the three-year period beginning after December 1993 and not more than $7.5 million for intervening three-year periods (i.e., three- year running average up to three-year period preceding the current tax year).

1521. For the ACE adjustment, discuss the relationship between ACE and unadjusted AMTI.

Correct Answer: A positive AMT adjustment occurs if the ACE amount exceeds unadjusted AMTI. A negative AMT adjustment occurs if unadjusted AMTI exceeds the ACE amount. A negative adjustment is limited to the aggregate of the positive adjustments under ACE for prior years, reduced by the previously claimed negative adjustments.

1522. The tax benefit received from a tax credit is never affected by the tax rate of the taxpayer.

*a. True b. False

1523. The tax benefits resulting from tax credits and tax deductions are affected by the tax rate bracket of the taxpayer.

a. True *b. False

1524. Nonrefundable credits are those that reduce the taxpayer’s tax liability but are not paid when the amount of the credit (or credits) exceeds the taxpayer’s tax liability.

*a. True b. False

1525. The credit for child and dependent care expenses is an example of a refundable credit.

a. True *b. False

1526. Any unused general business credit must be carried back 3 years and then forward for 20 years.

a. True *b. False

1527. A LIFO method is applied to general business credit carryovers, carrybacks, and utilization of credits earned during a particular year.

a. True *b. False

1528. The purpose of the tax credit for rehabilitation expenditures is to encourage the relocation of businesses from older, economically distressed areas (i.e., inner city) to newer locations.

a. True *b. False

1529. Qualified rehabilitation expenditures include the cost of acquiring the building, but not the cost of acquiring the land.

a. True *b. False

1530. The tax credit for rehabilitation expenditures for certified historic structures differs from that for qualifying structures that are not certified historic structures.

*a. True b. False

1531. Some (or all) of the tax credit for rehabilitation expenditures will have to be recaptured if the rehabilitated property is disposed of prematurely or if it ceases to be qualifying property.

*a. True b. False

1532. If a taxpayer is required to recapture any tax credit for rehabilitation expenditures, the recapture amount need not be added to the adjusted basis of the rehabilitation expenditures.

a. True *b. False

1533. The purpose of the work opportunity tax credit is to encourage employers to hire individuals from specified target groups traditionally subject to high rates of unemployment.

*a. True b. False

1534. Employers are encouraged by the work opportunity tax credit to hire individuals who have been long-term recipients of family assistance welfare benefits.

*a. True b. False

1535. The work opportunity tax credit is available only for wages paid to qualifying individuals during their first year of employment.

a. True *b. False

1536. An employer’s tax deduction for wages is affected by the work opportunity tax credit.

*a. True b. False

1537. The incremental research activities credit is 20% of the qualified research expenses that exceed the base amount.

*a. True b. False

1538. All taxpayers are eligible to take the basic research credit.

a. True *b. False

1539. Qualified research and experimentation expenditures are not only eligible for the 20% tax credit, but also can be expensed in the year incurred.

*a. True b. False

1540. The low-income housing credit is available to low-income tenants who reside in qualifying low-income housing.

a. True *b. False

1541. A taxpayer who qualifies for the low-income housing credit claims the credit over a 20-year period.

a. True *b. False

1542. The disabled access credit was enacted to encourage small businesses to make their businesses more accessible to disabled individuals.

*a. True b. False

1543. The disabled access credit is computed at the rate of 50% of all access expenditures incurred by the taxpayer during the year.

a. True *b. False

1544. If the cost of a building constructed and placed into service by an eligible small business in the current year includes the cost of a wheelchair ramp, the cost of the ramp qualifies for the disabled access credit.

a. True *b. False

1545. A small employer incurs $1,500 for consulting fees related to establishing a qualified retirement plan for its 75 employees. As a result, the employer may claim the credit for small employer pension plan startup costs for $750.

a. True *b. False

1546. BlueCo incurs $900,000 during the year to construct a facility that will be used exclusively for the care of its employees’ pre-school age children during normal working hours. The credit for employer- provided child care available to BlueCo this year is $225,000.

a. True *b. False

1547. Cardinal Company incurs $800,000 during the year to construct a facility that will be used exclusively for the care of its employees’ pre-school age children during normal working hours. Assuming Cardinal claims the credit for employer-provided child care this year, its basis in the newly constructed facility is $640,000.

a. True *b. False

1548. The earned income credit, a form of a negative income tax, is a refundable credit.

*a. True b. False

1549. The earned income credit is available only if the taxpayer has at least one qualifying child in the household.

a. True *b. False

1550. A taxpayer’s earned income credit is dependent on the number of his or her qualifying children.

*a. True b. False

1551. An eligible taxpayer may elect to receive advance payments of the earned income credit from his or her employer.

*a. True b. False

1552. Unless a taxpayer is disabled, the tax credit for the elderly or disabled is available only if the taxpayer is at least 59 1/2 years old.

a. True *b. False

1553. A taxpayer who meets the age requirement and receives no Social Security benefits will be entitled to the full tax credit for the elderly.

a. True *b. False

1554. In computing the foreign tax credit, the greater of the foreign income taxes paid or the overall limitation is allowed.

a. True *b. False

1555. Unused foreign tax credits can be carried back three years and forward fifteen years.

a. True *b. False

1556. All foreign taxes qualify for the foreign tax credit.

a. True *b. False

1557. Because current U.S. corporate income tax rates are higher than many foreign corporate income tax rates, the overall limitation does not yield a lower foreign tax credit than the amount of foreign taxes actually paid.

*a. True b. False

1558. If a taxpayer chooses to claim a foreign tax credit, part of the foreign income taxes paid can also be claimed as a deduction.

a. True *b. False

1559. An individual generally may claim a credit for adoption expenses in the year in which the expenses are paid.

a. True *b. False

1560. The child tax credit is based on the number of the taxpayer’s qualifying children under age 17.

*a. True b. False

1561. The maximum child tax credit under current law is $1,500 per qualifying child.

a. True *b. False

1562. The maximum credit for child and dependent care expenses is $2,100 if only one spouse is employed and the other spouse is a full- time student.

*a. True b. False

1563. Expenses that are reimbursed by a taxpayer’s employer under a dependent care assistance program can also qualify for the credit for child and dependent care expenses.

a. True *b. False

1564. For purposes of computing the credit for child and dependent care expenses, the qualifying employment-related expenses are limited to an individual’s actual or deemed earned income.

*a. True b. False

1565. A taxpayer may qualify for the credit for child and dependent care expenses if the taxpayer’s dependent is under age 17.

a. True *b. False

1566. Child care payments to a relative are not eligible for the credit for child and dependent care expenses if the relative is a child (under age 19) of the taxpayer.

*a. True b. False

1567. Child and dependent care expenses include amounts paid for general household services.

*a. True b. False

1568. The education tax credits (i.e., the American Opportunity credit and the lifetime learning credit) are available to help defray the cost of higher education regardless of the income level of the taxpayer.

a. True *b. False

1569. Both education tax credits are available for qualified tuition expenses, and in certain instances, also may be available for room and board.

a. True *b. False

1570. The American Opportunity credit is available per eligible student, while the lifetime learning credit is calculated per taxpayer.

*a. True b. False

1571. Qualifying tuition expenses paid from the proceeds of a tax- exempt scholarship do not give rise to an education tax credit.

*a. True b. False

1572. Even though an item of income is not subject to income tax withholding by a taxpayer’s employer, it may be subject to the income tax.

*a. True b. False

1573. For the current year, the base amount for the Social Security portion (old age, survivors, and disability insurance) is different from that for the Medicare portion of FICA.

*a. True b. False

1574. If an employee holds two jobs during the year, an overwithholding of FICA tax will result.

a. True *b. False

1575. In the event that overwithholding of FICA tax occurs because the taxpayer has more than one employer, the excess amount should be claimed as a credit on the Federal income tax return of the employee.

*a. True b. False

1576. John owns and operates a real estate agency as a sole proprietor. On a full-time basis, he employs his 17-year old daughter as a receptionist and his 22-year old son as a bookkeeper. Both children are subject to FICA withholding.

a. True *b. False

1577. Sam and Betty are married, have four dependent children, and both are employed. For withholding purposes, they may allocate their total allowances between themselves as they see fit.

*a. True b. False

1578. Certain situations exist where the wage-bracket table method of determining income tax withholdings cannot be used.

*a. True b. False

1579. On October 2, 2013, Ross quits his job with Blue Corporation. If requested by Ross, Blue Corporation must furnish a Form W-2 to Ross within 45 days after the date of the request or the final wage payment, whichever is later.

a. True *b. False

1580. Juan refuses to give the bank where he maintains a savings account his Social Security number. Juan is subject to backup withholding for the interest earned on the savings account.

*a. True b. False

1581. Only self-employed individuals are required to make estimated tax payments.

a. True *b. False

1582. The calculation of FICA and the self-employment tax both involve two components: the Social Security portion and the Medicare portion, each portion of which is imposed on the same base amounts.

a. True *b. False

1583. An expatriate who works in a country with an income tax rate higher than the U.S. rate probably will find the foreign earned income exclusion preferable to the foreign tax credit.

a. True *b. False

1584. An employee with outside income may be able to avoid the penalty for underpayment of estimated tax by having his employer increase income tax withholdings.

*a. True b. False

1585. Certain high-income individuals are subject to three additional Medicare taxes beginning in 2013—on wages, unearned income, and tax credits claimed.

a. True *b. False

1586. The additional Medicare taxes assessed on high-income individuals carry differing tax rates depending on the tax base.

*a. True b. False

1587. Roger is considering making a $6,000 investment in a venture that its promoter promises will generate immediate tax benefits for him. Roger, who does not anticipate itemizing his deductions, is in the 30% marginal income tax bracket. If the investment is of a type that produces a tax credit of 40% of the amount of the expenditure, by how much will Roger’s tax liability decline because of the investment?

a. $0. b. $1,800. c. $2,200. *d. $2,400. e. None of the above.

1588. Ahmad is considering making a $10,000 investment in a venture which its promoter promises will generate immediate tax benefits for him. Ahmad, who normally itemizes his deductions, is in the 28% marginal tax bracket. If the investment is of a type where the taxpayer may claim either a tax credit of 25% of the amount of the expenditure or an itemized deduction for the amount of the investment, what treatment normally would be most beneficial to Ahmad and by how much will Ahmad’s tax liability decline because of the investment?

a. $0, take neither the itemized deduction nor the tax credit. b. $2,500, take the tax credit. *c. $2,800, take the itemized deduction. d. Both options produce the same benefit. e. None of the above.

1589. Refundable tax credits include the:

a. Foreign tax credit. b. Tax credit for rehabilitation expenses. c. Credit for certain retirement plan contributions. *d. Earned income credit. e. None of the above.

1590. The components of the general business credit include all of the following except:

a. Credit for employer-provided child care. b. Disabled access credit. c. Research activities credit. d. Tax credit for rehabilitation expenditures. *e. All of the above are components of the general business credit.

1591. Which of the following best describes the treatment applicable to unused business credits?

a. Unused amounts are carried forward indefinitely. *b. Unused amounts are first carried back one year and then forward for 20 years. c. Unused amounts are first carried back one year and then forward for 10 years. d. Unused amounts are first carried back three years and then carried forward for 15 years. e. None of the above.

1592. Molly has generated general business credits over the years that have not been utilized. The amounts generated and not utilized follow:

2009 $2,5 00 2010 7,50 0 2011 5,00 0 2012 4,00 0

In the current year, 2013, her business generates an additional $15,000 general business credit. In 2013, based on her tax liability before credits, she can utilize a general business credit of up to $20,000. After utilizing the carryforwards and the current year credits, how much of the general business credit generated in 2013 is available for future years?

a. $0. b. $1,000. *c. $14,000. d. $15,000. e. None of the above.

1593. Which of the following correctly describes the tax credit for rehabilitation expenditures?

a. The cost of enlarging any existing business building is a qualifying expenditure. b. The cost of facilities related to the building (e.g., a parking lot) is a qualifying expenditure. c. No recapture provisions apply. *d. No credit is allowed for the rehabilitation of personal use property. e. None of the above.

1594. Several years ago, Sarah purchased a structure for $150,000 that was originally placed in service in 1929. In the current year, she incurred qualifying rehabilitation expenditures of $200,000. The amount of the tax credit for rehabilitation expenditures, and the amount by which the building’s basis for cost recovery would increase as a result of the rehabilitation expenditures are the following amounts:

*a. $20,000 credit, $180,000 basis. b. $20,000 credit, $200,000 basis. c. $20,000 credit, $350,000 basis. d. $40,000 credit, $160,000 basis. e. None of the above.

1595. Several years ago, Tom purchased a structure for $300,000 that was originally placed in service in 1929. Three and one-half years ago he incurred qualifying rehabilitation expenditures of $600,000. In the current year, Tom sold the property in a taxable transaction. Calculate the amount of the recapture of the tax credit for rehabilitation expenditures.

a. $0. *b. $24,000. c. $36,000. d. $48,000. e. None of the above.

1596. Cardinal Corporation hires two persons certified to be eligible employees for the work opportunity tax credit under the general rules (e.g., food stamp recipients), each of whom is paid $9,000 during the year. As a result of this event, Cardinal Corporation may claim a work opportunity credit of:

a. $1,440. b. $2,880. *c. $4,800. d. $7,200. e. None of the above.

1597. Black Company paid wages of $180,000, of which $40,000 was qualified wages for the work opportunity tax credit under the general rules. Black Company’s deduction for wages for the year is:

a. $140,000. *b. $164,000. c. $166,000. d. $180,000. e. None of the above.

1598. In March 2013, Gray Corporation hired two individuals, both of whom were certified as long-term recipients of family assistance benefits. Each employee was paid $11,000 during 2013. Only one of the individuals continued to work for Gray Corporation in 2014, earning $9,000 during the year. No additional workers were hired in 2014. Gray Corporation’s work opportunity tax credit amounts for 2013 and 2014 are:

a. $4,000 in 2013, $4,000 in 2014. *b. $8,000 in 2013, $4,500 in 2014. c. $8,000 in 2013, $5,000 in 2014. d. $8,000 in 2013, $9,000 in 2014. e. None of the above.

1599. During the year, Green, Inc., incurs the following research expenditures:

In-house wages, supplies, computer $60,000 time Paid to Blue Foundation for 30,000 research

Green’s qualifying research expenditures for the year are:

a. $60,000. b. $75,000. *c. $79,500. d. $90,000. e. None of the above.

1600. Which, if any, of the following correctly describes the research activities credit?

a. The research activities credit is the greater of the incremental research credit, the basic research credit, or the energy research credit. b. If the research activities credit is claimed, no deduction is allowed for research and experimentation expenditures. *c. The credit is not available for research conducted outside the United States. d. All corporations qualify for the basic research credit. e. None of the above.

1601. Green Company, in the renovation of its building, incurs $9,000 of expenditures that qualify for the disabled access credit. The disabled access credit is:

a. $8,750. b. $4,500. *c. $4,375. d. $4,250. e. None of the above.

1602. Amber is in the process this year of renovating the office building (originally placed in service in 1976) used by her business. Because of current Federal Regulations that require the structure to be accessible to handicapped individuals, she incurs an additional $11,000 for various features, such as ramps and widened doorways, to make her office building more accessible. The $11,000 incurred will produce a disabled access credit of what amount?

a. $0. *b. $5,000. c. $5,125. d. $5,500. e. None of the above.

1603. Which, if any, of the following correctly describes the earned income credit?

a. Would be available regardless of the amount of the taxpayer’s adjusted gross income. b. Not available to a surviving spouse. c. A taxpayer must have a qualifying child to take advantage of the credit. *d. Is a refundable credit. e. None of the above.

1604. Rex and Dena are married and have two children, Michelle (age 7) and Nancy (age 5). During 2013, Rex earned a salary of $24,500, received interest income of $300, and filed a joint income tax return. Dena had $0 gross income. Their earned income credit for the year is:

a. $0. b. $406. *c. $4,966. d. $5,372. e. None of the above.

1605. Cheryl is single, has one child (age 6), and files as head of household during 2013. Her salary for the year is $19,500. She qualifies for an earned income credit of the following amount:

a. $0. b. $315. *c. $2,935. d. $3,250. e. None of the above.

1606. During 2013, Barry (who is single and has no children) earned a salary of $13,000. He is age 30. His earned income credit for the year is:

a. $0. *b. $102. c. $385. d. $487. e. None of the above.

1607. George and Jill are husband and wife, ages 67 and 65 respectively. During the year, they receive Social Security benefits of $4,000 and have adjusted gross income of $11,000. Assuming they file a joint return, their tax credit for the elderly, before considering any possible limitation due to their tax liability, is:

a. $1,125. b. $750. *c. $450. d. $375. e. None of the above.

1608. During the year, Green Corporation (a U.S. corporation) has U.S.- source income of $750,000 and foreign income of $500,000. The foreign- source income generates foreign income taxes of $240,000. The U.S. income tax before the foreign tax credit is $425,000. Green Corporation’s foreign tax credit is:

*a. $170,000. b. $240,000. c. $425,000. d. $500,000. e. None of the above.

1609. During the year, Purple Corporation (a U.S. Corporation) has U.S.-source income of $1,800,000 and foreign income of $600,000. The foreign-source income generates foreign income taxes of $150,000. The U.S. income tax before the foreign tax credit is $816,000. Purple Corporation’s foreign tax credit is:

a. $112,500. *b. $150,000. c. $204,000. d. $816,000. e. None of the above.

1610. In 2012, Juan and Juanita incur $9,800 in legal and adoption fees directly related to the adoption of an infant son born in a nearby state. Over the next year, they incur another $4,500 of adoption expenses. The adoption becomes final in 2013. Which of the following choices properly reflects the amounts and years in which the adoption expenses credit is available. 2012 2013

a. $9,800 $ 4,500 *b. None $12,970 c. None $14,300 d. $9,800 $ 3,170 e. None of the above.

1611. Which of the following statements regarding the adoption expenses credit is not true?

a. The adoption expenses credit is a nonrefundable credit. b. The adoption expenses credit starts to be phased out in 2013 beginning when a taxpayer’s modified AGI exceeds $194,580. c. No adoption expenses credit is a available in 2013 if a taxpayer’s modified AGI exceeds $234,580. *d. The adoption expenses credit is limited to no more than $12,500 per eligible child in 2013. e. All of the above statements are true.

1612. George and Martha are married and file a joint tax return claiming their two children, ages 10 and 8 as dependents. Assuming their AGI is $119,650, George and Martha’s child tax credit is:

a. $0. b. $1,000. *c. $1,500. d. $2,000. e. None of the above.

1613. Harry and Wilma are married and file a joint income tax return. On their tax return, they report $44,000 of adjusted gross income ($20,000 salary earned by Harry and $24,000 salary earned by Wilma) and claim two exemptions for their dependent children. During the year, they pay the following amounts to care for their 4-year old son and 6-year old daughter while they work.

ABC Day Care Center $3,200 Blue Ridge Housekeeping 2,000 Services Mrs. Mason (Harry’s 1,000 mother)

Harry and Wilma may claim a credit for child and dependent care expenses of:

a. $840. b. $1,040. *c. $1,200. d. $1,240. e. None of the above.

1614. Kevin and Sue have two children, ages 8 and 14. They spend $6,200 per year on eligible employment related expenses for the care of their children after school. Kevin earned a salary of $20,000 and Sue earned a salary of $18,000. What is the amount of the credit for child and dependent care expenses?

*a. $690. b. $713. c. $1,380. d. $1,426. e. None of the above.

1615. Which of the following statements concerning the credit for child and dependent care expenses is not correct?

a. A taxpayer is not allowed both an exclusion from income and the credit for child and dependent care expenses on the same amount. b. A taxpayer is not allowed both a deduction as a medical expense and the credit for child and dependent care expenses on the same amount. c. If a taxpayer’s adjusted gross income exceeds $43,000, the rate for the credit for child and dependent care expenses is 20%. *d. If a taxpayer’s adjusted gross income exceeds $15,000 but is not over $17,000, the rate for the credit for child and dependent care expenses is 35%. e. All of the above are correct.

1616. Jermaine and Kesha are married, file a joint tax return, have AGI of $82,500, and have two children. Devona is beginning her freshman year at State University during Fall 2013, and Arethia is beginning her senior year at Northeast University during Fall 2013 after having completed her junior year during the spring of that year. Both Devona and Arethia are claimed as dependents on their parents’ tax return. Devona’s qualifying tuition expenses and fees total $4,000 for the fall semester, while Arethia’s qualifying tuition expenses and fees total $6,200 for each semester during 2013. Full payment is made for the tuition and related expenses for both children during each semester. The American Opportunity credit available to Jermaine and Kesha for 2013 is:

a. $2,500. b. $3,000. *c. $5,000. d. $6,000. e. None of the above.

1617. Bob and Sally are married, file a joint tax return, have AGI of $108,000, and have two children. Del is beginning her freshman year at State College during Fall 2013, and Owen is beginning his senior year at Southwest University during Fall 2013. Owen completed his junior year during the Spring semester of 2011 (i.e., he took a “leave of absence” during the 2012-2013 school year). Both Del and Owen are claimed as dependents on their parents’ tax return. Del’s qualifying tuition expenses and fees total $5,000 for the Fall semester, while Owen’s qualifying tuition expenses were $6,100 for the Fall 2013 semester. Del’s room and board costs were $3,200 for the Fall semester. Owen did not incur room and board costs since he lived with his aunt and uncle during the year. Full payment is made for the tuition and related expenses for both children at the beginning of each semester. In addition to the children’s college expenses, Bob also spent $3,000 on professional education seminars during the year in order to maintain his license as a practicing dentist. Bob attended the seminars during July and August 2013. Compute the available education tax credits for Bob and Sally for 2013.

a. $3,100. b. $5,000. *c. $5,480. d. $5,600. e. None of the above.

1618. Which of the following statements is true regarding the education tax credits?

a. The lifetime learning credit is available for qualifying tuition and related expenses incurred by students pursuing only graduate degrees. b. The American Opportunity credit permits a maximum credit of 20% of qualified expenses up to $10,000 per year. c. The American Opportunity credit is calculated per taxpayer, while the lifetime learning credit is available per eligible student. d. Continuing education expenses do not qualify for either education credit. *e. None of the above statements is true.

1619. Realizing that providing for a comfortable retirement is up to them, Jim and Julie commit to making regular contributions to their IRAs, beginning this year. Consequently, they each make a $2,000 contribution to their traditional IRA. If their AGI is $35,000 on their joint return, what is the amount of their credit for certain retirement plan contributions?

*a. $2,000. b. $1,000. c. $400. d. $200. e. None of the above.

1620. Which of the following issues does not need resolution in an employer’s effort to comply with employment tax payment requirements?

a. Ascertaining which employees and wages are covered by employment taxes and are subject to withholding for income taxes. b. Arriving at the amount to be paid and/or withheld. c. Reporting and paying employment taxes and income taxes withheld to the IRS on a timely basis through the use of proper forms. *d. Each of the above issues needs to be resolved. e. None of the above is relevant to the employer.

1621. An employer calculates the amount of income tax withheld from salary or wages based on the information an employee provides on the following form:

a. Form W-2. b. Form W-3. *c. Form W-4. d. Form 941. e. None of the above.

1622. In terms of the withholding procedures, which statement does not reflect current rules?

a. Penalties can be imposed for filing false information with respect to wage withholding. b. An employer need not verify the number of exemptions claimed by an employee on Form W-4 (Employee’s Withholding Allowance Certificate). c. An employee may claim fewer than the number of withholding allowances allowed, but not more. *d. In preparing the income tax return for the year, the employee is bound by the number of exemptions claimed for withholding purposes. e. None of the above.

1623. Identify the statement below that is false.

*a. If an employer is not required to withhold income taxes from an employee’s wages, the wages are not taxable to the employee. b. In certain situations, income tax withholding by an employer is voluntary. c. For 2013, an employer must deposit with the government an amount of FICA tax that is twice the amount withheld from the employee’s salary (i.e., the employee’s and employer’s shares). d. If an excess amount of FICA has been withheld for an employee because the employee has multiple jobs, the employee may claim a credit for the excess amount withheld on his or her income tax return. e. None of the above.

1624. In describing FICA taxes, which (if any) of the following statements is incorrect?

a. The base amounts for 2014 probably will increase from the 2013 amounts. *b. The base amounts for the Social Security and Medicare portions are the same. c. If both spouses work, excess FICA taxes need not result. d. Excess FICA taxes can be claimed as an income tax credit. e. None of the above.

1625. Which of the following correctly reflects current rules regarding estimated tax payments for individuals?

a. Employees are not subject to the estimated tax payment provisions. b. Any penalty imposed for underpayment is deductible for income tax purposes. c. Married taxpayers may not make joint estimated tax payments unless they file a joint income tax return. *d. No quarterly payments are required if the taxpayer’s estimated tax is under $1,000. e. None of the above.

1626. During 2013, Eleanor earns $120,000 in wages as an employee of an accounting firm. She also earns $13,000 in gross income from an outside consulting service she operates. Deductible expenses paid in connection with the consulting service amount to $3,000. Eleanor also has a recognized long-term capital gain of $1,000 from the sale of a stock investment. She must pay a self-employment tax on:

a. $0. *b. $10,000. c. $13,000. d. $14,000. e. None of the above.

1627. Pat generated self-employment income in 2013 of $76,000. The self-employment tax is:

a. $0. *b. $5,369.23. c. $10,738.46. d. $11,628.00. e. None of the above.

1628. The ceiling amounts and percentages for 2013 for the two portions of the self-employment tax are:

Social Security portion Medicare portion

a. $110,100; 12.4% Unlimited; 2.9% b. $110,100; 15.3% Unlimited; 2.9% *c. $113,700; 12.4% Unlimited; 2.9% d. $113,700; 2.9% Unlimited; 13.3% e. None of the above.

1629. Steve has a tentative general business credit of $110,000 for the current year. His net regular tax liability before the general business credit is $125,000, and his tentative minimum tax is $100,000. Compute Steve’s allowable general business credit for the year.

Correct Answer: Steve’s allowable general business credit for the year is limited to $25,000, determined as follows:

Net income $125,000* tax Less: The great er of:

• (100,000) $1 00,00 0 (tent ative minim um tax)

• $2 5,000 [25% × ($125 ,000 – $25,0 00)]

Amount of $ 25,000 general business credit allowed

*Net income tax = $125,000 (regular tax liability) + $0 [alternative minimum tax ($100,000 tentative minimum tax – $125,000 regular tax liability)] – $0 (nonrefundable credits).

1630. In January 2013, Tammy acquired an office building in downtown Syracuse, New York for $400,000. The building was originally constructed in 1932. Of the $400,000 cost, $40,000 was allocated to the land. Tammy immediately placed the building into service, but quickly realized that substantial renovation would be required to keep and attract new tenants. The renovations, costing $600,000, were of the type that qualifies for the rehabilitation credit. The improvements were completed in October 2013. a. Compute Tammy’s rehabilitation tax credit for the year of acquisition. b. Determine the cost recovery deduction for 2013. c. What is the basis in the property at the end of its first year of use by Tammy?

Correct Answer: a. Tammy’s adjusted basis in the building before the rehabilitation expenditures and current year cost recovery is $360,000 ($400,000 – $40,000) minus the cost recovery for the period January through September. Because the rehabilitation expenditures of $600,000 exceed the greater of (1) the adjusted basis of the building before the rehabilitation ($360,000 minus the cost recovery for the period January through September), or (2) $5,000, Tammy is allowed a rehabilitation tax credit of $60,000 (10% × $600,000). b. The improvements are treated as separate property items for purposes of computing cost recovery. The recovery period for these improvements begins in October 2013 when the improvements are placed in service by Tammy. (Rev. Rul. 87-57, 1987-2 C.B. 687) The cost recovery period for the underlying structure begins in January 2013 when it was placed in service by Tammy. The straight-line method over 39 years under MACRS of § 168(c) must be used. Using Table 8.6 in Chapter 8 for straight-line depreciation for 39-year nonresidential real property, the appropriate cost recovery percentage for the building is 2.461% and the percentage for the improvements is 0.535%.

Cost recovery for the property for 2013 is as follows:

Cost recovery of building ($360,000 × $ 8,860 2.461%) Plus: Cost recovery of improvements Cost of improvements $600,000 Less: Credit (10%) (60,000) Cost recovery basis $540,000 Cost recovery of 2,88 improvements ($540,000 × 0.535%) 9 Total cost recovery $11,749 for 2013 c. Tammy’s basis in the property at the end of 2013 is determined as follows:

Land $ 40,000 Building: Cost [$400,000 – $360,000 $40,000 (land)] Less: Cost (8,860) recovery Adjusted basis of 351,140 building Improvements: Cost [$600,000 – $540,000 $60,000 (credit)] Less: Cost (2,889) recovery Adjusted basis of 537,111 improvements Total adjusted basis of property $928,251

1631. In May 2009, Cindy incurred qualifying rehabilitation expenditures of $500,000 on a certified historic structure and properly claimed the tax credit for rehabilitation expenditures. In March 2013, she sold the building at a loss. Calculate the rehabilitation expenditures credit recapture that she must report in 2013.

Correct Answer: Amount of original credit ($500,000 × $100,000 20%)

Amount of rehabilitation expenditures $ 40 credit recapture ($100,000 × 40%) ,000

Because the property was held by Cindy for more than three years but less than four years, the recapture percentage is 40%.

1632. In May 2013, Blue Corporation hired Camilla, Jolene, and Tyrone, all of whom are certified as long-term family assistance recipients. Each employee is paid $12,000 during 2013. Camilla and Tyrone continued to work for Blue Corporation in 2014, earning $14,000 each. Blue hired no additional employees during 2014. a. Compute Blue Corporation’s work opportunity tax credits for 2013 and 2014. b. Assume Blue Corporation pays total wages of $500,000 to its employees during 2013 and $560,000 during 2014. How much may Blue Corporation claim as a wage deduction for 2013 and 2014 if the work opportunity tax credit is claimed in both years?

Correct Answer: a. The work opportunity tax credit for 2013 is calculated as follows: 3 qualified employees × $10,000 limit $12,00 on wages for each employee × 40% 0

The work opportunity tax credit for 2014 is calculated as follows: 2 qualified employees in second year $10,00 of employment × $10,000 limit on 0 wages per employee × 50%

b. The wage deduction for 2013 is $488,000 [$500,000 (total wages) – $12,000 (credit)].

The wage deduction for 2014 is $550,000 [$560,000 (total wages) – $10,000 (credit)].

1633. Rick spends $750,000 to build a qualified low-income housing project, which is placed in service on January 1, 2013. He financed the project using his personal funds. What is the amount of the low-income housing credit that Rick may claim in 2013 (assuming a rate of 7.36%)? What is the total amount of the credit that Rick may claim as a result of the $750,000 expenditure?

Correct Answer: Rick may claim a credit of $55,200 in 2013 ($750,000 × 7.36%). In addition, he may claim a credit of $55,800 per year for each of the next nine years, beginning in 2014, for a total credit of $552,000.

1634. Golden Corporation is an eligible small business for purposes of the disabled access credit. During the year, Golden makes the following expenditures on a structure originally placed in service in 1988.

Removal of architectural barriers $ 8,500 Acquired equipment for disabled 6,250 persons $14,750

In addition, $8,000 was expended by Golden on a building originally placed in service in the current year to ensure easy accessibility by disabled individuals. Calculate the amount of the disabled access credit available to Golden Corporation.

Correct Answer: Eligible access expenditures $10,250 ($8,500 + $6,250); limited to $10,250 Less: Threshold amount (25 0) Disabled access credit base $10,000 Tax credit rate × 50% Disabled access credit $ 5,000

The expenditures of $8,000 incurred on the building originally placed in service in the current year do not qualify for the credit. The outlay is not considered an eligible expenditure because it is incurred on a structure placed in service after the enactment of the disabled access credit provision (November 5, 1990).

1635. Julia is 30 years old, unmarried with a 9 year-old daughter, and has earnings during 2013 of $19,700. Does she qualify for the earned income credit? If so, calculate the amount of credit that is available to her.

Correct Answer: Maximum credit available $3,250 for 2013 ($9,560 × 34%) Less: Credit phaseout Earned income $19,700 Base for phaseout (17,530) Excess $ 2,170 Phaseout rate × 15.98% (347) Available earned $2,903 income credit

1636. Jack and Jill are married, have three children, and have earnings during 2013 of $28,500. Do they qualify for the earned income credit? If so, calculate the amount of credit that is available to them.

Correct Answer: Earned income of $28,500 (but $13,430 subject to ceiling of $13,430) Rate × 45.00% Tentative credit $ 6,044 Less: Credit phase-out Earned income $28,500 Base for phase-out (22,870) Excess $5,630 Phase-out rate × 21.06% (1,186) Available earned $ 4,858 income credit

1637. Henry, age 68, and Wilma, age 69, are married retirees who received the following income and retirement benefits during the year.

Fully taxable pension from $8,800 Henry’s former employer Taxable interest 4,500 Social Security benefits 2,400

Assume Henry and Wilma file a joint return, have no deductions for AGI, and do not itemize their deductions. Are they eligible for the tax credit for the elderly? If so, calculate the amount of the credit, assuming the credit is not limited by their income tax liability.

Correct Answer: Base amount (married filing $7,500 jointly; both 65 or older) Less: Social Security benefits $2,400 1/2 × (AGI over $10,000) [1/2 × ($8,800 + $4,500 – 1,650 (4,050) $10,000)] Balance subject to credit $3,450 Rate × 15% Tax credit $ 518

1638. Summer Corporation’s business is international in scope and is subject to income taxes in several countries. Summer’s earnings and income taxes paid in the relevant foreign countries are:

Country Income Taxes A $1,000,00 $500,000 0 B 300,0 30,00 00 0 C 400,0 120,000 00 Total $1,700,00 $650,000 0

If Summer Corporation’s worldwide income subject to taxation in the United States is $2,400,000 and the U.S. income tax due prior to the foreign tax credit is $816,000, compute the allowable foreign tax credit. If, instead, the total foreign income taxes paid were $550,000, compute the allowable foreign tax credit.

Correct Answer: Overall limitation:

Foreign-source U.S. tax taxable income Overall × before = Worldwide taxable limitation FTC income

$1,700,000 × $816,000 = $2,400,000 $578,000

Therefore, because the overall limitation is less than the foreign taxes actually paid ($650,000), the foreign tax credit is $578,000.

However, if $550,000 of foreign taxes were paid, the foreign tax credit would be limited to the amount of foreign taxes actually paid, or $550,000.

1639. Dabney and Nancy are married, both gainfully employed, and have two children who are 3 and 6 years old. Dabney’s salary is $35,000 while Nancy’s salary is $40,000. During the year, they spend $7,000 for child care expenses that are required so both of them can work outside of the home. Calculate the credit for child and dependent care expenses.

Correct Answer: For two or more qualifying children, the maximum expense allowed for purposes of the credit for child and dependent care expenses is $6,000. Since their combined AGI is more than $43,000, the applicable rate for the credit is 20%. Thus, the credit allowed is $1,200 (20% × $6,000).

1640. Bradley has two college-age children, Clint, a freshman at State University, and Abigail, a junior at Northwest University. Both Clint and Abigail are full-time students. Clint’s expenses during the 2013 fall semester are as follows: $2,400 tuition, $250 books and course materials, and $1,600 room and board. Abigail’s expenses for the 2013 calendar year are as follows: $10,200 tuition, $1,200 books and course materials, and $3,600 room and board. Tuition and the applicable room and board costs are paid at the beginning of each semester. Bradley is married, files a joint tax return, claims both children as dependents, and has a combined AGI with his wife of $114,000 for 2013. Determine Bradley’s available education tax credit for 2013.

Correct Answer: In 2013, both Clint and Abigail qualify for the American Opportunity credit. Clint’s qualifying expenses are $2,650 ($2,400 tuition and $250 books and course materials); Abigail’s qualifying expenses are $11,400 ($10,200 tuition and $1,200 books and course materials).

Clint’s American Opportunity credit is $2,162.50 [100% of the first $2,000 of qualifying expenses plus 25% of the next $2,000 of qualifying expenses; $2,000 + ($650 × 25%)]. Abigail’s American Opportunity credit is $2,500 (100% of the first $2,000 of qualifying expenses plus 25% of the next $2,000 of qualifying expenses; $2,000 + ($2,000 × 25%)].

Although the American Opportunity credits are subject to a phaseout for higher income taxpayers, Bradley’s AGI of $114,000 is less than the phase-out starting point in 2013 ($160,000 for married taxpayers filing jointly).

So, the total education credit available for the year is $4,662.50 ($2,162.50 + $2,500).

1641. Phil and Audrey, husband and wife, are both employed by Laurel Corporation. Phil earns $115,000 in salary in 2013, and Audrey earns $70,000. How much FICA tax must they pay for 2013?

Correct Answer: Phil will pay $8,716.90 [(6.2% × $113,700) + (1.45% × $115,000)] and Audrey will pay $5,355.00 [(6.2% × $70,000) + (1.45% × $70,000)], for a total of $14,071.90 for the couple.

1642. Ken is married to a nonemployed spouse and has three children. He is employed by Skyview, Inc., and is paid a monthly salary of $5,150. Using these facts, determine the amount to be withheld by Skyview, Inc., for Federal income tax purposes under the wage-bracket tables and under the percentage method for 2013.

Correct Answer: Wage-bracket table method: Withholding on $5,150 monthly payroll period, six withholding allowances,* married taxpayer (from Table 13.6) $300.00

* The six withholding allowances include Ken, Ken’s spouse, 3 dependent children, and the special withholding allowance (which is available because Ken’s spouse is not employed).

Computation of Ken’s withholding using the percentage method (Tables 13.7 and 13.8):

Step 1 Multiply the amount of one monthly allowance ($325.00) by the employee’s total allowances (6) = $1,950.00.

Step 2 Reduce the employee’s wages ($5,150.00) by the product that was reached in Step 1 ($1,950.00) = $3,200.00.

Step 3 Compute the tax on the result that was derived in Step 2 (referring to Table 13.8). Withholding on $3,200.00 [($3,200.00 – $2,179.00) × 15%] + $148.70 = $301.85.

1643. Susan generated $55,000 of net earnings from the conduct of a tax preparation business that she operated during the tax-filing season. She also received wages of $63,500 from her full-time job. Compute the self-employment taxes due for 2013.

Correct Answer: Self-employment earnings subject to the Social Security portion of the self-employment tax:

Ceiling amount $113,700.00

Less: FICA wages (63,500. 00) Net ceiling $ 50,200. 00

Net self-employment income $ 50,792.50 ($55,000 × 92.35%)

Lesser of net ceiling or net $ 50,200.00 self-employment income

Social Security portion of the $ 6,224. tax ($50,200 × 12.4%) 80 Self-employment earnings subject to the Medicare portion of the self- employment tax: $50,792.50 × 1,4 2.9% 72.98 Total self-employment tax $ 7,697. 78

1644. Discuss the treatment of unused general business credits.

Correct Answer: Unused general business credits are initially carried back one year and applied to reduce the income tax liability during that year. Thus, the taxpayer may receive a tax refund as a result of the carryback. Any remaining unused credits are then carried forward 20 years. A FIFO method is applied to the carrybacks, carryovers, and utilization of credits earned during a particular year. This procedure minimizes the potential for loss of a general business credit benefit.

1645. Explain the purpose of the tax credit for rehabilitation expenditures and describe the general characteristics of its computation.

Correct Answer: The rehabilitation expenditures credit is intended to discourage businesses from moving from older, economically distressed areas (e.g., inner cities) to newer locations and to encourage the preservation of historic structures. To that end, taxpayers are allowed a tax credit for expenditures incurred to rehabilitate industrial and commercial buildings and certified historic structures. To calculate the credit, qualifying expenditures are multiplied by 10 percent for nonresidential buildings and residential rental property (other than historic structures) originally placed in service before 1936. For nonresidential and residential certified historic structures, a 20 percent rate is applied. To qualify for the credit, the buildings must be substantially rehabilitated.

1646. Explain the purpose of the disabled access credit and describe the general characteristics of its computation.

Correct Answer: The disabled access credit is designed to encourage small businesses to make their businesses more accessible to disabled individuals. The credit is only available to eligible small businesses and is based on eligible access expenditures made by such taxpayers. In general, the credit is calculated at the rate of 50% of the eligible expenditures that exceed $250 but do not exceed $10,250. The credit applies only to buildings placed in service before November 6, 1990.

1647. Describe the withholding requirements applicable to employers.

Correct Answer: Employers are required to withhold employment taxes (i.e., FICA, which commonly is referred to as Social Security tax) and appropriate amounts for income taxes from each employee’s compensation payments. Though not withheld from the employees’ compensation, the employer also is required to match the FICA portion withheld and fully absorb the cost of FUTA.

The FICA tax is comprised of two elements: the Social Security tax and the Medicare tax. In 2013, the withholding for the Social Security component is equal to the gross compensation paid (up to $113,700 in 2013) × 6.2%, while the entire amount of compensation paid is subject to the Medicare tax at the rate of 1.45%.

To determine the appropriate amount of income tax withheld:

• the employee must complete a Form W-4, which indicates the number of withholding allowances and any additional amounts to be withheld,

• determine the employee’s payroll period (e.g., bi-weekly, monthly), and

• compute the amount to be withheld by using either the wage-bracket tables or the percentage method.

1648. How does the FICA tax compare to the self-employment tax? How are these two taxes similar and how do they differ?

Correct Answer: These taxes, commonly referred to as “payroll taxes,” are levied to support the Social Security and Medicare benefits payable to taxpayers by the Federal government during their retirement years. The FICA tax is levied on salary and wages earned by an employee, while the self- employment tax is levied on earnings from self-employment (e.g., gross income from a trade or business less allowable trade or business deductions, the distributive share of any partnership income or loss derived from a trade or business activity, and the net income for rendering personal services as an independent contractor).

Both taxes have two components: the Social Security tax and the Medicare tax. For Social Security, withholdings from employees must continue until the maximum base amount is reached. In 2013, Social Security withholding (calculated at 6.2% times earned wages subject to Social Security) ceases once the employee has earned wages in the amount of $113,700. For the Medicare portion, the employer is required to withhold at the rate of 1.45% on all wages without limit. In addition, the employer is required to contribute an amount equal to the amount withheld from an employee’s earnings. For 2013, the self- employment tax is 12.4% of self-employment income up to $113,700 and 2.9% of the total amount of self-employment earnings.

1649. Realized gain or loss is measured by the difference between the amount realized from the sale or other disposition of property and the property’s adjusted basis at the date of disposition.

*a. True b. False

1650. In computing the amount realized when the fair market value of the property received cannot be determined, the fair market value of the property surrendered may be used.

*a. True b. False

1651. If Wal-Mart stock increases in value during the tax year by $6,000, the amount realized is a positive $6,000.

a. True *b. False

1652. If the buyer assumes the seller’s liability on the property acquired, the seller’s amount realized is decreased by the amount of the liability assumed.

a. True *b. False

1653. The fair market value of property received in a sale or other disposition is the price at which property will change hands between a willing seller and a willing buyer when neither is compelled to sell or buy.

*a. True b. False

1654. If a seller assumes the buyer’s liability on the property acquired, the buyer’s adjusted basis for the property is increased by the amount of the liability assumed.

a. True *b. False

1655. Expenditures made for ordinary repairs and maintenance of property are not added to the original basis in the determination of the property’s adjusted basis whereas capital expenditures are added to the original basis.

*a. True b. False

1656. Milton purchases land and a factory building for his business for $300,000 with $100,000 being allocated to the land. During the first year, Milton deducts cost recovery of $4,922. Milton’s adjusted basis for the building at the end of the first year is $195,078 ($200,000 – $4,922).

*a. True b. False

1657. In a casualty or theft, the basis of property involved is reduced by the amount of insurance proceeds received and by any resulting recognized loss.

*a. True b. False

1658. Monroe’s delivery truck is damaged in an accident. Monroe’s adjusted basis for the delivery truck prior to the accident is $20,000. If Monroe receives insurance proceeds of $21,000 and recognizes a casualty gain of $1,000, his adjusted basis for the delivery truck after the accident is $21,000.

a. True *b. False

1659. If insurance proceeds are received for property used in a trade or business, a casualty transaction can result in recognized gain, but cannot result in a recognized loss.

a. True *b. False

1660. If the amount of a corporate distribution is less than the amount of the corporate earnings and profits, the return of capital concept does not apply and the shareholders’ adjusted basis for the stock remains unchanged.

*a. True b. False

1661. Reggie owns all the stock of Amethyst, Inc. (adjusted basis of $100,000). If he receives a distribution from Amethyst of $90,000 and corporate earnings and profits are $15,000, Reggie has a capital gain of $5,000 and an adjusted basis for his Amethyst stock of $0.

a. True *b. False

1662. The amount of a corporate distribution qualifying for capital recovery treatment which exceeds the shareholder-recipient’s basis in the stock investment is treated as a capital gain.

*a. True b. False

1663. The adjusted basis for a taxable bond purchased at a premium is reduced if the amortization election is made. The amount of the amortized premium is treated as an interest deduction.

*a. True b. False

1664. Helen purchases a $10,000 corporate bond at a premium of $1,000 and elects to amortize the premium. On the later sale of the bond for $10,800, she has amortized $300 of the premium. Helen has a recognized gain of $800 ($10,800 amount realized – $10,000 adjusted basis).

a. True *b. False

1665. The amount received for a utility easement on land is included in the gross income of the taxpayer.

a. True *b. False

1666. A realized gain on the sale or exchange of a personal use asset is recognized, but a realized loss on the sale, exchange, or condemnation of a personal use asset is not recognized.

*a. True b. False

1667. A realized gain whose recognition is postponed results in the temporary recovery of more than the taxpayer’s cost or other basis.

*a. True b. False

1668. A realized loss whose recognition is postponed results in the temporary recovery of more than the taxpayer’s cost or other basis.

a. True *b. False

1669. Wade is a salesman for a real estate development company. Because he is the “salesperson of the year,” he is permitted to purchase a lot from the developer for $90,000. The fair market value of the lot is $150,000 and the developer’s adjusted basis is $100,000. Wade must recognize a gain of $10,000 ($100,000 developer’s adjusted basis – $90,000 cost to Wade), and his adjusted basis for the lot is $100,000 ($90,000 cost + $10,000 recognized gain).

a. True *b. False

1670. When a taxpayer has purchased several lots of stock on different dates at different purchase prices and cannot identify the lot of stock that is being sold, he should use either a weighted average approach or a LIFO approach.

a. True *b. False

1671. Lump-sum purchases of land and a building are allocated on the basis of the relative fair market values of the individual assets acquired.

*a. True b. False

1672. Purchased goodwill is assigned a basis equal to cost, which is calculated using the residual method associated with the purchase of a business.

*a. True b. False

1673. The holding period for nontaxable stock dividends that are the same type (i.e., common on common) includes the holding period of the original shares, but the holding period for nontaxable stock dividends that are not the same type (i.e., preferred on common) is new and begins on the date the dividend is received.

a. True *b. False

1674. For nontaxable stock rights where the fair market value of the rights is 15% or more of the fair market value of the stock, the taxpayer is required to allocate a portion of the stock basis to the stock rights.

*a. True b. False

1675. The carryover basis to a donee for property received by gift can be an amount greater than the donor’s adjusted basis.

*a. True b. False

1676. This year, Fran receives a birthday gift of stock worth $75,000 from her aunt. The aunt has owned the stock (adjusted basis $50,000) for 10 years and pays gift tax of $27,000 on the transfer. Fran’s basis in the stock is $75,000—the lesser of $77,000 ($50,000 + $27,000) or $75,000.

a. True *b. False

1677. The amount of the loss basis of a gift will differ from the amount of the gain basis only if at the date of the gift the adjusted basis of the property exceeds the property’s fair market value.

*a. True b. False

1678. The basis for depreciation on depreciable gift property received is the donor’s adjusted basis of the property at the date of the gift (assuming no gift taxes are paid). The rule applies regardless of whether the fair market value at the date of the gift is greater than or less than the donor’s adjusted basis.

*a. True b. False

1679. The holding period for property acquired by gift is automatically long term.

a. True *b. False

1680. The basis of inherited property usually is its fair market value on the date of the decedent’s death.

*a. True b. False

1681. If the fair market value of the property on the date of death is greater than on the alternate valuation date, the use of the alternate valuation amount is mandatory.

a. True *b. False

1682. If the alternate valuation date is elected by the executor in 2013, the total basis of inherited property will be more than what it would have been if the primary valuation date and amount had been used.

a. True *b. False

1683. If the alternate valuation date is elected by the executor of the estate, the basis of all of the property included in the decedent’s estate becomes the fair market value 6 months after the decedent’s death.

a. True *b. False

1684. If a husband inherits his deceased wife’s share of jointly owned property in a common law state, both the husband’s original share and the share inherited from the deceased wife are stepped-up or down to the fair market value at the date of the wife’s death.

a. True *b. False

1685. Parker bought a brand new Ferrari on January 1, 2013, for $125,000. Parker was fatally injured in an auto accident on June 23, 2013, when the fair market value of the car was $105,000. Parker was driving a loaner car from the Ferrari dealership while his car was being serviced. In his will, Parker left the Ferrari to his best friend, Ryan. Ryan’s holding period for the Ferrari begins on January 1, 2013.

a. True *b. False

1686. Transactions between related parties that result in disallowed losses might later provide a tax benefit to the related party buyer.

*a. True b. False

1687. For the loss disallowance provision under § 267, related parties include certain family members, a shareholder and his or her controlled corporation (i.e., greater than 50% in value of the corporation’s outstanding stock), and a partner and his or her controlled partnership (i.e., greater than 50% of the capital interests or profits interest in the partnership).

*a. True b. False

1688. If losses are disallowed in a related party transaction, the holding period for the buyer includes the holding period of the seller.

a. True *b. False

1689. Ben sells stock (adjusted basis of $25,000) to his son, Ray, for its fair market value of $15,000. Ray gives the stock to his daughter, Trish, who subsequently sells it for $26,000. Ben’s recognized loss is $0 and Trish’s recognized gain is $1,000 ($26,000 – $15,000 – $10,000).

a. True *b. False

1690. The basis of property acquired in a wash sale is its cost plus the loss not recognized on the wash sale.

*a. True b. False

1691. Realized losses from the sale or exchange of stock are disallowed if within 30 days before or 30 days after the sale or exchange, the taxpayer acquires substantially identical stock.

*a. True b. False

1692. Gene purchased an SUV for $45,000 which he uses 100% for personal purposes. When the SUV is worth $30,000, he contributes it to his business. The gain basis is $45,000, the loss basis is $30,000, and the basis for cost recovery is $45,000.

a. True *b. False

1693. If property that has been converted from personal use to business use has appreciated in value, its basis for gain will be the same as the basis for loss.

*a. True b. False

1694. The basis for gain and loss of personal use property converted to business use is the lower of the adjusted basis or the fair market value on the date of conversion.

a. True *b. False

1695. Stuart owns land with an adjusted basis of $190,000 and a fair market value of $500,000. If the property is going to be given to Stuart’s nephew, Alex, it is preferable for the transfer to be by inheritance rather than by gift.

*a. True b. False

1696. The taxpayer owns stock with an adjusted basis of $15,000 and a fair market value of $8,000. If the stock or cash is going to be given to her niece, it is preferable for the taxpayer to sell the stock and give the $8,000 of cash to her niece. The same preference would exist if the recipient were a qualified charitable organization.

*a. True b. False

1697. Broker’s commissions, legal fees, and points paid by the seller reduce the seller’s amount realized.

*a. True b. False

1698. Since wash sales do not apply to gains, it may be desirable to engage in this type of transaction before the end of the tax year.

*a. True b. False

1699. Albert purchased a tract of land for $140,000 in 2010 when he heard that a new highway was going to be constructed through the property and that the land would soon be worth $200,000. Highway engineers surveyed the property and indicated that he would probably get $180,000. The highway project was abandoned in 2013 and the value of the land fell to $100,000. What is the amount of loss Albert can claim in 2013?

a. $40,000. b. $60,000. c. $80,000. d. $100,000. *e. None of the above.

1700. Abby sells real property for $300,000. The buyer pays $5,000 in property taxes that had accrued during the year while the property was still legally owned by Abby. In addition, Abby pays $15,000 in commissions and $3,000 in legal fees in connection with the sale. How much does Abby realize (the amount realized) from the sale of her property?

a. $277,000. b. $282,000. *c. $287,000. d. $300,000. e. None of the above.

1701. Alice owns land with an adjusted basis of $610,000, subject to a mortgage of $350,000. Real estate taxes are $9,000 per calendar year and are payable on December 31. On April 1, 2013, Alice sells her land subject to the mortgage for $650,000 in cash, a note for $600,000, and property with a fair market value of $120,000. What is the amount realized?

a. $1,370,000. b. $1,372,219. c. $1,720,000. *d. $1,722,219. e. None of the above.

1702. Pedro borrowed $250,000 to purchase a machine costing $300,000. He later borrowed an additional $25,000 using the machine as collateral. Both notes are nonrecourse. Eight years later, the machine has an adjusted basis of zero and two outstanding note balances of $145,000 and $18,000. Pedro sells the machine subject to the two liabilities for $45,000. What is his realized gain or loss?

a. $0. b. $45,000. c. $163,000. *d. $208,000. e. None of the above.

1703. The bank forecloses on Lisa’s apartment complex. The property had been pledged as security on a nonrecourse mortgage, whose principal amount at the date of foreclosure is $750,000. The adjusted basis of the property is $480,000, and the fair market value is $750,000. What is Lisa’s recognized gain or loss?

*a. $270,000. b. ($750,000). c. $0. d. ($480,000). e. None of the above.

1704. Carlton purchases land for $550,000. He incurs legal fees of $10,000 and broker’s commission of $28,000 associated with the purchase. He subsequently incurs additional legal fees of $25,000 in having the land rezoned from agricultural to residential. He subdivides the land and installs streets and sewers at a cost of $800,000. What is Carlton’s basis for the land and the improvements?

a. $1,350,000. b. $1,378,000. c. $1,385,000. *d. $1,413,000. e. None of the above.

1705. Jamie bought her house in 2008 for $395,000. Since then, she has deducted $70,000 in depreciation associated with her home office and has spent $45,000 replacing all the old pipes and plumbing. She sells the house on July 1, 2013. Her realtor charged $34,700 in commissions. Prior to listing the house with the realtor, she spent $300 advertising in the local newspaper. Sammy buys the house for $500,000 in cash, assumes her mortgage of $194,000, and pays property taxes of $4,200 for the entire year on December 1, 2013. What is Jamie’s adjusted basis at the date of the sale and the amount realized?

a. $370,000 adjusted basis; $661,400 amount realized. *b. $370,000 adjusted basis; $661,100 amount realized. c. $370,000 adjusted basis; $665,200 amount realized. d. $325,000 adjusted basis; $663,200 amount realized. e. $325,000 adjusted basis; $694,000 amount realized.

1706. Yolanda buys a house in the mountains for $450,000 which she uses as her personal vacation home. She builds an additional room on the house for $40,000. She sells the property for $560,000 and pays $28,000 in commissions and $4,000 in legal fees in connection with the sale. What is the recognized gain or loss on the sale of the house?

a. $0. *b. $38,000. c. $70,000. d. $110,000. e. None of the above.

1707. On February 2, 2013, Karin purchases real estate for $375,000. The annual property taxes of $5,000 are payable on December 31. Realizing that she will pay the property taxes for the entire year, Karin remits $374,575 to the seller at closing. Karin’s adjusted basis for the real estate is:

a. $374,575. *b. $375,000. c. $375,425. d. $379,575. e. None of the above.

1708. Capital recoveries include:

a. The cost of capital improvements. b. Ordinary repair and maintenance expenditures. c. Payments made on the principal of a mortgage on taxpayer’s building. *d. Amortization of bond premium. e. All of the above.

1709. Steve purchased his home for $500,000. As a sole proprietor, he operates a certified public accounting practice in his home. For this business, he uses one room exclusively and regularly as a home office. In Year 1, $3,042 of depreciation expense on the home office was deducted on his income tax return. In Year 2, Steve sustained losses in his business; therefore, no depreciation was taken on the home office. Had he been allowed to deduct depreciation expense, his depreciation expense would have been $3,175. What is the adjusted basis in the home?

*a. $493,783. b. $496,825. c. $496,958. d. $500,000. e. None of the above.

1710. Sandra’s automobile, which is used exclusively in her trade or business, was damaged in an accident. The adjusted basis prior to the accident was $11,000. The fair market value before the accident was $10,000 and the fair market value after the accident is $6,000. Insurance proceeds of $3,200 are received. What is Sandra’s adjusted basis for the automobile after the casualty?

a. $0. *b. $7,000. c. $7,800. d. $10,200. e. None of the above.

1711. Joyce’s office building was destroyed in a fire (adjusted basis of $350,000; fair market value of $400,000). Of the insurance proceeds of $360,000 she receives, Joyce uses $310,000 to purchase additional inventory and invests the remaining $50,000 in short-term certificates of deposit. She received only $360,000 because of a co-insurance clause in her insurance policy. What is Joyce’s recognized gain or loss?

a. $0. b. $10,000 loss. *c. $10,000 gain. d. $40,000 gain. e. None of the above.

1712. Elvis owns all of the stock of White Corporation. The accumulated earnings and profits of White Corporation at the beginning of the year are a deficit of $20,000. The current earnings and profits are $30,000. Elvis’ basis for his stock is $250,000. He receives a distribution of $300,000 on the last day of the tax year. How much dividend income and/or capital gain should Elvis report?

a. $0. *b. Dividend income of $30,000 and capital gain of $20,000. c. Dividend income of $30,000 and capital gain of $0. d. Dividend income of $10,000 and capital gain of $20,000. e. None of the above.

1713. Karen owns City of Richmond bonds with a face value of $10,000. She purchased the bonds on January 1, 2013, for $11,000. The maturity date is December 31, 2022. The annual interest rate is 8%. What is the amount of taxable interest income that Karen should report for 2013, and the adjusted basis for the bonds at the end of 2013, assuming straight-line amortization is appropriate?

a. $0 and $11,000. *b. $0 and $10,900. c. $100 and $11,000. d. $100 and $10,900. e. None of the above.

1714. Jason owns Blue Corporation bonds (face value of $10,000), purchased on January 1, 2013, for $11,000. The bonds have an annual interest rate of 8% and a maturity date of December 31, 2022. If Jason elects to amortize the bond premium, what is his taxable interest income for 2013 and the adjusted basis for the bonds at the end of 2013 (assuming straight-line amortization is appropriate)?

a. $800 and $11,000. b. $800 and $10,900. c. $700 and $11,000. *d. $700 and $10,900. e. None of the above.

1715. A strip along the boundary of Joy’s land is condemned for a utility easement. She receives a payment of $7,500 from the utility company. Her basis in the land is $80,000. Which of the following is correct?

a. Joy must include the $7,500 in gross income. *b. Joy must reduce the basis of the land by $7,500. c. Joy must include the $7,500 in the gross income and increase the basis of the land by $7,500. d. Only a. and c. are correct. e. a., b., and c. are correct.

1716. Katie sells her personal use automobile for $12,000. She purchased the car three years ago for $25,000. What is Katie’s recognized gain or loss?

*a. $0. b. $12,000. c. ($13,000). d. ($25,000). e. None of the above.

1717. Noelle owns an automobile which she uses for personal use. Her adjusted basis is $45,000 (i.e., the original cost). The car is worth $22,000. Which of the following statements is correct?

a. If Noelle sells the car for $22,000, her realized loss of $23,000 is not recognized. b. If Noelle exchanges the car for another car worth $22,000, her realized loss of $23,000 is not recognized. c. If the car is stolen and it is uninsured, Noelle may be able to recognize part of her realized loss of $23,000. d. Only a. and b. are correct. *e. a., b., and c. are correct.

1718. Mary sells her personal use automobile for $20,000. She purchased the car two years ago for $17,000. What is Mary’s recognized gain or loss? It increased in value due to its excellent mileage, yet safe design.

a. $0. *b. $3,000. c. $17,000. d. $20,000. e. None of the above.

1719. Which of the following statements is false?

*a. A realized gain that is never recognized results in the temporary recovery of more than the taxpayer’s cost or other basis for tax purposes. b. A realized gain on which recognition is postponed results in the temporary recovery of more than the taxpayer’s cost or other basis for tax purposes. c. A realized loss that is never recognized results in the permanent recovery of less than the taxpayer’s cost or other basis for tax purposes. d. A realized loss on which recognition is postponed results in the temporary recovery of less than the taxpayer’s cost or other basis for tax purposes. e. All of the above.

1720. Nat is a salesman for a real estate developer. His employer permits him to purchase a lot for $75,000. The employer’s adjusted basis for the lot is $45,000, and its normal selling price is $90,000.

What is Nat’s recognized gain and his basis for the lot? Recognized gain Basis

a. $0 $ 75,000 b. $0 $ 90,000 c. $15,000 $ 75,000 *d. $15,000 $ 90,000 e. $30,000 $105,000

1721. Over the past 20 years, Alfred has purchased 380 shares of Green, Inc., common stock. His first purchase was in 1992 when he acquired 30 shares for $20 a share. In 1997, Alfred bought 150 shares at $10 a share. In 2012, Alfred acquired 200 shares at $50 a share. Alfred intends to sell 125 shares at $60 per share in the current year (2013). If Alfred’s objective is to minimize gain, what is his recognized gain?

*a. $1,250. b. $3,520. c. $5,950. d. $6,250. e. None of the above.

1722. Mona purchased a business from Judah for $1,000,000. Judah’s records and an appraiser provided her with the following information regarding the assets purchased:

Adjusted Basis FMV Land $195,000 $270,000 Building 310,000 450,000 Equipment 95,000 180,000

What is Mona’s adjusted basis for the land, building, and equipment?

*a. Land $270,000, building $450,000, equipment $180,000. b. Land $195,000, building $575,000, equipment $230,000. c. Land $195,000, building $310,000, equipment $95,000. d. Land $270,000, building $521,429, equipment $208,571. e. None of the above.

1723. Nontaxable stock dividends result in:

a. A higher cost per share for all shares than before the stock dividend. *b. A lower cost per share for all shares than before the stock dividend. c. An increase in the total cost of the old and new stock combined. d. A decrease in the total cost of the old and new stock combined. e. None of the above.

1724. Kevin purchased 5,000 shares of Purple Corporation stock at $10 per share. Two years later, he receives a 5% common stock dividend. At that time, the common stock of Purple Corporation had a fair market value of $12.50 per share. What is the basis of the Purple Corporation stock, the per share basis, and gain recognized upon receipt of the common stock dividend?

a. $50,000 basis in stock, $10 basis per share for the original stock and $0 basis per share for the dividend shares, $0 recognized gain. *b. $50,000 basis in stock, $9.52 basis per share, $0 recognized gain. c. $53,125 basis in stock, $10 basis per share for the original stock and $12.50 basis per share for the dividend shares, $3,125 recognized gain. d. $53,125 basis in stock, $10.12 basis per share, $3,125 recognized gain. e. None of the above.

1725. Etta received nontaxable stock rights on October 3, 2013. She allocated $16,000 of the $50,000 basis for the associated stock to the stock rights. The stock rights are exercised on November 8, 2013. The exercise price for the stock is $52,000. What is Etta’s basis for the acquired stock?

a. $0. b. $16,000. c. $52,000. *d. $68,000. e. None of the above.

1726. Mike’s basis in his stock in Tan Corporation is $75,000. He receives nontaxable stock rights (fair market value of $20,000) when the value of the stock is $100,000. What is the basis for the stock rights?

a. $0. *b. $12,500. c. $15,000. d. The basis is $0 unless the taxpayer elects to allocate a portion of the cost of the stock to the rights. e. None of the above.

1727. Which of the following statements is correct?

a. Under no circumstances does part of the stock basis have to be allocated to nontaxable stock rights. *b. If the fair market value of stock rights is equal to at least 15% of the fair market value of the stock, part of the stock basis must be allocated to nontaxable stock rights. c. An election may be made to allocate part of the stock basis to nontaxable stock rights only if the fair market value of the nontaxable stock rights is at least 15% of the fair market value of the stock. d. Only b. and c. are correct. e. Only a. and c. are correct.

1728. In 2009, Harold purchased a classic car that he planned to restore for $12,000. However, Harold is too busy to work on the car and he gives it to his daughter Julia in 2013. At this time, the fair market value of the car has declined to $10,000. Harold paid no gift tax on the transaction. Julia completes some of the restoration herself with out-of-pocket costs of $5,000. She later sells the car for $30,000. What is Julia’s recognized gain or loss on the sale of the car?

a. $0. *b. $13,000. c. $15,000. d. $18,000. e. None of the above.

1729. Ralph gives his daughter, Angela, stock (basis of $8,000; fair market value of $6,000). No gift tax results. If Angela subsequently sells the stock for $10,000, what is her recognized gain or loss?

a. $0. *b. $2,000. c. $4,000. d. $10,000. e. None of the above.

1730. Gift property (disregarding any adjustment for gift tax paid by the donor):

a. Has no basis to the donee because he or she did not pay anything for the property. *b. Has the same basis to the donee as the donor’s adjusted basis if the donee disposes of the property at a gain. c. Has the same basis to the donee as the donor’s adjusted basis if the donee disposes of the property at a loss, and the fair market value on the date of gift was less than the donor’s adjusted basis. d. Has no basis to the donee if the fair market value on the date of gift is less than the donor’s adjusted basis. e. None of the above.

1731. Shontelle received a gift of income-producing property with an adjusted basis of $49,000 to the donor and fair market value of $35,000 on the date of gift. Gift tax of $6,000 was paid by the donor. Shontelle subsequently sold the property for $31,000. What is the recognized gain or loss?

a. $0. *b. ($4,000). c. ($10,000). d. ($18,000). e. None of the above.

1732. Rob was given a residence in 2013. At the time of the gift, the residence had a fair market value of $201,000, and its adjusted basis to the donor was $141,000. The donor paid a gift tax of $10,000 on the taxable gift of $187,000. What is Rob’s basis for gain?

a. $140,000. *b. $144,200. c. $150,000. d. $200,000. e. None of the above.

1733. In addition to other gifts, Megan made a gift of stock to Jeri in 1976. Megan had purchased the stock in 1974 for $7,500. At the time of the gift, the stock was worth $20,000. If Megan paid $850 of gift tax on the transaction in 1976, what is Jeri’s gain basis for the stock?

a. $7,500. *b. $8,350. c. $9,017. d. $20,000. e. None of the above.

1734. Noelle received dining room furniture as a gift from her friend, Jane. Jane’s adjusted basis was $9,200 and the fair market value on the date of the gift was $7,000. Noelle decided she did not need the furniture and sold it to a neighbor six months later for $6,500. What is her recognized gain or loss?

*a. $0. b. ($500). c. ($2,700). d. $6,500. e. None of the above.

1735. The holding period of property acquired by gift may begin on:

a. The date the property was acquired by the donor only. b. The date of gift only. *c. Either the date the property was acquired by the donor or the date of gift. d. The last day of the tax year in which the property was originally acquired by the donor. e. None of the above.

1736. Nancy gives her niece a crane to use in her business with a fair market value of $61,000 and a basis in Nancy’s hands of $80,000. No gift tax was paid. What is the niece’s basis for depreciation (cost recovery)?

a. $0. b. $19,000. c. $61,000. *d. $80,000. e. None of the above.

1737. Which of the following is correct?

a. The gain basis for property received by gift is the lesser of the donor’s adjusted basis or the fair market value on the date of the gift. b. The loss basis for property received by gift is the same as the donor’s basis. c. The gain basis for inherited property is the same as the decedent’s basis. d. The loss basis for inherited property is the lesser of the decedent’s basis or the fair market value on the date of the decedent’s death. *e. None of the above.

1738. Tobin inherited 100 acres of land on the death of his father in 2013. A Federal estate tax return was filed and the land was valued at $300,000 (its fair market value at the date of the death). The father had originally acquired the land in 1970 for $19,000 and prior to his death had made permanent improvements of $6,000. What is Tobin’s basis in the land?

a. $19,000. b. $25,000. *c. $300,000. d. $325,000. e. None of the above.

1739. Al owns stock with an adjusted basis of $100,000 and a fair market value of $300,000. He gives the stock to Jane on July 1, 2012. When Jane dies, the fair market value of the stock is $900,000. Jane’s will provides that Al is to receive the stock. Which of the following is false?

a. If Jane dies on June 1, 2013, Al’s basis for the stock is $100,000. b. If Jane dies on August 1, 2013, Al’s basis for the stock is $900,000. *c. If Jane dies on June 15, 2013, Al’s basis is $300,000. d. If Jane dies on July 1, 2013, Al’s basis is $100,000. e. All of the above are true.

1740. Emma gives 1,000 shares of Green, Inc. stock to her niece, Margaret. Emma’s adjusted basis for the stock is $200,000 and the fair market value is $300,000. Seven months after the gift, Margaret is killed in an airplane accident. Emma inherits the stock which then is worth $350,000. What is the adjusted basis of the inherited stock to Emma?

a. $0. *b. $200,000. c. $300,000. d. $350,000. e. None of the above.

1741. Neal and his wife Faye reside in Texas, a community property state. Their community property consists of real estate (adjusted basis of $800,000; fair market value of $6 million) and personal property (adjusted basis of $390,000; fair market value of $295,000). Neal dies first and leaves his estate to Faye. What is Faye’s basis in the property after Neal’s death?

a. $800,000 real estate and $295,000 personal property. b. $800,000 real estate and $390,000 personal property. c. $3,400,000 real estate and $295,000 personal property. *d. $6,000,000 real estate and $295,000 personal property. e. None of the above.

1742. Robert and Diane, husband and wife, live in Pennsylvania, a common law state. They purchased land as joint tenants in 2009 for $300,000. In 2013, Diane dies and bequeaths her share of the land to Robert. The land has a fair market value of $450,000. What is Robert’s adjusted basis for the land?

a. $300,000. *b. $375,000. c. $450,000. d. $750,000. e. None of the above.

1743. Taylor inherited 100 acres of land on the death of his father in 2013. A Federal estate tax return was filed and this land was valued therein at $650,000, its fair market value at the date of the father’s death. The father had originally acquired the land in 1967 for $112,000 and prior to his death he had expended $20,000 on permanent improvements. Determine Taylor’s holding period for the land.

a. Will begin with the date his father acquired the property. *b. Will automatically be long-term. c. Will begin with the date of his father’s death. d. Will begin with the date the property is distributed to him. e. None of the above.

1744. Kelly inherits land which had a basis to the decedent of $95,000 and a fair market value of $50,000 on August 4, 2013, the date of the decedent’s death. The executor distributes the land to Kelly on November 12, 2013, at which time the fair market value is $49,000. The fair market value on February 4, 2014, is $45,000. In filing the estate tax return, the executor elects the alternate valuation date. Kelly sells the land on June 10, 2014, for $48,000. What is her recognized gain or loss?

*a. ($1,000). b. ($2,000). c. ($47,000). d. $1,000. e. None of the above.

1745. Arthur owns a tract of undeveloped land (adjusted basis of $145,000) which he sells to his son, Ned, for its fair market value of $105,000. What is Arthur’s recognized gain or loss and Ned’s basis in the land?

*a. $0 and $105,000. b. $0 and $145,000. c. ($40,000) and $105,000. d. ($40,000) and $145,000. e. None of the above.

1746. Paul sells property with an adjusted basis of $45,000 to his daughter Dean, for $38,000. Dean subsequently sells the property to her brother, Preston, for $38,000. Three years later, Preston sells the property to Hun, an unrelated party, for $50,000. What is Preston’s recognized gain or loss on the sale of the property to Hun?

a. $0. b. $5,000. *c. $12,000. d. ($5,000). e. None of the above.

1747. Karen purchased 100 shares of Gold Corporation stock for $11,500 on January 1, 2010. In the current tax year (2013), she sells 25 shares of the 100 shares purchased on January 1, 2010, for $2,500. Twenty-five days earlier, she had purchased 30 shares for $3,000. What is Karen’s recognized gain or loss on the sale of the stock, and what is her basis in the 30 shares purchased 25 days earlier?

a. $375 recognized loss, $3,000 basis in new stock. b. $0 recognized loss, $3,000 basis in new stock. *c. $0 recognized loss, $3,375 basis in new stock. d. $0 recognized loss, $3,450 basis in new stock. e. None of the above.

1748. Andrew acquires 2,000 shares of Eagle Corporation stock for $100,000 on March 31, 2009. On January 1, 2013, he sells 125 shares for $5,000. On January 22, 2013, he purchases 135 shares of Eagle Corporation stock for $6,075. When does Andrew’s holding period begin for the 135 shares?

a. January 22, 2013. b. January 1, 2013. c. March 31, 2009. *d. March 31, 2009, for 125 shares and January 22, 2013, for 10 shares. e. None of the above.

1749. The basis of personal use property converted to business use is:

a. Always the lower of its adjusted basis or fair market value on the date of conversion. b. Always its adjusted basis on the date of conversion. c. Always its fair market value on the date of conversion. d. Always the higher of its adjusted basis or fair market value on the date of conversion. *e. None of the above.

1750. Lynn purchases a house for $52,000. She converts the property to rental property when the fair market value is $115,000. After deducting depreciation (cost recovery) expense of $1,130, she sells the house for $120,000. What is her recognized gain or loss?

a. $0. b. $6,130. c. $37,630. *d. $69,130. e. None of the above.

1751. Robert sold his ranch which was his principal residence during the current taxable year. At the date of the sale, the ranch had an adjusted basis of $460,000 and was encumbered by a mortgage of $200,000. The buyer paid him $500,000 in cash, agreed to take the title subject to the $200,000 mortgage, and agreed to pay him $100,000 with interest at 6 percent one year from the date of sale. How much is Robert’s recognized gain on the sale?

Correct Answer: Amount realized:

Cash $500,000 Mortgage (property taken subject 200,000 to) Note receivable 100,000 $800,000 Adjusted basis (460,000) Realized and recognized gain $340,000

1752. Ken is considering two options for selling land for which he has an adjusted basis of $100,000 and on which there is a mortgage of $80,000. Under the first option, Ken will sell the land for $225,000 with a stipulation in the sales contract that he liquidate the mortgage before the sale is complete. Under the second option, Ken will sell the land for $145,000 and the buyer will assume the mortgage. Calculate Ken’s recognized gain under both options.

Correct Answer: Option 1 Option 2 Amount realized $225,000 $225,000 Less: Adjusted basis (100,000) (100,000) Recognized gain $125,000 $125,000

Since the liability assumption is included in the calculation of Ken’s amount realized, the recognized gain is $125,000, the same as for the cash sale.

1753. Annette purchased stock on March 1, 2013, for $200,000. At December 31, 2013, it was worth $210,000. She also purchased a bond on September 1, 2013, for $20,000. At year end, it was worth $15,000. Determine Annette’s realized and recognized gain or loss.

Correct Answer: Annette’s realized gain or loss is zero and her recognized gain or loss is zero. Since a sale or other disposition has not occurred, there is no realization or recognition on either the stock or the bond.

1754. Nigel purchased a blending machine for $125,000 for use in his business. As to the machine, he has deducted MACRS cost recovery of $31,024, maintenance costs of $5,200, and repair costs of $4,000. Calculate Nigel’s adjusted basis for the machine.

Correct Answer: Nigel’s adjusted basis for the machine is calculated as follows:

Cost $125,000 Less: Cost recovery (31,024 ) Adjusted basis $ 93,976

Neither the maintenance cost of $5,200 nor the repair cost of $4,000 are capital expenditures. These costs are deducted in the tax year incurred.

1755. Peggy uses a delivery van in her business. The adjusted basis is $39,000, and the fair market value is $34,000. The delivery van is stolen and Peggy receives insurance proceeds of $34,000. Determine Peggy’s realized and recognized gain or loss.

Correct Answer: Amount realized $34,000 Adjusted basis (39,00 0) Realized loss ($ 5,00 0) Recognized loss ($ 5,00 0)

Since the proceeds received from the insurance company are less than the adjusted basis, the realized loss of $5,000 is recognized.

1756. Jan purchases taxable bonds with a face value of $250,000 for $265,000. The annual interest paid on the bonds is $10,000. Assume Jan elects to amortize the bond premium. The total premium amortization for the first year is $1,600. a. What is Jan’s interest income for the first year? b. What is Jan’s interest deduction for the first year? c. What is Jan’s adjusted basis for the bonds at the end of the first year?

Correct Answer: a. Jan receives interest payments of $10,000 each year. This amount is included in her gross income because the bonds are taxable. b. Jan deducts the premium amortization of $1,600 for the first year because the bonds are taxable. c. Jan’s adjusted basis for the bonds at the end of the first year is $263,400 ($265,000 cost – $1,600 premium amortization).

1757. Boyd acquired tax-exempt bonds for $430,000 in December 2013. The bonds, which mature in December 2018, have a maturity value of $400,000. Boyd does not make any elections regarding the amortization of the bond premium. Determine the tax consequences to Boyd when he redeems the bonds in December 2018.

Correct Answer: When Boyd redeems the bonds in 2018, he has no realized or recognized gain or loss.

Amount realized $400,000 Adjusted basis for bonds (400,000) Realized gain $ – 0–

Recognized gain $ – 0–

Amortization of the premium on tax-exempt bonds is mandatory. Thus, the adjusted basis of the bonds at the maturity date is $400,000 ($430,000 cost – $30,000 premium amortized). Since the bonds are tax-exempt, the amount of interest income included in Boyd’s gross income (i.e., $0) is not affected by the amortization of the bond premium.

1758. Marilyn owns 100% of the stock of Lilac, Inc., with an adjusted basis of $45,000. She receives a cash distribution of $160,000 from Lilac when its earnings and profits are $90,000. a. What is Marilyn’s dividend income? b. What is Marilyn’s recognized gain or loss? c. What is Marilyn’s adjusted basis for her stock after the distribution?

Correct Answer: a. and b. The $160,000 distribution is accounted for as follows:

Dividend income (Lilac’s $ 90,00 earnings and profits) 0 Return of capital (Marilyn’s 45,000 basis in the stock) Capital gain (presuming the 25 stock is a capital asset) ,000 Total distribution $160,000

c. Marilyn’s adjusted basis for her stock is $0.

1759. Hilary receives $10,000 for a 15-foot wide utility easement along one of the boundaries to her property. The easement provides that no structure can be built on that portion of the property. Her adjusted basis for the property is $200,000 and the easement covers 15% of the total acreage. Determine the effect of the $10,000 payment on Hilary’s gross income and her basis for the property.

Correct Answer: Hilary does not report the $10,000 payment in her gross income. Instead, she reduces the basis for the property by the $10,000 payment from $200,000 to $190,000.

1760. Ollie owns a personal use car for which he originally paid $48,000. He trades the car in on a sports utility vehicle (SUV) paying the automobile dealer cash of $30,000. If the negotiated price of the SUV is $49,000, what is Ollie’s recognized gain or loss and his adjusted basis for the SUV?

Correct Answer: Ollie’s realized loss on the trade of his personal use car is calculated as follows:

Amount realized (trade-in $19,000 value) Adjusted basis (48,000 ) Realized loss ($29,000)

Since the car was a personal use asset, none of the realized loss of $29,000 is recognized. Ollie’s adjusted basis for the SUV is his cost of $49,000.

1761. Omar has the following stock transactions during 2013:

Date Number Number Selling of Stock purchased shares of Basis price sold shares Orange 1/2011 100 $1,000 Blue 6/2011 200 3,00 0 Yellow 4/2012 50 1,25 0 Blue 2/2013 150 1,80 0 Yellow 3/2013 175 5,25 0 Blue 7/2013 250 $3,500 Yellow 11/2013 200 7,200

a. What is Omar’s recognized gain or loss on the stock sales if his objective is to minimize the recognized gain and to maximize the recognized loss? b. What is Omar’s recognized gain or loss if he does not identify the shares sold?

Correct Answer: a. Since Omar’s objective is to minimize recognized gain and maximize recognized loss, he will identify the specific shares (i.e., specific identification method) being sold. He will select high basis shares to achieve his objective.

Sale of Blue stock

Amount realized $3,500 Basis: 200 shares from 6/2011 lot $3,000 ($15 per share) 50 shares from 2/2013 lot ($12 600 (3,600) per share) Realized loss ($ 100)

Recognized loss ($ 100)

Sale of Yellow stock

Amount realized $7,200 Basis: 175 shares from 3/2013 lot $5,250 ($30 per share) 25 shares from 4/2012 lot ($25 625 (5,875) per share) Realized gain $1,325

Recognized gain $1,325

b. Since Omar does not identify the shares sold, he is required to use the FIFO method.

Sale of Blue stock

Amount realized $3,500 Basis: 200 shares from 6/2011 lot $3,000 ($15 per share) 50 shares from 2/2013 lot ($12 600 (3,600) per share) Realized loss ($ 100)

Recognized loss ($ 100)

Sale of Yellow stock

Amount realized $7,200 Basis: 50 shares from 4/2012 $1,250 lot ($25 per share) 150 shares from 3/2013 4,500 (5,750) lot ($30 per share) Realized gain $1,450

Recognized gain $1,450

1762. Hubert purchases Fran’s jewelry store for $950,000. The identifiable assets of the business are as follows: Basis FMV Inventory $ 90,000 $ 97,000

Accounts receivable 55,00 50,00 0 0 Building 100,000 225,000 Land 280,000 300,000

Hubert and Fran agree to assign $110,000 to a 7-year covenant not to compete. How should Hubert allocate the $950,000 purchase price to the assets?

Correct Answer: The purchase price is allocated to the assets as follows:

Inventory $ 97,000 Accounts receivable 50,000 Building 225,000 Land 300,000 Covenant 110,000 Goodwill 168,000 $950,000

Under the residual method, $168,000 ($950,000 – $782,000) is assigned to goodwill.

1763. Marge purchases the Kentwood Krackers, a AAA level baseball team, for $1.5 million. The appraised values of the identified assets are as follows:

Prepaid season tickets $150,000 Stadium lease 400,000 Player contracts 500,000 Equipment 100,000

The Krackers have won the pennant for the past two years. Determine Marge’s adjusted basis for the assets of the Kentwood Krackers.

Correct Answer: The portion of the purchase price of $1.5 million assigned to the identified assets is as follows:

Prepaid season tickets $ 150,000 Stadium lease 400,000 Player contracts 500,000 Equipment 100,00 0 $1,150,000

The residual value of $350,000 ($1,500,000 – $1,150,000) is assigned to goodwill.

1764. Melody’s adjusted basis for 10,000 shares of Cardinal, Inc. common stock is $1,000,000. During the year, she receives a 5% stock dividend that is a nontaxable stock dividend. a. What is the amount of Melody’s gross income? b. What is Melody’s total basis for the stock? c. What is Melody’s basis per share?

Correct Answer: a. Melody has no gross income because the dividend is a nontaxable stock dividend. b. Melody’s total stock basis remains at $1,000,000. c. The basis per share decreases to $95.24 per share ($1,000,000/10,500 shares).

1765. In 2013, Felix gives 10,000 shares of stock to his daughter, Monica. The stock was acquired in 2004 for $200,000, and at the time of the gift, it had a fair market value of $600,000. Felix paid a gift tax of $240,000. [Assume the gift tax exclusion has been used by Felix— see Chapter 1.] a. Does the receipt of the stock result in gross income to Monica? b. What is Monica’s basis in the stock?

Correct Answer: a. The receipt of the gift does not result in gross income to Monica. b. Monica’s basis in the stock is calculated as follows:

*Fraction rounded to 68%. **The $586,000 is equal to the fair market value of the stock of $600,000 reduced by the per donee annual exclusion of $14,000.

1766. On September 18, 2013, Jerry received land and a building from Ted as a gift. Ted had purchased the land and building on March 5, 2010, and his adjusted basis and the fair market value at the date of the gift were as follows:

Asset Adjusted FMV Basis Land $150,000 $200,000 Building 90,000 100,00 0

Ted paid gift tax on the transfer to Jerry of $100,000. a. Determine Jerry’s adjusted basis and holding period for the land and building. b. Assume instead that the FMV of the land was $89,000 and the FMV of the building was $60,000. Determine Jerry’s adjusted basis and holding period for the land and building.

Correct Answer: a. Jerry’s total basis for the assets received from Ted is:

$150,000 + $90,000 +[($60,000/$286,000*) × $96,000] = $261,000

The basis is allocated to the land and building as follows:

Land: ($200,000/$300,000) × $261,000 = $174,000

Building: ($100,000/$300,000) × $261,000 = $87,000

Jerry’s holding period begins on March 5, 2010.

*The $286,000 is equal to the fair market value of the land and building of $300,000 reduced by the per donee annual exclusion of $14,000. The fraction was rounded to 21%.

b. Because the land and building have declined in value, none of the gift tax paid by Ted is considered in calculating Jerry’s adjusted basis. Jerry’s basis for gain is:

Land $150,000 Building 90,000

Jerry’s basis for loss (the lower of Ted’s adjusted basis or the FMV at the date of the gift) is:

Land $89,000 Building 60,000

Jerry’s holding period begins on March 5, 2010, for the gain basis and September 18, 2013, for the loss basis.

1767. Emma gives her personal use automobile (cost of $32,000; fair market value of $12,000) to her son, Louis, on July 3, 2013. She has owned the automobile since July 1, 2010. a. What is Louis’ basis for the car? b. When does his holding period begin?

Correct Answer: a. Louis’ gain basis is $32,000 and his loss basis is $12,000. b. Louis’ holding period for gain begins on July 1, 2010 and his holding period for loss begins on July 3, 2013.

1768. Faith inherits an undivided interest in a parcel of land from her father on February 15, 2013. Her father purchased the land on August 25, 1986 and his basis for the land was $325,000. The fair market value of the land is $12,500,000 on the date of her father’s death and is $11,000,000 six months later. The executor elects the alternate valuation date. Faith has nine brothers and sisters and each inherited a one-tenth interest. a. What is Faith’s adjusted basis for her one- tenth undivided interest in the land? b. What is her holding period for the land?

Correct Answer: a. Faith’s adjusted basis is $1,100,000 ($11,000,000 X 10%), which is her share of the fair market value of the land on the alternate valuation date. b. Since the property is inherited, Faith’s holding period is automatically long term.

1769. Elbert gives stock worth $28,000 (no gift tax resulted) to his friend, Jeff, on June 8, 2013. Elbert purchased the stock on September 1, 2006, and his adjusted basis is $22,000. Jeff dies on December 8, 2014, and bequeaths the stock to Elbert. At that date, the fair market value of the stock is $31,000. a. What is Jeff’s basis and holding period for the stock? b. What is Elbert’s basis and holding period for the stock?

Correct Answer: a. Jeff has a carryover basis of $22,000 and a carryover holding period of September 1, 2006. b. Elbert has a new basis of $31,000. Since Elbert inherited the stock, his holding period is automatically long term. Since the period between the date of the gift (June 8, 2013) and the date of Jeff’s death (December 8, 2014) is more than one year, the deathbed gift provision does not apply.

1770. Ed and Cheryl have been married for 27 years. They own land jointly with a basis of $300,000. Ed dies in 2013, when the fair market value of the land is $500,000. Under the joint ownership arrangement, the land passed to Cheryl. a. If Ed and Cheryl reside in a community property state, what is Cheryl’s basis in the land? b. If Ed and Cheryl reside in a common law state, what is Cheryl’s basis in the land?

Correct Answer: a. Cheryl’s basis in the land is $500,000 ($250,000 + $250,000). b. Cheryl’s basis in the land is $400,000 [($300,000 × 50%) + $250,000].

1771. On January 15 of the current taxable year, Merle sold stock with a cost of $40,000 to his brother Ned for $25,000, its fair market value. On June 21, Ned sold the stock to a friend for $26,000. a. What are the tax consequences to Merle and Ned? b. Would Ned recognize any gain if he sold the stock for $41,000?

Correct Answer: a. Merle realizes a loss of $15,000 [i.e., $25,000 (amount realized) – $40,000 (adjusted basis)] which is disallowed because the stock was sold to a related party. Ned realizes a gain of $1,000 [i.e., $26,000 (amount realized) – $25,000 (adjusted basis)] on the sale to a friend, but does not recognize any gain. Ned’s gain of $1,000 is less than Merle’s previously disallowed loss of $15,000. b. Ned would realize a gain of $16,000 [i.e., $41,000 (amount realized) – $25,000 (adjusted basis)]. Gain of $1,000 would be recognized [i.e., $16,000 (gain realized) – $15,000 (previously disallowed loss)].

1772. Monica sells a parcel of land to her son, Elbert, for $90,000. Monica’s adjusted basis is $100,000. Three years later, Elbert gives the land to his fiancée, Karen. At that date, the land is worth $104,000. No gift tax is paid. Since Elbert is going to be stationed in the U.S. Army in Germany for 3 years, they do not plan on being married until his tour is completed. Six months after receiving the land, Karen sells it for $110,000. At the same time, Karen sends Elbert a “Dear John” email. Calculate Karen’s realized and recognized gain or loss.

Correct Answer: Elbert’s adjusted basis for the land is his purchase price of $90,000. When Elbert gives the land to Karen, her adjusted basis is a carryover basis of $90,000. Karen’s gain on the sale is calculated as follows:

Amount realized $110,000 Adjusted basis (90,000 ) Realized gain $ 20,000

Recognized gain $ 20,000

Monica’s disallowed loss of $10,000 ($90,000 amount realized – $100,000 adjusted basis) could have been used as an offset by Elbert if he had sold the land at a realized gain. But, it cannot be used by Karen since she is not the original transferee (i.e., related-party buyer).

1773. Mitch owns 1,000 shares of Oriole Corporation common stock (adjusted basis of $15,000). On April 27, 2013, he sells 400 shares for $5,200, while on May 5, 2013, he purchases 200 shares for $3,600. a. What is Mitch’s recognized gain or loss resulting from these transactions? b. What is Mitch’s basis for the stock acquired on May 5, 2013? c. Could Mitch have obtained different tax consequences in a. and b. if he had sold the 400 shares on December 27, 2013, and purchased the 200 shares on January 5, 2014?

Correct Answer: a. To the extent of the substantially identical shares purchased during the 60-day period beginning 30 days before April 27 and ending 30 days after April 27, the transaction is a wash sale. The realized loss on the April 27 sale is $800 ($5,200 amount realized – $6,000 adjusted basis of 400 shares). Because Mitch acquired fewer shares than he sold, only a portion of the realized loss is disallowed. The disallowed loss is $400 [(200 shares acquired/400 shares sold) × $800] and the recognized loss is $400 ($800 – $400). b. Mitch’s adjusted basis for the stock acquired on May 5, 2013, is $4,000 ($3,600 purchase price + $400 disallowed loss). c. The tax consequences would have been the same. Mitch has a wash sale to the extent of the 200 shares purchased. To avoid the limitations of the wash sale, he should not purchase substantially identical stock within the 60-day window for a wash sale.

1774. Marsha transfers her personal use automobile to her business (a sole proprietorship). The car’s adjusted basis is $30,000 and the fair market value is $16,000. No cost recovery had been deducted by Marsha, since she held the car for personal use. Determine the adjusted basis of the car to Marsha’s sole proprietorship including the basis for cost recovery.

Correct Answer: In this circumstance, the car is dual basis property. The adjusted basis to the sole proprietorship for gain is $30,000 and the adjusted basis for loss is $16,000. The loss basis of $16,000 is used in calculating cost recovery.

1775. Jacob owns land with an adjusted basis of $140,000 and a fair market value of $115,000. Determine the amount of realized and recognized gain or loss to the seller and the adjusted basis for the buyer for each of the following. a. Jacob sells the land for $115,000 to a corporation in which he owns 60% of the stock. b. Jacob sells the land for $115,000 to a partnership in which he has a capital and profits interest of 60%.

Correct Answer: a. Jacob’s realized and recognized loss is calculated as follows.

Amount realized $115,000 Adjusted basis (140,000) Realized loss ($ 25,000)

Recognized loss $ –0–

Losses on sales to a controlled corporation (greater than 50%) are disallowed under the related party rules. The corporation’s adjusted basis for the land is its cost of $115,000. b. Jacob’s realized and recognized loss is calculated as follows.

Amount realized $115,000 Adjusted basis (140,000) Realized loss ($ 25,000)

Recognized loss $ – 0–

Losses on sales to a controlled partnership (greater than 50%) are disallowed under the related party rules. The partnership’s adjusted basis for the land is its cost of $115,000.

1776. When a property transaction occurs, what four questions should be considered with respect to the sale or other disposition?

Correct Answer: The following questions need to be answered.

• Is there a realized gain or loss?

• If so, is the gain or loss recognized?

• If the gain or loss is recognized, is it ordinary or capital?

• What is the basis of any replacement property that is acquired?

1777. Discuss the effect of a liability assumption on the seller’s amount realized and the buyer’s adjusted basis.

Correct Answer: If the buyer assumes the seller’s liability associated with the acquisition of property, both the seller’s amount realized and the buyer’s adjusted basis are increased by the amount of the liability assumed.

1778. Define fair market value as it relates to property transactions.

Correct Answer: The fair market value of property received in a sale or other disposition has been defined by the courts as the price at which property will change hands between a willing seller and a willing buyer when neither is compelled to sell or to buy.

1779. What is the general formula for calculating the adjusted basis of property?

Correct Answer: Adjusted basis is determined as follows:

Cost (or other adjusted basis) on date of acquisition + Capital additions – Capital recoveries = Adjusted basis

1780. What is the difference between the depreciation (or cost recovery) allowed and the depreciation (or cost recovery) allowable and what effect does each have on the adjusted basis of property?

Correct Answer: Normally, there is no difference between the depreciation (or cost recovery) allowed or allowable. The allowed depreciation (or cost recovery) is the amount actually taken, whereas the allowable depreciation (cost recovery) is the amount that could have been taken under the applicable depreciation (or cost recovery) method. The basis of the property is reduced by the cost recovery allowed, but this amount cannot be less than the allowable amount.

1781. What effect does a deductible casualty loss have on the adjusted basis of property?

Correct Answer: A deductible casualty loss reduces the basis of property.

1782. For a corporate distribution of cash or other property to a shareholder, when does dividend income or a return of capital result?

Correct Answer: To the extent of corporate earnings and profits, a distribution to a shareholder is treated as dividend income. When the distribution exceeds corporate earnings and profits, a distribution to a shareholder is treated as a return of capital (i.e., tax-free to the extent of shareholder basis and capital gain for any excess).

1783. Under what circumstances will a distribution by a corporation to its only shareholder result in a capital gain?

Correct Answer: Capital gain will result if the amount of the distribution exceeds the corporation’s earnings and profits and the shareholder’s basis in the stock.

1784. If a taxpayer purchases taxable bonds at a premium, the amortization of the premium is elective. However, if a taxpayer purchases tax-exempt bonds at a premium, the amortization of the premium is mandatory. Explain this difference in the treatment.

Correct Answer: If mandatory amortization were not required for tax-exempt bonds, a taxpayer who held such bonds to maturity would have a recognized loss to the extent of the premium. This is not consistent with the rule that interest earned on the bonds is tax-exempt. Mandatory amortization, therefore, results in the adjusted basis of the bonds ultimately being equal to the maturity value. Thus, no loss results upon maturity. Furthermore, the amortization of the premium on tax-exempt bonds is not deductible.

For the taxable bonds and if the taxpayer does not elect to amortize the premium, a recognized capital loss results to this extent at maturity. Typically, the taxpayer will elect to amortize the premium so that it can be claimed over the life of the bond as an ordinary (rather than capital) deduction.

1785. Maurice sells his personal use automobile at a realized loss. Under what circumstances can Maurice deduct the loss? What if the personal use asset was sold at a realized gain?

Correct Answer: Under no circumstance can Maurice recognize (deduct) a loss on the sale of a personal use asset. Note that if the automobile had been used in a trade or business or held for the production of income, the loss could have been deducted.

If the personal use asset was sold at a realized gain, the realized gain would be recognized.

1786. Describe the relationship between the recovery of capital doctrine and the realized and recognized gain and loss concepts.

Correct Answer: The relationship between the recovery of capital doctrine and the realized and recognized gain and loss concepts can be summarized as follows:

• A realized gain that is never recognized results in the permanent recovery of more than the taxpayer’s cost or other basis for tax purposes.

• A realized gain on which recognition is postponed results in the temporary recovery of more than the taxpayer’s cost or other basis for tax purposes.

• A realized loss that is never recognized results in the permanent recovery of less than the taxpayer’s cost or other basis for tax purposes.

• A realized loss on which recognition is postponed results in the temporary recovery of less than the taxpayer’s cost or other basis for tax purposes.

1787. Define a bargain purchase of property and discuss the related tax consequences.

Correct Answer: A bargain purchase can occur when an employer transfers property to an employee at less than the fair market value. It results in compensation for services. It also occurs when a corporation sells property to a shareholder at less than its fair market value. The result is a dividend. The amount included in income is the difference between the purchase price and the property’s fair market value. The basis of the property acquired in the bargain purchase is the property’s fair market value.

1788. If a taxpayer purchases a business and the price exceeds the fair market value of the listed assets, how is the excess allocated among the purchased assets?

Correct Answer: The excess is not allocated among the listed assets. Instead, the excess is assigned to goodwill.

1789. Tara owns common stock in Taupe, Inc., with an adjusted basis of $250,000. She receives a preferred stock dividend which is nontaxable. a. What effect does the preferred stock dividend have on Tara’s adjusted basis of the common stock? b. How is the basis of the preferred stock calculated? c. What effect does the preferred stock dividend have on Tara’s gross income?

Correct Answer: a. Part of the adjusted basis of the common stock must be allocated to the preferred stock thereby decreasing the basis of the common. b. The amount that is allocated to the preferred stock is based on the relative fair market values of the common stock and the preferred stock on the date of the distribution. c. Since the preferred stock dividend is nontaxable, it has no effect on Tara’s gross income.

1790. Lois received nontaxable stock rights with a fair market value of $6,000. The fair market value of the stock on which the rights were received is $24,000 (cost $14,000). Assume the rights are exercised by paying $32,000 plus the rights. Discuss how to calculate the basis of the old stock and the basis of the new stock.

Correct Answer: Since the fair market value of the stock rights is at least 15% of the fair market value of the stock, part of the basis of the stock must be allocated to the stock rights. Thus, the basis of the new stock includes the basis of the old stock that is allocated to the stock rights.

Basis of new stock is $34,800 [$32,000 + ($6,000/$30,000 × $14,000)]. Basis of old stock is $11,200 ($14,000 – $2,800).

1791. For gifts made after 1976, when will part of the gift tax paid by the donor be added to the donee’s basis?

Correct Answer: This result would occur if the fair market value at the date of the gift exceeds the donor’s adjusted basis. In this case, the portion of the gift tax paid that is related to the appreciation is added to the donor’s basis in calculating the donee’s gain basis for the property.

1792. How is the donee’s basis calculated for the gift of appreciated property for a gift made before 1977? Assume the donor pays gift tax.

Correct Answer: If the gift is made before 1977, the donee’s adjusted basis is the sum of the donor’s adjusted basis for the property plus all of the gift tax paid by the donor. However, the total cannot exceed the fair market value of the property at the date of the gift.

1793. Joseph converts a building (adjusted basis of $50,000 and fair market value of $40,000) from personal use to business use. Justin receives a building with a $40,000 fair market value ($50,000 donor’s adjusted basis) from his mother as a gift. Discuss the tax consequences with respect to Joseph’s and Justin’s adjusted basis.

Correct Answer: Upon conversion from personal use to business use, the building receives dual basis treatment. That is, Joseph’s gain basis for the building is $50,000, the adjusted basis on the date of the conversion from personal use to business use. Joseph’s loss basis is $40,000, the lower of the adjusted basis or the fair market value on the date of the conversion. Justin’s basis for the building upon receipt of the gift is treated in the same manner according to gift basis rules (i.e., dual basis treatment).

1794. Discuss the application of holding period rules to property acquired by gift and inheritance.

Correct Answer: The holding period for inherited property is always long-term. For gift property, if the donee’s basis is the donor’s adjusted basis (i.e., gain basis), the holding period starts on the date the property was acquired by the donor. If the donee’s basis is fair market value (i.e., loss basis), the holding period starts on the date of the gift.

1795. What is a deathbed gift and what tax consequences apply?

Correct Answer: A deathbed gift occurs when a donor makes a gift of appreciated property to a terminal person with the understanding that the donor (or the donor’s spouse) will inherit the property on the donee’s death. If the period between the date of the gift and the date of the donee’s death exceeds one year, the usual transfer-by-death basis rules apply. However, if this period is one year or less, the transfer-by-gift basis rules apply.

1796. Sam and Cheryl, husband and wife, own property jointly. The property has an adjusted basis of $400,000 and a fair market value of $500,000. a. Discuss the rules for the calculation of the adjusted basis of the property to Sam if he inherits his wife’s share of the property and Sam and Cheryl live in a community property state. b. If they live in a common law state?

Correct Answer: a. In a community property state, Sam’s basis for the property will be stepped-up to fair market value for both the decedent’s share and the survivor’s share of the community property (i.e., $250,000 + $250,000 = $500,000). b. In a common law state, his basis will be stepped-up only for the decedent’s share (i.e., $250,000 + $200,000 = $450,000).

1797. Explain how the sale of investment property at a loss to a brother is treated differently from a sale to a niece.

Correct Answer: The brother is a related party under the § 267 loss disallowance provision. Consequently, the realized loss on the sale of the investment property is disallowed. The brother’s basis for the investment property is its cost. However, if the brother sells the investment property at a realized gain, he can offset this gain with as much of the prior disallowed loss as is needed to reduce it to zero. Otherwise, the disallowed loss is wasted.

Because a niece is not treated as a related party under § 267, the realized loss on the sale of the investment property is recognized. The niece’s basis for the investment property is its cost.

1798. For disallowed losses on related-party transactions, who has the right of offset?

Correct Answer: The right of offset is available only to the related-party buyer (i.e., the original transferee). If the related-party buyer transfers the property to another party by either gift or inheritance, the right of offset is not available to that party.

1799. What is the easiest way for a taxpayer who is going to sell property that has declined in value to avoid the § 267 loss disallowance provision?

Correct Answer: In this circumstance, the easiest way for a taxpayer to recognize the realized loss (by avoiding the § 267 loss disallowance provision) is to sell the property to someone who is not a related party as defined in § 267 (i.e., sell in the marketplace).

1800. Tariq sold certain U.S. Government bonds and State of Oregon bonds at a loss to offset short-term capital gain from a previous transaction. He felt that the U.S. Government and State of Oregon bonds were “good” investments, so he repurchased identical securities within one week. Do these transactions constitute wash sales?

If the bond sales resulted in the recognition of gain (rather than loss), would the wash sale provisions prevent the gains from being recognized?

Correct Answer: The wash sale rules apply because Tariq purchased substantially identical securities within the meaning of § 1091. If the bonds were sold at a gain, however, the wash sale rules would not apply.

1801. Morgan owned a convertible that he had purchased two years ago for $46,000 and which he transfers to his sole proprietorship. How is the sole proprietorship’s basis for the car calculated? What additional information does Morgan need?

Correct Answer: Morgan needs to calculate both the gain basis and the loss basis of the convertible for the sole proprietorship. The gain basis is a carryover basis and the loss basis is the lower of Morgan’s adjusted basis or the fair market value on the date of the transfer. So Morgan needs to determine the FMV and to identify the beginning of the holding period. The holding period associated with the gain basis includes the two years that Morgan has owned the convertible. The holding period associated with the loss basis starts on the date of the transfer.

1802. Jamie is terminally ill and does not expect to live much longer. Pondering the consequences of her estate, she decides how to allocate her property to her nephews. She makes a gift of depreciated property (i.e., adjusted basis exceeds fair market value) to Will, a gift of appreciated property (i.e., fair market value exceeds adjusted basis) to Jim, and leaves appreciated property to Sam in her will. Each of the properties has the same fair market value. From an income tax perspective, which nephew is her favorite?

Correct Answer: Jamie appears to like Sam best. Sam receives the most beneficial tax treatment by receiving a stepped-up basis (i.e., fair market value on the date of Jamie’s death) in the inherited property. Therefore, he would recognize less gain than Jim. Further, he would not have to deal with the dual basis issue like Will if he decided to sell the property. Because Jim receives a gift of appreciated property, he will realize gain equal to the amount of appreciation if he decides to sell. This is because his basis (i.e., carryover) is equal to Jamie’s adjusted basis.

Jamie appears to be indifferent about Will. A gift of depreciated property receives a loss basis to Will of the lower of the adjusted basis or the fair market value on the date of the gift. This eliminates a possible loss deduction for Jamie and also prevents Will from taking a loss deduction for the decline in value while Jamie owned the property. On the other hand, Will’s gain basis is equal to Jamie’s adjusted basis for the property (and is greater than Sam’s basis). Therefore, if the property appreciates while owned by Will, he will have recognized gain on the sale only if the property appreciates to a fair market value in excess of Jamie’s adjusted basis for the property.

1803. Why is it generally undesirable to pass property by death when its fair market value is less than basis?

Correct Answer: Assuming the property is not personal use property (where neither the decedent nor the beneficiary is able to deduct any of the loss), the decedent should sell the property prior to his or her death.

1804. Identify two tax planning techniques that can be used to avoid the wash sale disallowance of loss.

Correct Answer: One technique to avoid a wash sale result is to not purchase substantially identical stock or securities. Another is to avoid the 60-day wash sale window.

1805. Gains and losses on nontaxable exchanges are deferred because the tax law recognizes that nontaxable exchanges result in a change in the substance but not the form of the taxpayer’s relative economic position.

a. True *b. False

1806. Abby exchanges an SUV that she has held for personal use plus $24,000 for a new SUV which she will use exclusively in her sole proprietorship business. This exchange qualifies for nontaxable exchange treatment.

a. True *b. False

1807. In a nontaxable exchange, recognition is postponed. In a tax-free transaction, nonrecognition is permanent.

*a. True b. False

1808. In a nontaxable exchange, the replacement property is assigned a carryover basis if there is a realized gain, but receives a new basis if there is a realized loss.

a. True *b. False

1809. The nonrecognition of gains and losses under § 1031 is mandatory for gains and elective for losses.

a. True *b. False

1810. Leonore exchanges 5,000 shares of Pelican, Inc., stock for 2,000 shares of Blue Heron, Inc., stock. Leonore’s adjusted basis for the Pelican stock is $300,000 and the fair market value of the Blue Heron stock is $350,000. Leonore’s recognized gain is $0 and her adjusted basis for the Blue Heron stock is $300,000.

a. True *b. False

1811. Livestock of different sexes can qualify for like-kind exchange treatment if the livestock has been held for over 24 months.

a. True *b. False

1812. To qualify as a like-kind exchange, real property must be exchanged either for other real property or for personal property with a statutory life of at least 39 years.

a. True *b. False

1813. The exchange of unimproved real property located in Topeka (KS) for improved real property located in Atlanta (GA) does not qualify as a like-kind exchange.

a. True *b. False

1814. Lola owns land as an investor. She exchanges the land for a warehouse which she leases to a tenant who uses it to store his business inventory. The exchange does qualify for like-kind exchange treatment.

*a. True b. False

1815. A building located in Virginia (used in business) exchanged for a building located in France (used in business) cannot qualify for like- kind exchange treatment.

*a. True b. False

1816. Pat owns a 1965 Mustang car which he uses for personal use. He purchased it four years ago for $22,000, and it currently is worth $27,000. He exchanges it for a 1979 Triumph Spitfire convertible worth $27,000. Pat’s recognized gain is $0 and his adjusted basis for the convertible is $22,000.

a. True *b. False

1817. Kate exchanges land held as an investment for land and a building owned by Clark, to be used in her business. If Clark is Kate’s father, her realized gain of $150,000 must be recognized because they are related parties.

a. True *b. False

1818. An exchange of two items of personal property (personalty) that belong to different general business asset classes qualifies for nonrecognition under § 1031 as long as both properties are used in the taxpayer’s trade or business.

a. True *b. False

1819. If boot is received in a § 1031 like-kind exchange, the recognized gain cannot exceed the realized gain.

*a. True b. False

1820. Shari exchanges an office building in New Orleans (adjusted basis of $700,000) for an apartment building in Baton Rouge (fair market value of $900,000). In addition, she receives $100,000 of cash. Shari’s recognized gain is $100,000 and her basis for the apartment building is $800,000 ($700,000 adjusted basis + $100,000 recognized gain).

a. True *b. False

1821. When boot in the form of cash is given in a like-kind exchange, recognized gain is the greater of the boot or the realized gain.

a. True *b. False

1822. The surrender of depreciated boot (fair market value is less than adjusted basis) in a like-kind exchange can result in the recognition of loss.

*a. True b. False

1823. Cole exchanges an asset (adjusted basis of $15,000; fair market value of $25,000) for another asset (fair market value of $19,000). In addition, he receives cash of $6,000. If the exchange qualifies as a like-kind exchange, his recognized gain is $6,000 and his adjusted basis for the property received is $21,000 ($15,000 + $6,000 recognized gain).

a. True *b. False

1824. The basis of boot received in a like-kind exchange is its fair market value, unless the realized gain is a smaller amount.

a. True *b. False

1825. Terry exchanges real estate (acquired on August 25, 2007) held for investment for other real estate to be held for investment on September 1, 2013. None of the realized gain of $10,000 is recognized, and Terry’s adjusted basis for the new real estate is a carryover basis of $80,000. Consequently, Terry’s holding period for the new real estate begins on August 25, 2007.

*a. True b. False

1826. If boot is received in a § 1031 like-kind exchange that results in some of the realized gain being recognized, the holding period for both the like-kind property and the boot received begins on the date of the exchange.

a. True *b. False

1827. If a taxpayer exchanges like-kind property under § 1031 and assumes a liability associated with the property received, the taxpayer is considered to have received boot in the transaction.

a. True *b. False

1828. An involuntary conversion results from the destruction (complete or partial), theft, seizure, requisition or condemnation, or the sale or exchange under threat or imminence of requisition or condemnation of the taxpayer’s property.

*a. True b. False

1829. Section 1033 (nonrecognition of gain from an involuntary conversion) applies to both gains and losses.

a. True *b. False

1830. The amount realized does not include any amount received by the taxpayer that is designated as severance damages by both the government and the taxpayer.

*a. True b. False

1831. Under the taxpayer-use test for a § 1033 involuntary conversion, the taxpayer has less flexibility in qualifying replacement property than under the functional-use test.

a. True *b. False

1832. Milt’s building which houses his retail sporting goods store is destroyed by a flood. Sandra’s warehouse which she is leasing to Milt to store the inventory of his business also is destroyed in the same flood. Both Milt and Sandra receive insurance proceeds that result in a realized gain. Sandra will have less flexibility than Milt in the type of building in which she can invest the proceeds and qualify for postponement treatment under § 1033 (nonrecognition of gain from an involuntary conversion).

a. True *b. False

1833. Sidney, a calendar year taxpayer, owns a building in Columbus, OH, in which he conducts his retail computer sales business. The building is destroyed by fire on December 12, 2013, and two weeks later he receives insurance proceeds of $600,000. Due to family ties, Sidney decides to move to Columbia, SC. He reinvests all of the insurance proceeds in a building in Columbia where he opens a retail computer sales business on April 2, 2014. By electing § 1033, Sidney has no recognized gain and a basis in the new building of $450,000 ($600,000 cost – $150,000 postponed gain).

*a. True b. False

1834. Dennis, a calendar year taxpayer, owns a warehouse (adjusted basis of $190,000) which is destroyed by a tornado in October 2013. He receives insurance proceeds of $250,000 in January 2014. If before 2017, Dennis replaces the warehouse with another warehouse costing at least $250,000, he can elect to postpone the recognition of any realized gain.

*a. True b. False

1835. If a taxpayer reinvests the net proceeds (amount received – related expenses) received in an involuntary conversion in qualifying replacement property within the statutory time period, it is possible to defer the recognition of the realized gain.

*a. True b. False

1836. The holding period of replacement property where the election to postpone gain is made includes the holding period of the involuntarily converted property.

*a. True b. False

1837. A realized gain on an indirect (conversion into money) involuntary conversion of business property can be postponed, but a realized loss on an indirect involuntary conversion of business property cannot be postponed.

*a. True b. False

1838. Gil’s office building (basis of $225,000 and fair market value $275,000) is destroyed by a hurricane. Due to a 30% co-insurance clause, Gil receives insurance proceeds of $192,500 two months after the date of the loss. One month later, Gil uses the insurance proceeds to purchase a new office building for $275,000. His adjusted basis for the new building is $307,500 ($275,000 cost + $32,500 postponed loss).

a. True *b. False

1839. Casualty losses and condemnation losses on the involuntary conversion of a personal residence receive the same tax treatment.

a. True *b. False

1840. If the recognized gain on an involuntary conversion equals the realized gain because of a reinvestment deficiency, the basis of the replacement property will be more than its cost (cost plus realized gain).

a. True *b. False

1841. If there is an involuntary conversion (i.e., casualty, theft, or condemnation) of the taxpayer’s principal residence, the realized gain may be postponed as a § 1033 involuntary conversion or excluded as a § 121 sale of a principal residence.

*a. True b. False

1842. The taxpayer must elect to have the exclusion of gain under § 121 (sale of principal residence) apply.

a. True *b. False

1843. At a particular point in time, a taxpayer can have two principal residences for § 121 exclusion purposes.

a. True *b. False

1844. To qualify for the § 121 exclusion, the property must have been used by the taxpayer for the 5 years preceding the date of sale and owned by the taxpayer as the principal residence for the last 2 of those years.

a. True *b. False

1845. A taxpayer who sells his or her principal residence at a realized loss can elect to recognize the loss even if a qualified residence is acquired during the statutory time period.

a. True *b. False

1846. Wyatt sells his principal residence in December 2013 and qualifies for the § 121 exclusion. He sells another principal residence in November 2014. Under no circumstance can Wyatt qualify for the § 121 exclusion on the sale of the second residence.

a. True *b. False

1847. Taxpayer owns a home in Atlanta. His company transfers him to Chicago on January 2, 2013, and he sells the Atlanta house in early February. He purchases a residence in Chicago on February 3, 2013. On December 15, 2013, taxpayer’s company transfers him to . In January 2014, he sells the Chicago residence and purchases a residence in Los Angeles. Because multiple sales have occurred within a two-year period, § 121 treatment does not apply to the sale of the second home.

a. True *b. False

1848. The maximum amount of the § 121 gain exclusion on sale of a principal residence is $250,000 for a single individual and $500,000 for a married couple.

*a. True b. False

1849. Deidra has owned and occupied her principal residence for 10 years. Two and one-half years ago she married Doug who moved into her house. Doug has never owned a home. When Deidra is transferred to another city, she sells the house and has a realized gain of $425,000. Deidra can exclude the realized gain of $425,000 from her gross income under § 121 if she and Doug file a joint return.

*a. True b. False

1850. Alex used the § 121 exclusion three months prior to his marriage to June. If June sells her principal residence four months after their marriage, she cannot use the § 121 exclusion.

a. True *b. False

1851. Kitty, who is single, sells her principal residence, which she has owned and occupied for 8 years, for $375,000. The adjusted basis is $64,000 and selling expenses are $22,000. She purchases another principal residence three months later for $200,000. Her recognized gain is $39,000 and her basis for the new principal residence is $200,000.

*a. True b. False

1852. Matt, who is single, sells his principal residence, which he has owned and occupied for 5 years, for $435,000. The adjusted basis is $140,000 and the selling expenses are $20,000. Three days after the sale he purchases another residence for $385,000. Matt’s recognized gain is $25,000 and his basis for the new residence is $385,000.

*a. True b. False

1853. A taxpayer whose principal residence is destroyed in a fire can use both the § 121 (sale of residence gain exclusion) and the § 1033 (involuntary conversion postponement of gain) provisions.

*a. True b. False

1854. Owen and Polly have been married for five years. Owen sells investment property to Polly for a realized gain of $140,000. Owen’s gain of $140,000 is not recognized and Polly’s basis for the property she purchased is her cost.

a. True *b. False

1855. Which of the following statements is correct?

a. In a nontaxable exchange in which gain is realized, the transaction results in a permanent recovery of more than the taxpayer’s cost or other basis for tax purposes. b. In a nontaxable exchange in which loss is realized, the transaction results in a permanent recovery of less than the taxpayer’s cost or other basis for tax purposes. *c. In a tax-free transaction in which gain is realized, the transaction results in the permanent recovery of more than the taxpayer’s cost or other basis for tax purposes. d. All of the above. e. None of the above.

1856. In order to qualify for like-kind exchange treatment under § 1031, which of the following requirements must be satisfied?

a. The form of the transaction is a sale or exchange. *b. Both the property transferred and the property received are held either for productive use in a trade or business or for investment. c. The exchange must be completed by the end of the second tax year following the tax year in which the taxpayer relinquishes his or her like-kind property. d. Only a. and b. e. a., b., and c.

1857. Which, if any, of the following exchanges qualifies for nonrecognition treatment as a § 1031 like-kind exchange?

a. Partnership interest for a partnership interest. b. Inventory for inventory. c. Securities for personalty. *d. Business realty for investment realty. e. None of the above.

1858. Brett owns investment land located in Tucson, Arizona. He exchanges it for other investment land. In which of the following locations may the other investment land be located and enable Brett to qualify for § 1031 like-kind exchange treatment?

a. Mexico City, Mexico. b. Toronto, Canada. c. Paris, France. d. Only a. and b. *e. None of the above.

1859. Lily exchanges a building she uses in her rental business for a building owned by Kendall, her brother, which she will use in her rental business. The adjusted basis of Lily’s building is $120,000 and the fair market value is $170,000. Which of the following statements is correct?

a. Lily’s recognized gain is $50,000 and her basis for the building received is $120,000. b. Lily’s recognized gain is $50,000 and her basis for the building received is $170,000. *c. Lily’s recognized gain is $0 and her basis for the building received is $120,000. d. Lily’s recognized gain is $0 and her basis for the building received is $170,000. e. None of the above is correct.

1860. Latisha owns a warehouse with an adjusted basis of $200,000. She exchanges it for a strip mall building worth $225,000. Which of the following statements is correct?

a. If the warehouse was used in Latisha’s business to store inventory and the strip mall building is to be rented to tenants, her recognized gain is $25,000 and her basis for the strip mall building is $225,000. b. If the warehouse was used in Latisha’s business to store inventory and the strip mall building is to be used as a retail outlet for her business, her recognized gain is $0 and her basis for the strip mall building is $200,000. c. If the warehouse is used by Latisha to store personal use items such as excess furniture and the strip mall building is to be rented to tenants, her recognized gain is $25,000 and her basis for the strip mall building is $225,000. *d. Only b. and c. are correct. e. a., b., and c. are correct.

1861. Which of the following statements is correct?

a. The receipt of boot in a § 1031 like-kind exchange can result in the recognition of gain. b. The receipt of boot in a § 1031 like-kind exchange cannot result in the recognition of loss. c. The giving of boot in a § 1031 like-kind exchange can result in the recognition of gain. d. Only a. and b. *e. a., b., and c.

1862. Bud exchanges a business use machine with an adjusted basis of $22,000 and a fair market value of $30,000 for another business use machine with a fair market value of $28,000 and $2,000 cash. What is Bud’s recognized gain or loss?

a. $0. *b. $2,000. c. $6,000. d. $8,000. e. None of the above.

1863. Maud exchanges a rental house at the beach with an adjusted basis of $225,000 and a fair market value of $200,000 for a rental house at the mountains with a fair market value of $180,000 and cash of $20,000. What is the recognized gain or loss?

*a. $0. b. $20,000. c. ($20,000). d. ($25,000). e. None of the above.

1864. Melvin receives stock as a gift from his uncle. No gift tax is paid. The adjusted basis of the stock is $30,000 and the fair market value is $38,000. Melvin trades the stock for bonds with a fair market value of $35,000 and $3,000 cash. What is his recognized gain and the basis for the bonds?

a. $0, $30,000. b. $5,000, $33,000. c. $5,000, $30,000. d. $8,000, $33,000. *e. None of the above.

1865. Moss exchanges a warehouse for a building he will use as an office building. The adjusted basis of the warehouse is $600,000 and the fair market value of the office building is $350,000. In addition, Moss receives cash of $150,000. What is the recognized gain or loss and the basis of the office building?

a. $0 and $350,000. *b. $0 and $450,000. c. ($150,000) and $300,000. d. ($200,000) and $350,000. e. None of the above.

1866. Pam exchanges a rental building, which has an adjusted basis of $520,000, for investment land which has a fair market value of $700,000. In addition, Pam receives $100,000 in cash. What is the recognized gain or loss and the basis of the investment land?

a. $0 and $420,000. b. $100,000 and $420,000. *c. $100,000 and $520,000. d. $280,000 and $700,000. e. None of the above.

1867. If boot is received in a § 1031 like-kind exchange and gain is recognized, which formula correctly calculates the basis for the like- kind property received?

a. Adjusted basis of like-kind property surrendered + gain recognized – fair market value of boot received. b. Fair market value of like-kind property surrendered + gain recognized + fair market value of boot received. c. Fair market value of like-kind property received – postponed gain. *d. Only a. and c. e. None of the above.

1868. In determining the basis of like-kind property received, postponed losses are:

a. Added to the basis of the old property. b. Subtracted from the basis of the old property. *c. Added to the fair market value of the like-kind property received. d. Subtracted from the fair market value of the like-kind property received. e. None of the above.

1869. Molly exchanges a small machine (adjusted basis of $85,000; fair market value of $78,000) used in her business and investment land (adjusted basis of $10,000; fair market value of $15,000) for a large machine (fair market value of $93,000) to be used in her business in a like-kind exchange. What is Molly’s recognized gain or loss?

a. $0. *b. $5,000. c. ($2,000). d. ($7,000). e. None of the above.

1870. In October 2013, Ben and Jerry exchange investment realty in a § 1031 like-kind exchange. Ben bought his real estate in 2003 while Jerry purchased his in 2006. In addition to the realty, Ben receives Pearl, Inc. stock worth $10,000 from Jerry. Ben’s realized gain is $30,000. On what date does the holding period for Ben’s realty received from Jerry begin? When does the holding period for the stock he receives begin?

*a. 2003, 2013. b. 2003, 2003. c. 2006, 2006. d. 2006, 2013. e. None of the above.

1871. Dena owns 500 acres of farm land in southeastern Maryland. Her adjusted basis for the land is $480,000 and there is a $400,000 mortgage on the land. She exchanges the land for an office building owned by Chris in Newark, New Jersey. The building has a fair market value of $900,000. Chris assumes Dena’s mortgage on the land. What is the amount of Dena’s recognized gain or loss on the exchange?

a. $0. *b. $400,000. c. $500,000. d. $820,000. e. None of the above.

1872. On October 1, Paula exchanged an apartment building (adjusted basis of $375,000 and subject to a mortgage of $125,000) for another apartment building owned by Nick (fair market value of $550,000 and subject to a mortgage of $125,000). The property transfers were made subject to the mortgages. What amount of gain should Paula recognize?

*a. $0. b. $25,000. c. $125,000. d. $175,000. e. None of the above.

1873. Nancy and Tonya exchanged assets. Nancy gave Tonya her personal residence with an adjusted basis of $280,000 and a fair market value of $560,000. The house has a mortgage of $200,000 which is assumed by Tonya. Tonya gave Nancy a yacht used in her business with an adjusted basis of $250,000 and a fair market value of $360,000. What is Tonya’s realized and recognized gain?

a. $310,000 realized and $310,000 recognized gain. b. $310,000 realized and $0 recognized gain. *c. $110,000 realized and $110,000 recognized gain. d. $110,000 realized and $0 recognized gain. e. None of the above.

1874. If the taxpayer qualifies under § 1033 (nonrecognition of gain from an involuntary conversion), makes the appropriate election, and the amount reinvested in replacement property is less than the amount realized, realized gain is:

*a. Recognized to the extent of the deficiency (amount realized not reinvested). b. Recognized to the extent of realized gain. c. Recognized to the extent of the amount reinvested in excess of the adjusted basis. d. Permanently not subject to taxation. e. None of the above.

1875. Joyce, a farmer, has the following events occur during the tax year. Which of the events qualify as an involuntary conversion under § 1033 (nonrecognition of gain from an involuntary conversion)?

a. Her farm tractor is hauled to the city dump because it is worn out. b. She sells 10 acres of pasture land at a loss of $40,000 because she has reduced the size of her dairy herd in preparation for her retirement. *c. Her personal residence, adjusted basis of $100,000, is condemned to make way for an interstate highway. She recovers condemnation proceeds of $175,000. d. She sells 10 acres of pasture land at a loss of $40,000 because she has reduced the size of her dairy herd due to a reduction in milk prices. e. None of the above.

1876. Betty owns a horse farm with 500 acres of land (adjusted basis of $600,000). Fifty acres of the land are condemned by the state for $400,000 in order to build a municipal stadium. Since the fair market value of Betty’s farm is significantly decreased by the proximity to the future stadium, the state awards Betty $300,000 in severance damages. Betty does not use the $300,000 to restore the usefulness of the farm and all of the $700,000 ($400,000 + $300,000) proceeds are invested in the stock market. What is her recognized gain or loss associated with the receipt of the severance damages?

*a. $0. b. $100,000. c. $300,000. d. $340,000. e. None of the above.

1877. An office building with an adjusted basis of $320,000 was destroyed by fire on December 30, 2013. On January 11, 2014, the insurance company paid the owner $450,000. The fair market value of the building was $500,000, but under the co-insurance clause, the insurance company is responsible for only 90 percent of the loss. The owner reinvested $410,000 in a new office building on February 12, 2014, that was smaller than the original office building. What is the recognized gain and the basis of the new building if § 1033 (nonrecognition of gain from an involuntary conversion) is elected?

a. $0 and $320,000. b. $0 and $410,000. *c. $40,000 and $320,000. d. $130,000 and 410,000. e. None of the above.

1878. Which of the following statements is correct with respect to qualified replacement property in a § 1033 involuntary conversion?

a. If the functional use test applies, a warehouse used to store inventory can be replaced with a smaller building to be used to sell inventory. b. If the taxpayer use test applies, an office building rented to tenants can be replaced with an office building to be used in the taxpayer’s business. *c. If the like-kind exchange test applies, a building used by the taxpayer for manufacturing can be replaced with an office building to be used in the taxpayer’s business. d. Only b. and c. e. a., b., and c.

1879. Jared, a fiscal year taxpayer with a August 31st year-end, owns an office building (adjusted basis of $800,000) that was destroyed by fire on December 24, 2013. If the insurance settlement was $950,000 (received March 1, 2014), what is the latest date that Jared can replace the office building in order to qualify for § 1033 nonrecognition of gain?

a. December 31, 2013. b. August 31, 2014. c. December 31, 2015. *d. August 31, 2016. e. None of the above.

1880. Which of the following statements is correct for a § 1033 involuntary conversion of an office building which is destroyed by fire?

a. An election can be made to postpone gain on a § 1033 involuntary conversion only if the proceeds received are reinvested in qualifying property no later than two years after the end of the tax year in which a proceeds inflow is received that is large enough to produce a realized gain. b. The postponement of realized gain in a § 1033 involuntary conversion is elective. c. The functional use test is satisfied if a business warehouse is replaced with another business warehouse. d. The taxpayer use test is satisfied if a shopping mall rented to tenants is replaced with an office building to be rented to tenants. *e. All of the above are correct.

1881. Which of the following satisfy the time period requirement for postponement of gain as a § 1033 (nonrecognition of gain from an involuntary conversion) involuntary conversion?

a. Al’s business warehouse is destroyed by a tornado on October 31, 2013. Al is a calendar year taxpayer. He receives insurance proceeds on December 5, 2013. He reinvests the proceeds in another warehouse to be used in his business on December 29, 2015. b. Heather’s personal residence is destroyed by fire on October 31, 2013. She is a calendar year taxpayer. She receives insurance proceeds on December 5, 2013. She purchases another principal residence with the proceeds on October 31, 2015. c. Mack’s office building is condemned by the city as part of a road construction project. The date of the condemnation is October 31, 2013. He is a calendar year taxpayer. He receives condemnation proceeds from the city on that date. He purchases another office building with the proceeds on December 5, 2016. d. Lizzy’s business automobile is destroyed in an accident on October 31, 2013. Lizzy is a fiscal year taxpayer with the fiscal year ending on June 30th. She receives insurance proceeds on December 5, 2013. She purchases another business automobile with the proceeds on June 1, 2016. *e. All of the above.

1882. Sam’s office building with an adjusted basis of $750,000 and a fair market value of $900,000 is condemned on November 30, 2013. Sam is a calendar year taxpayer. He receives a condemnation award of $875,000 on March 1, 2014. He builds a new office building at a cost of $845,000 which is completed and paid for on December 31, 2016. What is Sam’s recognized gain on receipt of the condemnation award and basis for the new office building assuming his objective is to minimize gain recognition?

a. $0; $720,000. *b. $30,000; $750,000. c. $30,000; $845,000. d. $150,000; $750,000. e. None of the above.

1883. A factory building owned by Amber, Inc. is destroyed by a hurricane. The adjusted basis of the building was $400,000 and the appraised value was $425,000. Amber receives insurance proceeds of $390,000. A factory building is constructed during the nine-month period after the hurricane at a cost of $450,000. What is the recognized gain or loss and what is the basis of the new factory building?

a. $0 and $450,000. b. $0 and $460,000. c. ($10,000) and $440,000. *d. ($10,000) and $450,000. e. None of the above.

1884. If the taxpayer qualifies under § 1033 (nonrecognition of gain from an involuntary conversion) and the amount reinvested in replacement property exceeds the amount realized, the basis of the replacement property is:

a. The cost of the replacement property. b. The fair market value of the involuntarily converted property minus the postponed gain. *c. The cost of the replacement property minus the postponed gain. d. The amount realized. e. None of the above.

1885. Myrna’s personal residence (adjusted basis of $100,000) was condemned, and she received a condemnation award of $80,000. Myrna used the condemnation proceeds to purchase a new residence for $90,000. What is her recognized gain or loss and her basis in the new residence?

a. $0; $70,000. *b. $0; $90,000. c. ($20,000); $90,000. d. ($20,000); $70,000. e. None of the above.

1886. Fran was transferred from Phoenix to Atlanta. She sold her Phoenix residence (adjusted basis of $250,000) for a realized loss of $50,000 and purchased a new residence in Atlanta for $375,000. Fran had owned and lived in the Phoenix residence for 6 years. What is Fran’s recognized gain or loss on the sale of the Phoenix residence and her basis for the residence in Atlanta?

*a. $0 and $375,000. b. $0 and $425,000. c. ($50,000) and $325,000. d. ($50,000) and $375,000. e. None of the above.

1887. Evelyn, a calendar year taxpayer, lists her principal residence with a realtor on February 7, 2013, enters into a contract to sell on July 12, 2013, and sells (i.e., the closing date) the residence on August 1, 2013. The realized gain on the sale is $225,000. Which date is the appropriate ending date in determining if the residence has been owned and used by the Evelyn as the principal residence for at least two years during the prior five-year period?

a. February 7, 2013. b. July 12, 2013. *c. August 1, 2013. d. December 31, 2013. e. None of the above.

1888. During 2013, Howard and Mabel, a married couple, decided to sell their residence. The residence has a basis of $162,000 and has been owned and occupied by them for 11 years. The house was sold in May for $395,000 with broker’s commissions and other selling expenses being $24,000. They purchased a new residence in June for $400,000. What is the adjusted basis of the new residence?

a. $0. b. $141,000. c. $162,000. d. $191,000. *e. None of the above.

1889. During 2013, Ted and Judy, a married couple, decided to sell their residence, which had a basis of $300,000. They had owned and occupied the residence for 20 years. To make it more attractive to prospective buyers, they had the outside painted in April at a cost of $6,000 and paid for the work immediately. They sold the house in May for $880,000. Broker’s commissions and other selling expenses amounted to $53,000. Since they both are age 68, they decide to rent an apartment. They purchase an annuity with the net proceeds from the sale. What is the recognized gain?

a. $0. b. $17,000. *c. $27,000. d. $527,000. e. None of the above.

1890. During 2013, Zeke and Alice, a married couple, decided to sell their residence, which had a basis of $200,000. They had owned and occupied the residence for 20 years. To make it more attractive to prospective buyers, they had the inside painted in April at a cost of $5,000 and paid for the work immediately. They sold the house in May for $800,000. Broker’s commissions and other selling expenses amounted to $50,000. They purchased a new residence in July for $400,000. What is the recognized gain and the adjusted basis of the new residence?

a. $45,000 and $400,000. *b. $50,000 and $400,000. c. $100,000 and $600,000. d. $550,000 and $800,000. e. None of the above.

1891. Carl sells his principal residence, which has an adjusted basis of $150,000 for $200,000. He incurs selling expenses of $20,000 and legal fees of $2,000. He had purchased another residence one month prior to the sale for $380,000. What is the recognized gain or loss and the basis of the replacement residence if the taxpayer elects to forgo the § 121 exclusion (exclusion of gain on sale of principal residence)?

a. $0 and $380,000. b. $0 and $408,000. c. $28,000 and $352,000. *d. $28,000 and $380,000. e. None of the above.

1892. Ross lives in a house he received as a gift from his father. His father had lived in the house for 12 years. The adjusted basis of the house to his father was $160,000 and the fair market value at the time of the gift was $140,000. Ross sells this residence after living in it for 18 months for $150,000 and purchases a new home for $125,000. He incurs selling expenses of $7,000. What is Ross’ recognized gain or loss and basis for the new residence?

a. ($17,000); $125,000. b. ($17,000); $142,000. c. $3,000; $125,000. d. $3,000; $128,000. *e. None of the above.

1893. Paula inherits a home on July 1, 2013, that had a basis in the hands of the decedent at death of $290,000 and a fair market value of $500,000 at the date of the decedent’s death. She decides to sell her old principal residence, which she has owned and occupied for 9 years, with an adjusted basis of $125,000 and move into the inherited home. On September 16, 2013, she sells the old residence for $600,000. Paula incurs selling expenses of $30,000 and legal fees of $2,000. She decides to add a pool, deck, pool house, and recreation room to the inherited home at a cost of $100,000. These additions are completed and paid for on November 1, 2013. What is her recognized gain on the sale of her old principal residence and her basis in the inherited home?

a. $0; $500,000. *b. $193,000; $600,000. c. $443,000; $600,000. d. $475,000; $600,000. e. None of the above.

1894. Weston sells his residence to Joanne on October 15, 2013. Indicate which of the following statements is correctly associated with § 121 (exclusion of gain on sale of principal residence).

a. Selling expenses decrease the seller’s amount realized and increase the buyer’s adjusted basis. b. Repair expenses of the seller decrease the seller’s amount realized and have no effect on the buyer’s adjusted basis. *c. Capital expenditures made by the seller prior to the sale increase the seller’s adjusted basis and have no effect on the buyer’s adjusted basis. d. Only a. and c. e. a., b., and c.

1895. Eric and Faye, who are married, jointly own a house in which they have resided for the past 17 years. They sell the house for $375,000 with realtor’s fees of $10,000. Their adjusted basis for the house is $80,000. Since they are in their retirement years, they plan on moving around the country and renting. What is their recognized gain on the sale of the residence if they use the § 121 exclusion (exclusion of gain on sale of principal residence) and if they elect to forgo the § 121 exclusion?

With exclusion Elect to forgo

a. $0 $0 b. $35,000 $35,000 *c. $0 $285,000 d. $35,000 $285,000 e. $285,000 $225,000

1896. Lenny and Beverly have been married and living together in Lenny’s home for 6 years. He lived in the home alone for 20 years prior to their marriage. They sell the home, which has an adjusted basis of $120,000, for $700,000. Lenny and Beverly plan to use the § 121 exclusion (exclusion of gain on sale of principal residence). In Beverly’s prior marriage to Dan, Dan sold his principal residence and used the § 121 exclusion. Beverly and Dan filed joint returns during their seven years of marriage. They had lived in Dan’s house throughout their marriage. Dan’s sale had occurred one year prior to the divorce. Lenny and Beverly purchase a replacement residence for $650,000 one month after the sale. What is the recognized gain and basis for the new home?

a. $0; $80,000. b. $80,000; $150,000. *c. $80,000; $650,000. d. $330,000; $650,000. e. None of the above.

1897. Which of the following is incorrect?

a. The deferral of realized gain on a § 1031 like-kind exchange is mandatory. *b. The deferral of realized gain on an indirect (into cash and then into qualified property) § 1033 involuntary conversion is mandatory. c. The taxpayer can elect to forgo the exclusion of realized gain on a § 121 sale of residence. d. Both b. and c. are incorrect. e. a., b., and c. are incorrect.

1898. Which of the following types of exchanges of insurance contracts qualify for nonrecognition treatment under § 1035?

a. Exchange of life insurance contracts. b. Exchange of a life insurance contract for an endowment or annuity contract. c. Exchange of an endowment contract for an annuity contract. d. Only a. and b. *e. a., b., and c.

1899. Which of the following types of transactions qualify for nonrecognition treatment?

a. Exchange by a shareholder of stock in Chevron for stock in Shell. b. Investment of the proceeds from the sale of the stock of a publicly traded company in the common stock of a specialized small business investment company (SSBIC) within 60 days of the sale. c. Investment of proceeds from the sale of qualified small business stock in another qualified small business stock within 60 days of the sale. *d. Only b. and c. e. a., b., and c.

1900. As part of the divorce agreement, Tyler transfers his ownership interest in their personal residence to Lupe. The house had been jointly owned by Tyler and Lupe and the adjusted basis is $520,000. At the time of the transfer to Lupe, the fair market value is $800,000. What is the recognized gain to Tyler, and what is Lupe’s basis for the house?

*a. $0 and $520,000. b. $0 and $800,000. c. $140,000 and $520,000. d. $280,000 and $800,000. e. None of the above.

1901. Which of the following statements is correct with respect to § 1044 (rollover of publicly traded securities gain into specialized small business investment companies)?

a. Section 1044 provides for permanent exclusion of gain. b. To qualify under § 1044, the proceeds must be reinvested within one year of the sale. c. The statutory ceilings on § 1044 treatment are the same for individual and corporate taxpayers. d. Only b. and c. are correct. *e. None of the above.

1902. Which of the following might motivate a taxpayer to try to avoid like-kind exchange treatment?

a. Taxpayer has unused NOL carryovers. b. Taxpayer has unused general business credit carryovers. c. Taxpayer has suspended or current passive activity losses. d. Only a. and b. are correct. *e. a., b., and c. are correct.

1903. For the following exchanges, indicate which qualify as like-kind property. a. Inventory of a sporting goods store in Charleston for inventory of an appliance store in Savannah. b. Inventory of a ladies dress shop in Cleveland for inventory of a ladies dress shop in Richmond. c. Investment land in Virginia Beach for office building in Williamsburg. d. Used automobile used in a business for a new automobile to be used in the business. e. Investment land in Paris for investment land in San Francisco. f. Shares of Texaco stock for shares of Exxon Mobil stock.

Correct Answer: Only items c. (investment realty for investment realty or business realty) and d. (business personalty for business personalty of the same general business asset class) qualify. Items a. and b. do not qualify because they involve inventory. Item e. does not qualify because foreign realty is exchanged for domestic realty. Item f. does not qualify because shares of stock are not eligible for like-kind exchange treatment.

1904. Chaney exchanges a truck used in her business for making deliveries for a smaller more fuel-efficient truck to be used in her business for making deliveries. The adjusted basis for her truck is $32,000. The smaller truck has a fair market value of $33,000. In addition, Chaney receives cash of $4,000. a. Calculate Chaney’s realized and recognized gain or loss. b. Calculate Chaney’s basis for the assets she received.

Correct Answer: a. Amount realized ($33,000 + $37,000 $4,000) Adjusted basis (32,000) Realized gain $ 5,000

Recognized gain $ 4,000

The realized gain is recognized to the extent of the boot received of $4,000. b. Basis for smaller truck: Fair market value $33,000 Less: Postponed gain (1,000) Basis $32,000

Basis of the cash $ 4,000

1905. Sammy exchanges equipment used in his business in a like-kind exchange. The property exchanged is as follows:

Property Property Surrendered Received Adj. FMV Adj. FMV Basis Basis Equipment $44,000 $60,000$50,000 $43,000 Cash $ 5,000 $ 5,00 0 Liability on $12,000 $12,000 equipment

The other party assumes the liability. a. What is Sammy’s recognized gain or loss? b. What is Sammy’s basis for the assets he received?

Correct Answer: a. Amount realized: Equipment $43,000 Cash 5,000 Liability assumed 12,00 $60,000 0 Adjusted basis (44,000) Realized gain $16,000

Recognized gain $16,000

The recognized gain is the lesser of the boot received of $17,000 ($12,000 + $5,000) or the realized gain of $16,000.

b. Basis for equipment: Fair market value $43,000 Less: Postponed gain (– 0–) Basis $43,000

Basis of the cash $ 5,000

1906. Jake exchanges an airplane used in his business for a smaller airplane to be used in his business. His adjusted basis for the airplane is $325,000 and the fair market value is $310,000. The fair market value of the smaller airplane is $300,000. In addition, Jake receives cash of $10,000. a. Calculate Jake’s realized and recognized gain or loss and his adjusted basis for the assets received. b. Assume that the exchange is between Jake and Jake’s son. Calculate Jake’s realized and recognized gain or loss and his adjusted basis for the assets received if his son’s intention is to use the airplane in his trade or business.

Correct Answer: a. Amount realized ($300,000 + $310,000 $10,000) Adjusted basis (325,000) Realized loss ($ 15,000)

Recognized loss $ – 0–

Section 1031 postponement is mandatory. Thus, the realized loss of $15,000 is postponed.

Basis for airplane: Fair market value $300,000 Plus: Postponed loss 1 5,000 Basis $315,000

Basis of the cash $ 10,000

b. Amount realized ($300,000 + $310,000 $10,000) Adjusted basis (325,000) Realized loss ($ 15,000)

Recognized loss $ – 0–

Section 1031 postponement is mandatory. Even though the exchange is with a related party, § 1031 postponement applies. However, if Jake’s son should dispose of the airplane within two years of the date of the exchange, the realized loss of $15,000 would normally be recognized as of that date under § 1031. However, since this is a related party transaction, § 267 would then disallow the loss.

Basis for airplane: Fair market value $300,000 Plus: Postponed loss 15,00 0 Basis $315,000

Basis of the cash $ 10,000

1907. a. Orange Corporation exchanges a warehouse located in Michigan (adjusted basis of $560,000) for a warehouse located in Ohio (adjusted basis of $450,000; fair market value of $525,000). Indicate the amount of gain or loss that is recognized by Orange Corporation on the exchange, and the basis of the warehouse acquired. b. Assume that in addition to the warehouse Orange Corporation also received $100,000 in cash. Indicate the amount of gain or loss that is recognized by Orange Corporation on the exchange, and the basis of the warehouse acquired.

c. How would your answer in b. change if instead of receiving $100,000 in cash, the other party assumed Orange’s $100,000 mortgage on the Michigan warehouse?

Correct Answer: a. This is a nontaxable like-kind exchange. No gain or loss is recognized, and the basis for the new warehouse (in Ohio) is $560,000, the same as the basis for the old warehouse (in Michigan).

b. Amount realized: FMV of Ohio warehouse $525,000 Cash 100,000 $625,000 Adjusted basis of Michigan (560,000) warehouse Realized gain $ 65,000

Recognized gain $ 65,000

FMV of Ohio warehouse $525,000 Less: Postponed gain ( –0–) Basis of New Jersey $525,000 warehouse

c. The answer would the same as in b.

1908. Eunice Jean exchanges land held for investment located in Rolla, Missouri, for land to be held for investment located near Madrid, Spain. Her basis for the land given up is $450,000 and the fair market value of the land received is $500,000. Eunice Jean also receives cash of $45,000. a. What is Eunice Jean’s recognized gain? b. What is her basis for the land received?

Correct Answer: a. Amount realized ($500,000 + $545,000 $45,000) Adjusted basis (450,000) Realized gain $ 95,00 0

Recognized gain $ 95,00 0

Real property located in the United States (Rolla) exchanged for foreign real property (near Madrid) does not qualify as like-kind property. So the recognized gain is not limited to the boot received of $45,000.

b. FMV of Madrid land $500,000 Less: Postponed gain ($95,000 (– – $95,000) 0–) Basis for Madrid land $500,000

1909. For each of the following involuntary conversions, determine if the property qualifies as replacement property. a. Chuck’s restaurant building is destroyed by fire. He clears the site and builds another restaurant building. b. Diane’s warehouse which she used for storing inventory is destroyed by a tornado. She purchases another warehouse in which she will store inventory. c. Part of Andrew’s dairy farm land is condemned to make way for an interstate highway. He uses the condemnation proceeds to construct a barn to be used for storing cattle feed. d. Liz owns a shopping mall which is destroyed by a flood. Since the tenant occupancy rate was down, she uses the insurance proceeds to purchase an office building which she will rent to tenants. e. Eleanor’s Maserati Gran Turismo is stolen. The original cost was $125,000, and she had used it exclusively for personal use. Due to the limited supply of this model, it had appreciated in value. Eleanor received insurance proceeds of $130,000 and uses the proceeds to purchase a replacement Gran Turismo.

Correct Answer: All of the replacements qualify as replacement property for purposes of an involuntary conversion. Items a., b., and e. qualify under the functional use test; item c. qualifies under the like-kind test for condemned realty; and item d. qualifies under the taxpayer use test.

1910. Samuel’s hotel is condemned by the City Housing Authority on July 5, 2013, for which he is paid condemnation proceeds of $950,000. He first received official notification of the pending condemnation on May 2, 2013. Samuel’s adjusted basis for the hotel is $600,000 and he uses a fiscal year for tax purposes with a September 30 tax year-end. a. How much must Samuel reinvest in qualifying replacement property in order to postpone the recognition of realized gain? b. If Samuel reinvests the minimum amount required to avoid recognition of realized gain, what is his basis for the replacement property? c. What is qualifying replacement property? d. What is the earliest date that Samuel can acquire qualifying replacement property? e. What is the latest date that Samuel can acquire qualifying replacement property? f. How would the answer in e. change if Samuel’s hotel had been destroyed in a flood?

Correct Answer: a. Samuel must reinvest at least $950,000, an amount equal to the amount realized. This results in a postponed gain of $350,000 ($950,000 amount realized – $600,000 adjusted basis). b. Samuel’s basis for the replacement property would be:

FMV of replacement property $950,000 Less: Postponed gain (350,000) Basis $600,000

c. Since business real property has been condemned, the broader like-kind exchange rules apply. Thus, any realty (i.e., improved or unimproved) will suffice as replacement property.

d. The earliest date that Samuel can acquire another hotel is May 2, 2013, the date of the threat or imminence of requisition or condemnation of the property. e. The latest date that Samuel can acquire another hotel is September 30, 2016 (three years after the close of the tax year in which the proceeds received are large enough to produce a realized gain). f. The latest date that Samuel can acquire another hotel is September 30, 2015 (two years after the close of the tax year in which the proceeds received are large enough to produce a realized gain).

1911. Lucinda, a calendar year taxpayer, owned a rental property with an adjusted basis of $312,000 in a major coastal city. Her property was condemned by the city government on October 12, 2013. In order to build a convention center, Lucinda eventually received qualified replacement property from the city government on March 9, 2014. This new property has a fair market value of $410,000. a. What is Lucinda’s recognized gain or loss on the condemnation? b. What is her adjusted basis for the new property? c. If, instead of receiving qualifying replacement property, Lucinda was paid $410,000, what is the latest date that she can acquire qualifying replacement property?

Correct Answer: a. Because the conversion of Lucinda’s original property was directly into qualified replacement property, the nonrecognition of the realized gain of $98,000 ($410,000 amount realized – $312,000 adjusted basis) is mandatory. b. Due to the mandatory nonrecognition, the basis in the replacement property is a carryover basis of $312,000 ($410,000 – $98,000). c. The latest date that Lucinda can acquire qualifying replacement property is December 31, 2017 (three years after the close of the tax year in which the proceeds received are large enough to produce a realized gain).

1912. Patty’s factory building, which has an adjusted basis of $475,000, is destroyed by fire on April 8, 2013. Insurance proceeds of $500,000 are received on June 1, 2013. She has a new factory building constructed for $490,000, which she occupies on October 1, 2013. Assuming Patty’s objective is to minimize the tax liability, calculate her recognized gain or loss and the basis of the new factory building.

Correct Answer: Amount realized $500,000 Adjusted basis of building (475,000) Realized gain $ 25,00 0

Amount realized $500,000 Less: Reinvestment (490,000) Deficiency $ 10,00 0

Since Patty’s objective is to minimize the tax liability, she would elect § 1033 postponement treatment. Thus, her recognized gain would be $10,000. The basis of the new factory building would be $475,000 ($490,000 cost – $15,000 postponed gain).

1913. Evelyn’s office building is destroyed by fire on July 12, 2013. The adjusted basis is $315,000. She receives insurance proceeds of $350,000 on August 31, 2013. Calculate the amount that Evelyn must reinvest in qualifying property in order that her recognized gain be $20,000. Assume she elects § 1033 (nonrecognition of gain from an involuntary conversion) postponement treatment.

Correct Answer: Amount realized $350,000 Adjusted basis (315,000) Realized gain $ 35,000

Required reinvestment $350,000 Less: Deficiency (recognized gain) (20,000) Actual reinvestment $330,000

1914. Carlos, who is single, sells his personal residence on November 5, 2013, for $400,000. His adjusted basis was $125,000. He pays realtor’s commissions of $20,000. He owned and occupied the residence for 12 years. Having decided that he no longer wants the burdens of home ownership, he invests the sales proceeds in a mutual fund and enters into a 1-year lease on an apartment. The detriments of renting, including a crying child next door, cause Carlos to rethink his decision. Therefore, he purchases another residence on November 6, 2014, for $275,000. Is Carlos eligible for exclusion of gain treatment under § 121 (exclusion of gain on sale of principal residence)? Calculate Carlos’s recognized gain and his basis for the new residence.

Correct Answer: Carlos is eligible for § 121 exclusion treatment. At the date of the sale of his residence, he owned and occupied it as his principal residence for at least two years during the 5-year period ending on the date of sale.

Amount realized ($400,000 – $380,000 $20,000) Adjusted basis (125,000) Realized gain $255,000 § 121 exclusion (250,000) Recognized gain $ 5,000

Whether Carlos replaces his principal residence is not relevant in determining his qualification for the § 121 exclusion. His basis for his new residence is the cost of $275,000.

1915. On January 5, 2013, Waldo sells his principal residence with an adjusted basis of $270,000 for $690,000. He has owned and occupied the residence for 15 years. He pays $35,000 in commissions and $2,000 in legal fees in connection with the sale. One month before the sale, Waldo painted the exterior of the house at a cost of $5,000 and repaired various items at a cost of $3,000. On October 15, 2013, Waldo purchases a new home for $600,000. On November 15, 2014, he pays $25,000 for completion of a new room on the house, and on January 14, 2015, he pays $15,000 for the construction of a pool. What is the Waldo’s recognized gain on the sale of his old principal residence and what is the basis for the new residence?

Correct Answer: Amount realized ($690,000 – $35,000 – $653,000 $2,000) Adjusted basis (270,000) Realized gain $383,000 § 121 exclusion (250,000) Recognized gain $133,000

The $5,000 for painting and $3,000 for repairs are personal expenses that do not decrease the amount realized or increase the adjusted basis.

The adjusted basis of the new residence is calculated as follows:

Cost of house $600,000 Cost of new room 25,000 Cost of pool 15,00 0 Adjusted basis $640,000

1916. Katrina, age 58, rented (as a tenant) the house that was her principal residence from January 1, 2013 through December 31, 2014. She purchased the house on January 1, 2015, for $150,000 and continued to occupy it through June 30, 2016. She leased it to a tenant from July 1, 2016, through December 31, 2017. On January 1, 2018, she sells the house for $350,000. She incurs a realtor’s commission of $20,000. Calculate her recognized gain if her objective is to minimize the recognition of gain and she does not intend to acquire another residence.

Correct Answer: To qualify for § 121 exclusion treatment on the sale of a principal residence, Katrina must meet the following requirement:

Owned and used the residence as her principal residence for at least two years during the five-year period ending on the date of sale.

Katrina meets this requirement. The ownership and use requirements do not have to be the same period of time. She resided in the house from January 1, 2013 through June 30, 2016 (a period of three and one-half years). She owned it from January 1, 2015 through December 31, 2017 (a period of three years). It is not necessary that the property be Katrina’s principal residence at the date of the sale.

Her recognized gain is calculated as follows:

Amount realized ($350,000 – $330,000 $20,000) Adjusted basis (150,000) Realized gain $180,000 § 121 exclusion (180,000) Recognized gain $ –0–

1917. Use the following data to determine the sales price of Etta’s principal residence and the realized gain. She is not married. The sale of the old residence qualifies for the § 121 exclusion.

Selling expenses $ 45,000 Recognized gain 180,000 Cost of new residence 760,000 Adjusted basis of old residence 225,000 § 121 exclusion 250,000

Correct Answer: The sale of residence model can be used to calculate the sales price and the realized gain for Etta.

Amount realized: Sales price $700,000 Less: Selling (45,000) $655,000 expenses Adjusted (225,000) basis Realized $430,000 gain § 121 (250,000) exclusion Recognize $180,000 d gain

The adjusted basis of the new residence is its cost of $760,000 and has no effect on the prior calculations.

1918. Liz, age 55, sells her principal residence for $600,000. She purchased it twenty-two years ago for $175,000. Selling expenses are $30,000 and repair expenses to get the house in a marketable condition to sell are $15,000. Liz’s objective is to minimize the taxes she must pay associated with the sale. Calculate her recognized gain.

Correct Answer: Amount realized ($600,000 – $30,000) $570,000 Adjusted basis (175,000) Realized gain $395,000 § 121 exclusion (250,000) Recognized gain $145,000

The repair expenses of $15,000 do not affect the calculation.

1919. Mandy and Greta form Tan, Inc., by transferring the following assets to the corporation in exchange for 5,000 shares of stock each.

Mandy: Cash of $450,000 Greta: Land (worth $450,000; adjusted basis of $90,000).

How much gain must Tan recognize on the receipt of these assets?

Correct Answer: Tan has no recognized gain on the receipt of these assets. Section 1032 provides that a corporation does not recognize any gain or loss on the receipt of money or other property in exchange for its stock.

1920. Beth sells investment land (adjusted basis of $225,000) that she has owned for 6 years to her husband, Richard, for its fair market value of $195,000. a. Calculate Beth’s recognized gain or loss. b. Calculate Richard’s basis for the land. c. How would your answers in a. and b. change if Beth and Richard were not married (i.e., merely good friends)? d. Would the answer in a. and b. change if the selling price was $270,000?

Correct Answer: a. Amount realized $195,000 Adjusted basis (225,000) Realized loss ($ 30,000)

Recognized loss $ – 0–

Since Richard is Beth’s spouse, Beth’s realized loss of $30,000 is disallowed.

b. Because Richard is Beth’s spouse, he has a carryover basis in the land of $225,000 rather than the $195,000 he paid Ashley.

c. Beth’s recognized loss would be $30,000. Richard’s basis for the investment land would be $195,000.

d. The answers in a. and b. would be the same.

1921. After 5 years of marriage, Dave and Janet decided to get a divorce. As part of the divorce settlement, Janet transfers to Dave the house she purchased prior to their marriage. Janet’s adjusted basis for the house is $230,000 and the fair market value is $410,000 on the date of the transfer. What are the tax consequences to Janet and to Dave as a result of the transfer?

Correct Answer: Janet has a realized gain of $180,000 ($410,000 – $230,000). However, no gain is recognized as § 1041 provides that transfers of property between former spouses incident to divorce are nontaxable transactions.

Dave’s basis in the house is a carryover basis of $230,000 (i.e., the same as Janet’s adjusted basis).

1922. Walter owns stock in Target, Inc. (adjusted basis of $50,000) which he sells for $70,000 on March 21, 2013. On May 1, 2013, he uses the $70,000 to acquire stock in Lime, Inc., a specialized small business investment company. Calculate Walter’s recognized gain on the sale of the Target stock and his basis in the stock acquired.

Correct Answer: Amount realized $70,000 Adjusted basis (50,000) Realized gain $20,000

Recognized gain $ –0–

Under § 1044, Walter’s realized gain of $20,000 is postponed because he reinvested the sales proceeds in the stock of a specialized small business investment company within the 60-day period.

Walter’s basis for the stock acquired is calculated as follows:

Cost of new stock $70,000 Less: Postponed gain (20,000) Basis for new stock $50,000

1923. Discuss the logic for mandatory deferral of realized gain or loss for a § 1031 like-kind exchange.

Correct Answer: The property received is considered to be a continuation of the property exchanged (i.e., nothing of economic significance has occurred). Therefore, a realized gain or realized loss is not recognized, and the property received has a carryover basis and holding period.

1924. What requirements must be satisfied to receive nontaxable exchange treatment under § 1031?

Correct Answer: The following requirements must be satisfied to receive nontaxable exchange treatment under § 1031.

• The form of the transaction is an exchange. • Both the property transferred and the property received are held either for productive use in a trade or business or for investment. • The property is like-kind property.

1925. What kinds of property do not qualify under the like-kind provisions?

Correct Answer: The property exchanged may not qualify for § 1031 treatment for five reasons. First, the property involved in the exchange (i.e., transferred and received) must be business use or investment property. Thus, personal use property does not qualify. In addition, the types of property contained in the language of § 1031(a)(2) do not qualify as business use or investment property (i.e., inventory, partnership interests, stocks, bonds, notes, chooses in action, certificates of trust or beneficial interest, or other securities or evidences of indebtedness or interest). Second, one kind or class of property may not be exchanged for a different kind or class (i.e., real property for personal property or vice versa). Third, real property located in the United States exchanged for foreign real property (and vice versa) does not qualify as like-kind property. Fourth, livestock of different sexes do not qualify as like-kind property. Fifth, depreciable tangible personal property held for productive use in a business is like-kind property only if the exchanged properties are within the same general business asset class or the same product class.

1926. Can related parties take advantage of the like-kind exchange provisions?

Correct Answer: Yes. A special rule exists for related parties. In addition to satisfying the other requirements for like-kind exchange treatment, a holding period requirement must be met. The taxpayer and the related party must not dispose of the like-kind property received within the two-year period following the date of the exchange. If an early disposition does occur, the postponed gain is recognized as of the date of the early disposition.

1927. What requirements must be satisfied for a delayed swap to qualify for § 1031 like-kind exchange treatment?

Correct Answer: In a delayed exchange (nonsimultaneous), the transaction will qualify for like-kind exchange treatment if the following requirements are satisfied.

• Identification period. The property to be received must be identified within 45 days of the date when the old property was transferred. • Exchange period: The property to be received must be received by the earlier of the following: • Within 180 days of the date when the old property was transferred. • The due date (including extensions) for the tax return covering the period of the transfer.

1928. Discuss the relationship between the postponement of realized gain under § 1031 (like-kind exchanges) and the adjusted basis and holding period for the replacement property.

Correct Answer: Section 1031 results in the mandatory postponement of realized gain or realized loss on like-kind exchanges. Therefore, the basis for the replacement property is a carryover basis and the holding period is a carryover holding period.

1929. Discuss the relationship between realized gain and boot received in a § 1031 like-kind exchange.

Correct Answer: Realized gain serves as the ceiling on the amount of the gain that is recognized in a § 1031 like-kind exchange. If no boot is received, then none of the realized gain is recognized. If boot is received and its fair market value is less than the realized gain, then gain is recognized to the extent of the boot received. If boot is received and its fair market value is greater than the realized gain, then gain is recognized to the extent of the realized gain (i.e., full recognition occurs).

1930. Under what circumstance is there recognition of some or all of the realized gain associated with the giving of boot by the taxpayer in a like-kind exchange?

Correct Answer: Generally, the giving of boot by the taxpayer in a like-kind exchange does not trigger the recognition of realized gain (e.g., cash). However, if the basis of the boot is less than the fair market value of the boot, then gain is recognized to the extent of this excess (e.g., appreciated stock).

1931. What effect do the assumption of liabilities have on a § 1031 like-kind exchange?

Correct Answer: For the taxpayer who is transferring the liability, the liability increases the amount realized as it is treated as boot received. For the taxpayer who is assuming the liability, the liability increases this taxpayer’s adjusted basis for the property and is treated as boot given.

1932. Define an involuntary conversion.

Correct Answer: An involuntary conversion results from the destruction (complete or partial), theft, seizure, requisition, or condemnation, or sale or exchange under threat or imminence of requisition or condemnation of the taxpayer’s property.

1933. Discuss the treatment of realized gains from involuntary conversions.

Correct Answer: Realized gains from involuntary conversions are recognized unless the taxpayer elects postponement treatment under § 1033. In order to defer the realized gain, the taxpayer must reinvest an amount at least equal to the amount realized in qualifying property within the statutory time period. If there is an investment deficiency, realized gain is recognized to the extent of the deficiency. The ceiling on gain recognition is the realized gain. Note that for a direct conversion (i.e., into property), the deferral provision is mandatory (i.e., an election is not required).

1934. Discuss the treatment of losses from involuntary conversions.

Correct Answer: Business losses are § 1231 losses, personal casualty and theft losses are itemized deductions, and personal condemnation losses are not recognized.

1935. To be eligible to elect postponement of gain treatment for an involuntary conversion, what are the three tests for qualifying replacement property?

Correct Answer: The three tests for qualifying replacement property are as follows:

• Functional use test: This test applies to owners-users. The taxpayer’s use of the replacement property and the involuntary converted property must be the same. • Taxpayer use test: This test applies to owner-investors. The properties must be used by the taxpayer (owner-investor) in similar endeavors. • Like-kind test: This test applies when business real property or investment real property is condemned. In this case, the broader replacement rules for like-kind exchanges replace the narrower replacement rules discussed above.

1936. Edith’s manufacturing plant is destroyed by fire on the afternoon of November 3, 2013. The adjusted basis is $800,000. The insurance company offers a settlement of $700,000. After protracted negotiations, Edith receives $825,000 on June 9, 2014. Edith is a fiscal year taxpayer whose tax year ends on June 30th. What is the latest date that Edith can invest the proceeds in qualifying replacement property and elect to defer the gain under § 1033?

Correct Answer: Since the form of the involuntary conversion is a casualty, Edith has 2 years from the end of the tax year in which she received a proceeds inflow large enough to produce a realized gain. Since this occurred on June 9, 2014, she has until 2 years from June 30, 2014 (i.e., June 30, 2016).

1937. How does the replacement time period differ for the condemnation of real property used in a trade or business or held for investment when compared with that for other involuntary conversions?

Correct Answer: The plus two years is replaced with plus three years (i.e., an additional year to make the replacement is provided).

1938. Distinguish between a direct involuntary conversion and an indirect involuntary conversion.

Correct Answer: An involuntary conversion occurs when a taxpayer’s property is stolen, destroyed, or condemned. For a direct involuntary conversion, the taxpayer receives property rather than money. For an indirect involuntary conversion, the taxpayer receives money which can then be used to acquire replacement property.

1939. Louis owns a condominium in New Orleans which has been his principal residence for 12 years. He wants to be near Lake Ponchartrain since he enjoys water activities. Therefore, he sells the condominium. His original intent was to purchase a house in New Orleans near the lake. However, the cost of such properties far exceeded his sales proceeds. He was able to purchase a house on the lake in Covington, which is located across the causeway. He invested all of his sales proceeds in the Covington house. After two months of commuting over an hour to and from work each day, he decides to rent an efficiency apartment in New Orleans near his office. He spends the weekends and vacations at his home in Covington. a. Does Louis qualify for exclusion of gain under § 121? b. Does his Covington house qualify as his principal residence?

Correct Answer: a. Louis satisfies the § 121 exclusion requirement. He has owned and occupied the residence for at least two years during the 5-year period ending on the date of sale. b. No. The Covington home does not qualify as his principal residence. The principal residence is where the taxpayer lives most of the time. For Louis, this is the efficiency apartment in New Orleans.

1940. Byron, who lived in New Hampshire, acquired a personal residence ten years ago when he was 52 years old. During this period he has occupied the residence for only eight months (out of 12) each year due to winter vacations in Florida. Is Byron eligible for exclusion of gain under § 121?

Correct Answer: Yes, temporary absences such as vacations do not invalidate the requirement that the taxpayer use the house as a principal residence for at least two years during the five-year period ending on the date of the sale.

1941. Edward, age 52, leased a house for one year in Memphis with an option to buy as his personal residence. At the end of the lease, he purchased the house. He lived there for an additional 26 months before his employer transferred him to Tucson. Expecting to be in Tucson for 18 to 24 months, he rented the Memphis house for 18 months with an option to extend on a month to month basis for an additional 6 months. At the end of the 18-month period, Edward’s employer offered him a permanent position in Tucson as branch manager. The tenant who had been occupying Edward’s house in Memphis purchased it at the end of the 24- month extended lease period. Is Edward eligible to elect exclusion treatment under § 121?

Correct Answer: Yes. To qualify for § 121 exclusion treatment, Edward must have owned and used the residence as his principal residence for at least two years during the 5-year period ending on the date of sale. It is not necessary that the house be his principal residence at the date of sale. Edward has owned and occupied the residence for at least two years out of the 5-year period ending on the date of sale.

1942. Melissa, age 58, marries Arnold, age 50, on June 1, 2013. Melissa decides to sell her principal residence on August 1, 2013, which she has owned and occupied for the past 30 years. Arnold has never owned a house. However, while he was married to Kelly who died 6 months prior to his marriage to Melissa, Kelly used the § 121 election on the sale of her residence in January 2011 to reduce her realized gain from $123,000 to $0. Kelly used the sales proceeds to pay off Arnold’s gambling debts. Can Melissa elect the § 121 exclusion on the sale of her residence? What is the maximum § 121 exclusion available to Melissa and Arnold if they file a joint return?

Correct Answer: Melissa is eligible for a maximum § 121 exclusion of $250,000. Even if Melissa and Arnold file a joint return, the maximum § 121 exclusion still is $250,000. To increase the § 121 maximum exclusion amount from $250,000 to $500,000 on a joint return, Arnold would need to be eligible for the § 121 exclusion. He is ineligible on Melissa’s sale of her residence because he has not occupied the residence for at least two years.

1943. Under what circumstances may a partial § 121 exclusion be available even though the taxpayer has used the § 121 exclusion within the two-year period preceding the sale of the current residence?

Correct Answer: The relief provision which permits partial § 121 exclusion treatment is available in any of the following situations:

• Change in place of employment. • Health. • To the extent provided for in the Regulations, other unforeseen circumstances.

1944. Libby’s principal residence is destroyed by a tornado. She is single and her realized gain is $360,000. Is it possible for Libby’s recognized gain to be $0?

Correct Answer: Yes, it is possible for the Libby’s recognized gain to be $0. This can be achieved by using § 1033 in conjunction with § 121. To achieve the desired result, § 1033 postponement treatment needs to be elected and the taxpayer must invest in another principal residence with an amount at least equal to the amount realized reduced by the § 121 exclusion amount.

1945. Ramon sells land with an adjusted basis of $120,000 and a fair market value of $175,000 to Pauline, his wife, for $175,000. Discuss how the tax consequences would differ if Ramon and Pauline had never been married.

Correct Answer: Section 1041 provides that realized gains or losses on transfers of property between spouses are not recognized. Thus, none of Ramon’s realized gain of $55,000 ($175,000 amount realized – $120,000 adjusted basis) is recognized. Even though Pauline paid $175,000 for the land, her basis is a carryover basis of $120,000.

If Ramon and Pauline had never been married, Ramon’s realized gain of $55,000 ($175,000 amount realized – $120,000 adjusted basis) would be recognized. Pauline’s basis for the land would be her cost of $175,000.

1946. What types of exchanges of insurance contracts are eligible for nonrecognition treatment under § 1035?

Correct Answer: Exchanges of insurance contracts qualifying for nonrecognition treatment include the following:

• The exchange of life insurance contracts. • The exchange of a life insurance contract for an endowment or annuity contract. • The exchange of an endowment contract for another endowment contract that provides for regular payments beginning at a date not later than the date payments would have begun under the contract exchanged. • The exchange of an endowment contract for an annuity contract. • The exchange of annuity contracts.

1947. Define qualified small business stock under § 1045.

Correct Answer: Qualified small business stock is stock of a qualified small business that is acquired by the taxpayer at its original issue in exchange for money or other property (excluding stock) or as compensation for services. A qualified small business is a domestic corporation that satisfies the following requirements:

• The aggregate gross assets prior to the issuance of the small business stock do not exceed $50 million. • The aggregate gross assets immediately after the issuance of the small business stock do not exceed $50 million.

1948. The tax law requires that capital gains and losses be separated from other types of gains and losses because an alternative tax calculation may be used when taxable income includes net long-term capital gain.

*a. True b. False

1949. The tax law requires that capital gains and losses be separated from other types of gains and losses because there are limitations on the deduction of net capital losses.

*a. True b. False

1950. If a capital asset is sold at a gain, the holding period is important.

*a. True b. False

1951. An accrual basis taxpayer accepts a note receivable from a retail customer with a weak credit rating. The taxpayer immediately sells the note to a bank for less than the note’s stated value. The taxpayer has an ordinary loss.

*a. True b. False

1952. A business taxpayer sells depreciable business property with an adjusted basis of $40,000 for $32,000. The taxpayer held the property for more than a year. The taxpayer has an $8,000 capital loss.

a. True *b. False

1953. An individual taxpayer received a valuable painting from his uncle, a famous painter. The painter created the painting. After the taxpayer held the painting for two years, he sold it for a $400,000 gain. The gain is a long-term capital gain.

a. True *b. False

1954. Since the Code section that defines “capital asset” says what is not a capital asset, other Code sections have to help determine what is and what is not a capital gain or loss.

*a. True b. False

1955. Individuals who are not professional real estate developers may get capital gain treatment for sale of their real property if they engage only in limited development activities.

*a. True b. False

1956. The subdivision of real property into lots for resale when no substantial physical improvements have been made to the property never causes the gain from sale of the lots to be treated as ordinary income.

a. True *b. False

1957. A security that was purchased by an individual and qualifies as § 1244 stock becomes worthless. The taxpayer is single and the loss is $30,000. The loss is treated as an ordinary loss.

*a. True b. False

1958. For tax purposes, there is no original issue discount on a bond unless the bond is issued for less than its face value and the difference between the face value and the bond issue price is at least one-fourth of 1 percent of the redemption price at maturity multiplied by the number of years to maturity.

*a. True b. False

1959. If the holder of an option fails to exercise the option, the lapse of the option is considered a sale or exchange on the option expiration date.

*a. True b. False

1960. The only things that the grantee of an option may do with the option are exercise it or let it expire.

a. True *b. False

1961. When a patent is transferred, the most common forms of payment received by the transferor are a lump sum and/or periodic payment.

*a. True b. False

1962. A franchisor licenses its mode of business operation to a franchisee.

*a. True b. False

1963. A lease cancellation payment received by a lessee is generally treated as an exchange because the lease extinguished is usually a capital asset.

*a. True b. False

1964. Lease cancellation payments received by a lessor are always ordinary income because they are considered to be in lieu of rental payments.

*a. True b. False

1965. To compute the holding period, start counting on the day after the property was acquired and include the day of disposition.

*a. True b. False

1966. The holding period of property given up in a like-kind exchange includes the holding period of the asset received if the property that has been exchanged is a capital asset.

a. True *b. False

1967. Tom has owned 40 shares of Orange Corporation stock for five years. He sells the stock short for a total of $1,100. One month later, he closes the short sale by purchasing and delivering 40 shares of Orange Corporation stock for a total of $600. Tom has a $500 short-term capital gain.

*a. True b. False

1968. Short-term capital losses are netted against long-term capital gains and long-term capital losses are netted against short-term capital gains.

a. True *b. False

1969. Short-term capital gain is eligible for a special tax rate only when it exceeds long-term capital gain.

a. True *b. False

1970. A net short-term capital loss first offsets any 28% net long-term capital gain before it offsets either 25% net long-term capital gain or 0%/15%/20% net long-term capital gain.

*a. True b. False

1971. All collectibles short-term gain is subject to a potential alternative tax rate of 28%.

a. True *b. False

1972. An individual taxpayer with 2013 net short-term capital loss of $5,000 generally can deduct up to $3,000 for AGI and carry the balance forward to 2014.

*a. True b. False

1973. The tax law requires that capital gains and losses be separated from other types of gains and losses. Among the reasons for this treatment are:

a. Long-term capital gains may be taxed at a lower rate than ordinary gains. b. Capital losses that are short-term are not deductible. c. Net capital loss is deductible only up to $3,000 per year for individual taxpayers. *d. a. and c. e. None of the above.

1974. Recognized gains and losses from disposition of a capital asset may occur as a result of a:

a. Sale. b. Exchange. c. Casualty. d. Condemnation. *e. All of the above.

1975. The possible holding periods for capital assets include:

a. Short-term = held 14 months or less. b. Long-term = greater than six months. *c. Long-term = greater than 12 months. d. Short-term = greater than 12 months. e. None of the above.

1976. A business taxpayer sells inventory for $80,000. The adjusted basis of the property is $58,000 at the time of the sale and the inventory had been held more than one year. The taxpayer has:

a. No gain or loss. b. Sold a long-term capital asset. c. Sold a short-term capital asset. *d. An ordinary gain. e. None of the above.

1977. Stanley operates a restaurant as a sole proprietorship. Which of the following items are capital assets in the hands of Stanley?

a. The restaurant’s tables and chairs. b. A portable sound system used to play “theme music” for the restaurant. c. The restaurant building that is an asset of the sole proprietorship. *d. An interest-bearing savings account used to keep the restaurant’s excess cash. e. None of the above.

1978. Michael is in the business of creating posters (display art) for the movie industry. He creates a poster and sells it for a lump sum. He has:

a. Sold a capital asset. b. Sold an ordinary asset. c. No gain or loss. d. An ordinary gain. *e. b. and d.

1979. Ramon is in the business of buying and selling securities. Which of the following is a capital asset for Ramon?

a. The securities he buys and sells each day in the normal course of his business. *b. The securities he designates as held for investment at the end of the day of acquisition. c. The securities he holds more than 12 months. d. All the securities he owns. e. b., c., and d.

1980. Stella purchased vacant land in 2006 that she subdivided for resale as lots. All 10 of the lots were sold during 2013. The lots had a tax basis of $12,000 each and sold for $35,000 each. Stella made no substantial improvements to the lots. She acted as her own real estate broker; so there were no sales expenses for selling the lots. Which of the following statements is correct?

a. Stella must hold the lots for at least 10 years before she is eligible for the special capital gain treatment of § 1237. b. The $230,000 gain from the sale of the ten lots is all ordinary income. c. All of the $230,000 gain from the sale of the ten lots is long-term capital gain. d. To be eligible for the special capital gain treatment of § 1237, Stella must be a real estate dealer. *e. None of the above.

1981. A worthless security had a holding period of 6 months when it became worthless on December 10, 2013. The investor who had owned the security had a basis of $20,000 for it. Which of the following statements is correct?

a. The investor has a long-term capital loss of $20,000. *b. The investor has a short-term capital loss of $20,000. c. The investor has a nondeductible loss of $20,000. d. The investor has a short-term capital gain of $20,000. e. None of the above.

1982. Lana purchased for $1,410 a $2,000 bond when it was issued two years ago. Lana amortized $200 of the original issue discount and then sold the bond for $1,800. Which of the following statements is correct?

a. Lana has $10 of long-term capital loss. *b. Lana has $190 of long-term capital gain. c. Lana has no capital gain or loss. d. Lana has $190 of long-term capital loss. e. None of the above.

1983. On June 1, 2013, Brady purchased an option to buy 1,000 shares of General, Inc. at $40 per share. He purchased the option for $3,000. It was to remain in effect for five months. The market experienced a decline during the latter part of the year, so Brady decided to let the option lapse as of December 1, 2013. On his 2013 tax return, what should Brady report?

a. A $3,000 long-term capital loss. *b. A $3,000 short-term capital loss. c. A $3,000 § 1231 loss. d. A $3,000 ordinary loss. e. None of the above.

1984. Which of the following events causes the purchaser of an option to add the cost of the option to the basis of the property to which the option relates?

*a. The option is exercised. b. The option is sold. c. The option lapses. d. The option is rescinded. e. None of the above.

1985. Hiram is a computer engineer and, while unemployed, invents a switching device for computer networks. He patents the device, but does not reduce it to practice. Hiram has a zero tax basis for the patent. In consideration of $800,000 plus a $1 royalty per device sold, Hiram assigns the patent to a computer manufacturing company. Hiram assigned all substantial rights in the patent. Which of the following is correct?

a. Hiram automatically has long-term capital gain from the lump sum payment, but not from the royalty payments. b. Hiram automatically has long-term capital gain from the royalty payments, but not from the lump sum payment. *c. Hiram automatically has long-term capital gain from both the lump sum payment and the royalty payments. d. Hiram does not have automatic long-term capital gain from either the lump sum payment or the royalty payments. e. None of the above.

1986. Gold Company signs a 13-year franchise agreement with Silver. Silver retained significant powers, rights, and a continuing interest. Gold Company (the franchisee) makes noncontingent payments of $18,000 per year for the first four years of the franchise. Gold Company also pays a contingent fee of 2% of gross sales every month. Which of the following statements is correct?

a. Gold Company may deduct the $18,000 per year noncontingent payments in full as they are made. *b. Gold Company may deduct the monthly contingent fee as it is paid. c. Gold Company may deduct both the noncontingent annual fee and the contingent monthly fees as they are paid. d. Gold Company may not deduct either the noncontingent annual fee or the contingent monthly fees as they are paid. e. None of the above.

1987. A lessor is paid $45,000 by its commercial tenant as a lease cancellation fee. The tenant wanted to get out of its lease so it could move to a different building. The lessor had held the lease for three years before it was canceled. The lessor had a zero tax basis for the lease. The lessor has received:

*a. Ordinary income of $45,000. b. Long-term capital gain of $45,000. c. Short-term capital gain of $45,000. d. Neither gain nor loss. e. None of the above.

1988. Virgil was leasing an apartment from Marple, Inc. Marple paid Virgil $1,000 to cancel his lease and move out so that Marple could demolish the building. As a result:

*a. Virgil has a $1,000 capital gain. b. Virgil has a $1,000 capital loss. c. Marple has a $1,000 capital loss. d. Marple has a $1,000 capital gain. e. None of the above.

1989. On June 10, 2013, Ebon, Inc. acquired an office building as a result of a like-kind exchange. Ebon had given up a factory building that it had owned for 26 months as part of the like-kind exchange. Which of the statements below is correct?

a. The holding period of the factory building includes the holding period of the office building. b. The holding period of the office building starts on June 11, 2013. c. The holding period of the office building starts on June 10, 2013. *d. The holding period of the office building includes the holding period of the factory building. e. None of the above.

1990. Tan, Inc., sold a forklift on April 12, 2013, for $8,000 (its FMV) to its 100% shareholder, Ashley. Tan’s adjusted basis for the forklift was $12,000. Ashley’s holding period for the forklift:

a. Includes Tan’s holding period for the forklift. b. Begins on April 12, 2013. *c. Begins on April 13, 2013. d. Does not begin until Ashley sells the forklift. e. None of the above.

1991. Hank inherited Green stock from his mother when she died. The mother had a tax basis of $366,000 for the Green stock when she died and the Green stock was worth $437,000 at the date of her death. Which of the statements below is correct?

a. Hank’s holding period for the Green stock includes his mother’s holding period for the stock. b. Hank’s holding period for the Green stock does not include his mother’s holding period for the stock. c. Hank’s holding period for the Green stock is automatically long term. *d. b. and c. e. None of the above.

1992. Which of the following is correct concerning short sales of stock?

a. At the time the short sale is made, the taxpayer does not deliver to the purchaser the shares sold short. *b. At the time the short sale is made, the taxpayer delivers to the purchaser the shares sold short. c. At the time the short sale is made, the taxpayer may already own the shares sold short. d. At the time the short sale is made, the taxpayer always already owns the shares sold short. e. None of the above.

1993. Ryan has the following capital gains and losses for 2013: $6,000 STCL, $5,000 28% gain, $2,000 25% gain, and $6,000 0%/15%/20% gain. Which of the following is correct:

*a. The net capital gain is composed of $1,000 25% gain and $6,000 0%/15%/20% gain. b. The net capital gain is composed of $5,000 28% gain and $2,000 0%/15%/20% gain. c. The net capital gain is composed of $3,000 28% gain, $2,000 25% gain, and $2,000 0%/15%/20% gain. d. The net capital gain is composed of $1,000 28% gain and $6,000 0%/15%/20% gain. e. None of the above.

1994. In 2013, Mark has $18,000 short-term capital loss, $7,000 28% gain, and $6,000 0%/15%/20% gain. Which of the statements below is correct?

a. Mark has a $5,000 capital loss deduction. *b. Mark has a $3,000 capital loss deduction. c. Mark has a $13,000 net capital gain. d. Mark has a $5,000 net capital gain. e. Mark has a $18,000 net capital loss.

1995. In 2012, Jenny had a $12,000 net short-term capital loss and deducted $3,000 as a capital loss deduction. In 2013, Jenny has a $18,000 0%/15%/20% long-term capital gain and no other capital gain or loss transactions. Which of the statements below is correct?

a. Jenny has a 2013 $18,000 net capital gain. *b. Jenny has a 2013 $9,000 net capital gain. c. Jenny has a 2013 $9,000 net capital loss. d. Jenny has a 2013 $3,000 capital loss deduction. e. Jenny has a 2013 $9,000 capital loss deduction.

1996. In 2013, Satesh has $5,000 short-term capital loss, $13,000 0%/15%/20% long-term capital gain, and $7,000 qualified dividend income. Satesh is single and has other taxable income of $15,000. Which of the following statements is correct?

a. No more than $13,000 of Satesh’s taxable income is taxed at 0%. b. No more than $7,000 of Satesh’s taxable income is taxed at 0%. *c. No more than $15,000 of Satesh’s taxable income is taxed at 0%. d. None of Satesh’s taxable income is taxed at 0%. e. All of Satesh’s taxable income is taxed at 0%.

1997. Cason is filing as single and has 2013 taxable income of $36,000 which includes $34,000 of 0%/15%/20% net long-term capital gain. What is his tax on taxable income using the alternative tax method?

a. $0. *b. $200. c. $4,954. d. $300. e. None of the above.

1998. Sara is filing as head of household and has 2013 taxable income of $57,000 which includes $3,000 of net long-tem capital gain. The net long-term capital gain is made up of $1,000 25% gain and $2,000 0%/15%/20% gain. What is the tax on her taxable income using the alternative tax method?

a. $0. b. $8,453. c. $8,753. *d. $8,553. e. None of the above.

1999. Seamus had $16,000 of net short-term capital loss in 2012. In 2013, Seamus has $17,000 of long-term capital loss and $26,000 of long- term capital gain. Which of the following statements is correct?

a. Seamus had a $13,000 short-term capital loss carryover to 2013. b. Seamus has an $9,000 2013 net long-term capital gain. c. Seamus has a $4,000 2013 net short-term capital loss. *d. a. and c. e. None of the above.

2000. In 2013, an individual taxpayer has $863,000 of taxable income that includes $48,000 of 0%/15%/20% long-term capital gain. Which of the following statements is correct?

a. All of the LTCG will be taxed at 0%. b. All of the LTCG will be taxed at 15%. *c. All of the LTCG will be taxed at 20%. d. Some of the LTCG will be taxed at 15% and some at 20%. e. None of the above.

2001. Martha has both long-term and short-term 2012 capital gains and losses. The result of netting these gains and losses is a net long-term capital loss. Martha has no qualified dividend income. Also, Martha’s 2012 taxable income puts her in the 28% tax bracket. Which of the following is correct?

a. Martha will use Parts I, II, and III of 2012 Form 1040 Schedule D. b. Martha will not benefit from the special treatment for long- term capital gains. c. Martha will have a capital loss deduction. *d. All of the above. e. None of the above.

2002. Which of the following comparisons is correct?

*a. Corporations may carryback capital losses; individuals may not. b. Both corporation and individual long-term capital losses carryover as short-term capital losses. c. Corporations may carryforward capital losses indefinitely; individuals may only carryforward capital losses for five years. d. Both corporations and individuals may use an alternative tax rate on net capital gains. e. None of the above.

2003. Robin Corporation has ordinary income from operations of $30,000, net long-term capital gain of $10,000, and net short-term capital loss of $15,000. What is the taxable income for 2013?

a. $25,000. b. $27,000. c. $28,500. *d. $30,000. e. None of the above.

2004. Violet, Inc., has a 2013 $80,000 long-term capital gain included in its $285,000 taxable income. Which of the following is correct?

a. Violet will benefit from an alternative tax on net capital gains computation. *b. Violet’s regular tax on taxable income will be the same as its tax using an alternative tax on net capital gains approach. c. Violet’s $80,000 net capital gain is not taxable. d. Violet’s regular tax on taxable income will be greater than its tax using an alternative tax on net capital gain approach. e. None of the above.

2005. Theresa and Oliver, married filing jointly, and both over 65 years of age, have no dependents. Their 2013 income tax facts are:

Theresa’s wages $165,000 Oliver’s wages 33,000 Short-term capital gain 36,000 Long-term capital loss (41,000)

What is their taxable income for 2013?

Correct Answer: The couple’s taxable income is $172,600. Their long-term capital loss carryover is $2,000 ($5,000 – $3,000).

Wages ($165,000 + $33,000) $198,000 Short-term capital gain $36,000 Long-term capital loss (41,00 0) Net long-term capital loss ($ 5,00 0) Capital loss deduction (3,0 (limited to $3,000) 00) Adjusted gross income $195,000 Standard deduction (12,200) Additional standard (2,400) deduction (2 × $1,200) Exemptions (2 × $3,900) (7,8 00) Taxable income $172,600

2006. Carol had the following transactions during 2013: a painting held for two years and sold at a gain of $85,000; 100 shares of Gray stock held six months and sold for a loss of $6,000; 50 shares of Yellow stock held 18 months and sold for a gain of $36,000. Carol also had $264,000 of taxable income from other sources than these property transactions. What is Carol’s net capital gain or loss and what is her taxable income?

Correct Answer: Carol has taxable income of $379,000.

Long-term capital gain from $ 85,000 painting Long-term capital gain from 36,000 Yellow stock Net long-term capital gain $121,000 Short-term capital loss from Gray (6,000 stock ) Net long-term capital gain $115,000 Other taxable income 264,000 Total taxable income $379,000

2007. On January 10, 2013, Wally sold an option for $2,000 on vacant land he held as an investment. He had purchased the land in 2009 for $76,000. The option allowed the option holder to purchase the property for $122,000 plus the cost of the option. On March 1, 2013, the option holder exercised the option. What is the amount and nature of Wally’s gain or loss from disposition of the land?

Correct Answer: Wally’s proceeds from selling the land are $124,000 ($2,000 option proceeds + $122,000 sale proceeds). Wally’s gain is $48,000 ($124,000 – $76,000) and is all long-term capital gain because the asset was a capital asset held more than 12 months.

2008. Willie is the owner of vacant land that he purchased in 2009 for $1,400,000 and held for investment. On January 22, 2012, he was paid $145,000 for a thirteen-month option on the land by Susan. Susan could buy the land for an additional $1,200,000 by exercising the option. Susan had hoped to build a luxury home on the land, but was unable to get approval to build a big enough home to satisfy her needs. Consequently, Susan did not exercise her option and the option expired on February 22, 2013. (1) What is Willie’s basis, gain or loss, and type of gain or loss from these events? (2) What is Susan’s basis, gain or loss, and type of gain or loss from these events?

Correct Answer: (1) Willie held the land for investment; consequently it was a capital asset. Willie had no recognized gain or loss in 2012 from the receipt of the $145,000 option proceeds. When the option expired in 2013, the $145,000 option price is ordinary income because the option property was not stocks, securities, commodities, or commodity futures. The basis of the property remains $1,400,000.

(2) The tax status of an option is determined by what would be the tax status of the property to be acquired with the option. The land would have been used in a personal use activity and, therefore, would have been a capital asset. Consequently, the option on the land was a capital asset. The lapse of the option is considered a sale or exchange on the option expiration date. However, the $145,000 loss on the lapse of the option is not usable because losses on sale or exchange of personal use activity property are not deductible.

2009. Larry was the holder of a patent on a video game. During 2013, he sold all substantial rights in the patent for $365,000 in cash and a 3% royalty on the purchaser’s first $10,200,000 of sales each year related to the product in which the patent is incorporated. Larry had not reduced the patent to practice. He had a $86,000 basis for the patent. During 2013, he received $30,000 in royalties. What is the nature and amount of Larry’s gain?

Correct Answer: Larry was the holder of a patent and transferred all substantial rights to it. Consequently, § 1235 grants automatic long-term capital gain treatment to both the cash received and the royalties received. Larry recovers his $86,000 basis and has a $309,000 ($365,000 + $30,000 – $86,000) 0%/15%/20% long-term capital gain.

2010. Phil’s father died on January 10, 2013. The father had owned stock for 20 years with a basis of $45,000 that was transferred to Phil as a gift on August 10, 2012, when the stock was worth $430,000. His father paid gift tax of $31,000. This stock was worth $566,000 at the date of the father’s death. Phil sold the stock for $545,000 net of commissions on February 23, 2013. What is the amount and nature of Phil’s gain or loss from disposition of this property?

Correct Answer: Phil had a tax basis for the stock equal to its $45,000 basis at the date of his father’s gift of the stock increased by the gift tax paid on the stock’s unrealized appreciation in his father’s hands. He also had a long-term holding period because the father’s 20-year holding period is added to Phil’s holding period. Consequently, he had a $471,379 ($545,000 – $73,621) long-term capital gain when he sold the stock.

2011. On January 18, 2012, Martha purchased 200 shares of Blue Corporation stock for $2,000. On November 11, 2013, she sold short 200 shares of Blue Corporation stock which she borrowed from her broker for $2,300. On February 10, 2014, Martha closed the short sale by delivering the 200 shares of Blue Corporation stock which she had acquired in 2012. On that date, Blue Corporation stock had a market price of $4 per share. What is Martha’s recognized gain or loss and its character in 2013? In 2014?

Correct Answer: Since Martha owned substantially identical stock on the date of the short sale and did not close the short sale before January 31, 2014, she is deemed to have closed the short sale on November 11, 2013 (the date of the short sale). On her 2013 tax return, she would report a $300 long-term capital gain ($2,300 short sale price – $2,000 cost). On February 10, 2014, Martha has a $1,500 short-term capital loss [$2,300 basis for the shares sold – $800 (200 shares × $4 per share)] because the holding period of the shares used to close the short sale commences with the date of the short sale.

2012. Sharon has the following results of netting her short-term and long-term capital gains and losses for 2013: $56,000 short-term capital loss, and $82,000 net long-term capital gain ($21,000 0%/15%/20% long-term capital gain, and $61,000 25% long-term capital gain).

(a) What is her net capital gain or loss for 2013?

(b) If there is a net capital loss, how much of the loss and what type of loss carries over to 2014?

(c) If there is a net long-term capital gain, what is it made up of?

Correct Answer: (a) Sharon has a 2013 net long-term capital gain of $26,000 ($82,000 net long-term capital gain – $56,000 net short-term capital loss).

(b) There is no net capital loss.

(c) The $56,000 short-term capital loss first absorbs the $61,000 of 25% gain, leaving $5,000 of 25% long-term capital gain and $21,000 of 0%/15%/20% long-term capital gain.

2013. The chart below details Sheen’s 2011, 2012, and 2013 stock transactions. What is the capital loss carryover to 2013 and what is the net capital gain or loss for 2013?

Short-term Short-term Long-term Long-term Tax Capital Capital Capital Capital Year Gains Losses Gains Losses 2011 $ 4,000 $ 6,000 $ 2,000 $13,000 2012 $16,000 $14,000 $23,000 $28,000 2013 $55,000 $52,000 $67,000 $33,000

Correct Answer: There was a $2,000 net short-term capital loss and a $11,000 net long- term capital loss in 2011. All of the short-term capital loss and $1,000 of the long-term capital loss were used for the $3,000 capital loss deduction and $10,000 of long-term capital loss carried forward to 2012. The $10,000 long-term capital loss carryforward is grouped with the 2012 long-term losses. In 2012, there is $2,000 net short-term capital gain that is netted against the $15,000 ($23,000 long-term gain – $28,000 2012 long-term loss – $10,000 long-term loss carryforward) net long-term capital loss, resulting in a $13,000 net long-term capital loss for 2012. After the $3,000 capital loss deduction, $10,000 is carried forward as long-term capital loss to 2013. In 2013, the $10,000 is added to the long-term capital losses. There is a 2013 net short-term capital gain of $3,000 and a net long-term gain of $24,000 ($67,000 long-term gain – $33,000 2013 long-term loss – $10,000 long- term loss carryforward). The net result for 2013 is both net short-term capital gain of $3,000 and long-term capital gain of $24,000.

2014. Samuel, head of household with two dependents, has 2013 wages of $26,000, paid alimony of $3,000, has taxable interest income of $2,000, and a $12,000 0%/15%/20% net long-term capital gain. Samuel uses the standard deduction and is age 38. What is his 2013 taxable income and the tax on the taxable income?

Correct Answer: Samuel has $16,350 taxable income and the tax on that taxable income using the alternative tax on net long-term capital gain is $435.

Wages $26,000 Interest income 2,000 Net long-term capital gain 12,000 Gross income $40,000 Alimony paid (3,000) Adjusted gross income $37,000 Standard deduction (head of household) (8,950 ) Exemptions (3 × $3,900) (11,700) Taxable income $16,350

Tax on regular taxable income ($16,350 $435 – $12,000) Alternative tax on $12,000 net long- –0– term capital gain @ 0% Total tax $435

The regular tax liability on $16,350 taxable income would have been $1,815.

2015. Martha is single with one dependent and files as head of household. She had 2013 taxable income of $45,000 which included $16,000 of 0%/15%/20% net long-term capital gain. What is her tax on taxable income using the alternative tax on net long-term capital gain method?

Correct Answer: Martha has a tax of $3,713. Her tax on regular taxable income of $29,000 ($45,000 – $16,000) is $3,713. Her tax on the $16,000 of 0%/15%/20% at 0% is $0 (she is not out of the 15% regular tax bracket, so she pays 0% on the entire net long-term capital gain of $16,000). Martha would have paid tax of $6,113 on her taxable income if it did not include any net long-term capital gain.

2016. Harold is a head of household, has $27,000 of taxable income in 2013 from non-capital gain or loss sources, and has the following capital gains and losses:

28% long-term capital gain $ 4,300 28% long-term capital loss (2,000) 0%/15%/20% long-term capital 19,000 gain Short-term capital loss (1,700)

What is Harold’s taxable income and the tax on that taxable income?

Correct Answer: Harold has taxable income of $46,600 and the tax (as a head of household) on that taxable income is $3,430.

28% long-term capital $ 4,300 gain 28% long-term capital (2,000) loss Net 28% long-term capital $ 2,300 gain Net short-term capital (1,700) loss Remaining 28% long-term $ 600 gain 0%/15%/20% long-term 19,000 capital gain Net long-term capital $19,600 gain Other taxable income 27,000 Taxable income $46,600

Tax on $27,000 regular taxable income $3,413 Tax on 28% gain portion of net long- term capital gain; Harold is still in 90 15% bracket ($600 × 15%) Tax on 0%/15%/20% net long-term capital gain; Harold is still in 15% – bracket 0– [($46,600 – $27,000 – $600) × 0%] Total tax using the alternative tax $3,503

The regular tax liability on the $46,600 of taxable income would have been $6,353.

2017. Mike is a self-employed TV technician. He is usually paid as soon as he completes repairs, but occasionally bills a customer with payment expected within 30 days. At the end of the year he has $2,500 of receivables outstanding. He expects to collect $1,200 of this and write off the remainder. Mike is a cash basis taxpayer and had net earnings from his business (not including the effect of the items above) of $55,000. He also had $3,500 interest income, $200 gambling winnings, and sold corporate stock for $7,000. The stock had been purchased in 2010 for $8,200. Mike is single, has no dependents, and claims the standard deduction. What is his 2013 taxable income? (Ignore the self- employment tax deduction.)

Correct Answer: Since Mike is a cash basis taxpayer, he may not deduct bad debts from accounts receivable because he would not have included the accounts receivable in his gross income.

Net business income $55,000 Interest income 3,500 Gambling winnings 200 Long-term capital loss ($7,000 – (1,200) $8,200) Adjusted gross income $57,500 Standard deduction (6,100) Personal exemption (3,900) Taxable income $47,500

2018. Judith (now 37 years old) owns a collection of porcelain dolls that she acquired when she was a grade schooler. She had forgotten about them until her mother sent them to her. Her mother had discovered them in a box in her attic while she was cleaning out her house before selling it. Judith had originally acquired all the dolls as gifts from her parents, so she has no way to establish a basis for the dolls. Using information from the Internet, she prepares a careful inventory of the dolls that includes their name, when they were first available for sale, their current value, and other pertinent information. She then lists them for sale on the Internet. To her surprise, she quickly gets an offer of $5,000 for all of them and sells them. Judith has no other gain or loss transactions for the year and is in the 28% marginal tax bracket. What issues do these facts create?

Correct Answer: Judith has to determine the holding period, tax status, basis, gain or loss from the disposition of the dolls and, if they are sold at a gain, the tax rate applicable to the gain. At the time of the sale, it appears that Judith is holding the dolls as an investment and, therefore, they are a capital asset. Her original intent was to hold the dolls as a personal use activity. However, when she discovered what they were worth, her intent seems to have become investment. She has no determinable basis for the dolls, so their basis is zero. They have been held long-term, so the $5,000 gain is a long-term capital gain. The alternative tax rate applicable to the gain is 15% because Judith’s regular taxable income puts her above the 15% regular tax bracket. The gain is not subject to the collectibles 28% alternative tax rate because the dolls are neither a work of art nor an antique.

2019. Hilda lent $2,000 to a close personal friend to help the friend avoid overdrawing the friend’s checking account. The friend was supposed to repay the $2,000 within a month. Instead, the friend declared personal bankruptcy and Hilda will never recover any of the $2,000. What are the tax implications of these events for Hilda?

Correct Answer: Assuming Hilda is not in the trade or business of lending money, the loan was a capital asset for Hilda. Her basis was $2,000 and her loss is $2,000. This is a nonbusiness bad debt and the tax law treats this as a short-term capital loss.

2020. Ranja acquires $200,000 face value corporate bonds for $186,000 when the bonds are issued. He holds the bonds as an investment for two years and then sells them for $198,000. He amortizes $2,000 of the OID. What tax issues does Ranja have with respect to these bonds?

Correct Answer: The bonds have original issue discount of $14,000 ($200,000 – $186,000). Ranja must amortize this discount while he holds the bonds. The discount amortization of $2,000 increases Ranja’s basis for the bonds. Consequently, when he sells the bonds, his basis is $186,000 plus the $2,000 discount amortization, so his long-term capital gain is $10,000 ($198,000 – $188,000).

2021. In early 2012, Wanda paid $33,000 for an option on a parcel of land she intended to hold as an investment. After a survey of the land (paid for by the grantor) determined that the parcel was much smaller than the grantor said it was, she let the option lapse when it expired in 2013 after 14 months. How should Wanda treat these events in 2012? 2013?

Correct Answer: If an option holder (grantee) fails to exercise the option, the lapse of the option is considered a sale or exchange on the option expiration date. Thus, the loss is a capital loss if the property subject to the option is (or would be) a capital asset in the hands of the grantee. Wanda has no gain or loss in 2012 because the option had not yet expired. Wanda has a $33,000 long-term capital loss in 2013 when the option expires because the land would have been a capital asset if Wanda had exercised the option.

2022. Jambo invented a new flexible cover for a popular brand of cellphone, but did not have the finances to begin production of the cover. Instead, he sold all his rights to the invention (after patenting it) for $450,000 plus $.10 for each cover sold by the company that purchased the patent. Jambo had a zero tax basis for the invention. What is the character of his gain from disposition of the patent?

Correct Answer: Jambo is the holder of a patent because he is an individual and either created the patented invention or the invention was not yet reduced to practice. Therefore, the $450,000 received for the patent and each $.10 when (and if) he receives it are treated as long-term capital gain automatically under § 1235.

2023. “Collectibles” held long-term and sold at a gain are subject to maximum tax rate of 28%. An individual taxpayer recently sold an antique car for $40,000. The car had been held for several years and $30,000 was originally paid for it. Explain why the car is or is not a collectible.

Correct Answer: The definition of “collectibles” is quite ambiguous. Consequently, the antique car is a collectible if it is a work of art or an antique. Also, the § 408(m) regulations add “historical object” to the list of collectibles. The car fits these categories, so it is a collectible for tax purposes.

2024. When an individual taxpayer has a net long-term capital gain that includes both 28% gain and 0%/15%/20% gain, which of these gains will be taxed first when the alternative tax on net long-term capital gain method is used and what difference does it make?

Correct Answer: The 28% gain is taxed after the regular taxable income is taxed, after the 25% gain is taxed, and before the 0%/15%/20% gain is taxed. Taxing the 28% gain first may mean that some or all of the 0%/15%/20% gain will not be eligible for the 0% tax if the taxpayer’s taxable income after taxing the regular taxable income, the 25% gain, and the 28% gain puts the taxpayer out of the regular 15% bracket.

2025. Section 1231 applies to the sale or exchange of business properties, but not to personal use activity casualties.

*a. True b. False

2026. Rental use depreciable machinery held more than 12 months is an example of a § 1231 asset.

*a. True b. False

2027. If there is a net § 1231 loss, it is treated as an ordinary loss.

*a. True b. False

2028. Section 1231 property includes nonpersonal use property where casualty gains exceed casualty losses for the taxable year.

*a. True b. False

2029. Section 1231 property generally includes certain purchased intangible assets (such as patents and goodwill) that are eligible for amortization and held for more than one year.

*a. True b. False

2030. Section 1231 property generally does not include artistic compositions.

*a. True b. False

2031. Section 1231 property generally does not include accounts receivables arising in the ordinary course of business.

*a. True b. False

2032. An individual business taxpayer owns land on which he grows trees for logging. The land has been held more than 10 years and the trees growing on the land were planted eight years ago. Normally, the timber would be inventory for this taxpayer, but the tax law allows the taxpayer to elect to treat cutting the timber as the disposition of a § 1231 asset.

*a. True b. False

2033. A sheep must be held more than 18 months to qualify as a § 1231 asset.

a. True *b. False

2034. Casualty gains and losses from nonpersonal use assets are not netted against casualty gains and losses from personal use assets.

*a. True b. False

2035. If § 1231 asset casualty gains and losses net to a gain, the gain is treated as a § 1231 gain.

*a. True b. False

2036. Involuntary conversion gains may be deferred if the proceeds of the involuntary conversion are reinvested.

*a. True b. False

2037. Personal use property casualty gains and losses are not subject to the § 1231 rules.

*a. True b. False

2038. In the “General Procedure for § 1231 Computation: Step 2. § 1231 Netting,” if the gains exceed the losses, the net gain is offset by the “lookback” nonrecaptured § 1231 losses.

*a. True b. False

2039. Nonrecaptured § 1231 losses from the six prior tax years may cause current year net § 1231 gain to be treated as ordinary income.

a. True *b. False

2040. A personal use property casualty loss is generally deductible only to the extent it exceeds 10% of AGI.

*a. True b. False

2041. The Code contains two major depreciation recapture provisions—§§ 1245 and 1250.

*a. True b. False

2042. The maximum § 1245 depreciation recapture generally equals the accumulated depreciation.

*a. True b. False

2043. Section 1245 may apply to amortizable § 197 intangible assets.

*a. True b. False

2044. For § 1245 recapture to apply, accelerated depreciation must have been taken on the property.

a. True *b. False

2045. Section 1250 depreciation recapture will apply when accelerated depreciation was used on property used outside the United States and the property is sold at a gain.

*a. True b. False

2046. The maximum amount of the unrecaptured § 1250 gain (25% gain) is the depreciation taken on real property sold at a recognized gain.

*a. True b. False

2047. Section 1231 lookback losses may convert some or all of § 1245 gain into ordinary income.

a. True *b. False

2048. Section 1245 depreciation recapture potential does not carryover from the deceased taxpayer to the beneficiary taxpayer.

*a. True b. False

2049. The § 1245 depreciation recapture potential does not reduce the amount of the charitable contribution deduction under § 170.

a. True *b. False

2050. Property sold to a related party purchaser that is depreciable by the purchaser may cause the seller to have ordinary gain.

*a. True b. False

2051. Depreciation recapture under § 1245 and § 1250 is reported on Form 4797.

*a. True b. False

2052. Part III of Form 4797 is used to report gains from the sale of depreciable business equipment sold at a gain and held more than one year.

*a. True b. False

2053. White Company acquires a new machine for $75,000 and uses it in White’s manufacturing operations. A few months after White places the machine in service, it discovers that the machine is not suitable for White’s business. White had fully expensed the machine in the year of acquisition using § 179. White sells the machine for $60,000 in the tax year after it was acquired, but held the machine only for a total of 10 months. What was the tax status of the machine when it was disposed of and the amount of the gain or loss?

a. A capital asset and $60,000 gain. *b. An ordinary asset and $60,000 gain. c. A § 1231 asset and $60,000 gain. d. A § 1231 asset and $60,000 loss. e. None of the above.

2054. Which of the following assets held by a manufacturing business is a § 1231 asset?

a. Inventory. b. Office furniture used in the business and held less than one year. *c. A factory building used in the business and held more than one year. d. Accounts receivable. e. All of the above.

2055. Which of the following assets held by a cash basis accounting firm is a § 1231 asset?

a. An account receivable from a client. b. A desk used in the business and held more than one year. c. An investment in Orange Company common stock. d. A computer used in the business, held more than one year, but fully depreciated under § 179 when acquired. *e. b. and d.

2056. A barn held more than one year and used in a business is destroyed in a tornado. The barn originally cost $356,000 and was fully depreciated using straight-line depreciation. The barn was insured for its $543,000 replacement cost minus a deductible of $1,000. Which of the statements below is correct concerning these facts?

a. The barn was a long-term personal use asset. b. There is a casualty loss from disposition of the barn. c. The recognized gain from disposition of the barn is $186,000. *d. The recognized gain from disposition of the barn is subject to special netting rules. e. c. and d.

2057. Which of the following would be included in the netting of § 1231 gains and losses?

a. Personal use property net casualty gain. b. Section 1231 loss. c. Section 1231 gain. d. All of the above. *e. b. and c.

2058. Vertigo, Inc., has a 2013 net § 1231 loss of $64,000 and had a $32,000 net § 1231 gain in 2012. For 2013, Vertigo’s net § 1231 loss is treated as:

*a. Ordinary loss. b. Ordinary gain. c. Capital loss. d. Capital gain. e. None of the above.

2059. Vertical, Inc., has a 2013 net § 1231 gain of $67,000 and had a $22,000 net § 1231 loss in 2012. For 2013, Vertical’s net § 1231 gain is treated as:

a. $45,000 long-term capital gain and $22,000 ordinary loss. b. $67,000 ordinary gain. *c. $45,000 long-term capital gain and $22,000 ordinary gain. d. $67,000 capital gain. e. None of the above.

2060. Verway, Inc., has a 2013 net § 1231 gain of $55,000 and had a $62,000 net § 1231 loss in 2012. For 2013, Verway’s net § 1231 gain is treated as:

a. $55,000 ordinary loss. *b. $55,000 ordinary gain. c. $55,000 capital loss. d. $55,000 capital gain. e. None of the above.

2061. The following assets in Jack’s business were sold in 2013:

Asset Holding Period Gain/(Loss) Office Equipment 6 years $1,100 Automobile 8 months ($ 800) ABC Stock 2 years $1,400 (capital asset)

The office equipment had a zero adjusted basis and was purchased for $8,000. The automobile was purchased for $2,000 and sold for $1,200. The ABC stock was purchased for $1,800 and sold for $3,200. In 2013 (the year of sale), Jack should report what amount of net capital gain and net ordinary income?

a. $1,700 LTCG. b. $600 LTCG and $300 ordinary gain. *c. $1,400 LTCG and $300 ordinary gain. d. $2,500 LTCG and $800 ordinary loss. e. None of the above.

2062. An individual had the following gains and losses during 2013 on property held for the long-term holding period: sale of Orange common stock ($8,000 gain); sale of real property used in the taxpayer’s business ($1,800 loss); destruction of real property used in the taxpayer’s business by fire ($1,000 loss). Which of the following statements is correct?

a. The fire loss would reduce the real property sale loss. b. The fire loss would reduce the stock sale gain. c. The sale of real property loss would be netted against the stock sale gain. *d. The sale of real property is a § 1231 loss. e. None of the above.

2063. Which of the following is correct?

a. Improperly classifying a § 1231 loss as a capital loss might affect adjusted gross income. b. Improperly classifying a capital loss as a § 1231 loss might affect adjusted gross income. c. Misclassifying a § 1231 gain as a short-term capital gain might affect adjusted gross income. d. Misclassifying a short-term capital gain as a § 1231 gain might affect adjusted gross income. *e. All of the above.

2064. Spencer has an investment in two parcels of vacant land. Parcel 1 is a capital asset and parcel 2 is a § 1231 asset. Spencer already has short-term capital loss for the year he would like to offset with capital gain. Spencer has § 1231 lookback loss that exceeds the gain from the disposition of either land parcel. Spencer only wants to sell one land parcel and each of them would yield the same amount of gain. The gain that would be recognized exceeds the short-term capital loss Spencer already has. Which of the statements below is correct?

a. Spencer will have a net capital loss no matter which land parcel he sells. *b. Spencer will have a net capital loss if he sells parcel 2. c. Spencer will have a net capital loss if he sells parcel 1. d. Spencer will have a net capital gain if he sells either parcel 1 or parcel 2. e. None of the above.

2065. Copper Corporation sold machinery for $47,000 on December 31, 2013. The machinery had been purchased on January 2, 2010, for $60,000 and had an adjusted basis of $41,000 at the date of the sale. For 2013, what should Copper Corporation report?

*a. Ordinary income of $6,000. b. A § 1231 gain of $3,000 and $3,000 of ordinary income. c. A § 1231 gain of $6,000. d. A § 1231 gain of $6,000 and $3,000 of ordinary income. e. None of the above.

2066. Which of the following creates potential § 1245 depreciation recapture and potential § 1231 gain?

*a. Depreciable office furniture held more than one year and sold for more than its original cost. b. Amortizable goodwill held more than one year and disposed of for less than its adjusted basis. c. Land held more than one year and sold for more than was paid for it. d. A note receivable held more than one year and sold for less than was paid for it. e. None of the above.

2067. Blue Company sold machinery for $45,000 on December 23, 2013. The machinery had been acquired on April 1, 2011, for $69,000 and its adjusted basis was $34,200. The § 1231 gain, § 1245 recapture gain, and § 1231 loss from this transaction are:

*a. $0 § 1231 gain, $10,800 § 1245 recapture gain, $0 § 1231 loss. b. $0 § 1231 gain, $0 § 1245 recapture gain, $14,800 § 1231 loss. c. $0 § 1231 gain, $34,200 § 1245 recapture gain, $0 § 1231 loss. d. $0 § 1231 gain, $10,800 § 1245 recapture gain, $34,200 § 1231 loss. e. None of the above.

2068. Red Company had an involuntary conversion on December 23, 2013. The machinery had been acquired on April 1, 2011, for $49,000 and its adjusted basis was $14,200. The machinery was completely destroyed by fire and Red received $10,000 of insurance proceeds for the machine and did not replace it. This was Red’s only casualty or theft event for the year. As a result of this event, Red initially has:

a. $10,000 § 1231 loss. b. $10,000 § 1245 recapture gain. *c. $4,200 casualty loss. d. $4,200 § 1231 loss. e. None of the above.

2069. Orange Company had machinery destroyed by a fire on December 23, 2013. The machinery had been acquired on April 1, 2011, for $49,000 and its adjusted basis was $14,200. The machinery was completely destroyed and Orange received $30,000 of insurance proceeds for the machine and did not replace it. This was Orange’s only casualty or theft event for the year. As a result of this event, Orange has:

a. $4,200 ordinary loss. *b. $15,800 § 1245 recapture gain. c. $14,200 § 1245 recapture gain. d. $30,000 § 1231 gain. e. None of the above.

2070. Which of the following events could result in § 1250 depreciation recapture?

a. Sale at a loss of a depreciable business building held more than one year. b. Sale at a gain of a business building held more than a year on which straight-line depreciation was taken. c. Sale at a loss of a depreciable business building held for 9 months. d. Sale at a gain of depreciable equipment held more than a year on which straight-line depreciation was taken. *e. None of the above.

2071. Which of the following real property could be subject to § 1250 depreciation recapture?

a. Property placed in service after 1986 on which straight-line depreciation was taken. *b. A building on which § 168(k) depreciation was taken. c. Equipment on which accelerated depreciation was taken. d. Land which was not depreciated. e. a. and b.

2072. Assume a building is subject to § 1250 depreciation recapture because § 168(k) was used to depreciate it. The building is destroyed in a hurricane and this is the taxpayer’s only casualty or theft for the year. In which of the following situations could there be a § 1250 depreciation recapture gain?

a. There is a loss because the insurance recovery is less than the adjusted basis. *b. There is a gain because the insurance recovery exceeds the adjusted basis. c. Because of the length of time the building has been held, there is no remaining additional depreciation. d. There is no insurance recovery and the adjusted basis of the building is greater than zero. e. None of the above.

2073. Lynne owns depreciable residential rental real estate which has accumulated depreciation (all from straight-line) of $65,000. If Lynne sold the property, she would have a $53,000 gain. The initial characterization of the gain would be:

a. Section 1245 gain. *b. Section 1231 gain. c. Section 1250 gain. d. Section 1239 gain. e. None of the above.

2074. A retail building used in the business of a sole proprietor is sold on March 10, 2013, for $342,000. The building was acquired in 2003 for $400,000 and straight-line depreciation of $104,000 had been taken on the building. What is the maximum unrecaptured § 1250 gain from the disposition of this building?

a. $400,000. b. $322,000. *c. $104,000. d. $26,000. e. None of the above.

2075. Which of the following statements is correct?

a. When depreciable property is gifted to another individual taxpayer, the depreciation recapture potential is extinguished. *b. When depreciable property is inherited by a taxpayer, the depreciation recapture potential is extinguished. c. When corporate depreciable property is distributed as a dividend, the depreciation recapture potential is generally not recognized. d. When depreciable property is contributed to charity, the depreciation recapture potential has no effect on the amount of the charitable contribution deduction. e. All of the above are correct.

2076. Which of the following would extinguish the § 1245 recapture potential?

a. An exchange of depreciable business equipment for like-kind business equipment with gain realized, but not recognized. b. A nontaxable incorporation under § 351. c. A nontaxable contribution to a partnership under § 721. d. A nontaxable reorganization. *e. None of the above.

2077. Section 1239 (relating to the sale of certain property between related taxpayers) does not apply unless the property:

a. Was depreciated by the transferor. *b. Is depreciable in the hands of the transferee. c. Is a capital asset. d. Is real property. e. None of the above.

2078. An individual has a $40,000 § 1245 gain, a $35,000 § 1231 gain, a $33,000 § 1231 loss, a $3,000 § 1231 lookback loss, and a $15,000 long- term capital gain. The net long-term capital gain is:

a. $30,000. b. $40,000. c. $17,000. *d. $15,000. e. None of the above.

2079. An individual has the following recognized gains and losses from disposition of § 1231 assets (all the assets were vacant land): $15,000 gain, $10,000 loss, $25,000 gain, and $2,000 loss. The individual has a $5,500 § 1231 lookback loss. The individual also has a $16,000 net short-term capital loss from the disposition of stock. Which of the following statements is correct?

*a. The taxpayer has $5,500 ordinary gain and $6,500 net long- term capital gain. b. The taxpayer has $12,000 net long-term capital gain. c. The taxpayer has $28,000 ordinary gain and $16,000 net short- term capital loss. d. The taxpayer has $5,500 ordinary loss and $6,500 net long-term capital gain. e. None of the above.

2080. Section 1231 gain that is treated as long-term capital gain carries from the 2012 Form 4797 to the 2012 Form 1040, Schedule D, line ____.

a. 8. b. 9. c. 10. *d. 11. e. None of the above.

2081. Business equipment is purchased on March 10, 2012, used in the business until September 29, 2012, and sold at a $23,000 loss on October 10, 2012. The equipment was not suitable for the work the business had purchased it for. The loss on the disposition should have been reported in the 2012 Form 4797, Part:

a. I. *b. II. c. III. d. IV. e. This transaction would not be reported in the Form 4797.

2082. A business taxpayer sold all the depreciable assets of the business, calculated the gains and losses, and would like to know the final character of those gains and losses. The taxpayer had $353,000 of adjusted gross income before considering the gains and losses from sale of the business assets. The taxpayer had unrecaptured § 1231 lookback loss of $22,000. What is the treatment of the gains and losses summarized in the chart below after all possible netting and reclassification has been completed? What is the taxpayer’s adjusted gross income? (Ignore the self-employment tax deduction.)

Asset Purchase Sale Date Depreciation Gain Date (Loss) Machine 10/10/11 11/11/13 $323,000 $66,000 #1 Machine 10/02/10 11/11/13 65,000 (15,0 #2 00) Machine 09/23/09 11/11/13 183,000 23,00 #3 0 Machine 09/23/09 11/11/13 28,000 34,00 #4 0

Correct Answer: The taxpayer has adjusted gross income of $461,000 after including the effect of the property transactions. Machine #1’s $66,000 gain is all ordinary income due to § 1245 depreciation recapture. Machine #3’s $23,000 gain is all ordinary income due to § 1245 depreciation recapture. Machine #4 has $28,000 of ordinary income due to § 1245 depreciation recapture (equals depreciation taken) and $6,000 § 1231 gain ($34,000 – $28,000). Machine #2’s $15,000 loss is a § 1231 loss. There is a $9,000 net § 1231 loss ($6,000 gain – $15,000 loss) for the year. The net ordinary gain for the year is $108,000 ($66,000 + $23,000 + $28,000 – $9,000). There is no net § 1231 gain, so the $22,000 § 1231 unrecaptured lookback loss does not affect the character of the current year’s gains. Adjusted gross income is $461,000 ($353,000 + $108,000).

2083. A business taxpayer sold all the depreciable assets of the business, calculated the gains and losses, and would like to know the final character of those gains and losses. The taxpayer had $353,000 of adjusted gross income before considering the gains and losses from sale of the business assets. The taxpayer had unrecaptured § 1231 lookback loss of $12,000. What is the treatment of the gains and losses summarized in the chart below after all possible netting and reclassification has been completed? What is the taxpayer’s adjusted gross income? (Ignore the self-employment tax deduction.)

Asset Purchase Sale Date Depreciation Gain Date (Loss) Machine 10/10/11 11/11/13 $323,000 $66,000 #1 Machine 10/02/10 11/11/13 65,000 (15,0 #2 00) Machine 09/23/09 11/11/13 183,000 23,00 #3 0 Machine 09/23/09 11/11/13 28,000 64,00 #4 0

Correct Answer: The taxpayer has adjusted gross income of $491,000 after including the effect of the property transactions. Machine #1’s $66,000 gain is all ordinary income due to § 1245 depreciation recapture. Machine #3’s $23,000 gain is all ordinary income due to § 1245 depreciation recapture. Machine #4 has $28,000 of ordinary income due to § 1245 depreciation recapture (equals depreciation taken) and $36,000 § 1231 gain ($64,000 – $28,000). Machine #2’s $15,000 loss is a § 1231 loss. There is a $21,000 net § 1231 gain ($36,000 gain – $15,000 loss) for the year. The $12,000 § 1231 unrecaptured lookback loss converts $12,000 of this gain to ordinary income, leaving $9,000 of the net § 1231 gain to be treated as long-term capital gain. The net ordinary gain for the year is $129,000 ($66,000 + $23,000 + $28,000 + $12,000). Adjusted gross income is $491,000 ($353,000 + $129,000 + $9,000).

2084. A business machine purchased April 10, 2011, for $98,000 was fully depreciated in 2011 using § 179 immediate expensing. On August 15, 2013, the machine was sold for $67,000. What is the amount and nature of the gain or loss from disposition of the machine?

Correct Answer: The machine was a § 1231 asset because it was held for more than 12 months. However, all of the $67,000 ($67,000 sales price – $0 adjusted basis) gain is ordinary gain due to § 1245 depreciation recapture.

2085. An individual taxpayer has the gains and losses shown below. There are $3,000 of § 1231 lookback losses. What is the net long-term capital gain?

Holding Period/Property Character of Amount Gain or Loss 5 years/vacant land § 1231 gain $7,000 2 years/business § 1245 gain 3,200 equipment 3 years/publicly traded Long-term 890 stock capital gain 8 months/publicly traded Short-term (1,870) stock capital loss

Correct Answer: The taxpayer has a net long-term capital gain of $4,890 and a net short-term capital loss of $1,870. The $3,200 of § 1245 gain is ordinary income and does not affect the net long-term capital gain computation. Since there is $3,000 of § 1231 lookback loss, $3,000 of the $7,000 § 1231 gain is treated as ordinary income and the remaining $4,000 of § 1231 gain is treated as long-term capital gain. The $1,870 of short-term capital loss offsets the $4,890 of long-term capital gain, resulting is a net capital gain of $3,020 (0%/15%/20% gain).

2086. Vanna owned an office building that had been held more than one year when it was sold for $567,000. The real estate had an adjusted basis of $45,000 for the land and $233,000 for the building. Straight- line depreciation of $162,000 had been taken on the building. What is the amount and initial character of the gain or loss from disposition of the real estate? Is any of the gain unrecaptured § 1250 (25%) gain?

Correct Answer: The real estate was used in business and held more than one year. Therefore, the property was a § 1231 asset. Since straight-line depreciation was taken, there is no § 1250 depreciation recapture because no accelerated depreciation was taken. The entire gain of $289,000 [$567,000 sale price – ($45,000 land adjusted basis + $233,000 building adjusted basis)] is § 1231 gain. Since the recognized gain is greater than the $162,000 of depreciation, there is $162,000 of unrecaptured § 1250 gain in the $289,000 recognized gain.

2087. The chart below describes the § 1231 assets sold by the Ecru Company (a sole proprietorship) this year. Compute the gain or loss from each asset disposition and determine the net § 1231 gain treated as long-term capital gain for the year. Assume there is a § 1231 lookback loss of $4,000.

Asset Acquired Sold CostDepreciation Sale Price Stamping 3/10/0 8/10/20 $40, $29,736 $32,0 machine 9 13 000 00 Factory 2/12/0 7/23/20 80 18,838 90, building 6 13 ,000 000 Tractor 5/16/011/13/201 52 52,000 30, 8 3 ,000 000 Overhead 11/12/02 2/25/20 74 74,000 18, crane 13 ,000 000

Correct Answer: The stamping machine ($21,736), tractor ($30,000), and overhead crane ($18,000) are each sold at a gain and the gain is ordinary due to § 1245 depreciation recapture. The factory building yields a § 1231 gain of $28,838. There is no § 1250 depreciation recapture because straight- line depreciation was used (i.e., the building was placed in service after 1986). $4,000 of the $28,838 gain is treated as ordinary income because of the $4,000 § 1231 lookback loss. Consequently, the net § 1231 gain treated as long-term capital gain is $24,838 ($28,838 – $4,000). The chart below provides detail on the computations:

Sale Gain Asset Acquired Sold CostDepreciationBasisPrice(Loss) Stamping 3/10/ 8/10/20$40, $29,736 $10,2$32,0$21,73 machi 09 13 000 64 00 6 ne Factory 2/12/ 7/23/20 8 18,838 61,16 90 28, build 06 13 0,00 2 ,000 838 ing 0 Tractor 5/16/11/13/2013 5 52,000 –0– 30 30, 08 2,00 ,000 000 0 Overhead 11/12/02 2/25/20 7 74,000 –0– 18 18, crane 13 4,00 ,000 000 0

2088. The chart below describes the § 1231 assets sold by the Tan Company (a sole proprietorship) this year. Compute the gain or loss from each asset disposition and determine the net § 1231 gain treated as long-term capital gain for the year. Assume there is a § 1231 lookback loss of $14,000.

Asset Acquired Sold CostDepreciation Sale Price Stamping 3/10/0 8/10/20 $40, $29,736 $ 2, machine 9 13 000 000 Factory 2/12/0 7/23/20 80 18,838 90, building 6 13 ,000 000 Tractor 5/16/0 11/13/201 52 52,000 60, 8 3 ,000 000 Overhead 11/12/02 2/25/20 74 74,000 18, crane 13 ,000 000

Correct Answer: The stamping machine is sold at a $8,264 loss which is a § 1231 loss. The factory building yields a § 1231 gain of $28,838. There is no § 1250 depreciation recapture because straight-line depreciation was used (i.e., the building was placed in service after 1986). The tractor has $60,000 of gain, $52,000 of ordinary gain due to § 1245 depreciation recapture (equal to the deprecation taken) and $8,000 of § 1231 gain. The $18,000 gain on the overhead crane is ordinary due to § 1245 depreciation recapture. $14,574 of net § 1231 gain ($28,838 + $8,000 – $8,264 – $14,000 § 1231 lookback loss) is treated as long-term capital gain. The chart below provides details on the computations:

Asset Acquired Sold CostDeprBasis Sale Gain ecia Price(Loss) tion Stamping 3/10/ 8/10/20$40,$29,$10,2$ 2($ 8 machi 09 13 000 736 64 ,000 ,264) ne Factory 2/12/ 7/23/20 8 161,16 90 28, build 06 13 0,008,83 2 ,000 838 ing 0 8 Tractor 5/16/11/13/2013 5 5 60 60, 08 2,002,00 ,000 000 0 0 –0– Overhead 11/12/02 2/25/20 7 7 18 18, crane 13 4,004,00 ,000 000 0 0 –0–

2089. Residential real estate was purchased in 2010 for $345,000, held as rental property, and depreciated straight-line. Assume the land cost was $45,000 and the building cost was $300,000. Depreciation totaled $34,089. The building and land were sold on June 10, 2013, for $683,000 total. What is the tax status of the property, the nature of the gain from the disposition, and is any of it § 1250 depreciation recapture gain or unrecaptured § 1250 gain?

Correct Answer: The adjusted basis of the property at the date of sale is $310,911 ($345,000 cost – $34,089 depreciation). The asset is a § 1231 asset because it was depreciable property or real property used in business (rental is a form of business) and it was held more than one year. The recognized gain is $372,089 ($683,000 sale price – $310,911 adjusted basis) and it is all § 1231 gain since only straight-line depreciation was taken on the building. Thus, there is no § 1250 depreciation recapture because there was no additional depreciation due to accelerated depreciation. However, there is potential unrecaptured § 1250 gain of $34,089 because the depreciation taken is less than the recognized gain. The $338,000 ($372,089 – $34,089) balance of the gain is potential 0%/15%/20% long-term capital gain.

2090. Williams owned an office building (but not the land) that was destroyed by a fire. The building was insured and Williams has a $156,000 gain because his insurance recovery exceeded his adjusted basis for the building. Williams may replace the building. Williams had taken $145,000 of depreciation on the building, has no § 1231 lookback loss, has no other § 1231 transactions for the year, and has no Schedule D transactions for the year. What is the final nature of Jamison’s gain for the year and what tax rate(s) apply to the gain if:

(a) He does reinvest the insurance proceeds?

(b) If he doesn’t reinvest the insurance proceeds?

Correct Answer: (a) Williams initially has a casualty gain of $156,000 from business use property. If he reinvests the insurance proceeds, he will be able to postpone this gain.

(b) If he does not reinvest, he will have a recognized gain. Since he has a net casualty gain, the gain is treated as § 1231 gain and that gain is treated as a long-term capital gain because he has no § 1231 lookback loss. Williams has a net long- term capital gain of $156,000 because he has no other Schedule D transactions. The unrecaptured § 1250 portion of the gain is $145,000 (equal to the depreciation taken on the destroyed property). That portion of the gain is subject to an alternative tax rate of 25%. The $11,000 ($156,000 – $145,000) remaining gain is subject to the 0%/15%/20% alternative tax rate.

2091. A business machine purchased April 10, 2012, for $62,000 was fully depreciated in 2012 using § 179 immediate expensing. On August 15, 2013, the sole proprietor who owned the machine gave it to his son. On that date, the machine’s fair market value was $57,000. The son did not use the machine in business or hold it as inventory and the machine was sold on November 22, 2013, for $53,000.What is the amount and nature of the gain or loss from disposition of the machine? Where is it reported in the son’s tax return?

Correct Answer: A gift does not extinguish potential § 1245 depreciation recapture potential. The son that received the machine had a $0 basis for the asset because he has a carryover basis from the donor. The father’s holding period tacks to the son’s holding period; therefore, the son had a long-term holding period on the date of the gift and potential § 1245 depreciation recapture of $57,000 [the lesser of the depreciation taken ($62,000) or the realized gain at the date of the gift ($57,000)]. However, since the machine was sold for only $53,000, there is only $53,000 of § 1245 depreciation recapture gain. The son should complete Form 4797 Part III for this transaction and then carry the gain to Part II as ordinary income.

2092. Betty, a single taxpayer with no dependents, has the gains and losses shown below. Before considering these transactions, Betty has $45,000 of other taxable income. What is the treatment of the gains and losses and what is Betty’s taxable income?

§ 1245 gain #1 $18,000 § 1245 gain #2 5,000 Business equipment long-term (8,000) casualty loss Business real property long- 12,000 term casualty gain § 1231 gain 13,000 § 1231 lookback loss (2,000)

Correct Answer: The § 1245 recapture gains are combined and result in a $23,000 ordinary gain. The nonpersonal use property casualty gain and loss are combined and result in a $4,000 net gain. The net gain is treated as a § 1231 gain and when combined with the other $13,000 § 1231 gain results in a $17,000 net § 1231 gain. Due to the $2,000 § 1231 lookback loss, $2,000 of the net § 1231 gain is an ordinary gain and the $15,000 balance of the gain is treated as a long-term capital gain. Since this is the only capital gain or loss, there is a $15,000 net long-term capital gain.

Other taxable income $45,000 Ordinary gain due to recapture 23,000 Ordinary gain due to § 1231 2,000 lookback Net long-term capital gain 15,000 Taxable income $85,000

2093. In 2013 Angela, a single taxpayer with no dependents, disposed of for $44,000 a business building which cost $100,000. $60,000 of depreciation had been taken on the building. Angela has a short-term capital loss of $3,000 this year. She has taxable income (not related to property transactions) of $125,000. She has no § 1231 lookback loss. What is the amount and nature of the gain or loss, what is Angela’s taxable income, and what is her tax on the taxable income?

Correct Answer: The adjusted basis of the building is $40,000 ($100,000 cost – $60,000 depreciation). The building is sold for a gain of $4,000 ($44,000 sale price – $40,000 adjusted basis). Since the building was held more than one year, it is a § 1231 asset and the gain is a § 1231 gain. Angela has a $4,000 net § 1231 gain treated as a long-term capital gain. The gain is netted against her $3,000 short-term capital loss, resulting in a $1,000 net long-term capital gain. Since the other taxable income is $125,000, the taxable income after adding this gain is $126,000 ($125,000 + $1,000). The tax on her $125,000 regular taxable income is $28,293. All of the gain included in her taxable income is unrecaptured § 1250 gain because the depreciation on the building exceeded the gain included in her taxable income. Consequently, the tax on the $1,000 net long-term capital gain is $250 ($1,000 × .25). Her total tax is $28,543 ($28,293 + $250).

2094. Charmine, a single taxpayer with no dependents, has already incurred a $10,000 § 1231 gain in 2013 and has no § 1231 lookback losses. The taxpayer purchased a business machine for $100,000 five years ago, $70,000 of depreciation has been taken on it, and the machine is now worth $90,000. How will the net § 1231 gain or loss be affected if the taxpayer trades in the business machine for a like-kind business machine and pays an additional $12,000 in cash to obtain the replacement machine? If Charmine already has $322,000 of taxable income which does not include a $10,000 §1231 gain or any capital gains or losses, what is her taxable income?

Correct Answer: The current year § 1231 gain will not be affected because no gain or loss is recognized on the exchange of the machine. A like-kind exchange causes recognized gain only when “boot” is received. No boot was received in the exchange, so the potential § 1245 depreciation recapture of $70,000 carries over to the replacement machine. The taxable income is $322,000 + $10,000 = $332,000.

2095. Why is it generally better to have a net § 1231 gain year followed by a net § 1231 loss year rather than a net § 1231 loss year followed by a net § 1231 gain year?

Correct Answer: It is generally better to have a net § 1231 gain year followed by a net § 1231 loss year rather than a net § 1231 loss year followed by a net § 1231 gain year because the § 1231 lookback loss rules will be avoided. The net § 1231 gain in the first year is treated as a long- term capital gain and, therefore, potentially eligible for the reduced long-term capital gain rates. The second year net § 1231 loss is deductible for AGI as an ordinary deduction.

2096. Describe the circumstances in which the potential § 1245 depreciation recapture is extinguished.

Correct Answer: Section 1245 depreciation recapture potential is extinguished in at least two circumstances: (1) when the property with the depreciation recapture potential is sold at a loss and (2) when the owner of the property with the depreciation recapture potential dies.

2097. Describe the circumstances in which the maximum unrecaptured § 1250 gain (25% gain) does not become part of the Schedule D netting process for an individual taxpayer?

Correct Answer: Unrecaptured § 1250 gain (25% gain) is some or all of the § 1231 gain that is treated as long-term capital gain and relates to a sale of depreciable real estate. The maximum amount of this 25% gain is the depreciation taken on the real property sold at a gain. That maximum amount is reduced in one or more of the following ways:

• The gain recognized from disposition is less than the depreciation taken. The 25% gain is reduced to the gain amount. Refer to Example 13. The depreciation taken was $131,440, but the gain recognized was only $111,440. Consequently, all of the gain recognized is potential 25% § 1231 gain.

• There is § 1250 depreciation recapture because the property is residential real estate acquired in 1981-1986 on which accelerated depreciation was taken. The § 1250 recapture reduces the 25% gain. Refer to Example 13 again. Of the $111,440 recognized gain, $0 was recaptured by § 1250 as ordinary income, leaving $111,440 of potential 25% gain.

• There is § 1245 depreciation because the property is nonresidential real estate acquired in 1981-1986 on which accelerated depreciation was used. There will be no 25% gain left because § 1245 will recapture all the depreciation or the gain, whichever is less. Refer to Example 12. There was $100,000 of depreciation taken, but all of it was recaptured as ordinary income by § 1245. Thus, there is no remaining potential 25% gain. The entire $20,000 § 1231 gain in Example 12 is potential 0%/15%/20% gain.

• Section 1231 loss from disposition of other § 1231 assets held long-term reduced the gain from real estate. According to the IRS, § 1231 losses first absorb potential 0%/15%/20% § 1231 gain and then 25% § 1231 gain.

• Section 1231 lookback losses convert some or all of the 25% gain to ordinary income. According to the IRS, § 1231 lookback losses first absorb 28% net § 1231 gain, then 25% § 1231 gain, and then 0%/15%/20% § 1231 gain.

2098. Depreciable personal property was sold at a gain in 2012. On what 2012 form would this transaction be reported, where initially in that form, and what will the form most likely do with the gain?

Correct Answer: The transaction will initially be reported on Form 4797, Part III. In that Part, the gain recaptured by § 1245 will be determined. Most likely, all of the gain will be treated as an ordinary gain because the gain does not exceed the original cost of the property.

2099. A doctor’s incorporated medical practice may end the last day of any month of the year.

a. True *b. False

2100. A C corporation that does not have a natural business year must use a calendar year as its tax year.

a. True *b. False

2101. A C corporation’s selection of a tax year, generally, is independent of the tax year of its principal shareholders.

*a. True b. False

2102. The DEF Partnership had three equal partners when it was formed. Partners D and E were calendar year taxpayers and Partner F’s tax year ended on June 30th before he joined the partnership. The partnership may use a calendar year and partner F may continue to use the tax year ending June 30th.

*a. True b. False

2103. The tax year of one of the principal partners may determine the partnership’s tax year.

*a. True b. False

2104. The Seagull Partnership has three equal partners. Partner A’s tax year ends June 30th, and Partners B and C use a calendar year. If the partnership uses the calendar year to report its income, Partner A is permitted to defer partnership income earned from July through December 2013 until he files his tax return for his year ending June 30, 2014.

*a. True b. False

2105. Red Corporation and Green Corporation are equal partners in the R & G Partnership. Red Corporation’s tax year ends September 30th, and Green Corporation is a calendar year taxpayer. The greatest aggregate deferral of income would occur if the partnership used a calendar year for tax purposes.

*a. True b. False

2106. A CPA practice that is incorporated earns 40% of its annual revenues in the months of March and April. Although the CPA practice is a professional services corporation (PSC), it may use a fiscal year ending April 30th.

*a. True b. False

2107. The ability of the CPA to timely prepare a tax return is a justification for the partnership’s use of a particular tax year.

a. True *b. False

2108. In 2004, a medical doctor who incorporated his practice elected a fiscal year ending September 30th. During the fiscal year ended September 30, 2013, he received a salary of $190,000. During the period from October 1, 2013 to December 31, 2013, the corporation paid the doctor a total salary of $60,000, and paid him $240,000 of salary in the following 9 months. The corporation’s salary deduction for the fiscal year ending September 30, 2014, is limited to $240,000.

*a. True b. False

2109. Laura Corporation changed its tax year-end from July 31st to December 31st in 2013. The income for the period August 1, 2013 through December 31, 2013 was $35,000. The corporate tax rate is 15% on the first $50,000 of income, 25% on income from $50,001 to $75,000, and 34% on income from $75,001 to $100,000. A portion of Laura’s June – December 2013 income will be taxed at 34%.

*a. True b. False

2110. In 2013, T Corporation changed its tax year from ending each April 30th to ending each December 31st. The corporation earned $60,000 during the period May 1, 2013 through December 31, 2013. The annualized income for the short year is $90,000.

*a. True b. False

2111. Snow Corporation began business on May 1, 2013, and elected to use the calendar year for tax purposes. Brown Corporation, a calendar year corporation, sold all of its assets and liquidated as of April 30, 2013. Neither Snow Corporation nor Brown Corporation must annualize their income for their 2013 returns.

*a. True b. False

2112. Ted, a cash basis taxpayer, received a $150,000 bonus in 2013 when he was in the 35% marginal tax bracket. In 2014, when Ted was in the 28% marginal tax bracket, it was discovered that the bonus was incorrectly computed, and Ted was required to refund $40,000 to his employer. As a result of the refund, Ted can reduce his 2014 tax liability by $14,000 (.35 ´ $40,000).

*a. True b. False

2113. Generally, an advantage to using the cash method of accounting, as compared to the accrual method, is that under the cash method income is not recognized until it is collected, rather than being taxed as soon as the taxpayer has the right to collect the income.

*a. True b. False

2114. A calendar year, cash basis corporation began business on April 1, 2013, and paid $2,400 for a 24-month liability insurance policy. An accrual basis, calendar year taxpayer also began business on April 1, 2013, and purchased a 24-month liability insurance policy. Both the cash basis and accrual basis taxpayers’ deduction for insurance expense on the policy for 2013 is $900 (9/12 ´ $1,200).

a. True *b. False

2115. Alice, Inc., is an S corporation that has been in business for five years. Its annual gross receipts have never exceeded $1 million. The corporation operates a retail store and also owns rental property. The sales from the retail store and the rental income may be reported by the cash method, unless Alice previously elected the accrual method.

*a. True b. False

2116. A retailer must actually receive a claim for refund from the customer before a deduction can be taken for the refund.

*a. True b. False

2117. A C corporation provides lawn maintenance services to various businesses and homeowners. The corporation has average annual gross receipts of $3,500,000. The corporation may use the cash method of accounting.

*a. True b. False

2118. Franklin Company began business in 2009 and has consistently used the cash method to report income from the sale of inventory in income tax returns filed for 2009 through 2013. As a result of an audit by the IRS, Franklin was required to change to the accrual method of accounting beginning with 2014. The net adjustment due to the change is a positive adjustment to income. The adjustment may be spread equally over 2014 and the three following years.

a. True *b. False

2119. Sandstone, Inc., has consistently included some factory overhead as a current expense, rather than as a cost of producing goods. As a result, the beginning inventory for 2013 is understated by $40,000. If Sandstone voluntarily changes accounting methods effective January 1, 2013, the positive adjustment to the inventory is a § 481 adjustment and $10,000 must be added to taxable income for each year 2013, 2014, 2015, and 2016.

*a. True b. False

2120. A cash basis taxpayer sold investment land in 2013 for $200,000. He received $40,000 in the year of sale and $160,000 in 2014. The cost of the land was $80,000. Under the installment method, the taxpayer would report a $24,000 gain in 2013.

*a. True b. False

2121. In the case of a sale reported under the installment method, gain is recognized in each year the seller collects on the installment contract.

*a. True b. False

2122. If an installment sale contract does not charge interest on the sale of a capital asset, only capital gain will be recognized over the life of the contract.

a. True *b. False

2123. When an accrual basis taxpayer finances the construction of its building by borrowing, the interest is added to the cost of the building.

*a. True b. False

2124. In 2013, Cashmere Construction Company enters into a contract to build a beach cottage for Martha and Rob for a total price of $500,000. Cashmere estimates the total cost to complete the cottage to be $400,000. In 2013, Cashmere incurred $300,000 of costs on the contract, and in 2014 the contract was completed at a total cost of $425,000. Cashmere is not required to recognize any income from the contract until 2014.

*a. True b. False

2125. For a taxpayer who is required to use the percentage of completion method, the taxpayer can elect to defer the recognition of income and the related costs until the taxable year in which cumulative contract costs are at least 10 percent of the estimated contract costs.

*a. True b. False

2126. Blue Mart operates a large chain of retail stores. The company has four warehouses that are located in various parts of the country. The cost of operating the warehouses can be expensed immediately because it is a cost that is incurred even though the goods were not sold during the year.

a. True *b. False

2127. The lower of cost or market can be used in conjunction with both the FIFO and LIFO method.

a. True *b. False

2128. In applying the lower of cost or market for tax purposes, the market price is the replacement cost of the goods, rather than their expected selling price.

*a. True b. False

2129. The taxpayer does need the IRS’s permission to change from the FIFO inventory method to the LIFO method.

a. True *b. False

2130. Teal, Inc., used the lower of cost or market to value inventory in 2013. The ending inventory at cost was $400,000 and the ending inventory at market was $385,000. In 2014, Teal changed to the LIFO method. The company’s beginning LIFO inventory is $400,000.

*a. True b. False

2131. If a company uses the LIFO inventory method to report the cost of inventory and cost of goods sold on its financial statements, footnote disclosure of the income as calculated by the FIFO method does not violate the tax and financial accounting conformity requirement.

*a. True b. False

2132. The LIFO method is beneficial only when prices are rising and the taxpayer is increasing the quantities of inventory items on hand.

a. True *b. False

2133. Which of the following statements regarding a 52-53 week tax year is not correct?

a. Some tax years will include more than 366 calendar days. b. Whether the particular tax year includes 52 weeks or 53 weeks is not elective. c. The year-end must be the same day of the week in all years. *d. All of the above are correct. e. None of the above is correct.

2134. Gold Corporation, Silver Corporation, and Copper Corporation are equal partners in the GSC Partnership. The partners’ tax year-ends are as follows:

Gold December 31st Silver April 30th Copper September 30th

a. The partnership is free to elect any tax year. b. The partnership may use any of the 3 year-end dates that its partners use. *c. The partnership must use a September 30th year-end. d. The partnership must use a April 30th year-end. e. None of the above.

2135. Gold Corporation, Silver Corporation, and Platinum Corporation are equal partners in the GSP Partnership, which was formed on July 1, 2013. Gold and Silver use a calendar tax year, and Platinum’s tax year ends June 30th. GSP is not a seasonal business.

*a. GSP must use a tax year ending December 31st, and Platinum can retain its tax year ending June 30th. b. GSP must use a tax year ending June 30th, and the partners must change their tax years to end on June 30th. c. GSP must use a tax year ending December 31st and Platinum must change its tax year to December 31st. d. GSP may elect its tax year without regard to the partners’ tax years. e. None of the above.

2136. In regard to choosing a tax year for a business owned by individuals, which form of business provides the greater number of options in regard to the tax year?

a. A C corporation formed by medical doctors to conduct their practice. *b. A C corporation that is in the retail grocery business. c. A real estate partnership. d. An S corporation engaged in manufacturing. e. All of the above have the same options.

2137. Which of the following statements regarding a 52-53 week tax year is correct?

*a. The year-end must be the same day of the week in all years. b. The year cannot contain more than 366 calendar days. c. Every four years, there will be only 51 weeks. d. The year cannot end on a Sunday. e. None of the above.

2138. Purple Corporation, a personal service corporation (PSC), adopted a fiscal year ending September 30th. The sole shareholder of the corporation is a calendar year taxpayer. During the fiscal year ending September 30, 2013, the shareholder-employee received $120,000 salary. The corporation paid the shareholder-employee a salary of $15,000 during the period beginning October 1, 2013 through December 31, 2013.

a. The corporation salary expense for the fiscal year ending September 30, 2014 is limited to $120,000. b. The corporation salary expense for the fiscal year ending September 30, 2014 is limited to $135,000. *c. The corporation salary expense for the fiscal year ending September 30, 2014 is limited to $60,000. d. The corporation must switch to a calendar year. e. None of the above.

2139. A C corporation is required to annualize its income:

a. The first year the corporation is in existence, if the first tax return includes less than 12 months. b. The last year the corporation is in existence. *c. The year the corporation changes its tax year. d. When there has been a greater than 50% change in the ownership of the stock. e. All of the above.

2140. In 2013, Godfrey received a $50,000 sales commission on a long- term contract. But in 2014, the customer filed bankruptcy and Godfrey’s employer was not able to collect from the customer. Under the bonus agreement, Godfrey was required to repay the employer $20,000 of the bonus. Godfrey was in the 35% marginal tax bracket in 2013 but he is in the 25% marginal tax bracket in 2014.

a. Godfrey can amend his 2013 tax return and reduce his taxable income by $20,000. b. Godfrey should deduct the $20,000 paid in 2014 and thus his tax savings will be $5,000. *c. Godfrey can reduce his 2014 tax liability by 35% × $20,000 = $7,000. d. Godfrey should not have reported the income in 2013 because of the contingencies. e. None of the above.

2141. Which of the following taxpayers is required to use the accrual method of accounting?

a. A retail business with average annual gross receipts of $800,000. b. A medical doctor with average annual gross receipts of $2 million. c. An insurance agency with average annual gross receipts of $2 million. d. All of the above are required to use the accrual method. *e. None of the above is required to use the accrual method.

2142. Karen, an accrual basis taxpayer, sold goods in December 2013 for $20,000. The customer was unable to pay cash. So the customer gave Karen a note for $20,000 that was payable in April 2014. The note bore interest at the Federal rate. The fair market value of the note at the end of 2013 was $18,000. Karen collected $20,500 from the customer in April 2014, $20,000 principal plus $500 interest. Under the accrual method, Karen must recognize income of:

a. $20,500 in 2014. b. $18,000 in 2013 and $2,500 in 2014. *c. $20,000 in 2013 and $500 in 2014. d. $20,500 in 2014. e. None of the above.

2143. The accrual method generally is required to report income for which of the following types of businesses:

a. From long-term construction contracts. b. Earned by an incorporated public accounting firm with gross receipts in excess of $5 million. c. Earned by a partnership that has a partner that is an S corporation. d. A grocery store with average annual gross receipts of $800,000. *e. None of the above.

2144. Which of the following must use the accrual method of accounting?

I. An incorporated property management company with average annual gross receipts of $50 million. II. An incorporated law firm with average annual gross receipts of $6 million. III. An unincorporated grocery store with average annual gross receipts of $1,200,000.

a. All of the above must use the accrual method. b. None of the above must use the accrual method. c. Only I and II must use the accrual method. *d. Only I and III must use the accrual method. e. Only III must use the accrual method.

2145. Andrew owns 100% of the stock of Crow’s Farm Inc., an S corporation, that raises cattle and corn. The farm’s annual gross receipts have never exceeded $3 million and the farm is not considered a tax shelter.

a. The farm must report its sales and cost of goods sold by the accrual method because inventories are material to the business. *b. The income from the farm may be reported by the cash method. c. The income from the sales of cattle may be reported by the cash method, but the income from the sales of corn must be reported by the accrual method. d. The income from the sales of corn may be reported by the cash method, but the income from cattle sales must be reported by the accrual method. e. None of the above.

2146. In the case of an accrual basis taxpayer, an item of income:

a. Is not recognized until cash is received. b. From services is never recognized until the services are performed. c. Is not recognized if the customer can return the goods. *d. Is recognized when all the events have occurred to fix the taxpayer’s right to receive the income and the amount of the income can be determined with reasonable accuracy. e. None of the above.

2147. Ivory Fast Delivery Company, an accrual basis taxpayer, frequently has claims for damages to property the company delivered. Often the claim is not filed until a month after the delivery. In the past, approximately 80% of the claims are paid by Ivory. In 2013, claims for $80,000 were filed. The company refused to pay $20,000 of the claims (because they were not valid), and paid $50,000. The remaining $10,000 in claims were processed and paid in January 2014. Also, in January 2014, claims for $8,000 were filed for deliveries made in 2013, and $6,000 was paid on these claims by March 15, 2014. Ivory has not elected to use the recurring item exception to economic performance. Under the all-events and economic performance tests, Ivory can accrue as an expense for 2013:

a. $68,000. b. $66,000. c. $60,000. *d. $50,000. e. None of the above.

2148. Color, Inc., is an accrual basis taxpayer. In December 2013, the company received from a customer a $500 claim for defective merchandise. Color paid the customer in January 2014. Also, in December 2013, the company received a bill of $800 for office supplies that had been purchased and used in November 2013. The bill was not paid until January 2014. In January 2014, the company received a claim for $600 for defective merchandise purchased in 2013. Color paid the customer the $600 in February 2014. Assuming Color uses the recurring item exception to economic performance, the company’s deductions for 2013 as a result of the above are:

a. $500. b. $600. c. $800. *d. $1,300. e. $1,900.

2149. Pink Corporation is an accrual basis taxpayer that uses the recurring item exception to the economic performance test for all relevant years. For 2013, the corporation’s income subject to state income tax was $500,000 and the state corporate tax rate was 6%. During 2013, the corporation paid $24,000 on its estimated state income tax liability for that year. The remaining $6,000 of 2013 state income tax was paid in April 2014. In June 2013, the corporation paid $9,000 on its year 2012 state income tax liability, as a result of an audit of the 2012 return that was conducted in 2013. The company has elected to use the recurring item exception to economic performance. As a result of the above, the corporation should deduct in 2013 on its Federal income tax return state income taxes of:

a. $24,000. b. $30,000. c. $33,000. *d. $39,000. e. None of the above.

2150. Which of the following statements regarding the matching principle is correct?

a. Tax accounting strictly follows the matching principle. b. The matching principle of financial accounting is an important component of the cash method of accounting. *c. The matching principle of financial accounting is sometimes relevant to timing deductions for an accrual basis taxpayer’s recurring items. d. The matching principle has no relevance to tax accounting. e. None of the above.

2151. Gray Company, a calendar year taxpayer, allows customers to return defective merchandise for a full refund within 30 days of the purchase. In 2013, the company refunded $400,000 for claims involving sales. The $400,000 consisted of $350,000 in refunds from 2013 sales and $50,000 in refunds from 2012 sales. All of the refunds from 2012 sales were for claims filed in 2012 and were paid in January and February 2013. At the end of 2013, the company had $12,000 in refund claims for sales in 2013 for which payment had been approved. These claims were paid in January 2014. Also in January 2014, the company received an additional $30,000 in claims for sales in 2013. This $30,000 was paid by Gray in February 2014. With respect to the above, Gray can deduct:

a. $350,000 in 2013. *b. $362,000 in 2013. c. $392,000 in 2013. d. $442,000 in 2013. e. None of the above.

2152. Generally, deductions for additions to reserves for estimated future costs (e.g., an allowance for estimated warranty costs) are not allowed for Federal income tax purposes because allowing the deduction would:

a. Result in a mismatching of revenues and expenses. b. Violate established public policy. *c. Violate the economic performance requirement. d. Violate the tax benefit rule. e. None of the above.

2153. In 2013, Swan Company discovered that it had for the past 10 years capitalized as a production cost certain expenses that are properly classified as administrative expenses. The total amount of the expense for 2012 was $300,000, $60,000 of the item was included in the ending inventory that year and $240,000 was deducted as cost of goods sold.

a. The company should amend its 2012 tax return and reduce its income by $240,000. *b. The company should change its accounting method in 2013, with a $60,000 negative § 481 adjustment which decreases its 2013 taxable income. c. The company should change its accounting method in 2013, and increase its 2013 income by $60,000, the amount of the positive § 481 adjustment to income. d. The company should change its accounting method in 2013 and recognize a $60,000 negative § 481 adjustment that will be spread equally over 2013-16. e. None of the above.

2154. The taxpayer has consistently, but incorrectly, used an allowance for bad debts. At the beginning of the year, the balance in the allowance account is $90,000.

a. If the IRS examines the taxpayer’s return and requires the taxpayer to change accounting methods, the taxpayer will be required to recognize an additional $90,000 of income (one-half in the current year and one-half in the following year) as the adjustment due to the change in accounting methods. *b. If the taxpayer voluntarily changes methods, the $90,000 adjustment can be spread over the current and three following years. c. If the taxpayer voluntarily changes methods, the $90,000 reserve can be used to absorb bad debts until the account balance is zero. d. If the IRS examines the taxpayer’s return, no adjustment to the reserve account will be required if the balance is consistent with prior bad debt experience. e. None of the above.

2155. When the IRS requires a taxpayer to change accounting methods:

a. The taxpayer may be subject to penalties and interest. b. The taxpayer generally is required to make the change as of the beginning of the earliest open year. c. The adjustments due to the change cannot be spread over subsequent years. d. Only a. and b. are correct. *e. a., b., and c. are correct.

2156. The taxpayer had consistently used the cash method of accounting even though inventories were a material income-producing factor to its business. The taxpayer decided to voluntarily change to the accrual method of accounting. The adjustment to income due to the change was that the correct beginning balances for the year of the change as follows: $60,000 for inventories, $30,000 for accounts receivable, and $12,000 for accounts payable. The adjustment due to the change in accounting method is:

a. A positive adjustment for $102,000. b. A positive adjustment for $90,000. *c. A positive adjustment for $78,000. d. A positive adjustment for $60,000. e. None of the above.

2157. The taxpayer had incorrectly been using the cash method of accounting. For 2013, the company voluntarily changed to the accrual method. The adjustment due to the change in method as calculated at the beginning of 2013 was $120,000 (positive). The adjustment as calculated as of the end of 2013 was $80,000 (positive). As a result of the change in method, the company must:

a. Increase its income for 2013 by $120,000. b. Increase its income for 2013 by $80,000. *c. Increase its income for 2013 by $30,000. d. Increase its income for 2013 by $40,000. e. None of the above.

2158. The accrual basis taxpayer sold land for $100,000 on December 31, 2013. He did not collect the $100,000 until January 2, 2014. The land was held as an investment.

a. If the accrual basis taxpayer’s basis in the land was $110,000, the loss would be recognized in 2014. b. If the accrual basis taxpayer’s basis in the land was $60,000, the gain must be reported in 2013. *c. If the accrual basis taxpayer’s basis in the land was $60,000, the gain must be reported in 2014, unless the taxpayer elects to not use the installment method. d. The accrual basis taxpayer must recognize the gain or loss in the year of sale. e. None of the above.

2159. The installment method can be used for which of the following sales with payments being made in the year following the year of sale?

a. A department store’s credit card sales. *b. An individual’s sale of common stock in a family owned business. c. An individual’s sale of General Electric common. d. Depreciable equipment sold for less than its original cost. e. All of the above.

2160. The installment method applies where a payment will be received after the tax year of the sale:

*a. By an investor who sold real estate at a gain. b. By an investor who sold real estate at a loss. c. By an appliance dealer who sold inventory at a gain. d. By an investor who sold IBM Corporation common stock at a gain. e. None of the above.

2161. In 2013, Beth sold equipment used in her business. Her basis in the property was $300,000 ($500,000 cost less $200,000 of depreciation). Beth sold the property for $400,000, with $100,000 due on the date of the sale and $300,000 (plus interest at the Federal rate) due in 2014. Beth’s recognized installment sale gain in 2014 is:

a. $0. b. $50,000. *c. $100,000. d. $200,000. e. None of the above.

2162. Abby sold her unincorporated business which consisted of equipment and goodwill. The equipment had an original cost of $200,000 and Abby had claimed $120,000 in depreciation (adjusted basis = $80,000). Abby had no basis in the goodwill. The sales price for the business was $250,000, with $150,000 for the equipment and $100,000 for the goodwill. The buyer agreed to pay $120,000 on June 30, 2013, and $130,000 (plus interest at the Federal rate) in two years. Abby’s gain to be reported in 2013 (exclusive of interest) is:

a. $40,000. b. $51,000. c. $102,000. *d. $118,000. e. $170,000.

2163. Hal sold land held as an investment with a fair market value of $100,000 for $36,000 cash and a note for $64,000 that was due in two years. The note bore interest of 11% when the applicable Federal rate was 7%. Hal’s cost of the land was $40,000. Because of the buyer’s good credit record and the high interest rate on the note, Hal thought the fair market value of the note was at least $74,000.

a. Hal can elect to treat the $36,000 as a recovery of capital. b. Hal must recognize $60,000 gain in the year of sale. c. Hal must recognize $36,000 gain in the year of sale. *d. Unless Hal elects not to use the installment method, Hal must recognize $21,600 gain in the year of sale. e. None of the above.

2164. Todd, a CPA, sold land for $300,000 cash on the date of sale plus a note for $500,000 due in one year. The interest rate on the note was equal to the Federal rate. The fair market value of the note was $400,000. Todd’s basis in the land was $80,000.

a. If Todd uses the cash basis to report the income from his practice, he cannot use the installment method to report the gain on the sale of the land. b. If Todd uses the accrual basis to report the income from his practice, he cannot use the installment method to report the gain from the sale of the land. *c. If Todd uses the installment method to report the gain, the contract price is $800,000. d. If Todd does not use the installment method, his gain in the year of sale is $620,000 ($700,000 – $80,000). e. None of the above.

2165. Juan, not a dealer in real property, sold land that he owned. His adjusted basis in the land was $700,000 and it was encumbered by a mortgage for $100,000. The terms of the sale required the buyer to pay Juan $200,000 on the date of the sale. The buyer assumed Juan’s mortgage and gave Juan a note for $900,000 (plus interest at the Federal rate) due in the following year. What is the gross profit percentage (gain ÷ contract price)?

a. $700/$1,100 = 63.64%. b. $500/$1,200 = 41.67%. c. $700/$1,200 = 58.33%. *d. $500/$1,100 = 45.45%. e. None of the above.

2166. Pedro, not a dealer, sold real property that he owned with an adjusted basis of $120,000 and encumbered by a mortgage for $56,000 to Pat in 2011. The terms of the sale required Pat to pay $28,000 cash, assume the $56,000 mortgage, and give Pedro eleven notes for $12,000 each (plus interest at the Federal rate). The first note was payable two years from the date of sale and each succeeding note became due at two-year intervals. Pedro did not "elect out" of the installment method for reporting the transaction. If Pat pays the 2013 note as promised, what is the recognized gain to Pedro in 2013 (exclusive of interest)?

a. $12,000. *b. $7,200. c. $4,800. d. $0. e. None of the above.

2167. Charlotte sold her unincorporated business for $600,000 in 2013. The sales contract allocated $120,000 to equipment, $300,000 to land, and $180,000 to goodwill. Charlotte had a $0 basis in the goodwill, the land cost $150,000, and the equipment originally cost $250,000 but it was fully depreciated. What is the amount of the gain eligible for installment sales treatment?

a. $0. *b. $330,000. c. $450,000. d. $600,000. e. None of the above.

2168. In 2013, Norma sold Zinc, Inc., common stock for $100,000 cash and a note receivable for $900,000. The note was due in 2014 with accrued interest at the Federal rate. Norma’s basis in the stock was $250,000. This was Norma’s only installment sale transaction. Which of the following statements is correct?

*a. Norma cannot use the installment method to report her gain if the stock is listed on the New York Stock Exchange. b. Norma must recognize $75,000 gain in 2013 and she will be liable for interest on taxes deferred under the installment method. c. Norma must recognize $75,000 gain in 2013 and she will not be liable for interest on the taxes deferred under the installment method if the stock is not publicly traded. d. Norma should treat the $100,000 received as a recovery of capital. e. None of the above.

2169. Albert is in the 35% marginal tax bracket. He sold a building in the current year for $450,000. Albert received $110,000 cash at closing, the buyer assumed Albert’s mortgage for $120,000, and the buyer gave Albert a 6% note for $220,000 due in two years. The Federal rate was 6%. Albert’s basis in the building was $180,000 ($500,000 cost – $320,000 accumulated straight-line depreciation). Assuming he did not elect out of the installment method, Albert’s § 1231 gain and gain taxed at the 25% rate in the year of sale are what amounts?

Section 1231 Gain Unrecaptured § 1250 Gain Taxed at 25%

a. $66,000 $0 b. $0 $66,00 0 *c. $90,000 $90,000 d. $90,000 $0 e. $0 $110,0 00

2170. Walter sold land (a capital asset) to an unrelated party for $100,000 cash and a 4% note for $150,000 due in three years. His basis in the land was $40,000. Walter and the purchaser are cash basis taxpayers. Which of the following statements is correct?

a. If the Federal rate is 3%, interest will be imputed at that rate. *b. If the Federal rate is 5%, interest will be imputed at that rate and the capital gain will be reduced. c. If the Federal rate is 4.5%, interest will be imputed at that rate and the capital gain will be increased. d. All of the above. e. None of the above.

2171. Taylor sold a capital asset on the installment basis and did not charge interest on the deferred payment due in three years.

a. Interest will be imputed, thus increasing the total gross income from the transactions. *b. Interest will be imputed, thus decreasing the capital gain. c. Interest will not be imputed because the contract is for less than five years. d. Interest will be imputed, thus increasing the buyer’s basis in the asset. e. None of the above.

2172. Related-party installment sales include all of the following except the first seller’s:

a. Brothers and sisters. b. Controlled corporations. c. Lineal descendants and ancestors. d. Uncles and aunts. *e. All of the above would be considered related parties.

2173. Father sold land to Son for $500,000 in 2013. Father’s basis in the land was $100,000. Son paid Father $50,000 and gave Father a note for $450,000 due in 2016. In 2014, Son sold the land for $600,000 cash. The note bore interest at the appropriate Federal rate and both Father and Son held the land as an investment.

a. Father must recognize $400,000 of income in 2014. b. The installment method is not permitted because this is a related-party transaction. c. Father’s gain is all ordinary income. *d. Father must recognize a $360,000 gain in 2014. e. None of the above.

2174. In 2013, Father sold land to Son for $50,000 cash and an installment note for $150,000 due in 2017. Father’s basis was $100,000. In 2014, after paying $8,000 interest but nothing on the principal, Son sold the land for $300,000 cash. What gain, if any, must Father recognize in 2014?

a. $0. *b. $75,000. c. $100,000. d. $200,000. e. None of the above.

2175. Kathy was a shareholder in Matrix, Inc., when she sold the corporation a commercial building. The building cost $500,000 and the balance in the accumulated depreciation account was $400,000. Matrix, Inc., paid $100,000 in the year of sale and gave Kathy a note for $400,000 plus adequate interest due in 2015.

a. Because Kathy is a shareholder in Matrix, she cannot report the gain by the installment method. *b. Generally, if Kathy owned 100% of the Matrix stock, Kathy cannot use the installment method. c. Generally, if Kathy owned only 60% rather than 100% of the Matrix stock, she could use the installment method. d. Kathy cannot use the installment method to report the gain because the realized gain is equal to the depreciation she claimed on the building. e. None of the above.

2176. Which of the following statements is true concerning the disposition of an installment note?

*a. Deferred gain is not recognized by the transferor if the installment note is transferred in a § 351 transaction. b. Deferred gain must only be recognized if the installment note was transferred as a gift to a related party. c. Transfer of an installment obligation to another party will not trigger immediate recognition of deferred gain. d. Deferred gain must be recognized if the note is transferred to the owner’s estate at his death. e. None of the above.

2177. Wendy sold property on the installment basis in 2012 for more than her basis in the property. Wendy was to receive installment payments at the end of each year for the next five years. In 2013, Wendy was killed in a car accident and the note was transferred to her estate.

*a. The estate must recognize the gain from all the amounts collected on the installment obligation in 2013. b. The income will be reported on Wendy’s 2013 income tax return as income in respect of a decedent. c. The entire gain must be recognized in 2012. d. Gain is recognized by Wendy and reported on her 2013 income tax return when the note is transferred into the estate. e. None of the above.

2178. Gold Corporation sold its 40% of the Ruby Corporation common stock. Gold received $10 million in the year of the sale and a note for $15 million, payable in three years with interest at the Federal rate. Gold’s basis in the stock was $5 million. Assume that Gold Corporation will report the gain by the installment method where the method is permitted.

a. The installment method is never permitted on the sale of stock. *b. If Ruby Corporation stock is traded on an established securities market, Gold must recognize a $20 million gain in the year of sale. c. If the Ruby Corporation stock is not traded on a national exchange, Gold must recognize a $20 million gain. d. All of the above are true. e. None of the above is true.

2179. In the case of a small home construction company that builds under long-term contracts, generally:

a. The percentage of completion method is required to report the income from the construction contracts. b. The percentage of completion method can be elected and generally will defer income until the contract is completed. *c. The completed contract method can be used and generally will defer income. d. The accrual method must be used because inventories are an income-producing factor. e. None of the above is true.

2180. Robin Construction Company began a long-term contract in 2013. The contract price was $800,000. The estimated cost of the contract at the time it was begun was $500,000. The actual cost incurred in 2013 was $350,000. The contract was completed in 2014 and the cost incurred that year was $125,000. Under the percentage of completion method:

a. Robin should report $300,000 of income in 2013. b. Robin should report $90,000 of income in 2014. c. Robin will receive interest (under the lookback method) on the underpayment of taxes in 2013. d. Robin should report $325,000 of income in 2013. *e. None of the above is correct.

2181. Under the percentage of completion method, if the actual costs are ____ the estimated costs, the taxpayer must pay interest on the underpayment of prior years’ taxes.

a. Greater than. *b. Less than. c. Equal to or greater than. d. Equal to. e. None of the above.

2182. Camelia Company is a large commercial real estate contractor that reports its income by the percentage of completion method. In 2013, the company entered into a contract to construct a building for $900,000. Camelia estimated that the cost of constructing the building would be $600,000. In 2013, the company incurred $150,000 in costs under the contract. In 2014, the company incurred an additional $500,000 in costs to complete the contract. The company’s marginal tax rate in all years was 35%.

a. Camelia must report $300,000 of income in 2013. b. Camelia is not required to report any income from the contract until 2014 when the contract is completed. *c. Camelia must recognize $75,000 of income in 2013. d. Camelia should amend its 2013 tax return to decrease the profit on the contract for that year. e. None of the above.

2183. In the case of a taxpayer who uses the lower-of-cost-or-market inventory method:

a. Taxpayers may not use the lower of cost or market method for tax purposes. b. Market price means the expected selling price. c. Taxpayers may deduct a reserve for anticipated inventory price changes. *d. Each inventory item must be valued at the lower of its cost or its market value. e. None of the above.

2184. Aspen stores is a large retail chain. The company has four warehouses that are located in various parts of the country. The goods are stored at the warehouses and then moved to the retail stores for sale.

a. The costs of operating the warehouses can be deducted in the year the costs are incurred because it is a loss incurred from not selling goods. b. The costs of operating the warehouses can be deducted in the year the costs are incurred because they did not add to the value of the goods. c. The costs of operating the warehouses can be capitalized or expensed by electing one method or the other. d. The warehouses are on-sight storage facilities and, therefore, their costs must be added to the cost of goods on hand. *e. None of the above.

2185. A manufacturer must capitalize the following costs relative to inventories:

I. Maintenance cost for the factory building. II. Health insurance for production workers. III. Storage cost for finished goods.

a. Only I. b. Only I and II. c. Only II and III. *d. I, II, and III. e. None of the above.

2186. In determining the cost of goods manufactured and thus the cost of the ending inventory:

a. A production supervisor’s salary must be treated as a period cost (not added to production costs and inventory) because the salary is paid regardless of the amount produced. *b. The personnel department participates in hiring production workers, and therefore a portion of its costs must be allocated to production. c. Property taxes on the factory building are treated as period costs because the same taxes are due regardless of the volume of production. d. Only the costs of direct materials and direct labor must be capitalized. e. None of the above is correct.

2187. The taxpayer is an appliance dealer and has the following items of inventory on hand at the end of the year:

Replacement Expected Item Cost Cost Selling Price 20 Big Screen $12,000 $14,000 $18,000 TV’s 200 DVD 20,0 16,000 18,000 Players 00 100 Stereo 24,0 21,000 35,000 Systems 00 $56,000 $51,000 $71,000

Under the lower-of-cost-or-market inventory method, the ending inventory value is:

a. $71,000. b. $56,000. c. $51,000. *d. $49,000. e. None of the above.

2188. Duck Company has valued its inventories at FIFO cost for the past 10 years. The company would like to change to the LIFO method, effective in 2014.

*a. The election to change can be made with the 2014 tax return and the beginning inventory for 2014 will be the same as the FIFO inventory at the end of 2013 and no § 481 adjustment is required. b. The beginning inventory value for 2014 must be computed as though the company had been using LIFO in all prior years and a § 481 adjustment is required. c. The taxpayer must apply in 2013 for permission to change methods effective in 2014. d. Duck must amend all prior years’ tax returns to compute income by the LIFO method. e. None of the above.

2189. In the case of a change from the lower-of-cost-or-market FIFO to the LIFO inventory method:

a. The taxpayer must value the beginning inventory to be the same as if the company had used the LIFO method for all the years the company was in business. b. The taxpayer must seek written permission from the IRS in order to make the change. *c. The adjustment due to the change in accounting method must be spread over 3 years, the year of change and the two subsequent years. d. All of the above. e. None of the above.

2190. In 2013, George used the FIFO lower of cost or market inventory method. As of December 31, 2012, the inventory cost was $50,000 and its market price was $40,000. At the time of filing his 2013 income tax return, George changed to the LIFO method. The ending inventory at cost on December 31, 2013, was $75,000 and the market price of the goods totaled $35,000. Which of the following statements is correct?

*a. The beginning inventory for 2013 is $50,000, and George must spread a $10,000 adjustment ($50,000 – $40,000) evenly over 2013, 2014, and 2015. b. The beginning inventory for 2013 is $40,000. c. The beginning inventory for 2013 is $50,000, and George must spread a $10,000 adjustment over the three previous years. d. The change is invalid since the taxpayer did not apply for the change by the end of the tax year of change. e. None of the above.

2191. Crow Corporation has used the LIFO inventory method for the past 10 years. During that time, the prices Crow pays for the inventory have increased by 50%. Its inventory value when it first adopted LIFO was $5,000,000. The company began using a just-in-time inventory system the same year it adopted LIFO, and although sales have increased, the quantities of goods on hand at year end has not changed in the past ten years. The corporation’s marginal tax rate has been 35% in all of the years. As a result of the LIFO election:

a. The company has deferred $5,000,000 of income tax. b. The company has deferred $1,750,000 ($5,000,000 × .35) of income tax. c. The LIFO election did not defer any income tax because the quantity of goods on hand has not changed. *d. The company has deferred $875,000 [(.50)($5,000,000)(.35)] of income tax. e. None of the above.

2192. The use of the LIFO inventory method for tax purposes:

a. Will not defer income if the prices are increasing and quantities of inventories are decreasing. b. Is required for tax purposes if the taxpayer uses the LIFO method for reporting to stockholders and creditors. *c. Will result in a recapture of deferred income if the quantities of inventories decrease. d. Does not affect the company’s financial accounting for inventories and cost of goods sold. e. None of the above.

2193. Mallard Auto Parts, Inc. has on hand 1,000 fenders for 1953 Studebakers. Mallard purchased the fenders in 1965 for $30 each and the selling price is $400 each. Only rarely does Mallard sell a Studebaker fender and it is highly unlikely that more than 100 of the remaining fenders will ever be sold. However, Mallard has ample storage space and feels an obligation to Studebaker owners. Therefore, the company will not salvage the fenders and will continue to sell them for $400 each. Scrap value of the fenders is $5 each. Under the lower of cost or market inventory method:

a. Mallard can expense the 900 excess fenders. b. Mallard can expense all 1,000 of the fenders because of the unlikelihood they will be sold. c. The fenders should be valued at $7,500 [(100 × $30) + (900 × $5)]. d. The fenders should be valued at $5,000 (1,000 × $5). *e. None of the above.

2194. The Multi Department store takes physical inventories at each of its 300 stores on various dates between August 1 and September 30th each year. The company’s tax year ends on the Monday closest to January 31st. The company’s reduction in inventory due to breakage and theft after the last physical inventory in September 2013:

a. Cannot be determined until the physical inventory is actually taken and therefore breakage that occurs in December 2013 will not be deductible until the year ending in January 2014. b. Must be delayed until the inventory has been taken as a result of the all-events test. *c. Can be estimated and deducted for the year ending in January 2014. d. Can be estimated and deducted as of the end of the tax year, but only if the taxpayer uses the lower of cost or market inventory method. e. None on the above.

2195. Dr. Stone incorporated her medical practice and elected to use a fiscal year ending September 30th. For the fiscal year ending September 30, 2013, the corporation earned $40,000 profits each month, before Dr. Stone’s salary and income tax. Dr. Stone received a salary that averaged $30,000 per month. Next year (fiscal year ending September 30, 2014), Dr. Stone expects the average monthly profits before salary and taxes to be $48,000. What is the minimum salary Dr. Stone can receive for the last three months of calendar year 2013 to ensure that the corporation can deduct salary equal to the corporation’s before salary income for the fiscal year ending September 30, 2014?

Correct Answer: The corporation must pay Dr. Stone a salary of $144,000 ($48,000 × 3) for the last three months of 2013. This means that she is receiving the same as her average 2013 fiscal year monthly salary. Then, in 2014, her salary can be raised so that the total income for the fiscal year, $576,000 (12 × $48,000) is paid as salary.

2196. The Yellow Equipment Company, an accrual basis C corporation, is a manufacturer’s representative and works on a commission basis (15% of sales that it places) and does not carry inventory. In November 2013, Yellow made a sale and collected a commission for $20,000. In June of 2014, the customer had not received the equipment from the manufacturer and canceled the order. As a result, Yellow was required to refund the $20,000 commission to the manufacturer. Yellow’s taxable income in 2013 was $70,000, and in 2014 Yellow’s taxable income was $25,000 after deducting the refund. The applicable tax rate schedule is 15% on the first $50,000 of income and 25% on income in excess of $50,000. What is the effect of the refund on Yellow’s 2014 tax liability?

Correct Answer: 2014 taxable income before $45,000 the refund ($25,000 + $20,000) Applicable tax rate × .15 Tax before refund $6,750 Repayment $20,000 2013 marginal tax rate × .25 Reduction in tax from refund (5,000) 2014 tax liability $1,750

The $20,000 received in 2013 must be included in Yellow’s gross income for that year and thus increased the tax liability by 5,000 ($20,000 × 25%). Under § 1341, the repayment that is deductible in 2014 can likewise reduce the tax liability by $5,000, even though under the general rules of the annual accounting period, the repayment would reduce the tax by only $3,750 ($25,000 × 15%).

2197. Ramon sold land in 2013 with a cost of $80,000 for $200,000. The sales agreement called for a $50,000 down payment and a $50,000 payment plus 8% interest to be received on the first day of each year for the next three years. What would be the consequences of the following (treat each part independently and assume Ramon uses the installment method whenever possible): a. In 2013, Ramon gave one of the $50,000 installment obligations to a close relative. b. In 2013, Ramon transferred the installment obligations ($50,000) to his 100% owned corporation. c. Ramon collected the $50,000 plus $12,000 interest on January 1, 2014, and died on January 2, 2014.

Correct Answer: a. The gift is a taxable disposition and, thus, Ramon must recognize a $30,000 gain ($120,000/$200,000 × $50,000). b. The transfer to the controlled corporation is not a taxable disposition. The corporation will recognize the gain when it collects the amount due. c. Death is not a taxable disposition. Ramon’s beneficiaries will recognize the gain (as income in respect of a decedent) when the payments are collected. Ramon recognizes a $30,000 gain ($120,000/$200,000 × $50,000) plus $12,000 interest on his final return for 2014.

2198. Brown Corporation elected dollar-value LIFO in 2008. Its ending inventory at base year cost and its LIFO indexes are as follows:

(1) (2) (3) = (1)/(2) Ending Ending Inventory Inventory@ At End of the LIFO Base Year Year Prices Index Prices

2008 $1,000,000 1.00 $1,000,000 2009 1,100,000 1.06 1,037,736 2010 1,200,000 1.10 1,090,909 2011 1,300,000 1.11 1,171,171 2012 1,250,000 1.20 1,041,667 2013 1,550,000 1.23 1,260,163

Compute the LIFO inventory at the end of 2013.

Correct Answer: The company experienced a decrease in inventory in 2012. This eliminated the 2011 layer and decreased the 2010 layer added.

(1) (2) (3) = (1)/(2) (4) (5) Ending Inventory Ending (4) × (2) At End of the Year LIFO Inventory@ Layer Layer Prices Index Base Year Prices Added @Current @ Base 2008 $1,000,000 1.00 $1,000,000 $1,000,000 $ 1,000,000 2009 1,100,000 1.06 1,037,736 4 37,736 0,000 2010 1,200,000 1.10 1,090,909 5 53,173 8,491 2011 1,300,000 1.11 1,171,171 8 80,262 9,091 2012 1,250,000 1.20 1,041,667 ( (129,504)155,405) 2013 1,550,000 1.23 1,260,163 218,496 268 ,750 $1 Ending Inventory ,300,927

*2012 Reduction Reduction at Current At Base $129,504 Index Prices

$ 80,262 $ 89,09 From 2010 1.11 1

49,2 54 From 2009 42 1.10 ,167 $129,504 $143,258

2199. Brown Corporation had consistently reported its income by the cash method. The corporation should have used the accrual method because inventories are material to the business. In 2013, Brown timely filed a request to change to the accrual method. At the beginning of 2013, Brown had accounts receivable of $75,000. Also, Brown had merchandise on hand with a cost of $150,000 and accounts payable for merchandise of $45,000. The accounts receivable, inventory, and accounts payable balance per books were zero. Determine the adjustment to income due to the change in accounting method and the amount that is allocated to 2013.

Correct Answer: Adjustment due to the change:

Accounts receivable $ 75,000 Inventory 150,000 Accounts payable (45,000 ) Total adjustment $180,000

The change is from a clearly incorrect method to a correct method. Therefore, the positive adjustment must be spread over four years. The company must add $45,000 ($180,000 divided by 4) to 2013 income.

2200. John sold an apartment building for $600,000. His basis in the building was $360,000 and it was subject to $30,000 of depreciation recapture. John received $120,000 in the year of sale, the buyer assumed John’s mortgage payable of $240,000, and the buyer gave John an 8% (the current Federal rate) note of $210,000 due in 5 years. The interest on the note was payable each June 30th, beginning in the year following the year of the sale. John incurred $30,000 of selling expenses which he paid in the year of sale. Compute John’s installment sales gain that should be reported in the year of sale.

Correct Answer: Selling price $600,000 Less: Selling expenses (30,000) Less: Basis (360,000) Total gain $210,000 Less: Depreciation recapture (30,000) Installment sale gain (all $180,000 § 1231 gain) Contract price: Selling price $600,000 Less: Seller’s (240,000) liability assumed Contract price $360,000

Gross profit ratio $180,000/$360,000 = .50 Gain in the year of sale: Depreciation recapture $30,000 Installment gross .50 × $120,000 60,000 profit Total gain in the year $90,000 of sale

2201. Terry, Inc., makes gasoline storage tanks. All production is done under contract. The company makes three basic models, but each model must be adapted to customer specifications for the location of outlets, insulation, and paint. It takes from three to six months to complete a tank. How should Terry account for the income for the business?

Correct Answer: Terry, Inc. could have the percentage of completion or the accrual method apply. If Terry would like to avoid the percentage of completion method and instead use the accrual method, it should argue that the products are not “unique.” Terry would argue that the modifications for individual customers are minor and add very little to the costs of production.

2202. Computer Consultants Inc., began business as an adviser to chains of retail stores. The company assisted the stores in the selection of hardware and the development of software used by retail chain stores. Later the company developed software and sold it to its customers. The company also began selling some of the equipment to the customers. That is, the company would bid on a job to purchase and install equipment and the software. The company has consistently reported its income by the cash method. At the end of the year, the company has substantial accounts receivables from clients and a small amount of inventory on hand. What advice can you offer the company regarding its accounting method?

Correct Answer: Inventories are an income-producing factor for the business. Therefore, the company is required to use the accrual method of accounting. The company should voluntarily change to the accrual method, before the IRS requires the change. With a voluntary change, the company can avoid penalties and interest. Also, the company will be allowed to spread the gain over the current year and the three following years.

2203. The buyer and seller have tentatively agreed to a contract for the sale of a building that the buyer will use in its business. The buyer will pay the seller $100,000 (principal and interest) each year for 5 years. The seller’s cost of the asset is $200,000, and he will report the capital gain using the installment method. The buyer and seller are now negotiating the interest rate that will be used to compute the interest included in each $100,000 payment. The relevant Federal rate is 5%, but the market rate on similar contracts is in the area is 7%. a. Why would the seller bargain for a 5% interest rate for the contract rather than a 7% interest rate? b. How does the interest rate affect the buyer’s future taxable income?

Correct Answer: a. The total payments the seller will receive is not affected by the interest rate included in the contract. However, the interest rate will affect the seller’s ordinary income and capital gain. The seller would bargain for a 5% interest rate rather than 7% because the lower interest rate will result in less ordinary income and more capital gain, with no change in the total taxable income from the contract. Also, with the lower interest rate, more of each payment is allocable to the recovery of capital and thus more of the gain is deferred. b. The interest rate affects the buyer’s interest and cost recovery deductions. Each dollar allocated to interest will result in a deduction for the buyer, each year for the five years of the installment contract. However, the amount allocated to the building must be recovered over the MACRS cost recovery period, which may be more than thirty years. Therefore, the present value of the tax benefits of the interest deductions are much greater than the cost recovery deductions.

2204. The company has consistently used the LIFO inventory method and has deferred over $1 million of income from using that method. However, in the last two years, the prices it pays for goods has been decreasing. Therefore, the company is considering changing to the FIFO inventory method. What would be some tax consequences of the change?

Correct Answer: The company could voluntarily change to FIFO, but this would require that the company restate its beginning inventory for the year of change to equal its cost using the FIFO method. Because the LIFO deferral is $1 million, this means that the beginning inventory would be increased by $1 million and the company would have a $1 million addition to its income that can be spread equally over the year of the change and three subsequent years. Thus, deciding whether the change would be beneficial would require further analyses of the projected price changes and inventory levels. The recapture of the LIFO reserve would be a critical consideration.

2205. Yard Corporation, a cash basis taxpayer, received $10,000 from a customer in 2012. In 2012, the customer filed a claim for a refund of the fee. In 2013, Yard refunded the customer $6,000. In 2012, Yard paid $5,000 in estimated state income tax. In May 2013, Yard received a state income tax refund of $2,000 for overpayment of its 2012 income tax. Yard was in the 35% marginal tax bracket in 2012 and in the 15% marginal tax bracket in 2013. What are the tax effects of the 2013 payment to the customer and the collection of the state income taxes overpaid?

Correct Answer: The payment to the customer is eligible for § 1341 treatment. Because the amount received from the customer in 2012 was taxed at 35%, the refund to the customer in 2013 will reduce the tax for that year by $2,100 ($6,000 × .35). On the other hand, the state income taxes were deducted when the marginal tax rate was 35%, but the recovery of the prior deduction is taxed at only 15% under § 111.

2206. What incentives do the tax accounting rules provide for taxpayers to voluntarily change from an incorrect method of accounting that has reduced the company’s tax liability in prior years?

Correct Answer: The incorrect method that reduced taxable income in prior years generally will result in a positive § 481 adjustment to income. The taxpayer who voluntarily changes methods is permitted to spread the adjustment over the current and three succeeding years. On the other hand, if the IRS examines the return and makes the adjustment, the IRS will compute the adjustment as of the beginning of the earliest open year, require that all of the adjustment must be paid upon completion of the examination, and interest and perhaps penalties may be due.

2207. Mogo Manufacturing Company accounts for its inventories by the FIFO method. The company has consistently allocated building depreciation to production and general administration on the basis of the number of square feet occupied. According to the measurements used, manufacturing requires 90% of the square footage and general administration utilizes 10% of the total square feet. This year, 2013, the accountant realized that 5 years ago an addition was made to the portion of the building used for general administration, and the depreciation allocation had not been adjusted. What are the tax accounting implications of this discovery?

Correct Answer: The company has consistently used an incorrect accounting method to allocate its costs to production. The incorrect method has allocated too much of the depreciation to production, and thus to ending inventories. The company should apply for permission to change its method of allocating depreciation. The adjustment due to the change will be “negative” (reducing taxable income). Because Mogo is changing from an incorrect to a correct method, the negative adjustment can be used to reduce the 2013 income, assuming the request for changing methods is timely filed.

2208. After 2008, income averaging is allowed for Federal income tax purposes.

a. True *b. False

2209. Qualified plans have higher startup and administrative costs than nonqualified plans.

*a. True b. False

2210. A restricted property plan is considered a deferred compensation plan.

*a. True b. False

2211. Group life insurance is considered a deferred compensation plan.

a. True *b. False

2212. Forfeitures may be allocated to the accounts of the remaining participants in defined contribution plans.

*a. True b. False

2213. In a profit sharing plan, a separate account is not maintained for each participant.

a. True *b. False

2214. A defined contribution plan is exempt from funding requirements.

*a. True b. False

2215. In a stock bonus plan, contributions are dependent on the employer’s profits.

a. True *b. False

2216. A cash balance plan is a hybrid form of pension plan that is similar in many aspects to a defined contribution plan.

a. True *b. False

2217. The payout to an employee in a cash balance plan is based upon a formula based on years of service.

a. True *b. False

2218. A failure to make a minimum required distribution to a participant in any taxable year results in a 50% nondeductible excise tax on any excess of the amount that should have been distributed over the amount that actually was distributed.

*a. True b. False

2219. The minimum annual distributions must be made over the life of the participant or the life of the participant and a designated individual beneficiary.

*a. True b. False

2220. A defined benefit plan must reduce the $205,000 (in 2013) maximum benefits payable by one-tenth for each year of participation under 10 years that an employee has performed.

*a. True b. False

2221. If a taxpayer receives an early distribution from a qualified retirement plan, a 10% additional tax is levied on the full amount of any distribution includible in gross income.

*a. True b. False

2222. A taxpayer who receives a distribution can avoid current taxation by rolling the distribution into another qualified employer retirement plan or into an IRA.

*a. True b. False

2223. Under a defined benefit plan, the annual benefit payable to an employee is limited to the smaller of $205,000 (in 2013) or 100% of the employee’s average compensation for the highest 3 years of employment.

*a. True b. False

2224. A 20% excise tax is imposed on nondeductible contributions by an employer to a qualified plan.

a. True *b. False

2225. Any pre-tax amount elected by an employee as a plan contribution to a § 401(k) plan that does not exceed the statutory limit is not includible in gross income in the year of deferral and is 100% vested.

*a. True b. False

2226. Dana contributes $2,000 too much to a § 401(k) plan which is not returned within 2 1/2 months after the close of the tax year. The employer will have to pay a tax of $200.

*a. True b. False

2227. If an employer’s contribution to a SEP IRA is less than $51,000 in 2013 (or 25% of the employee’s earned income, if less), the employee can contribute the difference.

a. True *b. False

2228. Traditional IRA contributions made after an individual reaches the age of 65 are treated as excess contributions and are subject to a nondeductible 6% excise penalty tax.

a. True *b. False

2229. A participant has an adjusted basis of $0 in any nondeductible contributions to a traditional IRA.

a. True *b. False

2230. If a married taxpayer is an active participant in another qualified retirement plan, the traditional IRA deduction phaseout begins at $95,000 of AGI for a joint return in 2013.

*a. True b. False

2231. If an individual is ineligible to make a deductible contribution to a traditional IRA, nondeductible contributions of any amount can be made to a traditional IRA.

a. True *b. False

2232. The maximum annual contribution to a Roth IRA for an unmarried taxpayer who is age 35 is the smaller of $5,500 or the individual’s compensation for the year.

*a. True b. False

2233. Contributions to a Roth IRA can be made up to the due date (excluding extensions) of the taxpayer’s income tax return.

*a. True b. False

2234. An individual, age 40, who is not subject to the phase-out provision may contribute a nondeductible amount to a Roth IRA up to $5,500 per year in 2013.

*a. True b. False

2235. A participant who is at least age 59 1/2 can make a tax-free qualified withdrawal from a Roth IRA after a five-year holding period.

*a. True b. False

2236. Distributions from a Roth IRA that are subject to taxation are treated first as from earnings and last as from contributions.

a. True *b. False

2237. Low- and middle-income taxpayers may make nondeductible contributions up to $4,000 per child per year to a Coverdell Education Savings Account (CESA).

a. True *b. False

2238. An individual is considered an active participant in an employer- sponsored retirement plan merely because an individual’s spouse is an active participant for any part of a plan year in applying the IRA phase-out provision.

a. True *b. False

2239. For the spousal IRA provision to apply, a joint return must be filed.

*a. True b. False

2240. A direct transfer of funds from a qualified retirement plan to an IRA is subject to the withholding rules.

a. True *b. False

2241. In a direct transfer from one qualified retirement plan to another qualified retirement plan, the employer does not have to withhold 20% of the amount of the direct transfer.

*a. True b. False

2242. Income is taxed if a taxpayer’s control over the amount earned is subject to substantial restrictions.

a. True *b. False

2243. A NQDC plan cannot discriminate in favor of officers or other highly compensated employees.

a. True *b. False

2244. A company is denied a deduction for a golden parachute payment to an employee, but not for a golden parachute payment to an independent contractor.

a. True *b. False

2245. The $1 million deduction limitation on executive compensation is decreased by any nondeductible golden parachute payments made to an employee.

*a. True b. False

2246. The “spread” on an incentive stock option is subject to the alternative minimum tax.

*a. True b. False

2247. An employer obtains a tax deduction at the same time and to the extent that ordinary income is recognized by the employee who receives nonqualified stock options.

*a. True b. False

2248. If a NQSO has a readily ascertainable value, an employee recognizes income on the grant date.

*a. True b. False

2249. A major disadvantage of a NQSO is that an employee must recognize ordinary income on the exercise of the option or at the date of the grant without receiving cash to pay the tax.

*a. True b. False

2250. Which of the following characteristics does not describe a defined benefit pension plan?

a. Actuarial calculations are required to determine the employer’s annual contribution. *b. Upon retirement, the employee’s pension amount depends on the value of the employee’s account. c. Forfeitures must reduce subsequent funding costs and cannot increase the benefits any participant can receive under the plan. d. A separate account is not required to be maintained for each participant. e. None of the above.

2251. Which of the following characteristics is not a characteristic of a stock bonus plan?

*a. Can be in the form of a defined benefit plan. b. Contributions need not be dependent on employer’s profits. c. Exempt from funding requirements. d. Not subject to plan termination insurance. e. None of the above.

2252. Which of the following characteristics is not a characteristic of a cash balance plan?

a. Similar in many aspects to a defined benefit plan. *b. Employee bears the investment risk and rewards. c. Establishes allocations to individual employee accounts. d. Payout depends on how much builds up over time in the employee’s individual account. e. None of the above.

2253. Which of the following characteristics describes a defined benefit plan?

a. Subject to PBGC plan termination insurance rules. b. An account for each participant is not established. c. Defines the amount the employer is required to contribute. *d. Only a. and b. e. a., b., and c.

2254. A retirement plan covers 72% of the non-highly compensated individuals. The plan benefits 48 of the 131 employees. Which is true?

a. The 70% coverage requirement is not met. b. The plan meets the minimum participation test. *c. The plan meets the 70% coverage requirement, but fails the minimum participation test. d. The plan does not meet the 70% coverage requirement, but does meet the minimum participation test. e. None of the above is true.

2255. Zackie has five years of service completed as of February 5, 2013, which is his employment anniversary date. If his defined benefit plan [not a § 401(m) arrangement] uses the graded vesting rule, determine Zackie’s nonforfeitable percentage.

a. 40%. *b. 60%. c. 80%. d. 100%. e. None of the above.

2256. Deidre has five years of service completed as of February 5, 2013, her employment anniversary date. If the defined benefit plan [not a § 401(m) arrangement] uses the cliff vesting schedule, determine Deidre’s nonforfeitable percentage.

a. 0%. b. 60%. c. 80%. *d. 100%. e. None of the above.

2257. Dianna participates in a defined benefit plan that uses a fixed formula providing an employee with a benefit of 2% for each year of service, up to a maximum of 30 years. The total percentage accumulated before retirement is applied to the average of her three highest years of salary. Dianna works for 21 years, and the average of her three highest years of salary is $290,000. Calculate the amount of retirement benefits she will receive each year.

a. $0. b. $102,800. c. $110,000. d. $250,000. *e. None of the above.

2258. Brown, Inc., uses the three-to-seven year graded vesting approach for its retirement plan. Peter has five years of service completed as of February 5, 2013, his employment anniversary date. Determine Peter’s nonforfeitable percentage.

a. 40%. *b. 60%. c. 80%. d. 100%. e. None of the above.

2259. Danielle, who is retired, reaches age 70 1/2 in 2012, and she will also be age 71 in 2012. She has a $150,000 balance in her traditional IRA. If her life expectancy is 15.3 years, what distribution, if any, must be made by April 1, 2013?

a. $0. *b. $9,804. c. $19,608. d. $150,000. e. None of the above.

2260. Heather, age 48, is the sole remaining participant of a money purchase pension plan. The plan is terminated and a $240,000 taxable distribution is made to Heather. The early distribution penalty tax, if any, for 2013 is:

a. $0. b. $12,000. *c. $24,000. d. $30,000. e. None of the above.

2261. Scott, age 68, has accumulated $850,000 in a defined contribution plan, $100,000 of which represents his own after-tax contributions. If the full amount is distributed in 2013, his early distribution penalty is:

*a. $0. b. $75,000. c. $85,000. d. $127,500. e. None of the above.

2262. In 2013, Jindal Corporation paid compensation of $42,300 to the participants in a profit sharing plan and then contributed $12,800 to the plan. Jindal’s deductible amount and any contribution carryover are as follows:

a. $0 deductible; $12,800 carryover. b. $8,460 deductible; $4,340 carryover. *c. $10,575 deductible; $2,225 carryover. d. $12,690 deductible; $110 carryover. e. None of the above.

2263. Fred is a self-employed accountant with gross earned income of $140,000 per year (after the deduction for one-half of any self- employment tax). He has a profit sharing plan (i.e., defined contribution plan). What is the maximum amount Fred can contribute to his retirement plan?

*a. $28,000. b. $35,000. c. $40,000. d. $140,000. e. None of the above.

2264. The compensation paid by Purple Corporation to the plan participants of a profit sharing plan in 2013 was $38,300. During 2013, Purple Corporation contributed $10,000 to the plan. Purple’s deductible amount for 2013 is what amount, if any?

a. $0. b. $7,660. *c. $9,575. d. $10,000. e. None of the above.

2265. A participant, who is age 38, in a cash or deferred arrangement plan [§ 401(k)] may contribute up to what amount in 2013?

a. $12,000. b. $16,500. c. $17,000. *d. $17,500. e. None of the above.

2266. Merrill is a participant in a SIMPLE § 401(k) plan, and he elects to contribute 4% of his $40,000 compensation to the account, while his employer contributes 3%. What amount will vest immediately, if any?

a. $0. b. $1,200. c. $1,600. *d. $2,800. e. None of above.

2267. Susan is a self-employed accountant with a qualified defined contribution plan (a Keogh plan). She has the following income items for the year:

Earned income from self-employment $50,000 Dividend income 8,000 Interest income 2,000 Net short-term capital gain 12,00 0 Adjusted gross income $72,000

What is the maximum amount Susan can deduct as a contribution to her retirement plan in 2013, assuming the self-employment tax rate is 15.3%?

a. $9,235. b. $12,000. c. $46,000. *d. $46,468. e. None of the above.

2268. Joyce, age 40, and Sam, age 42, who have been married for seven years, are both active participants in qualified retirement plans. Their total AGI for 2013 is $120,000. Each is employed and earns a salary of $65,000. What are their combined deductible contributions to traditional IRAs?

*a. $0. b. $3,000. c. $4,000. d. $8,000. e. None of the above.

2269. Dana, age 31 and unmarried, is an active participant in a qualified retirement plan. Her AGI is $116,000. What amount, if any, may Dana contribute to a Roth IRA in 2013?

a. $0. b. $3,225. *c. $4,033. d. $5,500. e. None of the above.

2270. Sammy, age 31, is unmarried and is not an active participant in a qualified retirement plan. His modified AGI is $55,000 in 2013. The maximum amount that Sammy can deduct for a contribution to a traditional IRA is:

a. $2,800. b. $3,500. c. $5,000. *d. $5,500. e. None of the above.

2271. Frank established a Roth IRA at age 25 and contributed a total of $131,244 to it over 38 years. The account is now worth $376,000. How much of these funds can Frank withdraw tax-free?

a. $0. b. $131,244. c. $244,756. *d. $376,000. e. None of the above.

2272. Mary establishes a Roth IRA at age 50 and contributes the maximum amount per year to the Roth IRA for 15 years. The account is now worth $199,000, consisting of $75,000 in contributions plus $124,000 in accumulated earnings. How much can Mary withdraw tax-free?

a. $0. b. $75,000. c. $124,000. *d. $199,000. e. None of the above.

2273. Jana has $225,000 of earned income in 2013. Calculate the amount she can contribute to a SEP.

a. $22,500. b. $24,500. *c. $51,000. d. $55,000. e. None of the above.

2274. In 2015, Kathy receives a $4,000 distribution from her Coverdell Education Savings Account (CESA), which has a fair market value of $10,000. Total contributions to her CESA have been $7,000. Kathy’s AGI is $25,000. If Kathy uses $3,000 of the $4,000 distribution for qualified education expenses, what amount should she include in her gross income?

a. $0. b. $450. c. $750. d. $1,200. *e. None of the above.

2275. Juanita receives a $2,000 distribution from her Coverdell Education Savings Account (CESA). The distribution consists of $1,500 of contributions and $500 of earnings. Juanita pays $1,700 in qualified higher education expenses for the year. Calculate the amount to be included in Juanita’s gross income.

a. $0. *b. $75. c. $425. d. $500. e. None of the above.

2276. Which of the followings is not a characteristic of a Keogh plan?

a. A trust is established. b. Considered to be a qualified plan. c. Contribution to plan by extension due date. *d. Favorable 10-year forward averaging is not available. e. All of the above are characteristics.

2277. Which statement is true with respect to golden parachute payments?

a. Refers to excess severance pay. b. Does not include payments from a qualified profit sharing plan. c. A deduction is denied to the employer. d. A 20% excise tax is imposed on the recipient. *e. All of the above are true.

2278. Emmanuel, an executive, receives a $600,000 payment under a golden parachute agreement. Emmanuel’s base amount from Blue Corporation is $140,000. What amount, if any, is deductible by the corporation?

a. $0. *b. $140,000. c. $460,000. d. $600,000. e. None of the above.

2279. James, an executive, receives a $600,000 payment under a golden parachute agreement. James’s base amount from Silver, Inc., is $140,000. What is the total tax James must pay, assuming a 39.6% individual tax rate?

a. $0. b. $92,000. c. $210,000. *d. $329,600. e. None of the above.

2280. Saysha is an officer of a local bank that merges with a national bank, resulting in a change of ownership. She loses her job as a result of the merger, but she receives a cash settlement of $390,000 from her employer under her golden parachute. Her average annual compensation for the past five tax years is $110,000. Calculate any nondeductible excise tax Saysha must pay, if any.

a. $0. *b. $56,000. c. $110,000. d. $280,000. e. Some other amount.

2281. Larry negotiates a $2.5 million contract with Red, Inc., a publicly-held corporation that receives TARP funds, to become their CEO for 2013. What amount is deductible by Red, Inc., in 2013?

a. $0. b. $1,000,000. c. $2,064,000. d. $2,500,000. *e. None of the above.

2282. Nick negotiates a $4.5 million contract per year with a major college football program to become its head coach. What amount is deductible by the program in 2013 his first full year of employment.

a. None. b. $1,000,000. c. $3,000,000. *d. $4,500,000. e. None of the above.

2283. The special § 83(b) election (i.e., where income is taxed in the year of the grant) with respect to a restricted stock plan may be advantageous in which of the following situations in 2013?

a. The employer is an unstable company. *b. The bargain element is relatively small. c. A minimum amount of appreciation is expected in the future. d. The restriction probably will not be satisfied. e. None of the above.

2284. Pony, Inc., issues restricted stock to employees in July 2013, with a two-year vesting period and an SRF. An employee must remain a full-time employee of Pony for two years after the restricted stock is issued. The stock is trading at $10 per share when the stock is issued. An employee, Sam, decides to make the § 83(b) election with his 1,000 shares. At the end of 2013, the stock is trading at $13 per share. How much income, if any, must Sam recognize in 2013?

a. $0. b. $10,000 capital gain. *c. $10,000 ordinary income. d. $13,000 capital gain. e. $13,000 ordinary income.

2285. Pony, Inc., issues restricted stock to employees in July 2013, with a two-year vesting period and an SRF. An employee must remain a full-time employee of Pony for two years after the restricted stock is issued. The stock is trading at $10 per share when the stock is issued. An employee, Sam, decides to make the § 83(b) election with his 1,000 shares. At the end of 2013, the stock is selling for $13 per share. What amount, if any, can Pony take as a compensation deduction?

a. $0. *b. $10,000 in 2013. c. $13,000 in 2013. d. $10,000 when stock is sold. e. $13,000 when stock is sold.

2286. Pony, Inc., issues restricted stock to employees in July 2013, with a two-year vesting period and an SRF. An employee must remain a full-time employee of Pony for two years after the restricted stock is issued. The stock is trading at $10 per share when Sam is issued 1,000 shares, and he proceeds to make a § 83(b) election. At the end of 2013, the stock is selling for $13 per share. Sam remains a full-time employee of Pony for the required two-year vesting period at which time the stock is worth $30 per share. Sam sells his 1,000 shares in 2017 at $36 per share. What amount and type of income will Sam recognize in 2017?

*a. $26,000 capital gain. b. $26,000 ordinary income. c. $23,000 capital gain. d. $23,000 ordinary income. e. $36,000 capital gain.

2287. Pony, Inc., issues restricted stock to employees in July 2013, with a two-year vesting period and an SRF. An employee must remain a full-time employee of Pony for two years after the restricted stock is issued. The stock is trading at $10 per share when Sam is issued 1,000 shares, and he does not make a § 83(b) election. At the end of 2013, the stock is selling for $13 per share. Sam remains a full-time employee of Pony for the required two-year vesting period, at which time the stock is worth $30 per share (in 2015). Sam sells his 1,000 shares in 2017 at $36 per share. What amount and type of income will Sam recognize in 2017?

a. $6,000 ordinary income. *b. $6,000 capital gain. c. $26,000 capital gain. d. $6,000 capital gain; $20,000 ordinary income. e. $20,000 capital gain; $6,000 ordinary income.

2288. If the special election under § 83(b) is made as a result of a restricted property transaction, which statement is false?

a. A factor supporting making the § 83(b) election is the expectation the property will appreciate substantially. b. Ordinary income is recognized on the excess of the FMV over the amount paid for the property on the date received. c. Any appreciation on the property after receipt is treated as capital gain. *d. A deduction is allowed to the employee for any taxes paid on the original amount included in gross income if the property is subsequently forfeited. e. None of the above is false.

2289. What statement is false with respect to an incentive stock option (ISO)?

a. To qualify as an ISO, the option price must equal or exceed the fair market value of the stock at the time the option is granted. *b. The spread is not subject to the alternative minimum tax. c. If the employee meets the holding period requirement, the employer does not obtain a business deduction. d. The spread is the excess of the FMV of the share at the date of exercise over the option price. e. None of the above are false.

2290. Under a nonqualified stock option (NQSO) plan which is granted to Damon on March 15, 2011, he may purchase 200 shares of stock from his employer at $15 per share. At that date, the option does not have a readily ascertainable fair market value. Eight months later on the date of exercise the fair market value of the stock is $20. On December 1, 2013, Damon sells 100 shares for $24 each. Which of the following would be the result of these transactions on the date of exercise and the date of sale?

*a. Ordinary income of $1,000 and a long-term capital gain of $400. b. Ordinary income of $1,200 and a long-term capital gain of $300. c. Ordinary income of $2,400 and a long-term capital gain of $0. d. Ordinary income of $1,000 and a short-term capital gain of $400. e. None of the above.

2291. Yvonne exercises incentive stock options (ISOs) for 100 shares of Apple Corporation stock at the option price of $100 per share on May 21, 2013, when the fair market value is $120 per share. She sells the 100 shares of stock 3 1/2 years later for $140. Determine the recognized gain on the sale and classify it as capital or ordinary.

a. $2,000 LTCG. b. $2,000 ordinary income. *c. $4,000 LTCG. d. $4,000 ordinary income. e. None of the above.

2292. Yvonne exercises incentive stock options (ISOs) for 100 shares of Apple Corporation stock at the option price of $100 per share on May 21, 2013, when the fair market value is $120 per share. She holds the stock for only seven months and sells the shares for $140 per share. Determine the recognized gain on the sale and classify it as capital or ordinary.

*a. $2,000 ordinary income and $2,000 STCG. b. $2,000 ordinary income and $2,000 LTCG. c. $4,000 ordinary income. d. $4,000 LTCG. e. None of the above.

2293. Yvonne exercises incentive stock options (ISOs) for 100 shares of Apple Corporation stock at the option price of $100 per share on May 21, 2013, when the fair market value is $120 per share. She holds the stock for only two years and sells the shares for $115 per share. Determine the recognized gain on the sale and classify it as capital or ordinary.

a. $2,000 ordinary income and $2,000 STCG. b. $2,000 ordinary income and $2,000 LTCG. c. $4,000 ordinary income. *d. $1,500 LTCG. e. None of the above.

2294. Which is not an advantage of a § 401(k) plan over a traditional IRA?

a. Deduction limitation. b. Early withdrawal option without penalty. c. Loans from the plan. *d. Record keeping. e. All of the above are advantages of § 401(k) plan.

2295. Which would not be considered an advantage of a nonqualified stock option plan over an incentive stock option (ISO) plan?

a. Granted at any price. b. May have any duration. c. Spread subject to alternative minimum tax. *d. Governed by § 83. e. All are considered advantages.

2296. In order to postpone income tax obligations as long as possible, retirement assets should be taken from which assets (or accounts) first?

a. Section 457 accounts of government employees since no penalties apply once the employee is separated from employment. b. Section 401(k) and nonprofit § 403(b) plans because the age 55 separation rule may apply. c. Traditional IRAs since a taxpayer must be at least age 59 1/2 to obtain penalty-free withdrawals. d. Roth IRAs since no required minimum distribution rules apply. *e. Taxable accounts.

2297. Determine the nonforfeiture percentage under these independent situations for a defined benefit plan using the graded vesting rule. a. Mari has five years of service completed as of February 5, 2013 (which is her employment anniversary date). b. Jake, age 41, has six years of service completed as of March 2, 2013 (which is his employment anniversary date). c. Sammie, age 26, has four years of service completed as of July 21, 2013 (which is his employment anniversary date).

Correct Answer: a. 60%. b. 80%. c. 40%.

2298. Joey has been an active participant in a defined benefit plan for 21 years. During his last 5 years of employment, Joey earned $32,000, $48,000, $55,000, $95,000, and $105,000, respectively (representing his highest-income years). a. Calculate Joey’s maximum allowable benefits from this qualified plan (assume there are fewer than 100 participants). b. Assume that Joey’s average compensation for his three high years is $203,000. Calculate Joey’s maximum allowable benefits.

Correct Answer: a. The limitation is the smaller of $205,000 in 2013 (indexed each year) or 100% of the average of his three highest annual salaries:

$55,000 + $95,000 + = $85,000 maximum

$105,000 allowable benefit. 3

b. Smaller of: $205,000 in 2013 (indexed each year) or 100% times highest three years average, or $203,000. Thus, $203,000 is Joey’s maximum allowable benefit.

2299. Determine the maximum annual benefits payable to a participant from a defined benefit plan in the following independent situations: a. Jacob, age 67, has been a participant for 14 years, and his highest average compensation for 3 years is $92,000. b. Sloane, age 66, has been a participant for 8 years (12 years of service), and her highest average compensation for 3 years is $119,000.

Correct Answer: a. $92,000 in 2013. The maximum allowable benefits payable to Jacob from a defined benefit plan is the smaller of $205,000 in 2013 (indexed each year) or 100% of his average compensation for his highest three years of employment. b. $119,000 in 2013. The defined benefit plan must reduce Sloane’s maximum benefit payable [e.g., $205,000 in 2013 (indexed each year)] by one-tenth for each year of participation under 10 years (e.g., $205,000 × 8/10 = $164,000). Thus, the $119,000 average compensation amount is below the ceiling.

2300. Chee is a key employee of an H.R 10 (Keogh) defined contribution plan (a profit sharing plan) created by a partnership. Answer each of the following independent questions.

a. During 2013, $7,400 is contributed to the H.R. 10 plan for Chee. What amount is deductible if Chee’s earned income is $45,000 (after the deduction for one-half of self employment tax)? b. In 2013, $8,020 is contributed to the H.R. 10 (Keogh) plan on Chee’s behalf. If Chee’s earned income is $36,000 (after the deduction for one-half of self employment tax), what deduction, if any is allowed? c. The partnership had a bad year in 2013, and only $820 was contributed to the H.R. 10 plan on Chee’s behalf. Chee earned only $700 during 2013. Calculate Chee’s allowable deduction.

Correct Answer: a. Lesser of $51,000 or 20% × $45,000, so a $9,000 ceiling applies. However, because only $7,400 was contributed, only $7,400 can be deductible. b. Lesser of $51,000 or 20% × $36,000. Therefore, $7,200 can be taken as a deduction for AGI. c. Lesser of $51,000 or 20% × $700. Therefore, $140 can be deductible.

2301. On July 1, 2009, Red Corporation sold 100 of its common shares (worth $15 per share) to its employee Bobby for $5 per share. The sale was subject to Bobby’s agreement to resell the shares to the corporation for $5 per share if his employment is terminated within the following four years. The shares were valued at $26 per share on July 1, 2013. He sold the shares for $30 per share on September 15, 2013. No special election under § 83(b) was made. Identify the correct answer. a. Bobby will be taxed on what amount, if any, on July 1, 2009? (1) Ordinary income of $1,500. (2) Short-term capital gain of $1,500. (3) Long-term capital gain of $1,000. (4) Tax-exempt income of $1,500. (5) None of the above. b. Bobby will be taxed on what amount, if any, on July 1, 2013? (1) Ordinary income of $2,500. (2) Long-term capital gain of $2,500. (3) Ordinary income of $2,100. (4) Long-term capital gain of $2,100. (5) None of the above. c. Bobby will be taxed on what amount, if any, on September 15, 2013? (1) Long-term capital gain of $2,000. (2) Short-term capital gain of $2,000. (3) Long-term capital gain of $400. (4) Short-term capital gain of $400. (5) None of the above. d. Red Corporation will obtain what deduction, if any? (1) Ordinary deduction of $500 on September 15, 2013. (2) Ordinary deduction of $2,100 on July 1, 2013. (3) Ordinary deduction of $2,500 on September 15, 2013. (4) Ordinary deduction of $500 on July 1, 2013. (5) Ordinary deduction of $1,500 on July 1, 2013. (6) None of the above. e. Assume the same facts, except that Bobby makes an election under § 83(b). He will be taxed on what amount, if any? (1) Ordinary income of $1,000 on July 1, 2009. (2) Ordinary income of $1,000 on July 1, 2013. (3) Ordinary income of $2,000 on July 1, 2013. (4) Long-term capital gain of $2,000 on September 15, 2013. (5) None of the above. f. Red Corporation in e. above will obtain what deduction? (1) Ordinary deduction of $1,000 on July 1, 2009. (2) Ordinary deduction of $1,000 on July 1, 2013. (3) Ordinary deduction of $2,000 on July 1, 2013. (4) Ordinary deduction of $2,000 on July 1, 2013. (5) None of the above. g. Suppose Bobby under e. above voluntarily terminates his employment on January 3, 2013, and goes into business for himself. (1) Bobby may take a $1,000 deduction on his 2013 tax return. (2) Bobby may apply for a refund of the $1,000 on his 2009 tax return. (3) Red Corporation must repay the taxes saved on the $1,000 deduction, and Bobby gets no refund of the taxes paid on the $1,000 previously included in gross income. (4) Bobby obtains a $500 loss on the amount he paid for the stock initially when he sells it back to the corporation (i.e., he gets his money back). (5) None of the above.

Correct Answer: a. 5; no income recognized given that the resale feature qualifies as a substantial risk of forfeiture. b. 3; 100 shares × ($26 – $5) = $2,100 ordinary income. c. 4; 100 shares × ($30 – $26) = $400 STCG. d. 2; deduction in the same amount as Bobby recognized as compensation income ($2,100). e. 1; 100 shares × ($15 – $5) = $1,000 on July 1, 2009. f. 1; deduction equal to what Bobby recognizes as income and at same time ($1,000). g. 3; no refund of taxes Bobby paid on $1,000 recognized on July 1, 2009, as a result of the special election. Bobby has a $0 gain or loss on the resale of the stock to the corporation.

2302. Roxy, Inc., grants 1,000 NQSO to an employee, Carol, entitling her to purchase Roxy stock at $10 per share (the current price of the stock). Roxy simultaneously grants 1,000 ISOs to another employee, Donna, entitling her to buy 1,000 shares of Roxy at $10 per share over a two-year period. One year later, 2013, the stock has risen to $20 per share, and Carol and Donna both exercise their options in full, receiving stock not subject to an SRF. a. What happens to Carol tax-wise in 2014? b. What happens to Donna tax-wise in 2014? c. What happens to Roxy, Inc., tax-wise in 2014? d. What happens to Carol tax-wise in 2016 when she sells her stock for $37? e. What happens to Donna tax-wise in 2016 when she sells her stock for $38? f. What happens to Roxy, Inc., tax-wise in 2016 with respect to these transactions?

Correct Answer: a. Carol recognizes $10 per share of ordinary income or $10,000 (1,000 × $10) in 2014. b. Donna does not recognize any income at the time of the exercise (2014). c. Roxy, Inc., is entitled to a compensation deduction of $10,000 associated with Carol. d. Carol’s tax basis per share is $20 ($10 + $10). So when she sells her stock for $37 per share, she will have a $17,000 capital gain [1,000 × ($37 – $20)]. e. Donna’s tax basis is $10 per share. So when she sells her stock for $38 per share, she will have a $28,000 capital gain [1,000 × ($38 – $10)]. f. Roxy gets no deduction since there is no ordinary income to Carol or Donna.

2303. On November 19, 2012, Rex is granted a nonqualified stock option to purchase 100 shares of Tan Company. On that date, the stock is selling for $8 per share, and the option price is $9 per share. Rex exercises the option on August 21, 2013, when the stock is selling for $10 per share. Five months later, Rex sells the shares for $11.50 per share. a. What amount is included in Rex’s gross income in 2012? b. What amount is included in Rex’s gross income in 2013? c. What amount and type of gain is recognized by Rex in 2014? d. What amount, if any, is deductible by Tan Company in 2013? e. What amount, if any, is recognized in 2014 if the stock is sold for $9.50 per share?

Correct Answer: a. No gain is recognized on the grant date since the options have no readily ascertainable fair market value. b. $100 ordinary income [($10 – $9) × 100 shares = $100]. c. $150 short-term capital gain [($11.50 – $10) × 100 shares = $150]. d. $100 is deductible by Tan Company in 2013, the same amount that is taxed as ordinary income to Rex. e. $50 short-term capital loss [($9.50 – $10) × 100 shares = $50].

2304. On January 1, 2010, Gail (an executive) receives a warrant to purchase one share of stock at $70 and on the same date the fair market value of the stock is $100. The warrant has no restrictions and has a readily ascertainable fair market value on a stock exchange of $30. She exercises the warrant on May 15, 2010, and sells the stock for $200 on December 20, 2013. a. Calculate the amount Gail would recognize in 2010, if any. b. Calculate the amount Gail would recognize in 2013, if any. c. Suppose she sells the warrant in 2014 for $39. What amount would Gail recognize?

Correct Answer: a. Ordinary income of $30 in 2010. b. Long-term capital gain of $100 in 2013 [$200 – ($30 + $70)]. c. Long-term capital gain of $9 ($39 – $30) in 2014 on the sale of the warrant.

2305. What is a defined contribution plan?

Correct Answer: A defined contribution pension plan (or money purchase) defines the amount the employer is required to contribute (e.g., a flat dollar amount, an amount based on a special formula, or an amount equal to a certain percentage of compensation). A separate account must be maintained for each participant. Benefits are based solely on (1) the amount of contribution, and (2) income from the funds that accrues to the participant’s account. Consequently, actuarial calculations are not required to determine the employer’s annual contribution. Upon retirement, an employee’s pension amount depends on the value of his or her account. Although it is not mandatory, a plan may require or permit employee contributions to the pension fund. Since employers record the expense for both tax and accounting purposes when funding an employee’s account, book-tax differences do not often occur.

2306. What is a profit sharing plan?

Correct Answer: A profit sharing plan is a deferred compensation arrangement established and maintained by an employer to provide for employee participation in a company’s profits. Contributions are paid from the employer to a trustee and are commingled in a single trust fund. Despite the name, an employer does not have to have current or accumulated profits to make a contribution. The contribution formula can be based on compensation, earnings, gross sales, or any other measurement chosen by the employer.

In a profit sharing plan, a separate account is maintained for each participant. The plan must provide a definite, predetermined formula for allocating the contributions made to the trustee among the participants. Likewise, the plan must include a definite, predetermined formula for distributing the accumulated funds after a fixed number of years, on the attainment of a stated age, or on the occurrence of certain events such as illness, layoff, retirement, or termination of the plan. A company is not required to contribute a definite, predetermined amount to the plan every year, although substantial and recurring contributions must be made to meet the permanency requirement. Forfeitures arising under this plan may be used to increase the individual accounts of the remaining participants as long as these increases do not result in prohibited discrimination. Benefits to employees may normally be distributed through lump-sum payouts in a profit sharing plan. A § 401(k) cash or deferred arrangement plan is the most common type of profit sharing plan.

2307. Explain to a small business owner some advantages and disadvantages of a simplified employee pension plan (SEP).

Correct Answer: Defined benefit and defined contribution plans have become complex and costly due to restrictive legislation over the past 15 years. Thus, a SEP has become an alternative to these traditional plans. Some advantages of the SEP are as follows:

• Less costly and complex.

• Can establish after the end of the tax year.

• Vesting immediately.

There are still some disadvantages of a SEP:

• There is a 25% deduction limit for a SEP rather than a 100% deduction for certain defined contribution plans.

• Participation rules are stricter.

• There is no 10-year forward averaging technique for certain lump-sum distributions.

• Someone else other than a covered participant must be the trustee.

2308. What is a highly compensated employee?

Correct Answer: An employee is a highly compensated employee if, at any time during the year or the preceding year, the employee satisfies either of the following:

• Was a 5 percent owner of the company. • Received more than $115,000 (in 2013) in annual compensation from the employer and was a member of the top-paid group of the employer. The top-paid group clause is applicable only if the employer elects to have it apply. An employee whose compensation is in the top 20 percent of the employees is a member of the top-paid group.

2309. Under what circumstances is it advantageous for an employee to elect to be taxed immediately as ordinary income on the FMV in excess of the amount paid for restricted property?

Correct Answer: The special election may be advantageous if:

• The bargain element is relatively small. • Substantial appreciation is expected in the future. • A high probability exists that the restrictions will be met.

2310. From an employee’s point of view, discuss the difference between the tax treatment accorded to a nonqualified stock option (NQSO) that has an ascertainable fair market value and one that does not.

Correct Answer: If a nonqualified stock option (NQSO) has a readily ascertainable fair market value (i.e., a warrant traded on an established market), the value of the option must be included in the gross income of the employee at the date of grant. No income is reported when the employee exercises the warrant. Capital gain or loss treatment occurs when the employee disposes of the optioned stock. The employee’s basis at the date of disposition is the amount paid for the stock plus any amount reported as ordinary income.

Where a NQSO does not have a readily ascertainable fair market value, the employee does not recognize income on the grant date. Instead, in the year of exercise, the employee reports as ordinary income the difference between the fair market value of the stock on the exercise date and the option price. The amount paid for the stock, plus the amount reported as ordinary income, becomes the employee’s tax basis in the stock. Any appreciation above such basis is treated as a capital gain upon disposition.

2311. Compare a § 401(k) plan with an IRA.

Correct Answer: An instructor can use Concept Summary 19.4 to cover this comparison. Most employees will find a § 401(k) plan more attractive than an IRA. Probably the biggest limitation of an IRA is the $5,500 maximum shelter in 2013 (ignoring the catch-up provision). Under § 401(k), employees are permitted to shelter compensation up to $17,500 (in 2013). The restrictions on deducting contributions to IRAs for many middle-income and upper-income taxpayers may cause many employees to utilize § 401(k) plans more frequently.

Another difference between § 401(k) plans and IRAs is the manner in which the money is treated. Money placed in an IRA may be tax deductible, whereas dollars placed in a § 401(k) plan are considered to be deferred compensation. Thus, a § 401(k) reduction may reduce profit sharing payments, group term life insurance, and Social Security benefits.

2312. If a person has funds from sources other than retirement assets when he or she retires, which retirement asset should be spent first?

Correct Answer: The most tax-efficient result is to postpone the income tax obligation for as long as possible. Generally, retirement assets should be taken from assets or accounts in the following order so that the tax-deferred growth continues:

• Taxable accounts.

• Section 457 accounts of government employees since no penalties apply once the employee is separated from employment.

• Section 401(k) and nonprofit § 403(b) plans because the age 55 separation rule may apply.

• Traditional IRAs since a taxpayer must be at least age 59 1/2 to obtain penalty-free withdrawals.

• Roth IRAs since no required minimum distribution rules apply.

2313. In determining whether an entity is to be recognized as a corporation for Federal income tax purposes, classification under state law is not conclusive.

*a. True b. False

2314. Under the check-the-box Regulations, an entity is allowed an annual election on how it will be treated (e.g., corporation, partnership) for Federal income tax purposes.

a. True *b. False

2315. In order to obtain limited liability, a sole proprietor incorporates his business. Under the check-the-box Regulations, the entity can elect to be treated as a partnership.

a. True *b. False

2316. Under the check-the-box Regulations, a multi-owner entity that defaults (i.e., makes no election) will be taxed as a partnership.

*a. True b. False

2317. Taxpayers who want both limited liability and the passthrough of losses should choose either the Subchapter S or the LLC entity form of doing business.

*a. True b. False

2318. In terms of the pass-through of tax attributes to the owners, both partnerships and C corporations follow the conduit approach.

a. True *b. False

2319. Like individuals, corporations can take advantage of § 1031 to elect to defer recognizing gain on like-kind exchanges.

*a. True b. False

2320. The foreign tax credit is available only to individuals and not to C corporations.

a. True *b. False

2321. Some corporations (e.g., personal service corporations) cannot use a calendar year for tax purposes.

a. True *b. False

2322. Although the corporate income tax is progressive, the lower brackets are phased out at higher taxable income levels.

*a. True b. False

2323. Like individuals, corporations can claim both deductions for and deductions from AGI.

a. True *b. False

2324. For corporations and individuals, net short-term capital gains are taxed at ordinary income rates.

*a. True b. False

2325. For income tax purposes, excess capital losses of individuals and corporations are treated in the same manner.

a. True *b. False

2326. Both individuals and corporations can carry over excess charitable contributions indefinitely.

a. True *b. False

2327. A C corporation donates inventory (basis of $9,000, fair market value of $10,000) to charity. Under certain circumstances, it can claim a charitable contribution deduction of $9,500.

*a. True b. False

2328. The domestic production activities deduction (DPAD) for a corporation is 9% of the greater of QPAI or TI.

a. True *b. False

2329. The domestic production activities deduction of an S corporation can be passed through to its shareholders.

*a. True b. False

2330. In computing the NOL of a corporation, the dividends received deduction cannot be claimed.

a. True *b. False

2331. A corporation’s election to forego a net operating loss carryback is irrevocable and cannot later be changed.

*a. True b. False

2332. The amount of the dividends received deduction is affected by the percentage of ownership held by the recipient corporation in the paying corporation.

*a. True b. False

2333. Emerald Corporation and Gold Corporation each own 500 shares of IBM Corporation common stock and each receives the same amount of cash dividends on their stock investment. The dividends received deduction each corporation can claim will be the same.

a. True *b. False

2334. In determining which organizational expenditures can be amortized, it is the year incurred (and not paid) that controls if the corporation uses the cash method of accounting.

*a. True b. False

2335. In forming a corporation in 2013, organizational expenditures are incurred and paid. These expenditures must be amortized over a period of 180 months.

a. True *b. False

2336. If a corporation does not elect to amortize its organizational expenditures, such expenses can never be deducted.

a. True *b. False

2337. Organizational expenditures include the costs of transferring assets to the new corporation.

a. True *b. False

2338. If a C corporation has taxable income in excess of $18,333,333, the corporate tax rate is 35% on all of its taxable income.

*a. True b. False

2339. Qualified personal service corporations are subject to a flat income tax rate of 35% and are not allowed to use the graduated rates available to regular corporations.

*a. True b. False

2340. The AMT tax rate applicable to corporations is the same as that applicable to individuals.

a. True *b. False

2341. Certain small business C corporations are not subject to the alternative minimum tax (AMT).

*a. True b. False

2342. An individual with a fiscal year of July 1-June 30 has a due date for filing its Federal income tax return of October 15.

*a. True b. False

2343. The Schedule M-1 to Form 1120 starts with taxable income and, after making various negative and positive adjustments, ends up with net income per books.

a. True *b. False

2344. Schedule M-2 of Form 1120 reconciles unappropriated retained earnings at the beginning of the year with unappropriated retained earnings at year-end.

*a. True b. False

2345. Currently all C corporations must file a Schedule M-3 with Form 1120.

a. True *b. False

2346. A city donates land to a corporation in return for the corporation’s construction of an assembly plant on the site. The fair market value of the land is taxed to the corporation as income.

a. True *b. False

2347. When forming a corporation under § 351, recognized gain is the lesser of realized gain or the amount of boot received.

*a. True b. False

2348. In the formation of a corporation under § 351, the shareholder’s basis in the stock received is decreased by loss recognized and boot received.

a. True *b. False

2349. In a § 351 transfer to a controlled corporation, the corporation’s basis for the property it receives is the shareholder’s basis increased by any boot given by the corporation.

a. True *b. False

2350. A corporation whose major source of income is tax-exempt interest probably will not have much E & P.

a. True *b. False

2351. A distribution to a shareholder could result in dividend income even if the corporation had a deficit in current E & P.

*a. True b. False

2352. Property distributions to shareholders can result in the distributing corporation recognizing gains and losses.

a. True *b. False

2353. A shareholder’s basis in property received as a dividend is its fair market value on the date of the distribution.

*a. True b. False

2354. Sarah, a sole shareholder, rents property to her corporation. The rent income she receives from the lease is a constructive dividend to Sarah.

a. True *b. False

2355. Unlike regular dividends, constructive dividends need not be distributed to a corporation’s shareholders on a pro rata basis.

*a. True b. False

2356. If a corporation has no E & P, its shareholders will not have constructive dividends.

*a. True b. False

2357. Satisfying the definition of a small business corporation is no longer necessary once the S election is initially made.

a. True *b. False

2358. Under certain circumstances, an S corporation can have more than 100 shareholders.

*a. True b. False

2359. If the shareholders of a calendar year C corporation elect S status on Friday, March 15, 2013, the election is effective for all of 2013.

*a. True b. False

2360. An S corporation can be a shareholder in another corporation.

*a. True b. False

2361. Canary Corporation, a calendar year S corporation, issues some of its stock to a nonresident alien on March 5, 2013. Canary becomes a C corporation as of March 6, 2013.

a. True *b. False

2362. In order to voluntarily revoke an S election, all of the shareholders must consent to the revocation.

a. True *b. False

2363. On March 8, 2013, all of the shareholders of Kingfisher Corporation (a calendar year entity) voluntarily revoke the S election (effective as soon as possible). For all of 2013, Kingfisher Corporation will be a C corporation.

*a. True b. False

2364. The taxable income (or loss) of an S corporation is allocated to each shareholder on a per-share and per-day of stock ownership basis.

*a. True b. False

2365. In computing the taxable income (or loss) of an S corporation, a domestic production activities deduction (DPAD) is allowed.

a. True *b. False

2366. In computing the taxable income (or loss) of an S corporation, an NOL carryover from a prior year is not allowed.

*a. True b. False

2367. The amortization of organizational expenditures and the dividends received deduction are allowed in determining the taxable income of an S corporation.

a. True *b. False

2368. As a separately stated item, the long-term capital losses of an S corporation will pass through to its shareholders as short-term capital losses.

a. True *b. False

2369. Examples of separately stated items of an S corporation include tax-exempt interest.

*a. True b. False

2370. Darlene, a 30% shareholder in a calendar year S corporation, sells her stock on June 7, 2013. If the corporation has an operating loss for 2013, none of this loss can be claimed by Darlene because she was not a shareholder as of December 31, 2013.

a. True *b. False

2371. Jacob is a shareholder in Robin Corporation, an S corporation. Jacob’s share of Robin’s activities for the current year is a short- term capital gain of $80,000 and an operating loss of $50,000. As a result of these transactions, Jacob should increase his basis in the Robin stock by $30,000.

*a. True b. False

2372. An S corporation makes a $20,000 cash distribution to a shareholder whose basis in the stock is $19,000. The distribution will result in a $1,000 negative basis in the stock.

a. True *b. False

2373. If a shareholder in an S corporation makes a loan to the corporation, this increases the basis in his stock investment.

a. True *b. False

2374. Partnerships are not considered to be separate taxable entities and, consequently, are not taxed at the entity level.

*a. True b. False

2375. Like S corporations, partnerships serve as conduits for their owners.

*a. True b. False

2376. Even though partnerships are not subject to income tax, they are required to file income tax returns.

*a. True b. False

2377. If property contributed to a partnership is subject to a liability (assumed by the partnership), gain is recognized to the contributing partner to the extent of the liability.

a. True *b. False

2378. If a partnership interest is received in exchange for services rendered, ordinary income must be recognized by the recipient of the capital interest.

*a. True b. False

2379. In return for a 10% interest in the Sapphire Partnership, Lily transfers cash of $50,000 and land (basis of $200,000; fair market value of $300,000). Lily’s basis in her Sapphire Partnership interest is $250,000.

*a. True b. False

2380. Changes in the liabilities (trade accounts payable, bank loans, etc.) of a partnership will not affect the basis of a partnership interest.

a. True *b. False

2381. A partner’s basis in the partnership interest is decreased by his or her share of the capital losses incurred by the partnership.

*a. True b. False

2382. A partner’s basis in the partnership interest is increased by his or her share of the tax-exempt income received by the partnership.

*a. True b. False

2383. Jenny contributes land (adjusted basis of $300,000; fair market value of $280,000) and cash of $40,000 to the newly formed Green Partnership. Green will have a basis in the land of $280,000.

a. True *b. False

2384. The holding period to the partnership of property contributed by a partner includes the period during which the property was held by the contributing partner.

*a. True b. False

2385. A partnership’s ordinary income or loss is computed without considering any charitable contributions. This item is separately stated and passes through to the individual partners.

*a. True b. False

2386. The domestic production activities deduction (DPAD) is determined at the partnership level and then passed through to each individual partner based on his or her partnership interest.

a. True *b. False

2387. Partners can carry over partnership losses that exceed the basis in their partnership interests.

*a. True b. False

2388. If the basis of a partnership interest is exhausted, a partner can use loans made to the partnership to absorb excess losses.

a. True *b. False

2389. Jordan and his two brothers are equal owners in Taupe Partnership. If Jordan sells business property (basis of $230,000; fair market value of $180,000) to Taupe for $180,000, he cannot recognize the $50,000 realized loss.

*a. True b. False

2390. Guaranteed payments are deductible by the partnership and are taxable to the partner receiving the payments.

*a. True b. False

2391. Guaranteed payments have no effect on the basis of a partner’s interest in the partnership.

*a. True b. False

2392. Which of the following, if any, incorrectly characterize the check-the-box Regulations?

a. A one-owner business is a sole proprietorship if default (no election is made) occurs. b. A one-owner business can elect to be taxed as a corporation. c. If default (no election is made) occurs, a limited liability company is taxed as a partnership. *d. The check-the-box Regulations can apply to entities that are already incorporated under state law. e. None of the above.

2393. Which of the following, if any, correctly describe the status or nature of limited liability companies?

a. Can elect partnership tax status even if incorporated under state law. *b. One of the main reasons why the check-the-box Regulations were issued. c. Cannot elect to be taxed as a corporation. d. Statutes creating these entities have been adopted by only a minority of the states. e. None of the above.

2394. Which, if any, of the following rules relate only to the income taxation of C corporations?

a. A domestic production activities deduction is allowed. b. Like-kind exchanges are nontaxable. c. No deduction is available for inventory donated to charity. d. Net long-term capital gains are subject to a special alternative tax. *e. None of the above.

2395. Which of the following rules are the same as to the income taxation of individuals and C corporations?

a. Recapture of depreciation on real property (§ 1250). *b. Excess short-term capital losses are carried over as short- term capital losses. c. Dividends received deduction. d. Due date for filing the Federal income tax return. e. None of the above.

2396. In comparing regular (C) corporations with individuals, which of the following, if any, relate only to (C) corporations?

a. A net short-term capital gain is taxed as ordinary income. b. An election can be made to defer recognition of gain on an involuntary conversion under § 1033. c. The carryover period for excess capital losses is unlimited. *d. Excess capital losses can be carried back. e. None of the above.

2397. In comparing C corporations with individuals, which of the following, if any, relate only to individuals?

*a. Excess long-term capital losses are carried forward as long- term capital losses. b. Excess charitable contributions can be carried forward. c. Excess charitable contributions can be carried back. d. Estimated income tax payments may have to be made. e. None of the above.

2398. In 2012, Creeper Corporation had a $4,000 net long-term capital loss which it could not carry back. For 2013, it had the following capital transactions:

Long-term capital gain $2,000 Short-term capital gain 3,000

As a result of these transactions, for 2013 Creeper has a:

a. Net short-term capital gain of $1,000. b. Net short-term capital gain of $3,000. *c. Net long-term capital gain of $1,000. d. Carryover to 2014 of $2,000 long-term capital loss. e. None of the above.

2399. Madison, a calendar year individual taxpayer, has the following results for 2013: profit from operations of $200,000 and long-term capital loss from investments of $20,000. As to the capital loss, Madison may:

a. Carry back the loss for three years to offset any capital gains. b. Deduct $3,000 and carry forward $17,000 for five years as a long-term capital loss. *c. Deduct $3,000 and carry forward $17,000 indefinitely as a long-term capital loss. d. Deduct $3,000 and carry forward $17,000 indefinitely as a short-term capital loss. e. None of the above.

2400. Gray is a calendar year taxpayer. In early December 2013, Gray’s Board of Directors authorizes a pledge of an $80,000 donation to the Red Cross to help provide food and shelter to victims of natural disasters. The donation is paid as follows: $10,000 in late December 2013, $40,000 in February 2014, and $30,000 in early April 2014. Disregarding percentage limitations, the maximum deduction for 2013 is:

a. $80,000 if Gray is an accrual basis individual. *b. $50,000 if Gray is an accrual basis corporation. c. $50,000 if Gray is a cash basis corporation. d. $50,000 if Gray is an accrual basis individual. e. None of the above.

2401. Citron Company is a wholesale distributor of children’s clothing. During the year, it donates clothes (cost of $60,000; retail value of $100,000) to Clothes for Needy (a qualified charitable organization). Disregarding percentage limitations, Citron’s charitable contribution deduction is:

*a. $80,000 if Citron is a C corporation. b. $60,000 if Citron is a C corporation. c. $100,000 if Citron is a C corporation. d. $80,000 if Citron is an individual. e. None of the above.

2402. Skinner Corporation, a calendar year C corporation, had the following transactions during the current year:

Income from operations $800,000 Expenses from operations 900,000 Dividends from Siskin Corporation 200,000 (20% ownership)

Skinner’s taxable income (or NOL) for the current year is:

a. $100,000 taxable income. b. $40,000 NOL. *c. $60,000 NOL. d. $100,000 NOL. e. None of the above.

2403. During the current year, Goose Corporation (a calendar year, cash basis taxpayer) receives cash dividends as follows:

Source of Dividends Ownership Dividends Percentage Emu Corporation 90% $90,000 Robin Corporation 21% 60,000 Crane Corporation 10% 30,000

Presuming the taxable income limitation does not apply, Goose Corporation’s dividends received deduction for the current year is:

a. $126,000. b. $135,000. c. $141,000. *d. $159,000. e. None of the above.

2404. In the current year, Auburn Corporation (a calendar year taxpayer), has the following income and expenses:

Gross income from operations $1,000,00 0 Expenses from operations 1,100,000 Dividends from Blond Corporation 200,000

Auburn Corporation owns 20% of the stock in Blond Corporation. The dividends received deduction for the current year is:

a. $200,000. *b. $160,000. c. $100,000. d. $80,000. e. None of the above.

2405. In the current year, Mockingbird Corporation (a calendar year taxpayer) has the following income and expenses:

Gross income from operations $1,900,00 0 Expenses from operations 2,000,000 Dividends from Hummingbird 400,000 Corporation

Mockingbird Corporation owns 10% of the stock of Hummingbird Corporation. The dividends received deduction for the current year is:

*a. $210,000. b. $280,000. c. $300,000. d. $320,000. e. None of the above.

2406. Which, if any, of the following costs cannot be expensed or amortized as organizational expenses?

*a. Cost of issuing or selling stock certificates. b. Cost of organizational meetings of the board of directors. c. Fee paid to the state for incorporating. d. Legal fees incurred for preparing the corporate charter and by-laws. e. None of the above.

2407. Teal Corporation is incorporated in November 2013. The following formation expenses are incurred in 2013:

Legal and accounting expenses incident $4,000 to organization Expenses of temporary directors and 2,000 organizational meetings of directors and shareholders Expenditures connected with the 3,000 transfer of assets to Teal

Except for the legal and accounting expenses which are paid in 2014, all other expenses are paid in 2013. If Teal Corporation uses the cash basis and adopts a calendar year for tax purposes, the amount of organizational expenditures it can elect to expense for 2013 is:

a. $2,000. b. $5,000. c. $6,000. d. $9,000. *e. None of the above.

2408. Alabaster Corporation, a calendar year taxpayer, was formed on July 1, 2013, and incurred organizational expenditures of $52,000. Its deduction for these expenditures for 2013 can be a:

a. Maximum of $5,000. b. Maximum of $2,000 + (6/180 × $50,000). *c. Maximum of $3,000 + (6/180 × $49,000). d. Minimum of 6/180 × $52,000. e. None of the above.

2409. Two unrelated, calendar year C corporations have the following taxable income for the current year:

Olive Corporation Magenta Corporation Taxable income $70,000 $90,000

Magenta Corporation is a qualified personal service corporation. Based on these facts, their corporate tax liability is:

a. $24,500 for Olive and $18,850 for Magenta. b. $24,500 for Olive and $31,500 for Magenta. c. $12,500 for Olive and $18,850 for Magenta. *d. $12,500 for Olive and $31,500 for Magenta. e. None of the above.

2410. In completing Schedule M-1 (reconciliation of income per books with income per tax return) of Form 1120, to net income per books:

a. Add tax-exempt interest. *b. Add excess of capital loss over capital gain. c. Add proceeds of term life insurance received by the corporation due to the death of a key employee. d. Subtract Federal income tax. e. None of the above.

2411. To improve its liquidity, the shareholders of Spoonbill Corporation make a capital contribution of $200,000 in cash. To facilitate the building of a manufacturing plant, the City of Wichita donates land (worth $800,000) to Spoonbill Corporation. As a result of these transactions, Spoonbill has:

*a. No income and a basis in the land of zero. b. Income of $1 million and a basis in the land of zero. c. Income of $800,000 and a basis in the land of $800,000. d. No income and a basis in the land of $800,000. e. None of the above.

2412. Which of the following will help avoid the thin capitalization problem?

a. Avoid any provision for interest. b. Do not provide for a definite maturity date. c. Make repayment contingent on the corporation’s earnings. d. Subordinate the debt to other liabilities. *e. None of the above.

2413. Aiden and Addison form Dove Corporation with the following investments:

From From Addison Aiden Cash $800,00 0 Building (basis of $400,000, fair $300,000 market value of $300,000) Land (basis of $200,000; fair 500,000 market value of $500,000)

Dove issues stock equally to Aiden and Addison. One of the tax consequences of these transfers is:

*a. Dove has a basis in the building of $400,000. b. Addison has a recognized loss on the building of $100,000. c. Addison has a basis in the Dove stock of $800,000. d. Addison has a recognized gain on land of $300,000 but no recognized loss on the building. e. None of the above.

2414. Camilla and Dean form Grouse Corporation with the following investment:

From From Dean Camilla Cash $300,000 Property (basis of $320,000; fair $330,000 market value of $330,000)

Camilla and Dean each receive 300 shares of stock in Grouse Corporation and, in addition, Dean receives $30,000 in cash. As a result of the transfer, Dean’s basis in the stock and Grouse’s basis in the property will be:

a. $330,000 and $300,000. *b. $300,000 and $330,000. c. $310,000 and $300,000. d. $290,000 and $310,000. e. None of the above.

2415. Jim, Betty, and Bill form Crow Corporation. Jim and Betty each contribute cash of $500,000, while Bill contributes land (basis of $550,000; fair market value of $600,000). Each shareholder receives one-third of Crow’s stock, and Bill also receives $100,000 in cash. Which, if any, of the following choices is a correct result of the incorporation?

a. Bill’s basis in the Crow stock is $650,000. b. Crow Corporation’s basis in the land is $550,000. c. Crow’s basis in the land is $650,000. *d. Crow’s basis in the land is $600,000. e. None of the above.

2416. Mel, Fred, and Mary form Canary Corporation. Mel and Fred each contribute cash of $250,000, while Mary contributes land (basis of $100,000; fair market value of $300,000). Each transferor receives one- third of Canary’s shares, and Mary also receives $50,000 in cash. Which, if any, of the following choices is a correct result of the incorporation?

a. Canary Corporation’s basis in the land is $100,000. *b. Canary’s basis in the land is $150,000. c. Canary’s basis in the land is $300,000. d. Mary’s basis in the Canary stock is $450,000. e. None of the above.

2417. Jacob and Ashley form Junco Corporation with the following investments:

From From Ashley Jacob Cash $200, 000 Equipment (basis of $400,000; 30 fair market value of $300,000) 0,000 Land (basis of $300,000; fair $600,000 market value of $600,000)

Jacob and Ashley each receive one-half of the Junco stock. In addition, Ashley receives cash of $100,000. One of the results of these transfers is:

a. Jacob has a recognized loss of $100,000. b. Ashley has a recognized gain of $300,000. *c. Junco Corporation has a basis in the land of $400,000. d. Ashley’s basis in the Junco Corporation stock is $500,000. e. None of the above.

2418. As of January 1, 2013, Donald, the sole shareholder of Sandpiper (a calendar year C corporation), had a basis in his stock of $50,000. During 2013, Sandpiper had an operating loss of $100,000 and distributed a $70,000 dividend to Donald. As of January 1, 2013, Sandpiper had accumulated E & P of $200,000. What is Donald’s basis in his Sandpiper stock as of January 1, 2014?

a. ($120,000). b. ($20,000). c. $0. *d. $50,000. e. None of the above.

2419. As of January 1, 2013, Amanda, the sole shareholder of Tern (a calendar year S corporation), had a basis in her stock of $50,000. During 2013, Tern had a net operating loss of $100,000 and distributed a $70,000 dividend to Amanda. What is Amanda’s basis in her Tern stock as of January 1, 2014?

a. ($120,000). b. ($20,000). *c. $0. d. $50,000. e. None of the above.

2420. As of January 1, 2013, Owl Corporation (a calendar year C corporation) has a deficit in accumulated earnings and profits (E & P) of $150,000. In 2013, it has current E & P of $200,000 and distributes a $220,000 dividend to its shareholders. A result of these transactions is:

*a. As of January 1, 2014, Owl’s deficit in accumulated E & P is $150,000. b. As of January 1, 2014, Owl’s accumulated E & P is $50,000. c. The shareholders recognize dividend income of $50,000. d. The shareholders recognize dividend income of $220,000. e. None of the above.

2421. Jerry is the sole shareholder of Bluejay Corporation, a calendar year C corporation. As of January 1, 2013, Bluejay has a deficit of $50,000 in accumulated E & P. During 2013, it has current E & P of $300,000 and distributes a cash dividend of $400,000 to its shareholder. As of January 1, 2013, Jerry’s basis in the Bluejay stock is $75,000. One result of these transactions is:

a. Dividend income to Jerry of $250,000. *b. Dividend income to Jerry of $300,000. c. Dividend income to Jerry of $400,000. d. As of January 1, 2014, Bluejay Corporation has a deficit in accumulated earnings and profits of $150,000. e. None of the above.

2422. In converting a corporation’s taxable income for the year to current E & P, which of the following adjustments are proper?

a. Subtract the domestic production activities deduction (DPAD). b. Add Federal income taxes paid. c. Subtract the dividends received deduction. d. Add charitable contributions in excess of the 10% limitation. *e. None of the above.

2423. Snipe Corporation, a calendar year taxpayer, has total E & P of $800,000. During the current year, Snipe makes property distributions to Tracy (the sole shareholder) as follows:

Adjusted Basis Fair Market Value Stock investment $100,000 $ 90,000 Land 270,000 300,000

As a result of these distributions:

*a. Snipe Corporation must recognize a gain of $30,000 and no recognized loss. b. Snipe Corporation must recognize a gain of $30,000 and recognize a loss of $10,000. c. Snipe Corporation recognizes neither gain nor loss. d. Tracy will have a basis of $100,000 in the stock investment and $270,000 in the land. e. None of the above.

2424. Which of the following statements, if any, is characteristic of constructive dividends?

a. They must be available to the shareholders on a pro rata basis. b. They can occur even if the corporation has no earnings and profits. c. They must be formally declared and paid (i.e., declaration date, record date, payment date). *d. They are not deductible by the corporation. e. None of the above.

2425. Nick, Kristin, Spencer, Giselle, and Herbert are equal shareholders in Cuckoo Corporation. Which of the following, if any, could not be a constructive dividend?

a. Nick rents a building to Cuckoo. b. Kristen sells land to Cuckoo. *c. Cuckoo pays Spencer no salary for services he performs. d. Cuckoo makes an advance to Giselle. e. Herbert makes an interest-free loan to Cuckoo.

2426. Hailey redeems some of the stock she holds in Mallard Corporation for $300,000. The stock has a basis to her of $100,000 and has been held by her as an investment for many years. Mallard Corporation has more than $2 million in earnings and profits. In the year of the redemption, Hailey is in the 33% income tax bracket. The redemption generates a tax of:

a. $99,000 if it does not qualify for sale or exchange treatment. *b. $45,000 if it does not qualify for sale or exchange treatment. c. $45,000 if it does qualify for sale or exchange treatment. d. $66,000 if it does qualify for sale or exchange treatment. e. None of the above.

2427. Silver Corporation has held an S election since its formation five years ago. Which, if any, of the following will cause Silver to lose its election?

a. Silver buys stock in another corporation. *b. A shareholder sells her Silver stock to a partnership. c. A shareholder sells his Silver stock to a citizen of Italy who is a resident of New York City. d. A shareholder dies and the Silver stock passes to her estate. e. None of the above.

2428. On January 1, 2013, Copper Corporation (a calendar year S corporation for many years) became a C corporation. A possible reason for this change in status is:

a. The corporation had too much passive investment income for 2013. b. A partnership became a shareholder on March 4, 2013. *c. A majority of the shareholders met on March 14, 2013, and revoked the S election. d. All of the shareholders met on April 12, 2013, and revoked the election. e. None of the above.

2429. Which, if any, of the following transactions incurred by an S corporation is not a separately stated item?

a. Tax-exempt income. b. Foreign tax credit. c. AMT adjustments and tax preference items. *d. Amortization of organizational expenditures. e. Domestic production activities deduction (DPAD).

2430. Laurie is a 70% shareholder in Martin Ltd., a calendar year S corporation. As of January 1, 2013, Laurie’s basis in her stock investment was $400,000. During 2013, she made an additional capital contribution to Martin of $90,000 and also loaned the corporation $80,000. During 2013, Martin Ltd. had an operating loss of $900,000. One of the results of these transactions as to 2013 is that:

a. Laurie can recognize a loss of $400,000. b. Laurie can recognize a loss of $490,000. c. Laurie can recognize a loss of $630,000. d. As of January 1, 2014, Laurie’s basis in her stock is a negative $60,000. *e. None of the above.

2431. Jean is a shareholder in Parrot Corporation, a calendar year S corporation. During the year, Jean’s share of the pass-through from Parrot is:

• Dividend distribution by $30,000 Parrot • Long-term capital gain 5,000 • Operating loss 20,000 • Tax-exempt interest 10,000 income

If Jean’s basis in the Parrot stock was $100,000 at the beginning of the year, her basis at the end of the year is:

a. $45,000. b. $55,000. *c. $65,000. d. $95,000. e. None of the above.

2432. Elijah contributes land (basis of $80,000; fair market value of $400,000) for a 20% interest in a partnership. The land is subject to a mortgage of $50,000 which the partnership assumes. As a result of the transfer, Elijah:

a. Must recognize a gain of $40,000. *b. Has a basis in the partnership of $40,000. c. Has a basis in the partnership of $70,000. d. Has a basis in the partnership of $360,000. e. None of the above.

2433. In determining a partner’s basis in the partnership interest, which of the following reflects a correct adjustment?

a. Add capital losses. b. Subtract tax-exempt income. *c. Subtract for decrease in partnership liabilities. d. Add partnership operating loss. e. None of the above.

2434. In determining the taxable income (or loss) of a partnership, which, if any, of the following items must be considered?

a. Domestic production activities deduction. b. Charitable contributions. c. Qualified dividend income. *d. Guaranteed payments to partners. e. None of the above.

2435. Travis and his three sisters are equal partners in the Heron Partnership. In 2011, Travis sells property (basis of $300,000) to Heron for its fair market value of $280,000. In 2013, Heron sells the property to a third party for $290,000. Which of the following statements correctly describes these transactions?

a. Travis has no recognized loss, and Heron has a recognized gain of $10,000. *b. Travis has no recognized loss, and Heron has no recognized gain or loss. c. Travis has a recognized loss of $20,000, and Heron has no recognized gain or loss. d. Travis has a recognized loss of $20,000, and Heron has a recognized gain of $10,000. e. None of the above.

2436. Using the legend provided, classify each statement accordingly.Tax rates are progressive.A net short-term capital loss can be used to offset a net long-term capital gain.An excess long-term capital loss carryover is first used to offset current short-term capital gains.A domestic production activities deduction is available.Excess capital losses cannot be offset against ordinary income.A cash basis taxpayer cannot deduct a charitable contribution until the year of payment.Net long-term capital gains are subject to a preferential alternative tax rate.The foreign tax credit is not available.Like-kind exchange provisions apply.Interest on municipal bonds is excluded from gross income.Net short-term capital gains are taxed as ordinary income.Estimated tax payments may have to be made.Applies to both the income taxation of individuals and of C corporations Applies to both the income taxation of individuals and of C corporations Applies only to the income taxation of C corporations Applies to both the income taxation of individuals and of C corporations Applies only to the income taxation of C corporations Applies to both the income taxation of individuals and of C corporations Applies only to the income taxation of individuals Applies to neither the income taxation of individuals nor of C corporations Applies to both the income taxation of individuals and of C corporations Applies to both the income taxation of individuals and of C corporations Applies to both the income taxation of individuals and of C corporations Applies to both the income taxation of individuals and of C corporations

[a] 1. Tax rates are progressive. [b] 2. A net short-term capital loss can be used to offset a net long-term capital gain. [c] 3. An excess long-term capital loss carryover is first used to offset current short-term capital gains. [d] 4. A domestic production activities deduction is available. [e] 5. Excess capital losses cannot be offset against ordinary income. [f] 6. A cash basis taxpayer cannot deduct a charitable contribution until the year of payment. [g] 7. Net long-term capital gains are subject to a preferential alternative tax rate. [h] 8. The foreign tax credit is not available. [i] 9. Like-kind exchange provisions apply. [j] 10. Interest on municipal bonds is excluded from gross income. [k] 11. Net short-term capital gains are taxed as ordinary income. [l] 12. Estimated tax payments may have to be made.

a. Applies to both the income taxation of individuals and of C corporations b. Applies to both the income taxation of individuals and of C corporations c. Applies only to the income taxation of C corporations d. Applies to both the income taxation of individuals and of C corporations e. Applies only to the income taxation of C corporations f. Applies to both the income taxation of individuals and of C corporations g. Applies only to the income taxation of individuals h. Applies to neither the income taxation of individuals nor of C corporations i. Applies to both the income taxation of individuals and of C corporations j. Applies to both the income taxation of individuals and of C corporations k. Applies to both the income taxation of individuals and of C corporations l. Applies to both the income taxation of individuals and of C corporations

2437. Using the legend provided, classify each statement accordingly.Excess long-term capital losses are carried forward as short-term capital losses.Excess charitable contributions can be carried over for five years.Excess capital losses can be carried over indefinitely.All of the appreciation on ordinary income property donated to charity can be claimed as a deduction.Gain on an involuntary conversion cannot be deferred.Dividends received deduction is not available.Could be exempt from the AMT.Recapture of depreciation on real property (i.e., § 1250) can be more severe.Concept of itemized deductions is not relevant.The allowable deduction for charitable contributions is subject to percentage limitations.Once taxable income reaches a certain level, the benefits of the lower tax brackets phase out.The tax return is due on or before the fifteenth day of the fourth month following the end of the tax year.Applies only to the income taxation of C corporations Applies to both the income taxation of individuals and of C corporations Applies only to the income taxation of individuals Applies to neither the income taxation of individuals nor of C corporations Applies to neither the income taxation of individuals nor of C corporations Applies only to the income taxation of individuals Applies only to the income taxation of C corporations Applies only to the income taxation of C corporations Applies only to the income taxation of C corporations Applies to both the income taxation of individuals and of C corporations Applies only to the income taxation of C corporations Applies only to the income taxation of individuals

[a] 1. Excess long-term capital losses are carried forward as short-term capital losses. [b] 2. Excess charitable contributions can be carried over for five years. [c] 3. Excess capital losses can be carried over indefinitely. [d] 4. All of the appreciation on ordinary income property donated to charity can be claimed as a deduction. [e] 5. Gain on an involuntary conversion cannot be deferred. [f] 6. Dividends received deduction is not available. [g] 7. Could be exempt from the AMT. [h] 8. Recapture of depreciation on real property (i.e., § 1250) can be more severe. [i] 9. Concept of itemized deductions is not relevant. [j] 10. The allowable deduction for charitable contributions is subject to percentage limitations. [k] 11. Once taxable income reaches a certain level, the benefits of the lower tax brackets phase out. [l] 12. The tax return is due on or before the fifteenth day of the fourth month following the end of the tax year.

a. Applies only to the income taxation of C corporations b. Applies to both the income taxation of individuals and of C corporations c. Applies only to the income taxation of individuals d. Applies to neither the income taxation of individuals nor of C corporations e. Applies to neither the income taxation of individuals nor of C corporations f. Applies only to the income taxation of individuals g. Applies only to the income taxation of C corporations h. Applies only to the income taxation of C corporations i. Applies only to the income taxation of C corporations j. Applies to both the income taxation of individuals and of C corporations k. Applies only to the income taxation of C corporations l. Applies only to the income taxation of individuals

2438. Using the legend provided, classify each statement accordingly.Cannot be a shareholder in another corporation.Cannot have a partnership as a shareholder.Amount the corporation borrows from a bank increases the basis its shareholders have in their stock.Corporation can have both common and preferred stock outstanding.The corporation’s domestic production activities deduction is claimed by the corporation (not the shareholders).A dividends received deduction is not allowed.A deduction for organizational expenditures is allowed.Upon its formation, § 351 (transfers to controlled corporations) can apply.A resident and citizen of Mexico can be a shareholder.The corporation may utilize an NOL carryover from a prior year.A shareholder cannot have a negative basis in his or her stock investment.Tax-exempt income received by the corporation will pass through to its shareholders as tax-exempt income.Applies to neither the income taxation of C corporations nor of S corporations. Applies only to the income taxation of S corporations. Applies to neither the income taxation of C corporations nor of S corporations. Applies only to the income taxation of C corporations. Applies only to the income taxation of C corporations. Applies only to the income taxation of S corporations. Applies to both the income taxation of C corporations and S corporations. Applies to both the income taxation of C corporations and S corporations. Applies only to the income taxation of C corporations. Applies only to the income taxation of C corporations. Applies to both the income taxation of C corporations and S corporations. Applies only to the income taxation of S corporations.

[a] 1. Cannot be a shareholder in another corporation. [b] 2. Cannot have a partnership as a shareholder. [c] 3. Amount the corporation borrows from a bank increases the basis its shareholders have in their stock. [d] 4. Corporation can have both common and preferred stock outstanding. [e] 5. The corporation’s domestic production activities deduction is claimed by the corporation (not the shareholders). [f] 6. A dividends received deduction is not allowed. [g] 7. A deduction for organizational expenditures is allowed. [h] 8. Upon its formation, § 351 (transfers to controlled corporations) can apply. [i] 9. A resident and citizen of Mexico can be a shareholder. [j] 10. The corporation may utilize an NOL carryover from a prior year. [k] 11. A shareholder cannot have a negative basis in his or her stock investment. [l] 12. Tax-exempt income received by the corporation will pass through to its shareholders as tax-exempt income.

a. Applies to neither the income taxation of C corporations nor of S corporations. b. Applies only to the income taxation of S corporations. c. Applies to neither the income taxation of C corporations nor of S corporations. d. Applies only to the income taxation of C corporations. e. Applies only to the income taxation of C corporations. f. Applies only to the income taxation of S corporations. g. Applies to both the income taxation of C corporations and S corporations. h. Applies to both the income taxation of C corporations and S corporations. i. Applies only to the income taxation of C corporations. j. Applies only to the income taxation of C corporations. k. Applies to both the income taxation of C corporations and S corporations. l. Applies only to the income taxation of S corporations.

2439. During 2013, Scarlet had taxable income of $150,000 and the following capital asset transactions:

Long term: Gain $4,000 Loss 3,000

Short term: Gain $2,000 Loss 1,000

How are these transactions handled if Scarlet is: a. An individual? b. A C corporation?

Correct Answer: a. Netting the long-term and short-term transactions yields a $1,000 net LTCG and a $1,000 net STCG. The $1,000 LTCG is subject to an alternative tax and the $1,000 STCG is taxed at ordinary income rates. b. The same netting process (as in part a.) takes place, but the end result is different. As the alternative tax is not available to corporations, the full $2,000 gain ($1,000 LTCG + $1,000 STCG) is taxed at ordinary income rates.

2440. During 2013, Brown had taxable income of $200,000 and the following capital asset transactions:

Long term: Gain $ 1,000 Loss 3,000

Short term: Gain $ 6,000 Loss 10,000

How are these transactions handled if Brown is: a. An individual? b. A C corporation?

Correct Answer: a. Netting the long-term and short-term transactions yields a net LTCL of $2,000 and a net STCL of $4,000. Of the $4,000 STCL, $3,000 is applied against ordinary income. To be carried over to future years is a LTCL of $2,000 and a STCL of $1,000. Carryovers are indefinite (have no time limit). b. The same netting process (as in part a.) takes place, but corporations cannot apply capital losses against ordinary income. They can be carried back for three years and forward five (or until exhausted, if sooner). The $6,000 of losses ($2,000 LTCL + $4,000 STCL) become short term when carried back and carried forward.

2441. During 2013, Violet had the following capital gains and losses:

Long-term capital gain $4,000

Short-term capital gain 5,000

In 2012, Violet had a net long-term capital loss of $6,000 which could not be used. Assume all capital gains and losses result from the sale of securities held as investments. How are these transactions handled if Violet is: a. An individual? b. A C corporation?

Correct Answer: a. If Violet is an individual, the 2012 capital loss of $6,000 carries over as long term. It will offset the 2013 long-term gain of $4,000 and $2,000 of the short-term gain of $5,000. The balance of $3,000 short-term capital gain is taxed at ordinary income rates. b. If Violet is a C corporation, the 2012 loss carries over as short term. In 2013, it will offset the short-term gain of $5,000 and $1,000 of the long-term gain. The end result, a $3,000 long-term capital gain, is taxed at ordinary income rates.

2442. In December 2013, Swallow Company’s board authorizes a cash donation of $60,000 to the Hamilton County School District. The amount authorized is paid to the county in two $30,000 amounts in January and April 2014. Presuming a calendar year and accrual method taxpayer, when is the charitable contribution deduction available if Swallow Company is a: a. Partnership? b. C (regular) corporation?

Correct Answer: a. Both amounts are separately stated partnership items in 2014, as the year of payment controls. b. For year 2013 (i.e., the year authorized), $30,000 can be deducted as Swallow Company is an accrual basis taxpayer. The second $30,000 installment cannot be deducted until 2014 as the payment was not made on or before March 15, 2014. At the option of the taxpayer, the $60,000 paid can be deducted for year 2014.

2443. Ruby Company, a clothing retailer, donates children’s clothing to Salvation Army (a qualified charity). The clothing is part of Ruby’s inventory and has an adjusted basis of $100,000 and a fair market value of $180,000. What is the amount of Ruby’s charitable donation if it is a: a. Partnership? b. Corporation?

Correct Answer: a. $100,000. Inventory is ordinary income property. As a result, the deduction is limited to basis and none of the appreciation can be considered. b. $140,000. A special exception permits corporations to deduct one-half of the appreciation if the charity uses the inventory to care for the needy, children, and certain other purposes. Thus, $100,000 (basis) + $40,000 (50% × $80,000 appreciation) = $140,000.

2444. Crimson Corporation owns stock in other C corporations as follows: 90% of Maize; 30% of Mocha; and 10% of Saffron. During the current year, it receives dividends from the corporations as follows: $80,000 from Maize; $60,000 from Mocha; and $100,000 from Saffron. Presuming no taxable income limitations, what is Crimson’s dividends received deduction?

Correct Answer: $198,000 [(100% × $80,000) + (80% × $60,000) + (70% × $100,000)].

2445. Cerulean Corporation owns 6% of the stock in Auburn Corporation. During the year, Auburn received cash dividends of $800,000 from this investment. Determine Auburn Corporation’s dividends received deduction based on the following assumptions: a. Taxable income before the deduction is $960,000. b. Taxable income before the deduction is $720,000. c. Taxable income before the deduction is $480,000.

Correct Answer: a. $560,000 (70% × $800,000). b. $504,000 (70% × $720,000). The taxable income limitation applies. c. $560,000 (70% × $800,000). The taxable income limitation does not apply as the full dividends received deduction results in a net operating loss.

2446. Drab Corporation, a calendar year and cash basis taxpayer, is formed in December 2013. In the same month, expenses are incurred as follows:

Attorney fees for drafting the $14,000 charter and by-laws Expenses for issuing and selling 16,000 stock CPA fees for establishing accounting 7,000 system Fees for transfer of assets to the 10,000 corporation Expenses of organizational meetings 3,000 of temporary directors and shareholders Incorporation fee paid to the state 2,000

a. How much qualifies as organizational expenditures that can be deducted or amortized over a period of 180 months or more? b. Would it matter if some of these expenses were not paid until 2014?

Correct Answer: a. $26,000. Attorney’s fees ($14,000), CPA fees ($7,000), organizational meetings ($3,000), and incorporation fee ($2,000). Remaining expenses do not qualify for amortization. b. No. It is the year incurred that controls.

2447. Four unrelated, calendar year corporations are formed on January 1, 2013, and have the following qualified organizational expenditures: $4,000 for Rust Corporation; $15,000 for Orange Corporation; $53,000 for Pearl Corporation; and $56,000 for Chestnut Corporation. What is the maximum deduction each can claim for 2013?

Correct Answer: $4,000 for Rust Corporation; $5,667 [$5,000 + 12($10,000/180)] for Orange Corporation; $5,400 or $2,000 [$5,000 – ($53,000 – $50,000)] + $3,400 [($53,000 – $2,000)/180 × 12] for Pearl Corporation; and $3,733 ($56,000/180 × 12) for Chestnut Corporation.

2448. In the current year, Rose Corporation, a calendar year corporation, has taxable income from operations of $95,000 and a long- term capital gain of $10,000. a. What is Rose Corporation’s Federal income tax liability? b. What if Rose Corporation is a personal service corporation?

Correct Answer: a. $24,200 (15% × $50,000) + (25% × $25,000) + (34% × $25,000) + (39% × $5,000). Capital gains are included in ordinary income. b. $36,750 (35% × $105,000).

2449. For 2012, Plover Corporation, a calendar year taxpayer, had net income per books (after tax) of $75,250. Other transactions having a tax effect include:

Income tax liability $22,250 Interest on tax-exempt bonds 10,000 Excess capital losses 7,500 Excess charitable contributions 5,000 (not deductible due to the 10% limit)

What is Plover Corporation’s taxable income for 2012?

Correct Answer: $100,000. Using the approach taken by Schedule M-1 of Form 1120, proceed as follows:

Net income per books (after tax) $ 75,250 Add Federal income tax 22,250 Add excess capital losses 7,500 Add excess charitable contributions 5,000 Deduct interest on tax-exempt bonds (10,000) Taxable income $100,000

2450. In order to get the Cardinal Corporation to build a plant in the area, Union County gives it land worth $900,000. The land was purchased by the county many years ago for $50,000. a. How much income, if any, must Cardinal Corporation recognize for receiving the land? b. What basis will Cardinal Corporation have in the land?

Correct Answer: a. $0. b. $0.

2451. Arthur forms Catbird Corporation with the following investment:

Fair Market Asset Basis Value Marketable $600,000 $400,000 securities Land 500,000 800,000

Arthur receives all of the stock of Catbird. a. What is Arthur’s basis in the Catbird Corporation stock? b. What is Catbird Corporation’s basis in the marketable securities? The land?

Correct Answer: a. $1,100,000 ($600,000 + $500,000). b. $600,000 for the marketable securities and $500,000 for the land.

2452. Sophia and William form the Bobwhite Corporation with the following investments:

Fair Market Basis Value From Sophia— Cash $300,000 $300,000

From William— Land 280,000 350,000

Each person is issued one-half of Bobwhite’s stock and, in addition, William receives cash of $50,000. a. How much gain, if any, must William recognize and what basis does he have in the Bobwhite stock? b. What is Bobwhite’s basis in the land?

Correct Answer: a. $50,000 gain [the lesser of boot received ($50,000) or realized gain ($70,000)]. Basis in the stock is $280,000 [$280,000 (original basis) + $50,000 (gain recognized) – $50,000 (boot received)]. b. $330,000 basis in the land [$280,000 (William’s basis) + $50,000 (gain recognized)].

2453. Rowena is the sole shareholder of Rail, a calendar year C corporation. As of January 1, 2013, Rail had a $300,000 deficit in accumulated E & P. For 2013, it had current E & P of $110,000 and distributed to Rowena a cash dividend of $170,000. As of January 1, 2013, Rowena has a basis in her Rail stock of $40,000.

a. What are the tax consequences to Rowena of the distribution? b. What will be Rail Corporation’s balance in its accumulated E & P account as of January 1, 2014?

Correct Answer: a. Rowena accounts for the $170,000 distribution as follows:

Dividend income (covered by current $110,000 E & P) Return of capital (i.e., stock 40,000 basis) Capital gain 20,0 00 Total distribution $170,000

b. ($300,000). Only a loss can add to a deficit—a dividend cannot.

2454. Azure Corporation, a calendar year taxpayer, has taxable income of $850,000 for the current year. Among the transactions that related to Azure during the year are the following:

Income taxes $289,000 Domestic production activities 30,000 deduction Dividends received deduction 20,000 Interest on State of Maryland 10,000 bonds Charitable contributions in 15,000 excess of 10% limitation Fine paid for improper waste 5,000 disposal

Convert Azure’s taxable income to current E & P.

Correct Answer: $601,000 determined as follows: $850,000 (taxable income) + $30,000 (domestic production activities deduction) + $20,000 (dividends received deduction) + $10,000 (tax-exempt interest) – $289,000 (income taxes) – $15,000 (excess charitable contribution) – $5,000 (fine).

2455. Starling Corporation is a calendar year S corporation. For the current year, Starling had the following transactions:

Sales $800,000 Cost of goods sold 500,000 Interest from City of Rochester 4,000 bonds Payroll 100,000 Charitable contributions 5,000 Long-term capital gains 3,000 Rent expense on business premises 40,000

a. What is Starling Corporation’s taxable income? b. What are the separately stated items?

Correct Answer: a. $160,000. $800,000 (sales) – $500,000 (cost of goods sold) – $100,000 (payroll) – $40,000 (rent) = $160,000 (taxable income). b. $4,000 (interest on Rochester bonds); $5,000 (charitable contributions); $3,000 (long- term capital gains).

2456. Murrelet Corporation is a calendar year taxpayer that holds S status. For the current year, it had the following transactions:

Sales $500,000 Cost of goods sold (400,000) Interest on City of Charlotte 5,000 bonds Capital gains and losses Long-term gain (securities held as 8,000 investments) Long-term gain (antique car collection) 20,000 Short-term loss (2,000) Charitable contributions (4,000) Dividends received from another 9,000 corporation (1% ownership) Amortization of organizational (1,000) expenditures Payroll expenses (80,000)

a. What is Murrelet Corporation’s taxable income? b. What are the separately stated items?

Correct Answer: a. An operating profit of $19,000 results, computed as follows:

Sales $500,000 Cost of goods sold (400,000) Organizational expenditures (1,000) Payroll expenses (80 ,000) Operating profit ($ 19,00 0)

b. Separately stated items are as follows:

Interest on City of Charlotte bonds $ 5,000 Long-term capital gain (15%/0% 8,000 category) Long-term capital gain (28% category) 20,000 Short-term capital loss (2,000) Charitable contributions (4,000) Dividends received 9,000

2457. Spencer owns 50% of the stock of Crossbill Corporation, a calendar year taxpayer that has elected Subchapter S status. As of January 1, 2013, Spencer’s basis in the stock is $40,000. Crossbill has an operating loss of $100,000 for 2013 and taxable income of $60,000 for 2014. a. What should Spencer report for 2013? b. What should Spencer report for 2014? c. What is Spencer’s basis in Crossbill Corporation stock as of January 1, 2014? As of January 1, 2015?

Correct Answer: a. Spencer’s loss deduction is limited to his stock basis of $40,000. Of his $50,000 share of the loss, $10,000 cannot be deducted in 2013 due to a lack of basis. b. $30,000 income (50% × $60,000) and $10,000 loss from 2013. c. $0 as of January 1, 2014, and $20,000 as of January 1, 2015. For the 2013 loss, Spencer’s stock basis cannot be reduced below zero. The 2014 taxable income of $30,000 is added to basis, while $10,000 is subtracted for the unused 2013 loss now used. Thus, $0 (January 1, 2014, basis) + $30,000 (taxable income for 2014) – $10,000 (unused 2013 loss) = $20,000 (basis of stock on January 1, 2015).

2458. Isabella contributes land to the White Partnership for a 20% interest in capital and profits. The land (adjusted basis of $900,000 and fair market value of $1.4 million) is subject to a mortgage of $600,000 which the partnership assumes. What is Isabella’s basis in the White Partnership?

Correct Answer: $420,000. $900,000 (basis in the land) – $480,000 (portion of mortgage assumed by other partners).

2459. Jack is a 40% partner in the Gull Partnership, a calendar year taxpayer. As of January 1, 2013, Jack’s basis in his partnership interest is $100,000. Gull’s operating results are: $400,000 loss for 2013 and $300,000 profit for 2014. a. What is Jack’s basis for his partnership interest as of January 1, 2014? b. What is Jack’s basis for his partnership interest as of January 1, 2015?

Correct Answer: a. $0. $100,000 (1/1/13 basis in partnership interest) – $160,000 (Jack’s share of loss) = $0 (basis cannot be negative). b. $60,000. $0 (1/1/14 basis in partnership interest ) + $120,000 (Jack’s share of profit) – $60,000 (unabsorbed loss from 2013) = $60,000 (1/1/15 basis in partnership interest).

2460. Joshua is an equal partner with his parents in the Stilt Partnership. In 2010, he sells a tract of undeveloped land (basis of $250,000) to Stilt for its fair market value of $240,000. In 2013, Stilt resells the land to the local school district for $255,000. a. What are the tax consequences of the 2010 sale? b. Of the 2013 sale?

Correct Answer: a. Joshua has a realized loss of $10,000. However, none of the loss can be recognized due to the related-party rules of § 267. b. Stilt appears to have a realized gain of $15,000 [$255,000 (sale price) – $240,000 (cost basis)]. However, only $5,000 of this gain is recognized as Stilt can utilize Joshua’s disallowed loss of $10,000 to offset the remaining gain.

2461. Why were the check-the-box Regulations issued?

Correct Answer: The confusion over the income tax status of limited liability companies led to the issuance of these Regulations. Under the Regulations, the presence (or absence) of corporate characteristics are no longer controlling in determining corporate status.

2462. Regarding the check-the-box Regulations, comment on each of the following: a. An entity is a corporation under state laws b. A limited liability company defaults (no election is made). c. A one-person business defaults (no election is made).

Correct Answer: a. If an entity is incorporated under state law, then it is a corporation. The check- the-box Regulations are not applicable to such situations. b. Under the Regulations, it will be treated as a partnership for Federal income tax purposes. c. A sole proprietorship is the result for Federal income tax purposes, since no election was made.

2463. A calendar year taxpayer that in 2012 elected (under the check- the-box Regulations) to be taxed as a corporation wants to be taxed as a partnership in 2013. Is this permissible?

Correct Answer: No. New elections are permitted but only after a 60-month waiting period.

2464. The Carson brothers plan to open an auto repair facility. They anticipate losses in the early years of the business and are concerned about potential liability. What type of business entity should they choose?

Correct Answer: To achieve both the pass-through of losses and limited liability, the choice should be either an S corporation or a limited liability company (LLC). A C corporation offers limited liability but no pass-through, while a partnership provides a pass-through but no limited liability.

2465. Corporate taxpayers lose the benefits of lower tax brackets and the rates cease to be progressive once taxable income reaches a certain level. Explain this statement.

Correct Answer: The lower tax rates of 15% and 25% begin to phase out once the $335,000 bracket is reached. The benefit of the lower brackets is recovered by imposing a 39% rate. When taxable income reaches $15,000,000, a 38% rate recovers any income taxed at 34%. Once taxable income reaches $18,333,333, all benefits of the lower rates (i.e., below 35%) have been recouped. Thus, progression is eliminated and every dollar is subject to a marginal tax rate of 35%.

2466. In what manner do the tax rules dealing with capital gains differ for corporations and individuals?

Correct Answer: Corporations do not have access to the alternative tax available for net long-term capital gains. Thus, such gains are taxed as ordinary income and are not subject to a maximum rate of 15% (0% in some cases) available to individuals.

2467. In contrasting the tax treatment of excess capital losses of corporations with individual taxpayers: a. What advantages do corporate taxpayers enjoy? b. Disadvantages?

Correct Answer: a. Capital losses can be carried back three years. b. Cannot apply up to $3,000 offset against ordinary income. Carryovers limited to five years (not indefinite).

2468. The rules dealing with the recapture of depreciation on real estate are different for corporations than for individuals. Explain.

Correct Answer: Corporations selling depreciable real estate may have ordinary income in addition to that required by § 1250. Under § 291, the additional ordinary income element is 20% of the excess of the § 1245 recapture potential over the § 1250 recapture. Consequently, the § 1231 gain is correspondingly decreased by the additional recapture.

2469. Regarding the tax treatment of charitable contributions, corporations enjoy two major advantages over individuals. Elaborate.

Correct Answer: If certain requirements are satisfied, accrual basis corporations can deduct a contribution in the year it was authorized, rather than the year paid. Also, it is possible for corporate taxpayers to deduct one- half of the appreciation on certain ordinary income property.

2470. How does the domestic production activities deduction differ for corporations than for individuals?

Correct Answer: In the case of corporations, the domestic production activities deduction cannot exceed 9% of the lesser of QPAI or taxable income. In applying this limitation as to individuals, AGI is substituted for taxable income.

2471. Kestrel is a C corporation that is engaged solely in buying and reselling goods it acquires from Skimmer, another C corporation. Although Kestrel conducts no manufacturing activity and is exclusively involved in a marketing function, it may still be entitled to a domestic production activities deduction (DPAD). Explain how this could be possible.

Correct Answer: Skimmer does the manufacturing and the two corporations are members of the same affiliated group. Presuming the conditions of § 199 are satisfied, all members of the group could be eligible for a DPAD.

2472. The domestic production activities deduction (DPAD) is handled differently for a partnership than for a C corporation. Explain

Correct Answer: For a C corporation, the DPAD is claimed at the entity level. In the case of a partnership, however, the DPAD is a passthrough item and is claimed at the partner level.

2473. In determining the NOL of a C corporation, fewer adjustments are necessary than in the case of individual taxpayers. Why is this the case?

Correct Answer: In the case of C corporations, no adjustments are required for nonbusiness items (e.g., medical expenses) and artificial deductions (e.g., personal and dependency exemptions). A corporation’s loss more clearly approximates a true economic loss.

2474. When might an NOL carryback to prior years be ill-advised?

Correct Answer: A carryback would not be advisable if the corporation had little, if any, taxable income for the prior years and anticipates large amounts in the immediate future. Keep in mind, however, that the election to forgo the carryback is irrevocable.

2475. John owns stock in Green Corporation, which, in turn, owns stock in IBM. When IBM earns income, it is subject to the corporate income tax (i.e., the first tax). When IBM pays a dividend, Green Corporation must pay a corporate income tax on its share (i.e., the second tax). When Green Corporation pays a dividend, John must pay an individual income tax on what he receives (i.e., the third tax). Thus, the same income is subject to triple taxation. a. How does the tax law mitigate the tax on Green Corporation? b. How does the tax law mitigate the tax imposed on John?

Correct Answer: a. By allowing the dividends received deduction. Thus, Green Corporation can deduct a portion of the dividends received from IBM. b. Qualified dividends are subject to tax at the top rate applicable to net long-term capital gain (15%/0%). Consequently, the dividends John receives from Green Corporation are taxed at 15% (if he is in a 25% or higher income tax bracket) or 0% (if he is in the 10% or 15% income tax bracket).

2476. A corporation that claims the dividends received deduction may completely or only partly avoid any tax on the dividends involved. What accounts for the difference in tax consequences?

Correct Answer: The applicable deduction percentage makes the difference. If the percentage is 100%, no tax results. If, however, 80% or 70% applies, 20% or 30% of the dividend will be subject to tax. The deduction percentage depends on the amount of stock owned in the payer corporation [i.e., 100% (80% or more ownership), 80% (20% or more but less than 80% ownership), 70% (less than 20% ownership).

2477. Blue Corporation and Red Corporation are unrelated C corporations and hold as investments the same number of shares of stock in BP Corporation. As such, they each receive an equal amount of dividends from BP Corporation. If each claims a different dividends received deduction, what could cause the variation?

Correct Answer: The corporation claiming the lesser deduction probably is subject to the taxable income limitation. See the facts involving Osprey Corporation in Example 15 in the text.

2478. Is there a logical explanation why S corporations are not allowed a dividends received deduction when computing taxable income?

Correct Answer: The major purpose of the dividends received deduction is to mitigate (or eliminate) the triple taxation that occurs with the use of the corporate form. Since S corporations do not incur the corporate income tax, no triple taxation exists.

2479. In completing a Schedule M-1 of Form 1120, the Federal income tax per books is added to net income per books to arrive at taxable income. Why?

Correct Answer: The Federal income tax is claimed for accounting purposes and is, therefore, reflected in book income. Because it is not allowed for tax purposes, it is not considered in arriving at taxable income.

2480. As of January 1, 2013, Crimson Corporation, a calendar year C corporation, has a deficit of $300,000 in accumulated E & P. For 2013, it has current E & P of $100,000, and during the year distributes $120,000 as a dividend to the shareholders. Comment on the following assumptions. a. The shareholders recognize no dividend income as Crimson still has negative E & P for the year of the distribution [($300,000) beginning deficit + $100,000 current E & P = ($200,000) ending deficit]. b. Crimson’s accumulated E & P on January 1, 2014, is ($320,000).

Correct Answer: a. Distributions are treated as first coming from current E & P. Thus, the shareholders will have dividend income of $100,000. The remaining $20,000 distributed is treated as a return of capital. See the results reached in Examples 24 and 25 of the text. b. Crimson Corporation’s balance in accumulated E & P as of January 1, 2013, is ($300,000). The $120,000 distribution negated the $100,000 of current E & P, and the $20,000 balance cannot add to the deficit in accumulated E & P

2481. In adjusting the taxable income of a corporation to arrive at current E & P, the dividends received deduction is added. Why?

Correct Answer: E & P represents a corporation’s economic capacity to pay dividends to its shareholders. Since the dividends provide additional funds, they add to a corporation’s ability to pay dividends.

2482. Explain how a constructive dividend result could occur in each of the following situations. a. A shareholder is employed by a corporation. b. A shareholder lends funds to the corporation. c. A corporation sells property to the shareholder. d. A corporation rents property from the shareholder. e. A shareholder drives a car owned by the corporation.

Correct Answer: a. If the salary paid to the shareholder- employee is deemed to be unreasonable, a constructive dividend results. This has the effect of converting the payment from compensation income to dividend income and makes it nondeductible to the payor corporation. b. The main problem with shareholder loans to the corporation is the thin capitalization danger. In this eventuality, the debt is reclassified as equity and any interest or debt repayment is a constructive dividend. c. If the property is sold for less than its fair market value, the difference is a constructive dividend. d. If the corporation pays more than a fair rental for the property, the difference is a constructive dividend. e. If the use of the car is personal, the fair rental value of such use is a constructive dividend.

2483. Describe how a Subchapter S election can be lost. a. Voluntarily. b. Involuntarily.

Correct Answer: a. A voluntary termination occurs when the holders of a majority of the shares choose to do so. b. An involuntary termination occurs when the entity ceases to qualify as a small business corporation. In certain cases, an involuntary termination occurs when the corporation has too much passive investment income.

2484. Prior to the end of its tax year, the shareholders of an S corporation either make additional capital contributions or advance funds in the form of loans. Could this shareholder activity be motivated by income tax considerations? Explain.

Correct Answer: Yes, it could be motivated by income tax considerations. An S corporation may be involved that is anticipating net losses for the year. If the shareholders lack basis in order to take advantage of the passthrough of such losses, they could be making additional capital contributions or loans to increase basis.

2485. Nick and Ava each own a 20% interest in the Tanager Partnership. For the current year, one of the pass-through items is a charitable contribution. On their individual income tax returns, Nick can deduct his share of the charitable contribution but Ava cannot. What could cause the disparity in results?

Correct Answer: Most likely, Ava is claiming the standard deduction, while Nick is itemizing his deductions from AGI. Less likely, both parties are itemizing, but Ava is subject to the percentage limitations imposed on charitable contributions, while Nick is not.

2486. What causes a partner’s basis in a partnership interest to fluctuate?

Correct Answer: A partner’s basis is increased by additional contributions and the sum of the partner’s share of the partnership’s:

• Taxable income (including capital gains).

• Tax-exempt income.

• The excess of the deductions for depletion over the basis of the partnership’s property subject to depletion.

Similarly, a partner’s basis is decreased by the distributive share of the following:

• Partnership losses (including capital losses).

• Partnership expenditures that are not deductible in computing taxable income or loss and that are not capital expenditures.

Changes in partnership liabilities also affect the basis of a partnership interest.

2487. Although some differences exist, the tax treatment of S corporations and partnerships are very similar. In this connection, comment on the following. a. Tax at the entity level. b. Conduit approach. c. Treatment by owners (i.e., shareholders, partners) of excess entity losses.

Correct Answer: a. Generally, the entity is not taxed. In limited situations (e.g., built-in gains, excessive passive investment income), S corporations may incur a tax. b. Income, deductions, credits do not lose their identity as they pass through to the owners. c. Excess losses area carried over by the owners and applied against future basis increases. In taking advantage of a loss, however, only in S corporations can owners use the basis in loans made to the entity.

2488. Margie owns a 30% direct interest in the capital and profits of a family partnership. During the year, Margie sells land (basis of $400,000) to the partnership for its fair market value of $380,000. Could there be a reason why Margie’s realized loss of $20,000 from the sale cannot be recognized? Explain.

Correct Answer: The loss can be recognized unless Margie and the partnership are related parties. Margie has no problem as to her direct ownership, but what about indirect ownership? Thus, who are the other partners and are they related parties?

2489. In some situations, a partnership makes guaranteed payments to one or more of its partners. a. What are these payments? b. What are their tax effects to the partnership? c. What are the tax effects to the partner?

Correct Answer: a. These are payments made by a partnership to one or more of its partners for services rendered or for the use of capital, to the extent they are determined without regard to the income of the partnership. b. Guaranteed payments are deductible by the partnership and taxed as ordinary income to the receiving partner. c. Guaranteed payments are taxed as ordinary income to the recipient partner. They have no effect on the basis in the partnership interest.

2490. For various reasons, Bill would like to incorporate his very successful business. Before doing so, however, he has the following reservations:

· Recognizing gain on appreciated business assets.

· Avoiding (or minimizing) any future corporate income tax.

· Retaining ownership of the building occupied by the business.

Suggest a resolution of Bill’s concerns.

Correct Answer: Through the proper use of § 351, Bill will avoid recognizing any gain on the incorporation of his business. The corporate income tax can be avoided (or minimized) by the judicious combination of salaries, rent, and interest payments to the shareholders on the part of the corporation. As an alternative, an election under Subchapter S can be made.

There is no requirement that all of a business be transferred to the newly formed corporation. Thus, Bill can retain title to the building. In fact, renting the property to the corporation will reduce taxable income and help avoid any corporate income tax (see prior comment).